Board II Review

March 28, 2018 | Author: ougeta | Category: Peripheral Neuropathy, Foot, Human Anatomy, Lower Limb Anatomy, Clinical Medicine


Comments



Description

Board II ReviewMedicine 1. Dsecription Infectious disease An infection is the detrimental colonization of a host organism by a foreign species. In infection, the infecting organism seeks to utilize the host's resources in order to multiply at the expense of the host. The infecting organism, or pathogen, interferes with the normal functioning of the host and can lead to chronic wounds, gangrene, loss of an infected limb, and even death. The host's response to infection is inflammation. Top 3 single agent disease killers are: HIV/AIDS, TB, and malaria. Childhood diseases include pertussis, poliomyelitis, diphtheria, measles, and tetanus. Tropical diseases include trypanosomiasis, Chagas disease, schistosomiasis, leishmaniasis, lymphatic filariasis, and onchocerciasis. Calor, rubor, dolor, tumor and loss of function. Constitutional s/s: chills, malaise, loss of appetite and GI distress feverà 102 F (must do blood cultures to r/o sepsis) CBCà left shift where there is an increase in immature neutrophils and the total WBC countis elevated above 10,000 Low platelet count (>150,000) ABGs that show acidosis, and abnormal Creatinine and BUN that shows early kidney disease Variable by the infecting organism, location of the infection and extent HIV antiviral agents 1. Nucleoside reverse transcriptase inhibitors: Slows or prevents the formation of DNA copies of HIV in infected cells • All can causes fatal lactic acidosis, fat redistribution and hyperlipidemia Drugs Description/MOA Indication Contraindications Dose Zidivudine Ist line DOC HIV, pregnanst women-given after the 13-14th wk of gestation, 200mg po tid (AZT) or PO and Parenteral reduces transmission to the fetus from 26%-8% (Retrovir) Anemia, neutropenia, N/V, headaches, fatigue, confusion myopathy and toxic hepatitis Stavudine Used as a 2nd line substitute Similar to AZT 40 mg po bid (D4T-Zerit) for AZT, often combied with DON’T COMBINE W/AZT ddI or 3TC Peripheral neuropathy Didnosiine Used in combination with Combination drug 200mg po bid (ddI-Videx) AZT or D4T More toxic, causes peripheral neuropathy, pancreatitis, GI disturbances, lactic acidosis and retinal depigmentation Lamivudine Used in combination with Combination drug, also effective for HBV 150mg po bid (3TC-Epivir) AZT or D4T Less toxic and better tolerated with less side effects Zalcitabine Used in combination with Combination drug only, never used alone N/A (ddC-Hivid) AZT or D4T Least potent, causes peripheral neuropathy, rash, stomatitis, esophageal ulceration, pancreatitis and fever Combinations: Zalcitabine lamivudine (Combivir) 1 tab po bid 2. Non-nulcleoside reverse transcriptase inhibitors • Used synergistically with nucleoside Rx: resistance develops too rapidly if used alone • Comon side effects: rash, fever, nausea and headache • Preparations o Nevirapine (Viramune) 200mg po tid o Delaviradine (Rescriptor) 400mg po tid 3. Protease inhibitors • Actions: : prevents cleavage of protein precursors essential for HIV maturation à inhibition of the infection of new cells and replication • Problems: o Can cause increased bleeding and hyperglycemia à chemical diabetes • Preparations o Sanquanavir (Invirase) Highly active against HIV including AZT resistant strains, well tolerated 600mg po tid o Ritonavir (Norvir) well absorbed and produces high levels in the serum and lymph nodes. Common side effects: N/D/D, parasthesis, altered taste and renal failure 600mg po bid o Indinavir (Crixivan) 800mg po qid Epidemiolgy Clincal presentation Labs Treatment 1 o o o o Nelfinavir (Viracept) 750mg po tid Aprenavir (Agenerase) Lopinivir Opinivir/ ritonavir (Kalestra) –fixed dose combination DOC for tx of HIV in adults: • 2 nucleoside inhibitors and 1 protease inhibitors o Combivir and nelfanivir (1200mg) • 2 nucleoside inhibitors and 1 non-nucleoside inhibitors o Combivir and delaviritide (Rescriptor) Drugs for Tuberculosis Mycobacterium TB cultures take two weeks or longer Most commonly prescribed drug regimen o Rifampin 600mg and isoniazid 300mg in a single daily dose for 18monà conventional therapy o 9 months is the short course of treatment 5% relapse rate is acceptable • • • Empirical Tx: anti-mycobacterial therapy: Initial 4 drug combo ***TQ o Isoniazid Rifampin o Pyrazinamide Ethambutol or streptomycin (if INH resistant) • Then INH and rifampin w/o pyrazinamide for the next 6mo Isoniazid **TQ Description Isonicotinic acid hydrazide 1952-synthetic bacteriocidal MOA Interferes with cell wall mycolic acid synthesis in the bacteria Indications Cornerstone of TB tx: all forms of TB and TB prophylaxis Pharmacology Rapidly and completely absorbed po, Contraindications Previous isoniazid associated hepatic injury Warnings Severe and fatal hepatitis, monitor and interview pts monthly, and run LFTs, enzymes are elevated in 10% of patients 10-20% of pts have 4-5X normalß discontinue trt Often pyridoxine deficiency (VitB6) is major s/e resulting in peripheral neuropathy Dose Adults: 300mg po qd single dose Rifampin MOA Indications Pharmacology Contraindications Susceptible Organisms Warnings Rx interactions Preparations Dose Bacteriocidal-blocks RNA polymerase and protein synthesis Staph, MAC (AIDS), H. flu, meningitis (in children) used with dapsone in leprosy PO only, well absorbed from GI; 600mg dose will yield 7mg/ml (peak serum conc) Hepatotoxic-alcoholics and preexisting liver problems are most prone Coagulase po/neg staph, Neisseria, H. flu, Cl difficile, useful in pts with Legionnaire’s Disease who have fialed to respond to erythromycin, may be combined with nafcillin or vancomycin in the tx of chronic staph osteomyelitis Hepatotoxic; GI disturbances; skin rash; thrombocytompenic purpura, may turn body fluids a red-orange and may permanently stain contact lenses or implants + cytoP50à will decrease the effectiveness of co-administered drugs: protease inhibitors, BCP, warfarin, quinidine, zidovudine, itraconazole, fluconazole, and ketokonazole Combo with Rifampinà Rifamate 600mg po qd along with Isoniazid B. Neurologic disorders 1. Peripheral neuropathies Peripheral causes of peripheral neuropathy D A N G T H R A P I S T Diabetic-Alcoholic-Nutritional-Guillame Barre-Toxic-Herditary-Reccurrent-Amyloidosis-Porphyria-Infectious- Systemic-Tumor 1. Diabetic peripheral neuropathy 2 Capasaicin to target substance P, Baclophen, m relaxants, TCA’s, anti-seizure meds (Tegretol), GABApentin,(post herpetic neuraligia req titrations up) OD: dizziness, drowsiness, alters ability to drive and use heavy machinery. Excreted unalteredà no kidney stress Neurontin 900mg/d (normal dose) initially, (1 tab) 300mg/d then (2 tabs) 600mg/d then 900mg/d (3 tabs) If Neurontin doesn’t work, use amytriptiline 25mg w/pm dose à + DOPA and NE receptors Diabetic Peripheral Neuropathy Peripheral neuropathy is an all too frequent complication of diabetes affecting sensory, motor and autonomic neurons of the peripheral nervous system and the organs these neurons innvervate. Hyperglycemia and the duration of the disease appear to be the primary factors in its development. Distal symmetrical polyneuropathy is the most common type of neuropathy in diabetics. It develops insidiously may affect the small sensorimotor nerve fibers, large sensorimotor nerve fibers or both. Small, unmyelinated C fibers are composed of autonomic and sensory axons that transmit thermal perception and sympathetic function. These are affected early in the disease process. Patients present with prominent paresthesias and autonomic nervous system dysfunction recognized by the presence of orthostatic hypotension, resting tachycardia and distal anhydrosis. Large myelinated axons include both motor and sensory nerves. They conduct proprioception, light touch, vibratory and pain sensations. Symptoms of large fiber involvement include tingling, burning, numbness, allodynia or deep lancinating pain. Sensory ataxia may occur as a result of diminished vibratory and proprioceptive sense. Sensory changes do not always correlate with nerve conduction deficits. Deep tendon reflex responses are attenuated or absent and there may be distal motor weakness. The neuropathy develops in a length-dependent fashion, progressing from distal to proximal in a “stocking and glove” distribution. Progression of nerve injury leads to the loss of protective threshold or the ability to detect small objects or stimuli resulting ultimately in the neurotrophic or insensate diabetic foot. This is the cause of diabetic ulceration in up to 85% of patients. The exact pathophysiology of nerve damage in diabetes remains unclear. A number of theories exist which include the polyol pathway, microcirculation complications secondary to the stimulation of protein kinase and the non-enzymatic glycosylation of proteins throughout the body. The polyol pathway has long been implicated. Peripheral nerve tissue does not require insulin for glucose uptake. Hyperglycemia results in increased cellular glucose levels within nerve tissue which require an alternative catabolic pathway to be cleared. Via oxidative reactions, glucose is converted to sorbitol and sorbitol converted to fructose. Initially it was believed that the accumulation of sorbitol and fructose lead to osmotic stress resulting in nerve injury. However, it is now thought that it is the oxidative stress resulting from the breakdown of intraneural glucose that metabolically compromises neurons and leads to nerve damage. Functional loss of axons seems to occur as a length-dependent loss result in an initial distal neuropathy Intracellular hyperglycemia stimulates the activation of protein kinase C or PKC. This enzyme facilitates the transfer of phosphate groups from a donar molecule. Although there are many isoenzymes of protein kinase C, the beta-2 form has been implicated as the mediator of microvascular damage. These elevated levels of protein kinase C beta-2 result in increased basement membrane matrix protein deposition, leukocyte activation and smooth muscle proliferation and contraction. This process results in decreased endoneural blood flow resulting in nerve damage. Exposure of proteins to high levels of glucose initiates a multi-step process resulting in non-enzymatic glycosylation of these proteins - referred to as advanced glycation end-products or AGEs. Proteins, lipids and nucleic acids are all affected with resultant alteration of metabolic function. The large protein complexes may also be difficult for the body to clear resulting in AGE accumulation in susceptible tissues. Interaction with collagen in endoneural vessel walls thickens the walls compromising microcirculation to the nerves. The wide reach of diabetic neuropathy therefore results in many changes in the lower extremity. Sensory involvement results in a loss of protective threshold and the development of a neurotrophic foot. It is recognized early on as a loss of protective threshold and dorsal column involvement characterized by a loss of vibratory and position sense. This is the most prominent factor in the development of foot ulceration and the clinical path to lower extremity amputation. Motor involvement affects initially the intrinsic musculature of the foot leading to what is sometimes referred to as an “intrinsic minus” foot. Atrophy of the intrinsic musculature results in digital contractures, plantar prominence of the metatarsals and abnormal distribution of the normal weight bearing load with ambulation. In advanced neuropathy a drop foot may develop secondary to anterior compartment muscle wasting in the lower leg. The gastrocsoleus complex, having lost its antagonistic muscle group, then gains mechanical advantage resulting in ankle joint equinus. This deformity adds further to the weight bearing load borne by the forefoot placing the patient at even greater risk for forefoot ulceration. Autonomic nervous system involvement in the lower extremity results in a profound vasodilation of all vessels to the lower extremity and sudomotor changes. The foot will present clinically as warm, erythematous and dry. Increased vascular flow to the foot results in demineralization of bone; it is literally “washed away.” This is a major contributing factor to the pathogenesis of Charcot joint disease. Medical treatment for diabetic peripheral neuropathy must first start with rigid glucose control and patient education regarding the risks and hazards associated with nerve damage. Superficial nerve pain can be managed with capsaicin creams. Deeper nerve pain may be managed with tricyclic anti-depressants medications such as amytriptyllene or anti-seizure medications such as gabapentin. Muscle relaxants may provide relieve of deep pain. Disease modifying drugs that would modulate the pathogenesis of neuropathy are in clinical trials and will open a new frontier in the prevention of neuropathic complications in the diabetic. Treatment in the lower extremity should be directed towards preventing ulceration. Accommodative shoe gear is indicated in all patients who have lost protective threshold. A laminated plastizote and poron insole provides the ability to off-load plantar prominences and provide absorption of abnormal shearing forces. Often an extra-depth shoe will provide ample room for the diabetic foot, however, if severe foot deformities are present, custom molded shoes are indicated. Physical therapy is an important adjuvant therapy in the diabetic with treatments directed towards increasing the patients balance and muscular strength. 2. Alcoholic • The cause of alcoholic neuropathy is controversial but may be because of the toxic effect of alcohol on nerve tissue. It is likely also associated with nutritional deficiencies and may be indistinguishable from nutritional-related neuropathies such as beriberi. Treatment 3 severe cases – recovery in 1-2 yrs. It may be described as presentation “uncomfortable. its presence is recognized when after appropriate thiamine and magnesium treatment. primarily in the LE. Toxic-drugs 4 . vitB12 neuropathy-some studies advocate iron deficiency as an underlying etiology Epedemiology <10% of the population are affected. In severe cases. vitE may also help – at least 400 IU/day is recommended. Nutritional Restless Leg Syndrome “Ekbom’s Syndrome” Description def of zinc or folate.20% of pts also have s/s in the UE. evaluation of cerebrospinal Changes fluid – increased protein w/ few monocytes Clinical onset variable. Moderate exercise may be helpful but strenuous night time exercises Mod! Activityà may actually exacerbate the symptoms. Mx of paralysis is palliative . Pts may suffer with periodic limb movements of sleep (PLMS) Treatment Ca/Mg supplements have been found to be useful. mild cases – complete recovery in 1-2 mths. and the patient Vitaminsà should be advised to eliminate caffeine and tobacco. muscle wasting with loss of function presentation or paralysis of the lower extremities. autonomic nervous system is variably Course of the involved. Clinical Symptoms of dry beriberi include pain. • Prolonged heavy use of alcohol. motor activity. most frequent antecedent infection is Campylobacter jejuni.5/100. improvement begins with gradual resolution of the paralysis over weeks to months. and potential brain damage and death.~33% of patients require intubation and assisted ventilation. burning feet. usually the distal limbs are first involved with weakness developing either acutely (within days) or subacutely Features and (within four weeks) symptoms are roughly symmetrical – loss of tendon reflexes. persistent urge to wiggle and move associated w/fibromyalgia. Anticonvulsantsàquiet the opiods àsevere Drugsà work in CNS be careful will lead to tolerance levodopa/carbidopa 25/100 mg tid hs may be effective – the controlled release preps may be a bit more beneficial involuntary mvmts cloazepam may be effective if the others fail to relieve the s/s symptomatic relief sedating anti-histamines like diphenhydramine 3. a major cause of gastroenteritis. brief plateau phase.Beriberi Thiamine essential co-factor in CHO catabolismà areas w/ most enzymatic activity in brain resulting in high Gl-demand à greatest risk Description Deficiency in thiamine (vitamin B1) There are two major manifestations of thiamine deficiency: cardiovascular disease (wet beriberi) and nervous system disease (“dry beriberi” and Wernicke-Korsakoff syndromeàwhich is alcohol-related brain damage affecting language and thinking). incidence is 1. limbs treated by supporting splints to prevent contracture. paralysis of the legs. tingling. believed to be 2o to a disturbance of the immune syndrome. painful. will relieve the symptoms regular exercise before bedtime may be helpful for some patients but most biofeedback and relaxation techniques fail to affect a cure the most effective treatment may be pharmacological dopaminergic drugs are the most effective agents: DOC is dopamine agonist pergolide (antiparkinson’s drug) dopaminergic agents that BZDsàaid in m relaxation and sleep induction. and there seems to be an autosomal dominant inheritance pattern Clinical The m. helps support the molecular mimicry theory. corticosteroids administered during the acute paralytic phase but their efficacy is difficult to determine. Epidemiology Beriberi has become rare in the US b/c most foods are now vitamin-enrichedà contains adequate amounts of thiamine. tingling. Attacks may recur. “abnormal” sensations and weakness. or alcoholism that is present for 10 years or more indicates high risk for alcoholic neuropathy 2.c s/s is a recurrent. 92% within three weeks and 94% within four weeks. motor weakness accompanied by sensory disturbance – from mild hypoesthesia to profound absence Disease course of the disease varies. or weakness. crawling. Guillame Barre Syndrome Landry’s Ascending Paralysis Description rare entity characterized by symmetrical motor and sensory paresis. however. Treatment 100 mg IV of thiamine should be administered parenterally – along w/ Mg (which is necessary for thiamine metabolism) if response to the thiamine is noted. all other symptoms return to normal with the exception of persistent memory lossàas many as 20 to 25% of patients with Korsakoff amnesia do not improve with treatment • The most common symptoms are numbness. it is associated with lesions in the mammillary bodies of the hypothalamus. 75% reach maximum disease progression within two weeks. Both types are most often caused by excessive alcohol consumption. Etiology unknown. passive and active exercises used to maintain joint range of motion 4. arms. it should be repeated daily during the acute phase of the diseaseàcerebellar ataxia will generally improve but recovery may be incomplete Korsakoff’s Syndrome Usually occurs as a chronic sequelae to Wernicke’s syndromeàconsists of memory loss. Treatment plasma phoresis along w/ immunomodulation via infusions of IgG shorten the duration of the disease. confusion and confabulation. the autonomic nerves (those that regulate internal body functions) may be involved. Schwann-cell surface membrane targeted → acute segmental inflammatory demyelinating polyneuropathy. grabbing sensations.000 Pathologic edema of the nerve → degeneration of axons and their myelin sheaths → Wallerian degeneration. or loss of sensation in hands and feet. breathing muscles and face may occur. particularly walking. infecting organism → induces both humoral and cell mediated immune responses d/t molecular mimicry react with ganglioside surface components of peripheral nerves Epidemiology leading cause of acute flaccid paralysis in western countries. agent) present in depilatories and some pesticides. however. clinical findings – enlarged nerve sheathes resembling Dejerine-Sottas Dz. increase • neurologic complication: tremors and personality changes. dysarthria and brisk DTRs behavioral changes are the first clinical sign: the dz further develops to widespread damage to the basal ganglia resulting in parkinsonismàthe rigidity and hypokinesia can be managed with L-dopa Neuropathy Barium Lead Mercury (Mad Hatter Syndrome)à Lithium Manganese [Urate]p 5. metallic mercury (occurring primarily from inhalation Treatment of mercurial vapors) and organomercurials (BAL). tremor of the limbs or postureurinary excretion with holding –increases w/voluntary mvmt the administration of Symptoms of Organomercurial Exposure: paresthesias of the limbs. absent pain and temperature sense. it can lower serum K resulting in the clinical picture of periodic paralysis vasculopathy caused by a protein-containing edematous fluid surrounding abnormally Treatment permeable capillariesàthe capillaries b/c engorged and develop endothelial proliferation (BAL) and (EDTA) for chelation and thrombià in focal necrosisàcortical neuronal necrosis occurs • Supportive: ventilation and Signs and Symptoms seizure control • convulsive seizures – generalized or focal àparalysis may follow these seizures oral penicillamine (with supplemental any type of neurologic sign may develop – cerebellar ataxia. dysarthria. visual. spastic weakness is followed by coma or death (permanent) neurotoxicity is reversible – resulting in tremor. it is unlike the barium sulfate used in XR exams. tremors and penicillamine incoordinationàw/ large exposure. demonstrates neuronal degeneration and characterized by the onset of distal lower limb weakness Dejerine Sottas Dz hypertrophic neuropathyà “onion bulb” Refsum’s disease lipid storage disorder associated w/ increased excretion of phytanic acid. spastic paraplegia optic atrophy and hypertrophic neuropathy typical of type I clinical picture of type I & has the additional complication of retinitis pigmentosum Type Type I Type II Type III Type IV Type V Type VI Type VII 5 . hemiplegia or decerebrate pyridoxine) may suffice in milder rigidity. osmotic cathartics (BAL-chelating trypanosomiasis. connected to CMT Dz b/c syndromes have overlapping sensory & spinal cord pathologies. Thalidomide (used to fight leprosy) INH (isoniazid) – used against tuberculosis Drugs used to treat skin conditions (Dapsone) Anticonvulsants (Phenytoin) Anti-alcohol drugs (Disulfram) Drugs to fight HIV Zidovudine (Retrovir. Cisplatin Drugs used to fight infections : Metronidazole (Flagyl) Nitrofurantoin. formerly AZT) Didanosine (Videx) Stavudine (Zerit) Zalcitabine (Hivid) Ritonavir (Norvir) Amprenavir (Agenerase) Drugs to lower cholesterol Lovastatin (Mevacor) Indapamid (Lozol) Gemfibrozil (Lopid) Toxic-environment Pollutant Arsenic Description Clinical Signs and Symptoms Medical use is essentially Affects GIT and the nervous systemà presents as a painful polyneuropathy nonexistent in the 90’s with the • skin changes occur concomitantly – white striae in the fingernails (Mees bands) exception of the management of TX of acute poisoning: emesis and/or gastric lavage. lethargy. ataxia. decreased vibratory sense. it is not Symptoms of Inorganic Salts and Mercury Vapor Exposure effective in chronic poisoning.• • • • • • • • Heart or blood pressure medications : Amiodarone Hydralazine Perhexiline Drugs used to fight cancer : Vincristine. associated with retinitis pigmentosum. Hereditary Herditary Motor and Sensory Neuropathies Characteristics CMT histologically demonstrates segmental demyelination and hypertrophic neuropathy. delirium and coma cases of lead poisoning It is dangerous in three forms – inorganic salts. symptoms – repeated attacks & remissions of distal motor and sensory loss in the hands and feet. onset b/t 5–15 yrs. mentality is dull – normal. high prevalence of breech presentation. occurs in 25% of pts with 3o syphilis. (mc 80-90% of all cases). delayed motor milestones slowly progressive scoliosis in thoracic region. +ve Romberg’s sign. no pus.surgical intervention to correct fixed deformities – tendon transfer. joints. Cryptococcosis Pigeons are the vectors. “Coccidiomycosis” and pleuritic pain. 40% of pts react to stress – either physiologic/emotion w/ constellation of s/s termed the dysautonomia crisisà characterized by increased HR. absent deep tendon reflex responses. characterized by atrophy of peroneals and intrinsic musculature of the hands and feet Charcot-Marie-Tooth 1 . autosomal recessive trait in Ashkenazi Jews. poor walking and inability to run Clinical: all sensory modalities affected. m > f. weak/absent suck and poor tone. peroneals & intrinsic muscles of the feet affected 1st→ lose ability to evert the foot. TX: PCN G Cryptoccocus Neoformans A yeast passed to humans via pigeon vectorsà can produce a subacute meningitis Poliomyelitis RNA virus. familial pes cavus & extension of the digits. strengthening exercises to “beef up” wasting muscles advanced cases . Sequelaeà persistent arthralgia. clinical dz state starts as a flu. bone. mononeuritis multiples and chorea Meningitis Bacterial Causes: H. difficulty feeding in the neonatal pd increasing the risk of aspiration pneumonia. prolonged nerve conduction velocity in involved motor nerves early stages . Infectious Tuberculosis/Leprosy Can result in multiple nerve palsies Neurosyphillis Etiological agent: T. wide base unsteady gait.Erythema multiforme or nodosum. use of Vitamin E Other hereditary disorders Riley-Day Syndrome AKA Familial Dysautonomia Clinical Presentation Roussy-Levy Syndrome Hereditary areflexic dystasia Friedreich’s Ataxia AKA Hereditary spinocerebellar ataxia Complete indifference to pain.demyelinating form characterized by Charcot Marie-Tooth 2 . autosomal dominant trait. positive Romberg’s sign Onset – early childhood. Dz d/t ↓ protein frataxin which is normally in the CNS heart & pancreas. diagnosis is by FTA-ABS or RPR. sensory findings – loss in position & vibratory sense & 2 pt discrimination. anterior motor horn cells are normal Onset . have AbN high Fe in cardiac tissue → reacts w/ O2 to produce free radicals →detrimental to neural and muscular tissues.“neuronal” form degeneration of the posterior columns of the spinal cord.males = females. Pallidum. Reflexes – diminished or absent deep tendon reflexes. anesthesias to light touch & pinprick in a stocking-glove distribution. and CNS. self-mutilation. Finally. autosomal recessive gene. tendency to stagger. prolonged nerve conduction velocity of involved motor groups. absent or hyporeflexic DTR. proprioception is lost → +ve Romberg’s sign. plump thigh. bones. Muscles – coarse muscle fibrillations like those in CMT Dz & slowly progressive muscle weakness. pts not b/c seriously impaired until the 4th decade muscular atrophy is symmetrical and distal. insensate foot measures should be taken – education & Rx like a diabetic. posterior group atrophy. adrenals. slender legs with claw toes “ostrich or stork legs” or “inverted champagne bottle” w/ symptoms of LMN disease +ve family history. Disseminated: Granulomas. LE Findings: feet display a symmetrical cavus foot deformity. muscle imbalance is an important factor. impaired papillary response to light Tx accommodative care aimed at preventing injury 2o to sensory loss. present from birth.Pneumonitis proceeds dissemination to CNS-meningitis. involves spinocerebellar tracts. onset is later in life with no enlargements of the conduction velocities and changes (onion bulbs) on biopsy peripheral nerves highly variable. touch. foot affected first. the corticospinal tracts & posterior columns. EDL muscles to digits→ cavus deformity & eventual clawing of the digits. paraplegia and decreased life expectancy Onset – infancy. diagnosis is confirmed by lumbar puncture and tx: PCN G or ampicillin +Brudzinski and Kernig sign Coccidioidomycosis SW: Valley fever or Dessert rheumatismTransmitted via dust particles. Clinical Presentation – static tremor of the hands most common inherited ataxias. cardiac failure is cause of death à paresthesias and shooting or “lightning-like” pains in extremities → wheelchair confinement at 20 yrs. Delayed hypersen. Can disseminates also to skin. Flu. temper tantrums. liver and other viscera 6 . peroneal muscle weakness is mc. initially unsteady gait. Afro-Americans exempt from CMT. followed by meningitis and then flaccid paralysis of the limbs and trunk. thermal regulation is poor. cerebellar signs – develop first. atrophy spread to anterior calf: TA. loss of anterior characterized by axonal degeneration of the peripheral horn cells and degeneration of the spinocerebeallar tracts. pain & temp preserved absent DTR. invertors exhibit its mechanical advantage unopposed → early symptoms of varus or high arched foot. night splinting & passive muscle stretching useful to prevent development of contractural deformities Dejerine-Sottas Syndrome AKA progressive hypertrophic polyneuritis 6. of the skin. excessive sweating and postural hypotension are usual findings. cystic gelatinous masses in the gray “Cryptococcus Neoformans” matter.passive stretching exercises. EHL. fall & unable to make sudden turns. rare. meningitides. heel to shin ataxia present speech explosive – slurred and staccato. trophic ulcers on lower extremity d/t indifference to noxious stimuli a forme fruste of Charcot-Marie-Tooth Dz. Tx: vaccine Herpes Zoster Has a dermatomal distribution of vesicles and demonstrates segmental weakness and pain for years Lyme disease Neurologic s/s: meningeal irritation w/ cranial neuritis. N.Description Classification Dyck & Lambert 1968 Incidence Onset Clinical Findings clinical s/s Diagnosis Treatment Peroneal Muscular Atrophy Charcot-Marie-Tooth Disease begins as slowly progressive Dz in feet & legs and spreads to the hands and forearms over several years. motor or sensory radiculoneuritis. sweating and emotional lability. slow nerve nerve. indifference to all pain. emotional lability. b/t ages 5-15yrs. surgical decompression Principle Lower Extremity Nerves of the Lumbar Plexus • Contains anterior rami of the first three lumbar spinal nerves. and sometimes the laryngeal muscles (which interferes with breathing). L1.5cc IM No TIG TIG (250mg) units IM at other site immunizations Td0. innervates skin over the medial aspect of the thigh.subgroup of compression neuropathies occurring secondarily to: gradual constriction of anatomic structures about a nerve. ends at level of the first metatarsal phalangeal joint Injury/Entrapment: injured mostly by laceration.insidious & mild. adductor brevis & gracilis(+Internal Rotate leg) →→Adducts thigh obturator externus → External Rotation thigh & ½ adductor magnus muscle → Adducts. L3 and L4 nerve roots SENSORY & MUSCULAR 7 . reports show up to 1 million cases/yr.Histoplasmosis Histoplasma Capsulatum Dsecription Epidemiolgy Clincal presentation Diagnosis Wound management Associated with bat and starling guano. entrapped as it exits the subsartorial (Hunter’s) canal proximal to the knee no motor weakness b/c nerve has no motor branches **may be injured when the GSV is harvested for CABG surgery formed within the psoas muscle. Tetanus antibody test Previous immunization history Non tetanus prone wound Tetanus prone wound Uncertain or <3 yrs prior tetanus Td0.skin of the anterior thigh and medial calf Muscular branches pectineus. leg and foot. mostly in developing countries. back. Course: accompanies the femoral artery in the femoral triangle → descends & diving medially under the sartorius muscle. allowing them to go into severe spasm. which begins w/mild spasms in the jaw (trismus). pain referred along the n. brevis and the gracilis posterior branch obturator externus and half of the adductor magnus sensory fibers supply cutaneous sensation to the upper medial thigh & anastomose with the saphenous nerve Movement. (granulomatous) Benign pulmonary Lymphadenopathy and Splenopathy Disseminated RES diseaseà infects macrophages –-> CNS fatal meningitis Tetanus Dz caused by the toxin of Clostridium tetani that affects the central nervous system. and facial muscles.5cc IM <10 yrs since last dose >3 yrs prior tetanus immunizations Nothing TIG (250mg) units IM at other site 8. The spores germinate. lying just lateral to the femoral artery and vein Sensory branches . Tetanospasmin selectively blocks inhibitory nerve transmission from the spinal cord to the muscles. or chronic compression of the nerve against an adjacent unyielding fibrous or skeletal structure • onset of s/s . Stiffness rapidly develops in the chest.S and internationally. 12cm inferior it bifurcates into an anterior and posterior branch anterior branch .supplies skin over the lateral/anterior surface of the thigh posterior branch – lateral/posterior thigh Course: exits vertebral canal → lateral edge of the psoas muscle → musuclar innervation to both the psoas and iliacus muscles → exits the pelvis by passing underneath the inguinal ligament. quadriceps femoris → extension of the leg at the knee largest and longest sensory branch of the femoral nerve. releasing active bacteria that multiply and produce a neurotoxin called tetanospasmin. abdominal muscles. neck muscles. Sartorius. adductor longus. sometimes resulting in death. supplying sensation to the medial aspect of the calf. as courses inferiorly → infrapatellar branch supplies the medial aspect of the knee terminal branch → courses inferiorly along w/ greater saphenous vein. . pectineus → flexion of the thigh. quadriceps femoris Movements Under Control of the Femoral Nerve: iliopsoas → hip flexion. Tetanus causes 5 deaths/yr in U. supplying sensation to the medial foot. distribution • Dx . Course. Systemicà hypothyroidism. courses forward along the brim of the pelvis to the lateral end of the inguinal ligament.enters pelvis immediately anterior to the sacroiliac joint →through the obturator canal bifurcating into an anterior and posterior division anterior branch adductor longus. Molds and dust are inhaled. Enters the thigh through a tunnel formed by the lateral attachment of inguinal ligament & ASIS. motor and sensory changes painful. at medial malleolus continues anteriorly with the greater saphenous vein. L3 & L4 SENSORY & MUSCULAR Saphenous Nerve SENSORY ONLY Course Injury or Entrapment 1st sensory branch of the lumbar plexus Course: emerges from the lateral border of the psoas major. L2 and L3 as well as a portion of L4 Nerve Lateral Femoral Cutaneous Nerve L2 & L3 SENSORY Femoral Nerve L2.electromyography and NCVS identifying and localizing the lesions • Tx . Flex & Extend Thigh Obturator Nerve L2. LE edema from various etiologies → compression of nerve under the laciniate ligament 9. Tumorà space occupying lesions → schwannoma or fibroma create pressure on the nerve Entrapment neuropathies Def . Dz – Sciatica .the ABH muscle belly to test the medial plantar nerve and the abductor digiti quinti to test the lateral plantar nerve. superior gluteal nerve → abduction and internal rotation of the thigh.7 milliseconds to perform the nerve conduction studies: PTN stimulated just proximal to the laciniate ligament w/ stimulating electrode. a delay in conduction is presented by a greater latency value. it gives off muscular tibial nerve” branches to the medial and lateral heads of the gastrocnemius & soleus. enters foot: passes through a space between the MUSCULAR medial malleolus and the flexor retinaculum. Tibial Nerve sciatic nerve bifurcated within the popliteal fossa→medial popliteal nerve→ tibial nerve. under the flexor retinaculum. the tibial branch gives off short muscular branches in the posterior thigh.reproduced with percussion of the posterior tibial nerve at the level of the laciniate ligament Diagnosis electro-diagnosis is quite useful.refers to isolated entrapment of any of the distal segments of the posterior tibial nerve. Courses inferiorly through the popliteal fossa. hamstring muscles in lower third of the thigh. describe an "afterburn" .pain down entire lower limb. Course: exits pelvis through the greater sciatic foramen both portions course inferiorly. FDB. entrapment between quadratus plantae and ABH muscle is associated w/ chronic heel spur syndrome and is the most common type of distal tarsal tunnel syndrome o pain present for up to a year & traditional Mx for heel spur syndrome has failed to relieve their symptoms. abduction and external rotation of the thigh Sciatic Nerve Comprised of peroneal portion from the posterior division of the anterior rami. Ø medial plantar nerve = "jogger's foot" isolated involvement occur 2o to compression of the nerve as it courses into the plantar aspect of the foot b/t the navicular tuberosity & abductor hallucis muscle belly Ø Lateral plantar nerve 1st branch to abductor digiti minimi quinti = "Baxter's nerve". NVS passing through the laciniate ligament gets attached to some of these septa Vascular Medial and posterior tibial nerves are normally well supplied with arterial blood.FHBand the1st lumbricale & Cutaneous intrinsic muscles of the foot & sensory innervation on innervation of medial distal 2/3 of plantar foot.S2 anterior rami. plantar of digits 1lateral aspect of the sole and to the lateral aspect of the 3 & medial side of 4th digit fourth and entire fifth toe Injury and Entrapment of Tibia Nerve Proximal Tarsal Tunnel Syndrome . & tibial portion from anterior divisions of the L4 . it divides into separate and distinct peroneal and tibial nerves. motor innervation to the muscles of AKA “posterior both the superficial and deep sural muscle groups. flexor hallucis longus and flexor digitorum longus.S2) Nerves Muscular Movements.continued burning discomfort at rest following a period of prolonged standing 8 . may be normal even when syndrome is present. supplies motor innervation to 4 muscles (LAFF) and lateral plantar surface. uses surface electrodes & invasive probes are not necessary Represents time required for the stimulus to travel to the end point. pain that rotate proximally = "Valleix phenomenon" Presentation Symptoms . gives classic H/O of post-static dyskinesia but also C/O increased pain following prolonged standing. medial plantar nerve enters foot courses along with the medial Lateral plantar nerve enter the foot → laterally to heel plantar artery. splitting into the medial and lateral plantar nerves.entrapment of medial and lateral branches of tibial nerve (dependent upon the level of the bifurcation). inferior gluteal nerve → MUSCULAR extension.1 ms of lateral plantar nerve – abnormal > 6. enlargement or varicosities of venous system w/i the tarsal tunnel. motor innervation to all other the ABH. factors contributing to “entrapment” include: Anatomic Flexor retinaculum has deep fibrous septa which blend with the periosteal covering of the medial side of the calcaneus. the peroneal component MUSCULAR supplies short head of the biceps femoris and a portion of adductor magnus. after giving off the 1st small calcaneal branch.Principle Lower Extremity Nerves of the Sacral Plexus Superior (L4 – S1) & Superior Gluteal → muscular innervation to gluteus medius. They regenerate easily & are extremely susceptible to ischemic injury or arterial insufficiency→ Tarsal Tunnel Syndrome. recording electrodes are placed over terminal end point of the respective nerves . enlarged navicular tuberosities d/t osseous impingement and anomolous insertion of the posterior tibial tendon. pronation syndromes → extreme PF & adduction of the talus during ambulation. seen in pts w/ enlarged ABH muscle bellies or osteophytic spurring of the medial arch of the foot. application of ankle tourniquet → venous engorgement →Tarsal Tunnel Syndrome Biomechanical compression entrapment of medial & lateral plantar nerves as they enter the foot.tibialis posterior. Treatment Surgical decompression Distal Tarsal Tunnel Syndrome . occurs as it courses beneath the laciniate ligament (flexor retinaculum). indicative of derangement of conductivity of the nerve distal latency of medial plantar nerve -abnormal > 6. Clinical C/O burning or overall “tiredness” in the foot. distal latency(millisecs) is evaluated for medial and lateral plantar nerves. In superior calf: motor innervation to deep posterior SENSORY & group . minimus and tensor fascia lata Inferior Gluteal Inferior gluteal → innervation to gluteus maximus (L5 . posterior to the fibular malleolus. upon entering the leg → gives off the small recurrent nerve which supplies sensation to patella → bifurcates into superficial and deep peroneal nerves. entrapment .this maneuver will put the nerve on stretch. occur 2o to constriction with resultant demyelination at the fibrous tunnel arising at the origin of the PL on the fibula. becomes superficial at level of the popliteal fossa → infero-laterally around the head of the fibula. Entrapment – 2o to fibrosis following a lateral inversion ankle injury. symptoms . Course: b/t bellies of the gastrocnemius. c/o paresthesias and hypesthesias. More proximal lateral (peroneal component) trunk of the sciatic nerve → EMG aberrations of the biceps femoris. placing the nerve on stretch. 3) direct percussion of the nerve with the foot held in plantarflexion and inversion Treatment. smaller lateral branch innervation to lateral aspect of the fifth digit. massage over the nerve after Sx to prevent entrapment Other “Nerve Conditions” in the Foot • Medial calcaneal nerve entrapment . medial dorsal cutaneous nerve traumatized or even severed w/surgical procedures which address the 1st MC joint Diagnosis: 3 clinical tests performed to reproduce the pts s/s & to confirm the dx → local infiltration of anesthesia should eliminate symptoms. surgical decompression with release of the nerve and its branches. supplying the antero-lateral aspect of the lower half of the leg and the dorsum of the foot and toes. peroneus tertius and EDB → responsible for dorsiflexion & frontal plane movement. originates inferior to the popliteal fossa.Common Peroneal Nerve L4 –L5-S1 SENSORY MUSCULAR Superficial Peroneal Nerve AKA the musculocutaneous nerve SENSORY MUSCULAR Deep Peroneal Nerve AKA anterior tibial nerve SENSORY MUSCULAR Sural Nerve SENSORY Only Course: branches laterally from the sciatic trunk within the popliteal fossa. 2) foot passively placed into PF and inversion. consists of education regarding properly fitting shoe gear. EHL.occurs as it exits the fascia w/i the calf approximately 10 cm above lateral malleolus. excision of an os trigonum or Shepard’s fracture(Fx of lateral tubercle of the posterior process of talus caused by compression of bone b/t the posterior malleoli and calcaneal tubercle). sensory branches → anterior to extensor retinaculum. severe inversion ankle injuries → stretched nerve & injury. palpating the nerve at level of the anterior ankle just medial to the dorsalis pedis artery Arises from union of medial sural cutaneous nerve (from tibial nerve) and sural communicating branch (from common peroneal nerve). complete compartmental fascial release if conservative management fails. intermediate dorsal cutaneous nerve (AKA Lemont’s nerve) located over the dorsal aspect of the foot → coursing just medial to the sinus tarsi. cutaneous innervation to postero-lateral aspect of the distal leg and lateral aspect of the foot. favorable results from isolated decompression are quite variable supplies motor innervation to TA.more unusual. communicates with the intermediate dorsal cutaneous nerve. terminal branches divide into a lateral and medial branch: largermedial branch supplies cutaneous innervation to the dorsal skin of the base of the fourth metatarsal. Joplin’s Neuroma compression injury or entrapment of medial plantar digital proper nerve. distal involvement with neuritic symptoms radiating along the plantar medial aspect of the hallux. cortisone injection at the level of entrapment. the CPN “dives” underneath the PL coursing between them muscle bellies of the peroneus longus and brevis Diagnosis nerve conduction studies. entrapped under the extensor retinaculum or irritated by the superior edge of the inferior extensor reticulum.seen in conjunction w/ infra-calcaneal heel spur syndrome. Injury and Entrapment: injury occurs at level of the ankle → minimal sensory deficit to lateral aspect of the foot Etiology .iatrogenic causes include “slip of the hand” during excision of a retrocalcaneal spur. Injury or Entrapment: vulnerable to external compression injuries → neuropraxia can result from simply crossing the legs. weakness of anterior muscle group. terminal branches provide branches to the tarsal joints. clinical signs of injury include weakness of the TA and extensor hallucis muscles resulting in the classic “dropfoot” deformity supplies motor innervation to the peroneus longus and peroneus brevis →eversion & plantarflexion of the foot.numbness and pain infero-medial to the first metatarsal -phalangeal joint. maximum point of contact with anterior tarsal tunnel syndrome occurred at the dorsal talonavicular joint. nerve irritated 2o to abnormal foot pronation and/or anomolous anatomy (a large abductor hallucis muscle belly. frequently seen in conjunction with valgus bunion deformity and a medial hallux interphalangeal joint tyloma etiology: pronation syndrome → apropulsive gait with “medial roll-off” during propulsion → fibrosis of the nerve. Course: descends inferiorly between the peroneal muscles dividing inferiorly into the medial and intermediate dorsal cutaneous nerves.→ minimal residual anesthesia. cutaneous innervation of small wedge of skin on dorsal surface of lateral hallux and medial aspect of the second toe Injuries or Entrapment = Anterior Tarsal Tunnel SyndromeàBlunt trauma occurs as it courses anterior to the ankle. c/o of paresthesias over the dorsum of the foot with numbness in the first intermetatarsal space. radiculopathy of L5 →EMG aberrations in gluteal & paraspinal muscle groups. calcaneal osteotomies and lateral ankle stabilization procedures. nocturnal pain relieved by movement of the foot. Clinical Findings: symptoms unequivocal. d/t motor innervation to EDB → wasting of the muscle belly and this area should be palpated and examined for any signs of decreased strength or atrophy Diagnosis: distal motor latencies w/ nerve conduction studies >7milliseconds (normal is 5 milliseconds). insertion of the medial slip of the plantar fascia more superiorly upon the calcaneus) 9 . tarsal spurs compress the DPN over the anterior ankle. provocative testing reproduce symptoms on PF of the ankle and concomitant extension of the toes. one of the most easily accessible sensory nerves of foot & considered to be the “nerve of choice” for biopsy . after wrapping around the head and neck of the fibula. winding inferiorly and distally. 1) pressure may be applied to the area where the nerve pops through the superficial fascia while pt dorsiflexs and everts the foot against resistance . most common nerve injured in the footInjury and Entrapment . EDL. associated w/ Charcot joints & Argyll Robertson pupils Anterior Motor Horn Diseases Dz that cause selective damage that affects voluntary movement & rarely attack other pathways within the spinal cord.symptoms . low-grade fever. positive if extension/doriflex of great toe and flex/fanning of lesser toes. they may also provide information regarding the sensory innervation to a muscle • To elicit a normal deep tendon reflex response all five components of the reflex arc must be in tact.first plantar intermetatarsal nerve Heuter’s neuroma . lower motor neuron weakness or paralysis → gradual tightening and muscle spasm. classic triad: SINà Scanning speech. Poliomyelitis Caused by three distinct viruses that actually target the anterior horn cells of the brain stem and spinal cord.e. and just prior to percussion The integrity is dependent upon spinal nerves L3 and L4 and ellicited on a slightly extended knee Integrity is dependent upon spinal nerves S1 and S2 à a reflex response is present when the foot responds in PF Patellar DTR Achilles DTR Pathologic reflex responses that elucidates UMN disease 1-Babinski sign: normal if age <2yrs.hypo-reflexic 0 absent • if the reflex response is absent. pts c/o increased deterioration of previously involved muscles involves the lateral columns and anterior gray matter which are closely connected within the spinal cord. thighs and neck. stroke lateral plantar foot proximal to distal then across ball of foot. Intention tremor and Nystagmus Tabes Dorsalis Degeneration of Dorsal columns d/t tertiary syphilis. develops yrs after the initial viral infection & is the result of expansion of the “territories” of the remaining functional motor units. the examiner should repeat the percussion using recruitment or what is called the Jendrassik maneuverà the patient is asked to interlock his hands in front of his chest. Mental retardation may be present in up to 60% of all patients with cerebral palsy 10 .fourth plantar IM nerve • 2. ALS involves voluntary motor system involving degeneration of the corticospinal tracts and the alpha motor neurons. nuchal rigidity. and maintaining a high fat diet may prolong survival rate Post Polio Syndrome Amyotrophic Lateral Sclerosis Cerebral Palsy Description Cerebral palsy is a group of disorders characterized by loss of movement or loss of other nerve functions. gondii ß most common in AIDS pts Subacute. CNS disorders Infections of the CNS à demyelinating CNS diseases Description Characteristics Clinical Presentation Histology Progressive Multifocal Viralencephalitisà Focal and relentlessly extensive neurologic symptoms and signs due to multifocal Leuckoencephalopathy polyoma JC virus lesions in the cerebral hemisphere.second plantar IM nerve Morton’s neuroma . positive if toes PF/flex CNS Disorders 1.associated w/clonus 3+\ hyper-reflexic 2+ average 1+low normal . These disorders are caused by injuries to the brain that occur during fetal development or near the time of birth. Demylination with viral inclusions CMV Occurs in the fetus or the Severe brain destruction à Subacute encephalitis w/CMV inclusions Periimmunosuppressed i. focal or diffuse neurological symptoms with multiple Toxoplasmosis abscess formation Multiple sclerosis Relapsing. rate of weight loss is prognostic factor for survival. AIDS pts microcephaly in fetuses ventricular necrosis-calcification Cerebral T. muscle tightness in hamstrings. muscle weakness develops in asymmetrical and scattered distribution Skeletal deformities occur 2o to disruption of normal agonist-antagonist balance of involved muscle groups.third plantar intermetatarsal nerve Islen’s neuroma . DTR responses are assoc with LMN or UMN lesions respectively. Houser’s neuroma . Symptoms – dependent upon the degree of UMN and LMN involvement. recent data suggest that hypermetabolism of muscular tissue is associated with the disease process. Symptoms – H/A malaise and myalgias – flulike symptoms. These are: Ø an intact afferent sensory nerveà a functional synapse at the spinal cord level Ø an intact motor nerveà an intact and functional neuromuscular junctionà a competent muscle • Deep tendon reflex responses are graded as follows: 4+ Brisk . response is slow as compared to plantar withdrawal response from tickling 2-Chaddock’s sign: stroke lateral foot from above lat mall moving distally 3-Oppenheim sign: use thumb and index finger as caliper to squeeze tibial crest applying pressure P>Distal 4-Gordons sign: squeeze posterior calf NOTE: all above positive if hallux extend/dorsiflex 5-Rossolimos sign: tap balls or distal pulps of digits. remitting course.. transmission via respiratory secretions or via fecal contamination. presents with both upper and lower motor neuron disease. including diseases of the spinal cord Description Deep Tendon Reflex Responses Although diminished or augmented. CNS disorders. evolving. immunization has ended epidemics.reproduced with percussion of this branch of the posterior tibial nerve. and many others. primary incoordination of movement or balance is observed. or other movements) this variety of CP results from a lesion in the basal ganglia and post natal kernicterus is the most common etiology Involves tremors. herpes simplex infections. Within a few months. quadraplegia .000 live births and in the US the prevalence has been reported to be between 390 and 590/100. meningitis.Etiology Epidemiology Clinical presentation Spastic CP (Pyriamidal) Dyskinetic (athetoid) CP Ataxic CP Hypotonia Rigidity Tremor Mixed foot deformities Treatment Cerebral palsy is caused by injury to the cerebrum (the largest portion of the brain. it contributes to the maintenance of normal posture. Cerebral palsy may pccure prenatal. It affects about 10%. balance and unconscious proprioception • It contributes to vestibular function helping to maintain equilibrium.this is used to temporarily relax spastic musclesà it is a protein exotoxin derived the Clostridium botulinum .involves all four limbs equally. blood vessel injuries. head injury that results in subdural hematoma. sensory abnormalities. diplegia also affecting the arms.13% of all CP .represents paralysis of the legs. Cerebellar disorders The Cerebellar Examination The cerebellum is responsible for the smooth coordination of voluntary. equinus . In addition. These toxic effects are limited to the peripheral nervous system 2. skilled movements. the toxin will then functionally denervate the muscle resulting in a flaccid paralysis.involved only one limb. direction and force of fine motor movements. and defects of hearing and vision. with any combination of the above symptoms. sensations. It exerts its effect at the neuromuscular endplate by preventing the exophytic release of acetylcholine. pes varus . Botox . forefoot equinus and metatarsus adductus Physical and occupational therapy. pes cavus. natal and postnatally occuring during early infancy as a result of illnesses (encephalitis. and voluntary muscle activities). Occurs secondary to cerebellar dysfunction àinability to control the rate. but to a lesser extent than the legs. ataxic.5 of every 1. Speech abnormalities are common and seizures may occur Classifications of cerebral palsy include spastic. alternating movements occurring either at rest or during voluntary movement The remaining 20% are classified as mixed. the adduct the foot Romberg’s This test assesses a patient’s proprioception – that is. range. and double hemiplegia .5 to 7.this is exhibited by muscles tiring easily. and abnormal movements.000.a gram-positive. hemiplegia 30% of all CP . one side of the body (spastic hemiplegia). and a cerebellar lesion generally results in awkwardness and uncoordination of volitional movements Heel-Knee Test This assesses the integrity of the EPS or spinocerebellar tractà it is the most reliable test of cerebellar function in the LE Patting This assesses the integrity of the extrapyramidal or spinocerebellar tract. and so on). It involves development of abnormal movements (twisting.1% of all CP . and mixed + Babinksi sign à 50% of cases Further calssified into: monoplegia .involved an arm and leg on the same side of the body (usually the arm is more affected than the leg. dyskinetic. the ability to sense where one is in sense and time: proprioception is Test carried on two tracts – conscious proprioception on the dorsal columns and unconscious proprioception on the spinocerebellar Damage Eyes Open Eyes Closed Cerebellar and Dorsal Columns sway Sway Cerebellar Only steady sway Clinical 0 = absent – no evidence of contractility 1 =trace – evidence of slight contracture – no joint motion Muscle 2 =poor – complete ROM w/ gravity eliminated 3 = fair – complete range of motion against gravity Testing 4 =good-complete ROM against gravity w/ some resistance 5 =normal-complete ROM against gravity w/full resistance • Movement disorders with cerrebellar lesions: Asthenia is present . the pt is asked to pat his foot repetitively against the Test floor. both legs (spastic diplegia) and both arms and legs (spastic quadriplegia). which is involved with higher mental faculties. Hyporeflexia is a concomitant finding shown by a decrease in deep tendon reflex responses • 11 . Some form of CP is reported to occur in between 1. unsteady gait. spore-forming anaerobe. there may be partial or full loss of movement. diplegia/paraplegia .20% of all CP . Extrapyramidal or Dyskinetic Varieties of Cerebral Palsy Affects about 20%. the muscle develops new acetylcholine receptors and the paralytic effects are reversed. compensation for this balancing difficulty is a wide base of gait 5% of all cases of CP this results from a lesion in the motor cortex and it is manifest by decreased or absent muscle tone 5% of all cases of CP this results in lead pipe or cog wheel rigidity and resistance to motion is greatest with slow stretch 1% of all cases of CP this results in rhythmic. it occurs with greater incidence in the lower birth weight infant The classical finding of CP is spasticity which may affect a single limb. jerking. loss of coordination.quadriplegia in which the arms are more severely involved than the legsà all of these conditions involve abnormalities in motor coordination and a scissored gait. to alternately tap the heel then the toe against the floor or asked to maintain the heel against the floor and abduct. penicillamine which facilitates the removal of the excess body copper 12 . The movements are worse when trying to repress thsm and disappear while sleeping Traditional treatment of placing the child in a dark room to minimize external stimulus may suffice in mild cases • phenothiazines or haloperidol can effectively control the chorea although valium is more practical • anti-microbial therapy of the underlying etiology is essential generally a course of penicillin or erythromycin if penicillin-allergicà the condition will usually resolve in 3 to 6 weeks Parkinson’s disease (PARK) Description Etiology Epidemiology Clinical presentation Treatment Pathology Epidemiology Clinical presentation Treatment most common pathologic feature is the loss of the dopaminergic neurons originating in the substantia nigra the mean onset is between 55 to 60 years of age. rigidity. progressive dementia and psychiatric and behavioral disturbances and this abnormality is confined to the brain and in advanced cases shown extensive atrophy of the cerebral cortex. bradykinesia and disturbances in gait and posture Pill rolling resting tremorà one of the most common presenting signs. 1% of the US population over 50 years of age are affectedà about 50.000 new cases are diagnosed each year and the incidence does not appear to be changing with time the classic features are the insidious onset of tremor.is occurs frequently in the hands and face. Vitus Dance Sydenham's chorea is considered a major diagnostic sign of rheumatic fever and it can occur up to six months following a Group A beta-hemolytic streptococcal infection (Streptococcus pyogenes) and it is an indication to initiate prophylactic antibiotic therapy to prevent subsequent development of other manifestations of rheumatic fever result from autoantibodies resulting from the strep infection which attack specific areas in the brain it occurs most commonly between ages 5 to 15 with peak incidence at age 8 and it is more common in girls The onset is usually insidious and the child may exhibit nervousness with emotional lability (irritable and/or anxious). Over time. jerky or explosive in nature nystagmus is usually present with cerebellar lesions intention tremor – occurs with initiation of voluntary movement that often intensifies as the goal is neared 3. propranolol at doses from 60-240 mg/day may reduce the tremor in roughly 75% of patients. impaired coordination and muscular weakness with reduced muscle tone. COMT inhibitors may increase CNS delivery of dopamine Wilson's disease • in the juvenile patient. aimless involuntary movements. Description Etiology Epidemiology Clinical presentation Treatment Basal Ganglia disorders-responsible for fine movements.a slowness and fatiguing of voluntary movementß all of these symptoms progress in severity leading to "masked facies" and disabling postural difficulties • levodopa .000 It affects both men and women equally. schizophrenic behavior and multiple movement disorders • treatment is with D. speed or power or movement • a patient may typically "overshoot" a desired point or stop before it is reached speech is slurred. beta-2 receptor antagonists are more effective than beta-1 receptor antagonists Sydenham's Chorea aka St.this is a loss of the ability to gauge distance. basal ganglia and cerebellum Assoc w/ a defect in chromosome 4à this gene codes for a protein called huntingtin that is found in neurons throughout the brain its normal function is unknown and the greater number of copies. Akinesiaàa lack or poverty of movement and Cogwheel ridgidity à shuffling festinating gait . hemmiballisms-this is a dramatic condition characterized by wild flinging or circling movements of the limbs (lesion associated with the subthalamus) Essential tremor and choreoathetotic gait haloperidol and phenothiazines may suppress the chorea. the more likely the patient is to develop the disease Transmitted via an autosomal dominant pattern with virtually complete penetrance it occurs in from 4 to 7/100.the most effective: a combination of levodopa and carbidopa (Sinemet) is usually used • dopamine agonists. but it is rarely possible to suppress of al the involuntary movements and it may subside with low levels of alcohol. bradykinesia . amantadin. symptoms characteristic of parkinsonism may occur • it is the result of is a defect in copper-binding ceruloplasmin leading to accumulation of copper in the tissues • clinical findings include signs and symptoms of basal ganglia dysfunction and hepatic cirrhosis • there is a pathognomonic appearance of copper at the periphery of the cornea (Kayser-Fleischer rings) • the children may exhibit arm flapping. sudden jerks and flinging movement of the limbs occur. a head tremor may be more frequent in women and it is generally present in the older population by age 65 Athetosis.• • • • dysmetria is present . composed of subthalamic nucleus and substantia nigra Huntington’s chorea Chronic progressive degenerative CNS disease of adult life characterized by choreic involuntary movement. however. this condition progresses and involuntary movements. anticholinergics • selegiline: a selective inhibitor of type B-monoamine oxidase which does not stop the progression but has been shown in clinical trials to delay the need for levodopa therapy • COMT inhibitors which include entacapone and tolcapone: COMT is an enzyme that breaks down levodopa in the periphery and when given in conjunction with levodopa. Cerebral Palsy. hips in full extension. pt places wt on unaffected side with one hand on his back Lumbar lordosis or thoracic kyphosis Pseudoclaudication relieved by flexing spine Straight leg raise test: pt supine and hip passively flexed with knee in full extension. vertebral foramina is a limited space and subject to impingement by bony spurs. joints.e.e. acute trauma. periosteum. the twisting of the pelvis causes sacro-iliac pain Ø Valsalva maneuver: dx space-occupying lesion or herniated disc. Spinal Radiculopathy Def. skin.Upper motor neuron damage Lower motor neuron damge 4. vibratory and decussate just after ascending only one to two and cervical dorsal roots (i. Radicular Pain*pain follows the distribution of he nerve involved Spinal cord evaluation of radicuopathy Ø Ø Ø Ø Ø Ø Ø Neri’s sign: pt takes small steps with knees semi-flexed to prevent stretching of the nerve root Minor’s sign: when pt rises from seated position. i. Radiculopathy – “pathology pertaining to the spinal nerve roots” as they exit the spinal column through the vertebral foramina.05 to 0. ascending pain and sensory fibers enter the spinal cord → ascend no more than 1 – 2 vertebral levels before temperature decussating into the contralateral side of the cord → fibers ascend up to the cord→ to the pain and temperature thalamus or “relay center” of the brain → cerebral motor cortex. the position sense vertebral levels beyond their entry→ sparing of light upper extremities) touch sensation with unilateral spinal cord lesion. control synapses in the anterior motor horn of the spinal cord Damage to the corticospinal tract produces pyramidal disease. if nerve pain then when knee is flexed pain goes away Ø Gaenslens test: differentiate lumbosacral from sacral iliac pain. some fibers carrying light touch impulses from the upper thoracic light touch.25 per 1000 births Ø Prevention: prenatal screening for AFP may demonstrate ↑levels in neural tube defects 20% incidence of false positivesà folate prevents neural tube defect 13 . infections and neoplasms. the spinocerebellar tract enters the stereognosis cerebellum via the superior and inferior peduncles primary motor pathway exiting the cerebral cortex descends to the brainstem → decussating at the junction of the (descending track) responsible for voluntary motor brainstem and spinal cord to provide contralateral motor input. pt is supine. defects occur most commonly in the lumbosacral spine Ø incidence – 0. light touch. pt supine with one leg in full extension and the other is lowered off the side of the table. unconscious pathway of unconscious proprioceptors in the spinocerebellar tract remains ipsilateral – proprioception and ascending fibers remain within the cord on the side of entry. parts of the body. repeated mechanical strain. Pathways The Dorsal Columns Spinal Cord diseases Spinal Cord Neuronal Pathways Damage Produces IPSILATERAL Loss of conscious proprioception. should be nonpainful Lasegues test: reproduction of pain when pt’s leg is elevated less than 30 degrees and foot dorsiflexed Bowstring test: to differentiate root pain from hip pain. tumors or protruding (herniated) vertebral discs. b/l compression of jugular vns result in increase intra-spinal pressure and reproduces radicular pain Ø EMG studies Spinal Dysraphisms Def – all forms of developmental abnormalities occurring in the midline of the backà failure of spinal column to close 2o to faulty development of vertebrae. degenerative joint disease and other arthritides affecting the spine. muscles and same side at the level of the brain stem → fibers cross the lower extremities) connective tissue – over or decussate →to the thalamus & relayed to other fasciculus cuneatus carries conscious proprioception. congenital anomalies. Frequent causes of lower extremity radiculopathy include: disc herniation. Normally vertebral column should be closed by 4th intrauterine week. vibratory and position sense CONTRALATERAL loss of Pain and temp CONTRALATERAL loss CONTRALTERAL loss of motor control Spinothalamic Tracts Spinocerebellar Tract Corticospinal Tract (Pyramid) Description Sensory Information Pathway fasciculus gracilis – carries receptors in and about taken by ascending vibratory and position sense fibers: impulses from lower thoracic.e. bony canal should be closed by the 12th intrauterine week. bones. and outside of the pyramidà extra pyramidal disease symptoms (EPS). fibers enter the spinal cord and ascend on the ipsilateral lumbar and sacral dorsal roots (i. generally palliative. proprioception & cutaneous sensorium. Involves variable loss of motor function. Injury to the lumbosacral nerve roots. muscles that abduct the glenohumeral jt remain strong. cardiac Dystrophy involvement may → congestive heart failure or arrhythmias. caused by a central lumbar disk herniation. Cardiomyopathy. Girdle minimal facial involvement. Frontal balding. Limbautosomal recessive – 60%. lumbosacral dermal sinus serve as a pathway for infection & concomitant weakness & atrophy of the leg present as meningitis. Involves a cervical lesion. ambulation continues for 20 yrs after the initial diagnosis. onset – 3rd & 4th decade → better prognosis meral Clinical Sym . mild but labile hypertension common Dx CK levels . hypertrophy of calf muscles < 1/3 of pts.early in Dz 2o to peroneal & anterior tibial muscle weakness. posterior arches of L5 & S1 are most common. an incidental finding on radiographic imaging Occurs 2o to faulty closure of the vertebral arches + involvement of meninges (meningcele) or spinal cord itself (meningomyelocele). Pseudohypertrophy-replacement and Slower progression Cardiac infiltration of the muscle cells by fat. seen in adults that causes a myotonia (sustained involuntary contraction) of a muscle group. Mutations of the sarcoglycan complex (transmembrane proteins of the sarcolemma that interact with dystrophies dystrophin) are the cause of these diseases. cardiac & respiratory involvement rare. biceps & triceps affected later w/ deltoid spared. asymptomatic. shoulder girdle and proximal arm muscles. with variable motor and sensory loss in the lower limbs. trapezium & rhomboid muscles → winging of scapular & sloping Dystrophy shoulders. Sensory loss – variable. with contralateral loss of pain and temperature sensation. characterized by areflexic bowel and/or bladder. scapular stabilization to decrease scapular winging. specifically the proximal muscles of the trunk muscular and lower limbs. Breathing exercise Diseases of the NMJ: Myasthenia Gravis Description and Characteristics Autoimmune disorder caused by the production of autoantibodies to the Ach receptor on the motor end plate. Sacral sensory sparing exists. and lower limbs (motor & sensory loss in LE is variable). occur 2o to diastematomyelia (division of spinal cord into two segments within the sagittal plane) d/t bony or cartilaginous island projecting from the posterior vertebral arches into the spinal cord. involves muscles of facial expression and the proximal upper extremities. Associated with thymic hyperplasia or thymoma 14 . autosomal dominant inherited disorder. pain and/or temperature sensation. Diseases of the musculature and neuromusclular junction X-linked Muscular Dystrophies It is caused by mutations in the gene for dystrophin (deletions) Duchenne Muscular Dystrophy Becker Muscular Dystrophy Most common 1:10. *lumbosacral skin appendage – skin tag which resemble a tail Involves traction on the conus medullaris & is associated with a tight filum terminale → progressive neurological deterioration in the lower spinal cord. Myotonic Autosomal dominant trait. Sacral cord injury with or without involvement of the lumbar nerve roots. varus and “gritty” fat bound together by septal fibers present at the base of the spine cavovarus deformities & gait disturbances. weakness of serratus anterior. AFO for foot drop. decreased lumbosacral hypertrichosis (localized overgrowth of hair) at base of the spine. Dysesthesias (sensation of burning in hands or arms) esp in UE. characterized by areflexia in bladder.Condition Spina Bifida Occulta Spina Bifida with Meningocele or Meningmyelocele Tethered Cord Syndrome Anterior Cord Synd Brown-Séquard syndrome*** Central cord syndrome Conus medullaris syndrome Cauda equina syndrome Characteristics Incomplete closure of one or more of vertebral arches only in the sacral region. onset – 2nd . exophthalmos with (N) thyroid function. proximal muscular weakness starting w/ legs or arms progresses to all extremities. respiratory failure may occur after 30 yrs. M scapulohu = F. m. but altered dystrophin size Primarily affects walking due to extensive involvement of the pelvic muscles. inability to whistle. capillary angioma or naevus flammeus most common findings associated with decreased lower extremity DTR with spina bifida. EMG and muscle biopsy → mixed features of myopathy and neuropathy Tx . Eventually lose ability to ambulate w/o assistance. Decreased IgG and Abnormal GTT FacioSlowly progressive. with greater motor weakness in the upper extremities than in the lower extremities.characteristic weakness of facial.elevated CK not as high as Duchenne's. B/c is a nerve root injury rather than a true Spinal Cord injury the affected limbs are areflexic. foot drop . expressionless Muscular appearance noted w/I 1st decade. EMG consistent w/ myopathy. sullen.c.normal or slightly elevated. intact intellectual function. with preservation of proprioception Involves a greater ipsilateral loss of proprioception and motor function. S/s: Cataracts. muscle biopsy not specific Tx – PT. more likely to lose pain and/or temperature sensation than proprioception and/or vibration. Can affect systemic muscles of the heartß cause of death disease is less common Pathognomonic for this disease is Gower’s sign where the affected child rises from a sitting position by climbing on her legs Limb-Girdle Heterogenous autosomal muscular dystrophies that affect specific muscle groups. the spinal column defect is associated w/ a sac-like protrusion of skin and meninges the nature of the clinical symptoms will depend upon the degree and the level of the deformity in the sacral spine Clinical Symptoms Dematological Manifestations Symptoms unitlateral. Dystophy The disorder tends to increase in severity and appear at younger ages in successive generationsß Anticipation. occurs in 10 – 25% of population. bowel. Dx . midline lumbosacral subcutaneous lipoma – firm & muscles foot pathology – valgus.3rd decade.000 Least common Most severe: Fatal at 20 years old Least severe: Not fatal 5 years old onset Later in childhood or adolescence No dystrophin Some. short acting isosorbide dinatrate (sublingual tablets). squeezing Impending doom N/V. alcohol ingestion. smoking. vent arrhythmias only.. costochondritis. of costocondral Benign. Ca+ antagonists. intermediate acting isosorbide mononitrate. ripping Generalized >15 min varies à stroke Surgery and prevention Sharp pain associated with respirations Generalized >15 min Pleuritis. diaphoreses (sweating) Chest pain Location Time Arm. Befridilà acts on fast Ca channels and predisposed to ventricular arrhythmias and torsade de points …. vent arrhythmias only. Persistent hypertension is one of the risk factors for strokes. Major cardiac Symptoms Pressure. orthopnea (dyspnea that increases upon lying down). not useful in the tx of prinz metals or vasospastic angina Nitrates Antidysrythmiacs Class Ia Quinidine ß cinchonism –high anticholinergic activity. and aldosterone antagonists. thyrotoxicosis. or requiring elevated filling pressures in order to pump effectively.high anticholinergic activity. Treating the signs and symptoms of CHF involves maintaining a euvolemic stateà diuretic agents. will also increase HR and CO Very short acting Amyl nitrate. tender at Chest wall Days wks Inflam. and long acting transdermal nitrate Ca channel blockers Dighydropyridines: Nefedipine. vent arrhythmias only Class 1b: Weak Na Lidocaine. and hypertension. heart failure and arterial aneurysm. anemia. vent arrhythmias only and Disopyramide . Phenytoin. Delaying the progressionàACE inhibitors. surgery Sharp. thrombolytic tx. peripheral edema. does not effect AV conduction. an enlarged or pulsatile liver. overworked heart. esterNa channel blocker low anticholinergic activity. HTNà >140/90 and In patients with diabetes mellitus or kidney disease studies have shown that blood pressure over 130/80 mmHg should be considered a risk factor and may warrant treatment Systolic Diastolic BP Stage 1 140-159 90-99 Stage 2 160-179 100-109 Stage 3 180-209 110-119 Stage 4 >210 >120 Description Staging 15 . knife-like. The pooling of blood leads to congestion in body tissue. and positive inotropes. MD → muscular contractures <2 bad C. beta blockers. heart attacks. Genetic family hx of CHF. gallstones Surgery. varies margin shoulder Infection. can be used for ventricular arrhythmias: Amiodarone. B-blockers. Jaw 15 min Similar . Pulmonary edema.dipines… B blockers Not a vasodialator. between the visceral and pericardium Sitting up and forward Diagphragmatic pain at the right costal Upper abdomen. PCTA Sharp. heparin. dyspepsia (burning) Generalized Assoc with hiatal hernia Medication.angina >15 min Causes CAD ß ischemia Angina may precede Treatment Nitroglycerin Angina Pectoris Myocardial Infarction Dissecting aneurysm Pleurisy Esophagitis GERD Tietze’s Pericarditis Gallbladder Cholecystitis Nitrates. can be seenControl infection pneumonia Heart burn.<2yrs – (N). predominate venodilator to reduce CVP. severe. DO NOT use pneumatic compression devices. old age. can be used for all arrhythmias. control infection Description Causes Symptoms and signs Treatment Congestive heart failure Inability of the heart to pump a sufficient amount of blood throughout the body. stress. fatigue and paroxysmal nocturnal dyspnea.NB – Toe walking . will be degraded by organic hepatic nitrate reductase if taken orally. UTI. ASA. rales heard on chest auscultation. Sotolol Class 4 L-type Ca channel blockers-Verapamil Hypertension A medical condition where the blood pressure is chronically elevated. used to treat digitalis toxicity channel blockers Tocainide/mexiltine Class 2 Beta blockers vent arrhythmias only. and JVD. Antianginal drugs NOà increases cGMP. vasodilator agents. used in ischemic arrhythmias . Procainamide à local anesthetic. infection. can also be used to tx digitalis induced arrythmias Class 3 K channel blocker-refractory treatment. and is a leading cause of chronic renal failure. no/little excercise. Btretylium. and obesity. Cardiovascular disorders 1. arrhythmia. Symptoms of decompensated heart failure include dyspnea on exertion. The usual heart irritants can make CHF deadly: arterial plaque. assoc with pressure points joints anxiety Sharp Generalized varies Inflam. other causes of toe walking: CP → spastic paralysis. a-methyldopa (do LFTS. Carbonic anhydrase inhibitors Loop diuretics o Types: Fursosemide-Lasix o Adverse effects: causes an increase is K loss Ø Thiazide diuretics o Adverse effects Ø K sparing diuretics o Types: Spirinolactone. fibromuscular dysplasia(. resulting in natriuretic factor being secreted to promote salt excretion with the side-effect of raising total peripheral resistance. hardware. AK. Antihypertensives Diuretics Mannitol. leading to increased stress responses. ankleàSuspect occlusion is gradient is >20-30 between 2 levels TCPO2ß Wyss. Harrington and Burgess JBJS 1988 Normally 30-40mmhg 16 . Sedentary lifestyle. elevated K+. blurred vision caused by increased intracranial pressure and end-organ damage. and CXR Lifestyle modification with weight loss and diet. Guanadrel Direct vasodialators Apresozide. making blood vessels stiffer and w/ the reduced elasticity comes a smaller cross-sectional area in systole. uses UV ligh to create wave form s that are similar to Doppler. Nanitroprusside. hyperaldosteronemia.45 indicates healing potential TBI >0. but in absence of raised intracranial pressure.8à >0.chronic renal failure due to chronically high blood pressure HTN is rarely severe enough to cause symptoms. vasodialator Angiotensin receptor antag -artans—blocks the angiotensin 2 receptor. Amiodarone o Adverse effects Beta blocker Decreases HR. CO. so often multiple drugs are necessary to achieve blood pressure control.2 severe risk *more reliable than ABIs Digital pressures Normal-70-110mmhg. drugs (especially oral contraceptives) 1. it is called hypertensive emergency. Tobacco smoking . The increase in blood volume leads to hypertension.2 Intermittent Claudication-05-0. High salt intake. 3. and HR. Arterial. and so a raised mean arterial blood pressure. Stress.High levels of saturated fat in the diet. biphasic-may be normal in pts with DM. hydralazine. Alcohol abuse.6 low risk. Cushing’s disease. <30-40mmhg indicates poor healing Segmental pressures Thigh. nitroglycerin infusion Ace inhibitors -prils-à cough and hyperkalemia via its inhibition of aldosterone that causes hypoK and hyperNa Ca channel blockers Decreases TRP. monophasic-slower Q-indicates significant abnormality Plesmography Check the microvascular status. urinalysis (proteinuria). Minoxidil (à hirsutism) Diozozide. Kidney function: Serum creatinine and BUN are elevated. Malignant hypertension (or accelerated hypertension) is distinct as a late phase in the condition. the number of collagen fibres in artery and arteriole walls increases.damage to the retina nephropathy . These pressures without signs of end-organ damage (such as renal failure) are termed accelerated hypertension. Inability of the kidneys to excrete sodium. BK. may ppt CHF and angina A2 agonists Clonidine. prodrug) à does not cause reflex tachycardia A antagonists Phentolamine and Phenoxybenzamine (non-selective) used in the tx of pheochromocytoma …. OK in pts with renal dz b/c no dye is used ABIß Wagner in 1979 Normal-09-1. <0. Cerebrovascular accident (CVAs or strokes) Myocardial infarction (heart attack) Cardiomyopathy (heart failure due to chronically high blood pressure) Hypertensive retinopathy . intrinsic renal disease. These only surface with a SBP> 240 mmHg and/or a diastolic blood pressure over 120 mmHg. lymphatic Review of arterial and venous testing Duplex ultrasound 8-10MHz of U/S Triphasic-faster Q wave form resembles a teepee. 2. EKG. losing a kilogram of mass generally reduces blood pressure by 2 mmHg. and SBP S/Eà hypoglucose. Each added drug may reduce the SBP by 5-10 mmHg.zocin—selective à will cause reflex tachycardia and 1st dose syncope Post-ganglionic sympatholytics Reserpine. has no cough Ø Ø 2. and may present with headaches. Low birth-weight.Essential HTN Risk factors in development of HTN Secondary HTN Pathophysiology Complications Diagnostic evaluation Treatment Types No identifiable etiology Ageà Over time. DM Identifiable etiology Renovascular disease (atherosclerosis. coarctation of the aorta. The aim of treatment should be blood pressure control (<140/90 mmHg. An overactive renin / angiotension system leads to vasoconstriction and retention of sodium and water. When end-organ damage is possible or already ongoing. An overactive sympathetic nervous system. Pulse volume recording Measures the actual volume passing through a specific area MRA Contraindicated with a pacemaker. venous.). CO. hyperTGs. Obesityà In obese subjects. Platelet count 150. with reduced mortality and recurrence 3-4. Hemostasis: prevention of bleeding • Primary hemostasis à blood vessels and the platelets Secondary hemostasis à Coagulation proteins 2.000/mciro liter 2. Lee-White clotting time (whole blood coagulation time) 6-10 min 3. Bleeding time (finger stick) 1-4 min 4.5-3. Prothrombin Time: 11-13s guide for coumadin therapy ß International normalized ratio Recommended INR ranges Situation INR Prevention of venous thrombosis 2-3 Treatment of venous thrombosis 2-3 Treatment of PE.Normal clotting 1.000-400. and Afib 2-3 Prevention of recurrent DVT 2-3 Treatment post MI 2-3 Prevention of stroke. VIIa.5 Mechanical heart valves 2-3 17 . Xa. TIA. which is stabilized by factor XIII Coagulation factor of Human Blood Factor Synonym Deficiency I (Fibrinogen) Afibrinogenemia II (Prothrombin) Hypoprothrombinemia III (tissue thromoboplastin) IV Calcium ions Hypocalcemia V Labile factor Accelerator globulin Procaccelerin parahemophilia VII Stable factor. by thrombin. IXa. Prevention of thrombosis • Naturally occurring anticoagulants and platelet inhibitors. kallekrein and plasmin by forming stable complexes with theses enzymes • Heparin Slows the conversion of IIà IIa Potentiates ATIII Decreases platelet activity and adhesiveness Preventing activation of IX and VII Coagulation studies 1. Proconvertin or Prothrombin conversion accelerator-SPCA Hypoconvertenemia VIIIa AHF Antihemophilic factor Classic hemophilia VIIIb vWF Von Willebrand disease IX (plasma thromboplastin component-PTC) Christmas factor Hemophilia B-Christmas disease X Stuart Factor Stuart Stuart def XI plasma thromboplastin antecedent-PTA PTA def XII Hagemon factor Glass contact factor Hagemon trait XIII fibrin stabilizing factor Fibrinase XIII def Kallekrein pre-kininogen Fletcher factor HMW kininogen Fitzgerald factor Reactions of the extrinsic pathway are initiated by contact of blood with injured tissue Intrinsic Pathway III + VIIà VIIa à activates Xà Xa Extrinsic Pathway XIIà XIIa by surface contactà XIà XIaà IXà IXa + VIIIaà XàXa Xaà IIà IIaà Fibrinogenà Fibrinà clot!! ß Plasmin ß Plasminogen ß t-PA (clot breakdown) Regulation of hemostasis: Natural anticoagulants • Antithrombin III: Inhibits thrombin (IIa). Cellular injury à vessel vasoconstrictionà platelet adhesionà stabilization and reinforcement of the plug by the intrinsic and extrinsic systems à fibrinolysis Phases of the classic theory of coagulation Phase 1: Generation of tissue thromboplastin (Factor III) ß intrinsic pathway Phase 2: Activation of thrombplastin-end product of intrinsic and extrinsic pathways The final common pathway begins with the activation of Factor X Phase 3: Conversion of prothrombin to thrombin by Factor Xa Phase 4: Conversion of fibrinogen into fibrin.5 Prevention of embolism 2. no systemic plasminogen activation): t-PA (activase) DICà condition in which clotting and hemorrhage occur simultaneously within the intravascular compartment: Overall pathology: Unrestricted coagulation occurs affecting all systems à thrombotic eventsà rapid consumption of clotting factors (trying to destroy the clots)à bleeding Associated with three pathologic process 1) Endothelial cell damage (burns. decrease blood viscosity. X. Takayasu’s arteritis. Afib with embolism. coldness…etc Athersclerosis Obliterans (ASO) of the lower extremity Plaques tend to occur on the posterior aspect of the LE arteries .5-2. decreases blood viscosity by decreasing RBC aggregation. of VitK dependent factors: II. CNS surgery. L&D. (+) oxidative metabolism in the skeletal m. Plain X-ray films of the LE particularly if lesions are present. CBC. endovascular Description 18 . Coagulation Factor assays: II. kidney and serum chemistries Cardiac testing: 12 lead EKG if surgery is being considered. PDE inhibitors Persantine-PDE III inhibitorà decreases [cAMP] which leads to decreased platelet aggregation Pletal (clopidrel) PDE III inhibitorà decreases [cAMP] which leads to decreased platelet aggregation 3. Trental-pentoxyfylline (400mg ot id). also releases plasminogen activators thereby promoting fibrinolysis Fibrinolytic therapy • Prevent thrombosis and provide a rapid return to normal Q. decreasing TXA2. Blood chemistry: Fasting sugar. X Antiplatelet-type drugs 1. VII. Q to distal tissues is maintained thorough collateral vessels Clinical Intermittent claudication Rest pain Ischemic ulceration and gangrene Manifestations Ischemic neuropathy Disuse atrophy Muscular weakness and joint stiffness Laboratory studies NIVAs. XII Anticoagulants: Indications: venous thrombosis. lipid profile. VII. and improves rheorrheology. PTT 25-35s used a guide for heparin therapy (pts should have a PTT 1. and femorotibial reconstruction. MI. V. PE. eye surgery. IX. use with caution with liver and pancreatic disease Types: Heparin: LMWH-enoxaprin (Lovenox) and Coumadin-(-) the prod. HbA1c. Ticlid-250mg po bid 4. pregnancy. Arteriogram if intervention is considered Differential Atherosclerosis. phlegmasia cerula dolens. fibromuscular dyplasia. DVT. arthritis Treatment Lifestyle modifications: d/c smoking. ideally within ½-1 ½ post MI • First Generation Plasminogen activators: (no longer used) Streptokinase and Urikinase • Second generation (fibrin target specific. platelet concentration ü Heparin. Other areas: common femoral a. IX. popliteal Diagnosis artery entrapment. when thrombotic events are present: cyanosis. MI) causing the 2) Release and activation of tissue thromboplastin which activates the coagulation cascade 3) And direct activation of X (snakes. XI. femorropopliteal. thereby preventing end organ damage • 27% reduce the mortality of MI • Treatment within 6 hours of MI. m/c bifurcation sites. physiologic and hemodynamic states.Tissue heart valves 5. Cystic adventitial disease of the popliteal a. venoms) ß involves most often the extrinsic system Treatment ü Remove the offending agent ü Restore the appropriate balance between coagulation and fibrinolysis ü Maintain organ viability ü Fluid replacement ü Antibiosis as needed ü FFP. lumbar canal stenosis. ergotamine use. aortoiliac-femoral. stress testing. degenerative joint disease of the jip and back. surgery. LFts. profunda femoris. severe HTN. and surgical prophylaxis Contraindications: allergy active bleeding. brain injury. Persantine Surgical treatment: Lumbar sympathetctomy.5x normal) 6. decreasing elevated plasma levels of fibrinogen. ASA : (-) COX. causing decreased platelet aggregation/increased vasodilatation 2. GI ulceration. atheromatous emboli. distal superficial a at Hunter’s canal and the tibioperoneal trunk. Pharmocologic tx: 3 goals: 1) Increase walking distance 2) Decrease the need for surgical revascularization or endovascular therapy3) Prevent the progression of atherosclerosis and reduce cardiac and CV mortality and morbidity. arterial embolism. diet control. Pletal-cilostazol and Antiplatelet drugs ASO drugs: Ticlid. Trental (pentoxyfilline)-alters the rheology of RBCs (increases flexibility. Throboangitis obliterans. Exercise tx: Increase collateral Q. Giant cell arteritis. ASA. Fibrinogen 200-400mg/dl 7. chronic pernio. Ankle edema > 1cm Multiple dilated veins varicosities. with Edema> 2cm. or w/o claudication multiple vein varicaosities. mild pigmentation and mild liposclerosis Severe swelling. o Incidence – uncertain.S. The build-up of toxins and metabolites leads to changes as visible dry.therapy (PTA. confined to the deep veins of the calf. Prevent thrombus extension: Heparin bolus 5000-20. 20% KIM and extend into proximal deep veins of the thigh. Grade I II III S/S Physical findings Mild swelling. stent placement) amputation Some hospital order/prescriptions for acute arterial occlusion 1. skin changes Incompetent perforating veins. May dose by weight: 80 U/kg IV bolus the 18 U/kg/hour infusion 3. The proteins polymerize forming sclerosis of the surrounding tissue (lipodermatosclerosis-gives the leg an inverted champagne bottle resemblance). that presents as a painful.000 U followed by 2000-5000 U/h infusion a. superoxide radicals and cytokins leading to skin breakdown and ulcers 1) Edema: This may be the first manifestation of early CVI.000 hospitalizations each year in the U.S. Augmentation on distal compression No augmentation on distal release No augmentation on proximal compression Augmentation on proximal release Treatment – Nonsurgical The cornerstone for control of CVI is the control of venous hypertensionß external compression. particularly because it is under recognized and misdiagnosed o Most thrombi in leg are small & asymptomatic. Sclerotheraphy-success is dependent upon producing an inflammatory response in a limited section of a vein (SotradecalSTD: Soidum tetradecyl sulfate 3% and monolate: monoehtanolamine oleate Radio-Frequency Endovenous Occlusion-Closure ß aternative to saphenofemoral ligation and/or stripping DEEP VEIN THROMBOSIS INTRODUCTION o Deep venous thrombosis (DVT) is common among hospitalized patients characterized by the formation of thrombi in the leg veins. 2) Venous dilation and varicosities 3) Leg pain: The most frequent type of pain is limb heaviness or ache which occurs after prolonged standing (involvement of the superficial venous system) but can also show a deep venous insufficiency à venous claudication that is felt as a bursting or cramping pain (related to an iliofemoral venous obstruction) or pain associated with valvular incompetence that is felt as sudden heaviness or ache that is quite debilitating. 200. Predisposing risk factors of DVT I am clotted Immobility/inactivity Longevity of surgery Estrogen Atrial fibrillation/ age Obesity DVT history Morbidity/ MI/malignancy Tobacco/tourniquet Coagulable state Tumor/trauma 1. heaviness. Pain: Demorol 100mg IM q4h prn pain or 25-50mg IV q4h prn pain 2. intra-operative and post-operative situations & inactivity after discharge. 19 . calf pain. Long-term anticoagulation: Coumadin 5mg po qd x 2-4 days then adjust to INR of q2-3 or PT (2-2.5x control) Chronic venous insufficiency Definition Patho-physiology: Long-standing incompetence of the perforating communicating veins of the LE that drain 90% its blood. à fatal pulmonary embolism (PE). 300. The Perthes test to examine the patency of the perforating valves Duplex scanning imaging is the test of choice in evaluating venous disease. marked skin pigmentation. scaly dermatitis. decreasing gradually to 12 to 17mmHg below the knee (sustained pressure).600. Activated WBC’s (cell trapping theory) release their proteolytic enzymes. noninvasive. severe liposclerosis and the presence of an ulcer Clinical findings Clinical Classification Diagnosis NIVA* Biphasic Trendelenberg or retrograde filling test to assess the competence of the perforators and saphenofemoral valve. swollen leg/arm or asymptomatic o Two major complications . Multiple dilated veins. The middle perforators are located just proximal and extend 7-10 cm above themedial malleolus The basic underlying pathologic mechanism in chronic venous insufficiency is venous hypertension. Incompetent perforating veins (severe). Normal skin and subQ Moderate-heavy swelling. and readily available. vein dilation Ankle edema < 1cm Dilated superficial veins.000 . PE was 3rd most frequent cause of death in the U. >70% of patients who die of PE are not suspected of having it before death.pulmonary embolism and post-phlebitic syndrome. It is highly sensitive. heaviness.000 people die each year of DVT and its complications. The amount of pressure at the ankle should be between 30 and 40 mmHg. The RBC’s break down and the hemoglobin oxidizes à brownish hemosiderin staining of the skin. for venous thromboembolism. Immobilization: o risk for pre-operative. if thrombus does not embolize à resolved by canalization. it is loosely attached to the vein walls. a swollen. blue discoloration. Significant thrombosis may be present w/ minimal findings. Fate of the thrombus . or pelvic surgery). Propagates in the direction of Q by addition of successive layers of cellular components & fibrin and when the thrombi reach large vein of the thigh. decreased fibrinolytic activity. o Treatment of DVT A painful. venous dilation (venous stasis) common in the elderly patient. increased estrogen o Produces venous dilation. painful limb w/ a questionable or negative duplex scan requires a venogram for adequate criteria to initiate treatment.patients undergoing orthopedic procedures (hip. Pregnancy o DVT d/t decreased fibrinolytic activity during the 3rd trimester. venous stasis resulting from venous dilation. tenderness: palpable firm cord (75%).in the paralyzed limb & post-CVA patient. Impedance plethysmography (IPG) is a NIVA. positive in 33% of cases. UA. Surgery o Greatest risk .g. Unusual presentation of DVT à Phlegmasia AIba Dolens & Phlegmasia Cerelea Dolens dDx OF DVT Muscle aches.high risk . patients w/ incapacitating cardiac & pulmonary disease (COPD). most reliable technique for the detection of DVT that is confirmed as an intraluminal filling defect however. or lysis by fibrinolytic sys. knee. Malignancy o DVT in unusual anatomical locations & develop thrombi resistant to anticoagulation therapy. UG tract. stasis from immobilization. organization. 95% specificity for dx of IFDVT. BCP-7. o Thrombogenic factors associated with surgery à release of tissue thromboplastin. and hypercoagulability (increase activation of clotting factors). Contrast venography is the gold standard. 7. healthy patients during long airplane rides or automobile trips 2. warmth. Age o Patients >60yrs @ highest risk for developing DVTß decreased activity due to systemic disease. & insensitive to infrapopliteal occlusions. cancer. PATHO-GENESIS Virchow’s triad: endothelial damage. increasing the potential for detachment & embolization. breast. and intracranial tumors à greatest risk for thrombosis Phelmasia cerulean dolens Phlegmasia alba dolens Venous gangrene “Milk leg” Pain. night cramps. reestablishing the balance btwn thrombosis/ fibrinolysis. BCPs. GI tract. MRA has 100% sensitivity & NPV & a specificity of 98. pain (50%). and posterior tibial veins. mechanical pressure of the uterus on the Inferior Vena Cava during the 3rd trimester and during delivery. increased focus on D-dimer assays. swollen leg and a positive duplex scan initiate therapy for DVT. A hand-held continuous-wave Doppler – for examining venous flow in the groin. o May develop into phlegmasia cerulea dolens (venous gangrene). marked edema Pale or white leg caused by extensive iliofemoral signs of arterial insufficiency. which are rapid. CLINICAL àedema. and + Homans sign (pain in the FEATURE calf with passive DF of the ankle /foot). LFT’s) 20 . 97% sensitivity. Obesity o Impairs fibrinolytic activity & promotes greater inactivity à an increased frequency of DVT in obese pts (who weigh more than 20% above standard) 6. & a reduction of the fibrinolytic system activity 3. since D-dimers are also found in other dz states (e. and start in the tibial veins and the soleal sinuses à popliteal & femoral veins. non-invasive/ inexpensive.5%. neurosurgery & major abdominal or pelvic surgery for malignancies. popliteal fossa. CHF. these patients compression secondary to profound edema or venous often have an underlying malignancy and many thrombosis-associated with pregnancy (post-partum) will require an amputation 5. MRAcould dx either thrombosis or compression of the vein. procedure puts patients at risk for a DVt Real time (duplex) u/s most accurate NIVA for dx of proximal DVT but less reliable in detecting small isolated thrombi. o Release of tissue thromboplastin at the time of placental separation. high sensitivity & specificity in the diagnosis of proximal DVT (proximal and popliteal veins). o Patients w/ neoplasms of the lung. erythema. inflammation) not specific. Patients w/ DVT à hospitalized and anticoagulated o Obtain a coagulation profile and routine labs (CBC. and >50% false positives. decreased functional levels of antithrombin III & decreased fibrinolytic activityà vasoconstriction that lead to clot formation DVT Pathology Venous thrombi form in areas of the venous system where the blood is naturally stagnant (stasis) around the valvular system. the dx should always be established by an objective invasive or noninvasive test. muscle tears Arterial insufficiency Bakers cyst Arthritis Neurogenic pain Cellulitis Myositis Lymphangitis Panniculitis Vasculitis Fibrositis Gout Fibromyalgia Superficial phlebitis varicose veins May-Thurner syndromeà Extrinsic compression within the pelvis Dx Because clinical findings are notoriously unreliable in DVT. o DVT also in pediatric population due to central venous lines (iatrogenic cause) 4. venous stasis. early labor & before placental detachment. D-dimers: commonly found in circulation when a thrombus is present. In the lower extremities the lymphatic channels are located in the superficial (subcutaneous) and deep compartments. popliteal and femoral. cause acute onset. sarcomas. malaria.10. and other systemic disease. Pt will eventually be discharged with po Coumadin.2 x control (1. Pulses are present.000 U. Lymphedema is caused by the inability of the lymphatic system to remove lymph from the interstitial space. chills.c in the extremities but also in the lower abdomen. the swelling is painless and is more resistant to pitting when compared with edema of venous nature. qd x 2-3d. The dx is based on clinical presentation and clinical findings. open prostatectomy Pneumatic compression stockings Orthopedic surgery. The superficial system.O. Dose: 10 mg P. in cases of severe hemorrhage who require rapid reversal à FFP 2 -4 units o Other treatment modalities include: Low-molecular-weight heparin (LMWH): Lovenox (enoxaparin) 1mg/kg q12h. The deep lymphatic system follows the deep veins: tibial. Stage I Mild: Reversible edema Pitting Reveresed by elevation Normal at waking Stage 2 Moderate: Spontaneous irreversible Need tretment to decreases swelling Pitting takes longer time to resolve Spongy tissue Stage 3 Severe: Lymphoclastic elephantiasis Non-pitting edema Hard. The central portion of the lesion may ulcerate. The presence of s/s indicating venous insufficiency. and the extremity is of normal color. and elevation of the LE for the 1st 48 . pneumonia. subsequent daily doses are adjusted to therapeutic INR of 2-3 or a PT of 1.000 .5 .5 mg IV o if rapid reversal is required d/t serious bleeding or the INR > 20 -à vitamin K 10 mg IV o Re-check INR in 6hrs. This condition should be treated aggressively w/ wide radical excision and chemotherapy. which eventually enlarges and produces satellite lesions.5-2 x control. is located along the GSV. o Reversal of Coumadin: o INR > 6 < 10 w/ no serious bleeding à vitK: 0. fibrous connective tissue. Red (dorsum) and ankle. o Administer local heat (AK pads) and analgesia as needed. typhoid.000 U SQ ql2h AND / OR pneumatic compression stockings LYMPHEDEMA Definition Introduction Lymphedema is defined as the accumulation of fluid (lymph) in any part of the body.000 .Heparinize w/ bolus of 5. and the face.72h. hospital intermittent compression pumps (immobilized with DVT prophylaxis of 5-10ku (immobile). painless. thighs pitting edema w/ no signs of ulceration radical mastectomy (lymphadenopathy).The 2 systems (superficial and deep) join at the level of the groin and transport the lymph to the thoracic duct. and neoplasms of bone. Because of this. which causes stasis and accumulation of the lymph. Diffuse cellulitis Lymphedema tardae After age 35 and a portal of entry on the foot Classifications Clinical Picture Diagnosis Treatment Complications of lymphedema Enlargement of the extremities or affected body part. No cure Malignant lymphangiosarcoma will develop from chronic distorted and enlarged lymph channels. surgery. TYPE OF OPERATION OR Condition PROPHYLAXIS Abdominal or thoracic surgery Heparin 5000 U SQ ql2h starting 2 h preoperatively AND / OR pneumatic compression stockings Eye surgery or neurosurgery. nonWomenß ovarian CA and post streaks on the feet. Heparin dosing based on patient weight (80 U/Kg of actual body weight by bolus followed by 18 U/Kg/h infusion) may achieve therapeutic anticoagulation more rapidly without an increased risk of bleeding o Recall that heparin does not break down the clot.15. It should be mentioned that no effective chemotx exists and the condition is easily blood-borne causing metastasis elsewhere. LEs. it gives the patient’s fibrinolytic system time to break clot down o Heparin Reversal – Protamine sulfate 10-50mg slow IV infusion over 10min. the external genitalia. (congenital familial or Milroy’s disease) Birth or early in life Secondary ß results from obstruction of the lymphatic channels secondary to mechanical occlusion due to tumors or metastasis or fibrosis from radiation (radiation fibrosis). 1mg/100U of heparin o PTT should be checked 4h after initiation of therapy until therapeutic anticoagulation is achieved. Limb is very large and swollen Clinical Stages Lypmhedema 21 . PREVENTION of DVT: The basis for prevention is effective prophylaxis. o Complete Bed Rest (don’t want to mobilize the clot). amputation. Coumadin may be initiated on admission to prevent PE (it takes 3-5d to start working). This is usually seen as a bluish cutaneous area resembling a bruise. HTZ. followed by a continuous infusion adjusted to a PTT of 1. high temp. arterial insufficiency.5-1 mg IV or if the INR > 10 < 20à vitamin K 3 . Bancrofti Other: influenza. outpatient compression therapy*.000 U/h).c. Fragmin o Thrombolytic therapy t-PA. And is caused by an inflammatory process in which the lymphatic channels become obstructed USA: streptococcus World: W. m. followed by radiation tx is often favored over radical excision and chemotherapy. Lymphedema precox Obstructive (most common) Inflammatory Before age 25 (9-19 years) Abrupt onset (hallmark) Recurrent lymphangitis cellulitis à Notice “puffiness” around the foot Menß prostate CA m. and then q6 . Vena cava interruption (Greenfield filter) Note: untreated DVT 25% à PE & 35% will be fatal. which empties into the left jugulosubclavian venous junction. Hodgkin’s disease.8h. DVI. Her notes: TQ: Tx for patient includes: Lasix. the scrotum. which is now available to most non-surgical and surgical patients. Thrombectomy. hip or knee replacement LMWH 30 mg ql2h or pneumatic compression stockings OR adjusted doses of heparin SQ to prolong PTT 1-3 sec OR warfarin beginning the night of surgery General medical patients including stroke and MI Heparin 5. systemic edema. which is more important and carries 80% of the lymph from the lower extremities. and allergic causes of edema should be ruled-out. radiculopathies. termed Sudek’s atrophy this is an inconsistent Based on finding that may occur also with disuse or immobilization syndromes. but sensitivity varies from 50% to 90%. if nerve involvement is suspected. cervical OA and HSV and other 15% are idiopathic. and autonomic dysfunction.chronic pain syndrome not associated with nerve damage Description RSD is a commonly misdiagnosed condition characterized by severe pain.4th wk to avoid HPA axis suppression). livido reticularis Dystrophic months and cyanosis are present. contractures. Steinbroker (clinical stages of RSD) Duration Description 3-6 months Pain. favoring of one limb. isonizid and barbiturates) Evidence of SNS over activity and the pain can be stopped with SNS blockers or sympathectomy o RSD. The skin is cool. diffuse osteopenia on x-rays. metastatic or ischemic disease 3. livedo reticularis in the contralateral limb. swelling. giving a dusky purple appearance and causing dependent rhubor. contractures. renal failure. active infection. Sympathetic nerve blockade (medical à Phenoxybenzamine or surgical decompression is a popular treatment modality for RSD. or if present limited to the skin of the fingers -No evidence of underlying disease A history of symptoms for at least 2 years Supported by measurement of ANA (indicates the presence of a systemic diease is present-is normally high in the very young 22 . and capsular retraction (physical therapy). Edema is increased. may present with a throbbing pain. hypersensitivity. throbbing and may decrease over stage 2 and may spread proximally. Triphasic sequence of color changes that is well-demarcated Whiteà (pallor) ischemia. can also be assoc w/metabolic bone dz. tenderness. or thrombophlebitis. III Atrophic Many years Atrophy of the subQ. disability and atrophic skin changes. Caused byà entrapment neuropathies. stroke.) More common in patients 40-60 years but can also occur in children and the elderly Diagnosis Plain radiographs of the involved extremity may show patchy osteopenia. MI. and vasomotor changes that lower or raise the temperature of the extremity. limitation of motion. DVT. SNS activity is decreased from stage 2. and idiopathic. Treatment Goals of therapy: pain control (Narcotics). face. ears and chest can also be affected.Pulses are present and normal unless underlying disease exists Diagnosis Allen and Brown classification in the Dx of Primary Raynaud’s Vasospastic attack precipitated by cold or stress -Bilateral involvement of the extremities Absence of gangrene. most commonly tingling and burning “pins and needles” Pathophysiology Heightened vasomotor tone and increased blood viscosity due to increased fibrinogen levels Clinical picture Vasopastic attacks of the fingers (sparing the thumbs) mainly (50%). Nerve conduction velocity studies may be performed. muscle spasms. arthroscopy. Thermography can detect temperature changes preceding trauma between the limbs. Amputation (rare) is required to control pain. often the only color change Blueà venous congestion and cyanosis Red à reactive hyperemia. dystrophic changes in the skin and nails.*Contraindications to Compression tx: CHF. Vasomotor REFLEX SYMPATHETIC DYSTROPHY (RSD) AND CAUSALGIA Represents of group of disorders and wide number of terms or syndromes employed to describe themà Current term is “Chronic Pain Syndrome” Two major types: o Causalgia-chronic pain syndrome associated with nerve injury. neoplasm. but this test is non-specific. Stage I Acute Vasospastic diseases Raynaud’s Disease Description Cutaneous color changes. muscle weakness. marked loss of function: irreversible impairments-muscle atrophy. however. Therapeutic blocks can be performed by injection of local anesthetics into the lumber sympathetic ganglia for LE RSD Oral corticosteroids (Predinone 60-80 mg qid for 2wks and then tapered/discontinued by the 3. About 60% of the causes of RSD are posttraumatic. II Another 3 Characterized by persistent pain. The skin is cyanotic and edema is less from stage 2. defined as burning pain (allodynia) and exaggerated subjective response to pain (hyperpathia) usually in the hand or foot after partial injury of a nerve or one of its major branches. eradicate infection. incoordination. Three-phase bone scan is specific for RSD. Etiology The precipitating causes of RSD can be grouped into traumatic. Minor or trivial trauma is the most common precipitating factor About 25% are associated w/ non-traumatic conditions such as failure to rehabilitate after stroke. joint rigidity. cyanosis. neoplasms. and depression or anxiety (TCAs). or improve function. non-traumatic. less than half have attacks of the toes. Patho-physiology: Current theories: two different mechanisms: altered sympathetic outflow and regional neurogenic inflammation Clinical Picture PAIN OUT OF PROPORTION to the inciting injury or event (reactive hyperemia. produced by reversible spasm of arterioles/ small arteries brought on by exposure to cold or emotional upset. tremors. pain is burning. a/v thromboses and drug use (rifampin. sweaty. can be iatrogenic. the recurrence of RSD in the stump is common. The nose. early join mobilization prevention of contractures. and only a few with attacks of only the toes. elbows and knees. Esophogeal motility abnormalities. lethargy. Treatment of Raynaud’s phenomenon: 1. and impotence) i. necrotizing lesions of skin may be present. palpitations. Drug therapy a. Gottron's sign . Dx 23 . m. contractures & decreased DTR's Primary Idiopathic Dermatomyositis Description > 1/3 of all the immune myopathies. depression. anemia. Photo sensitivity. hips and thighs. Overlapping Dz process with scleroderma. muscle atrophy. Tolazoline 25-100 mg qd Scleroderma(Systemic sclerosis) Connective tissue disease SLE Connective tissue disease Sjogren’s syndrome (SS) Polymyositis and Dermatomyositis A. Prazocin 2-8 mg qd ii. Polymyositis and inclusion body myositis à Skeletal muscle breakdown and elevation of [CPK] and [aldolase] characterize these autoimmune disorders (anti-Jo 1 antibody) diagnostic markers Primary Idiopathic Polymyositis Description 1/3 of all inflammatory myopathies. dysphagia. Skin Changes localized or diffuse erythema. dizziness. Conservative: Reassurance that the disease is benign. dysarthria and pulmonary fibrosis. diarrhea. distal muscles are spared in 75% of the cases. 1. ischemic infarction of the kidneys. seizures. butterfly patterned(malar) rash. periungual erythema. arthritis and arthralgias. ESR (helpful but non-specificà normal in the primary. Photosensitivity. pericarditis and nephritis.25-0. arthritis. breast. avoid cold and smoking. arthralgias. psychosis. dyspepsia and pruritis) i. classic heliotrope rash on eyelids. rheumatoid arthritis and mixed connective tissue disorders. elevated in the secondary form) With no limb-threatening vasospamà conservative therapy with heparin and bed rest Secondary causes of Raynaud’s phenomenon CREST syndrome: Calcinosis. fatigue. thrombocytopenia and leukopenia Immune mediated disorder of exocrine sweat glands Keratoconjunctivitis sicca (dry eyes). gastrointestinal tract and myeloproliferative disorders. Rheumatoid arthritis Raynaud’s phenomenon as well as focal ischemic lesions with microinfarctions of the perilungal area and digital pulp. Calcium channel blockers (SE: dizziness. Guanethedine 10-50 mg qd c. normal CK levels on serial testing in pt w/ symptoms of myositis suggests increased likelihood of malignancy. rash) i. associated malignancies . pain described as "aching" type in buttocks. dysphagia. Methyldopa 1-2 mg qd iii. difficulty arising from a chair. an erythemetous skin rash (polymyositis). Telengectasiasà Distal ulcerations. Nifidipine 10-20mg tid slow release preparations are better tolerated ii. Rheumatologic disorders Myopathies (primary and secondary) Muscular Dystrophiesà progressive dz where the pattern of weakness of the muscle determines the dx Myopathies Inflammatory Myopathies:Dermatomyositis. Elevated CPK and aldolase Mixed connective tissue disease Joint. muscles. edema. Scelerodactyly. Bad prognosis: Childhood Polymyositis & 8 to 20% of these processes. Raynaud's phenomenon Secondary Polymyositis or Dermatomyositis with Neoplasm Description Pt of a paraneoplastic syndrome. Heliotrope of purple discoloration of the upper eyelids is characteristic of dermatomyositis. palpitations. weight loss and fever. ovary. GI tract and even the brain have been reported Polymyositis or Dermatomyositis Assoc. Ø an exfoliative dermatitis. Clinical Presentation Proximal limb weakness –esp. arthralgias. Phenoxybenzamine 10-30 mg qid iv. Reserpine 0. thighs and calves →tender w/ deep palpation. Xerostemia (dry mouth) and dry vagina in women Symmetrical muscle weakness.c. poikiloderma.pink to violaceous scaling typically over the knuckles. alopecia. muscle. minimize stress 2. The mc type of malignancy is the same that is most typical for the pt's age and sex. arthralgia or arthritis. edema. morning stiffness. an pulmonary and renal vasculature involvement Autoimmune disease. gastrointestinal Dermatomyositis Associated w/ Vasculitis tract and other organs. fatigue. bridge of the nose and cheeks (butterfly or "malar" distribution – periobital area). skin changes may precede muscle involvement.lung. shawl sign. maculopapular eruption or scaling. dyspepsia.5 mg qd ii. muscle/ skin changes are indistinguishable from above Presentation Dz processes. Alpha-adrenoreceptor antagonists (nausea. pulmonary and renal involvement. decreased HR. Dilitiazem 30-120 mg tid b. clinical or pathologic evidence of vasculitis in skin. Sympatholytic agents (postural LBP.Treatment and old). Diagnosis Most time serum CK levels elevated. Raynaud’s Phenomenon. flushing. anorexia. Swelling of the dorsum of the hands and fingers with sausage shaped appearance of the digits. F>M 2:1. 20% (6-60%) of Dz. consistent w/ localized edema and loss of soft tissue planes.) • Microbiologic studies. with irregularity of the subchondral 24 . hyporeflexia or areflexia. including a Gram stain and cultures Condition Normal Osteoarthritis Traumatic Arthritis SLE Rheumatoid arthritis Gout (acute) Pseudogout Bacterial arthritis (acute) Tuberculous arthritis Color/Clarity yellow/clear yellow/clear yellow to bloody/ clear to bloody yellow/clear to slightly cloudy yellow/cloudy yellow/cloudy yellow/clear yellow to bloody/ cloudy to purulent yellow/cloudy Viscosity high high high decr low low low low low Total WBC -%PMN 0-200/µl (0-25%) 200-2000/µl (0-25%) 200-5000/µl (0-50%) 2000-5000/µl (25-50%) 5000-50.à examination for the presence of microorganisms and crystals is the most critical part of the analysis. Adrenal Disorders Pituitary Disorders Vitamin Deficiency 2. pain most likely reflects the underlying bone disease. direct extension. attracting neutrophils. interfering with cartilage nutrition.000/µl (90-100%) Formed Elements none collagen fibrils cartilage fragments & RBC’s LE cells cholesterol crystals (occ) urate crystals CA pyrophosphate crystals bacteria on gram stain gram stain – 20% positive cultures – 80-90% positive 2000-100. hyperreflexia hypopPTH→ present w/ neuromuscular involvement. radiographic changes in subchondral bone are evident and the infection has advanced from the joint cavity into the bone. presence of Chvostek's sign (spasm of the facial muscles elicited by tapping the facial nerve in the region of the parotid gland) & Trousseau's phenomenon (spasmodic contractions of muscles provoked by pressure upon the nerves that supply them) endogenous elevations of glucocorticoids → severe muscle weakness and wasting acromegaly associated with muscle enlargement. weakness of pan-hypopituitaryism d/t coexistent adrenal or thyroid insufficiency severe malabsorption may → vitamin E deficiency myopathy. heparinized tube (green top) or EDTA. hypothyroidism → weakness & pain.w/ Connective Tissue Dz difficult. begins as synovial infection → contamination of the joint fluid and space HOW? Bacteria in synovium incite host defense mechanism releasing enzymes and toxins. synovial fluid glucose and protein determinations. by the time cartilage is destroyed. then chondrolytic enzymes are released. vitamin D deficiency d/t decreased intake or decreased absorption associated w/ renal disease may → chronic muscle weakness. cartilage is rapidly destroyed by this process beginning either in the center of the joint or at the periphery where impingement by the synovial lining occurs. and immunologic studies (rheumatoid factor.polymyositis considered. rapidly destroying articular cartilage. Arthridities Synovial Fluid Analysis • Joints normally have small amt of synovial fluid & depends on the size of the joint. pts c/o muscle "pain" d/t underlying bone disease. late changes in septic jt – osteoporosis appears in the subchondral bone d/t synovitis → hyperemia.c. serum CK elevated →DDx . growth plate is closed four types: hematogenous.000/µl (50-90%) 5000-50. the joint space becomes widened d/t increased fluid content and pressure w/i infected joint. complement. 30-50% of bone matrix must be destroyed before it is seen radiographically chronic changes develop if the infection is left untreatedà joint space become narrowed. the fluid collected using aseptic technique and placed in a sterile.etc.000/µl (50-90%) 5000-50. implantation and post-surgical acute changes are non-specific. the type of cells and the presence and type of crystals • Chemical analysis – mucin clot test. synovial fluid production & pressure increases → local ischemia. CK elevated 10x normal even w/ minimal muscular involvement.000/µl (30-90%) Infectious Arthritis Etiology Clinical Presentation Septic arthritis ßoccurs m. color and appearance • Microscopic examination – total number of cells present. most findings are non-specific and can be found in a number of disease entities. adults have muscle hypertrophy with cramps (Hoffmann's syndrome) hyperpPTH →muscle weakness and atrophy. fluid is inspected for viscosity. Endocrine and Metabolic Myopathies Thyroid Disorders Parathyroid Disorders hyperthyroidism → muscle weakness. in infant b/c metaphyseal vessels penetrate the growth plate. antinuclear antibodies. tetany.000/µl (35-85%) >50. In adults. is not usually cutures in synovial fluid analysis Neisseria gonorrhea It affects women more frequently than men (4:1) and its highest incidence is among sexually active adolescent girls. 500 mg PO q12h or ciprofloxacin (Cipro). elderly persons. Must culture all mucus membranes. and rashes (1-mm to 2-cm red macules). 1 mg/kd IV q8h plus Nafcillin. Haemophilus influenzae causes half of the cases following an upper respiratory tract infection • in children over two years of age. children and immunocompromised persons (Enterococcus species) Neonates. 1-2 million U IV q4h Ampicillin. hemophilia or immuno-suppression • mycobacterial infectious are on the rise primarily as a result of HIV and immuno-suppressed patients • Mycobacterium marinum is seen in patients involved in aquatic hobbies • candidal organisms can reach joints via direct injection or hematogenous spread • Sporothrix schenckii (sporotrichosis) is an occupational disease affecting gardeners • Salmonella ssp. 1/2 of the cases are causes by Staphylococcus aureus and the other half by streptococci and gram-negative bacilli • Staphylococcus . 500 mg q12h x 7 days plus Doxycycline (Vibramycin. patients with SLE and IVDA** Staphylococcus aureus Group A streptococci Non-group A streptococci Aqueous penicillin G. Migratory polyarticular arthritis involving several joints in rapid succession and then settles in one or two. Single joint arthritis follows dissemination of the gonococcal infection and is associated with symptoms of fever.most common following joint aspiration or injection. Staphylococcus aureus is most common • in adults and the elderly. usually of a single joint (in 90% to 95% of cases) that occurs with gonorrhea. This episode may end as a single joint becomes infected. Staphylococcus aureus is responsible for most septic joints • in children younger than two years. children < 2 years. it may be accompanied by gram-negative bacilli and anaerobes • Staphylococcus aureus.Manifestation bone surfaces. multiple joint aches (arthralgia). streptococci. and ankle. 1-2 g IV or IM/day. Doryx) 100 mg q12h Nafcillin sodium (Unipen) 1-2 g IV q4h x 2-4 wks. 1-2 g IV q4h Gram-negative bacilli 25 . The most commonly involved joints are the large joints such as the knee. then PO x 2-4 wks Children >2 yr and adults who have polyarticular infection or are immunocompromised Elderly persons. sequesta (dead bony fragments) develop btwn the borders of the joint space → ankylosis of the involved joint Rash Identifying the Cause (gonococcal vs non-gonococcal arthritis) • in infants. chills. wrist. have been isolated from sickle cell patients and those with systemic lupus erythematosus • Pseudomonas auringinosa may be most common in patients with IVDA along with MRSA • Staphylococcus epidermidis is most common in the early infection of prosthetic joints whereas other gram positive cocci may be associated with late infections Gonococcal arthritis “Lover’s heel” Gonococcal arthritis is an infection. There is also increased risk during menstruation and pregnancy. 1-2 g IV q4h plus gentamicin (Garamycin) 1 mg/kg IV q8h Third-generation cephalosporin or gentamicin. then cefuorxime (Ceftin). Description Etiology Epidemiology Clinical presentation Treatment: Patient Profile sexually active young adults* Microorganism Neiserria gonorhoeae Adult Antibiotic Regimen Ceftriaxone sodium (Rocephin). gram-negative bacilli are associated with injection of coagulation factors in hemophiliacs • polyarticular arthritis is most commonly caused by Staphylococcus aureus in patients with rheumatoid arthritis. Reiter's Syndrome or Reactive Arthritis • • • • referred to as “reactive arthritis” believed to be precipitated by a bacterial infection in the genital. urinary or gastrointestinal tracts.classic clinical triad or inflammation of the genital. and it will expand over the following days to form a large annular lesion • the central portion of the lesion may become extremely erythematous and indurated. Salmonella ssp. About 8% of pts develop cardiac involvement and the m. mostly in developing countries. headache and fatigue. motor or sensory radiculoneuritis. Mono. nervous system. lymphatic system. rubella. bones and joints. infecting 2 billion people or 1/3 of the world's population. preceded by a bout of non-gonococcal urethritis (often caused by chlamydia. some individuals respond by the development of reactive arthritis chronic form of arthritis characterized by . and Campylobacter fetus) Keratoderma Blenorrhagicum is the cutaneous manifestation of Reiter’s syndrome 26 . mumps and parvovirus Conservative regmen of rest and NSAIDS Seronegative spondyloarthropathies (RAPE) 1.intermittent or chronic arthritis and neurological stage arthritis is the hallmark of late disease and may occur years after the initial disease process • this stage will demonstrate severe progressive arthritis. chronic fatigue syndrome. ataxic gait. abnormality is fluctuating degrees of AV block • cardiac involvement usually lasts only a few weeks but may recur within several months after the infection Stage 3 Disseminated Disease . It is a multi-systemic condition involving the skin. vesicular or necrotic • it may develop multiple rings and is warm to the touch • w/i days.c. It is most commonly localized in the thoracic portion of the spine. encephalomyelitis. conjunctivitis and inflamed joints (arthritis) chiefly in young men with an HLA-B27 genotype (positive in 60 to 75% of patients).o. resulting in two million deaths. Tuberculin test (PPD) Rifampin 600mg and isoniazid 300mg in a single daily dose for 18monà conventional tx: 9 mo is the short course of tx M. urinary or gastrointestinal systems urethritis. Yersinia ssp. spastic paresis and polyradiculopathy and the arthritis is characterized by intermittent attacks of oligoarticular symptoms of the large joints – occurring most commonly in the knees TX • amoxicillin is the DOC for early disease – 500-1000 mg 3xd for 10-14d in early disease and up to 28d in disseminated disease • doxycycline – 100 mg p. a kind of tuberculous arthritis of the intervertebral jts. circulatory system (Miliary tuberculosis). dependent on histocompatability antigens. bid for 14 to 21 daysà not recommended for children less than 8 years of age • Ceftriaxone (Rocephin) – DOC for CNS involvement or persistent arthritis – 2 grams IV qg for 14 to 21 days Tuberculosis Dsecription Epidemiolgy Joint and bone involvement Diagnosis Treatment infection caused by Mycobacterium tuberculosis. preventative tx is only effective w/ INH Empirical Tx: anti-mycobacterial therapy:Initial 4 drug combo : INH Rifampin Pyrazinamide Ethambutol or streptomycin (INH resis) Then INH & rifampin w/o pyrazinamide for next 6mo Viral arthritis Description Etiology Treatment Presents as a self-limiting mild inflammatory nondestructive arthritis that lacks suppuration-usually begins as a migrating polyarthralgia that rarely lasts for more then 6 weeks Hep B. “Pott’s disease” is a presentation of extrapulmonary TB that affects spine. Shigella flexneri.Lyme disease Description Lyme disease is caused by the bacterium Borrelia burgdorferi (a sphirochete) and is transmitted to humans by the bite of infected blacklegged ticks (ixodes diamini) Clinical presentation Typical symptoms include fever. TB is one of the most deadly and common infectious diseases today. 9 million new cases of active dz. along with musculoskeletal symptoms – it generally occurs three to 10 weeks after exposure to the bacterium • Neurologic s/s are suggestive of meningeal irritation w/ cranial neuritis. Also called tuberculous spondyloarthropathy.c resistanceà isoniazid although there are isolates w/resistances to multiple drugs. pts with hematogenous spread will lead to secondary annular skin lesions at sites remote from the initial innoculation Stage 2 Early Disseminated Disease – the “cardiac and neurological stage” this stage demonstrated disseminated involvement of the nervous system. joints and heart Stages of the disease Stage 1 Erythema chronicum migrans usually occurs within seven to ten days of the tick bite • it begins as a patch or macule of erythema . mononeuritis multiples and chorea. occur annually. genitourinary system. which m.c affects the lungs (pulmonary TB) but can also affect the CNS (meningitis). UVA and UVB therapies. PUVA therapy . recurrent acute iritis in 1/3 of patient Diagnosis Increased ESR.very messy (strong odor and a tendency to stain clothing) . knees and elbows. osteolysis of the metatarsals and paravertebral ossification • destructive changes have a predilection for the DIPJ and PIPJ w/ relative sparing of the MPJ Symmetric polyarthritis-like rheumatoid affect the small joints of the hands and feet. Disadv . should not be used in acute or active eruptions • Disadv. S/E . Psoriatic Arthritis . followed by 100 mg/day for 1wk. Be careful for HPA axis suppression.infliximab . dermatitis and worsening of psoriasis (1 to 10% of patients) Coal Tar may be applied directly to the skin and is available in several strengths.of the matrix and lateral nail fold. osteolysis of the phalangeal and • tapering or "whittling" of the phalanges. very effective form of therapy. increase PMNs Calcipotriene synthetic version of vitamin D3 (NOT same as vitamin D dietary supplementation). injections may be repeated once a monthà one must use caution to prevent fat atrophy at the injection site à particularly important with plantar psoriasis methotrexate. cyclosporine .utilized in the cream form to treat chronic psoriatic lesions. adv: fewer S/E than topical steroids. kyphosis.burning. itching and skin irritation (10 to 15% of patients).2.introduction of a phototoxin enhances dose of 10 to 30 mg. ankles. good method for treating small lesions. +Schoeber’s test. Some physicians may elect to begin therapy with this modality rather than topical medications. rash. Topical Retinoids (Retin A) Sunlight Cortico-steroids UVB Phototherapy PUVA Therapy The "narrow band" part of the ultraviolet spectrum band that is most helpful for psoriasis. DIPJ arthritic (70%) changes are characteristic of the disease & occur with almost always concomitant nail plate involvement ankylosing spondylitis alone or with peripheral arthritis (5%) Treatment of Arthritis Associated with Psoriasisàrest.not always successful & may cause skin irritation and discoloration to both skin and clothing. 2-3x increase in fasting insulin. Anklosing Spondylitis Description Chronic inflammatory arthritis that affects the sacroiliac joint and to a lesser extent the rest of the spine. psoralen will make the skin more sensitive to the long-wave ultraviolet therapy. there is a high frequency arthritis. splinting.about 60% of patients have a good response w/i 4 mths (it is not (Dovonex®) recommended for use on the face) Efficacy .new anti--TNF antibody . hydroxyurea may be combined with methotrexate. Anthralin One preliminary study found that topical use of vitamin E was able to protect against this side effect. dry skin. these agents may be particularly effective on pedal lesions Use rule of 9s.calcipotriene is as effective as corticosteroids w/o the potential risk of atrophy of the subcutaneous fat. S/E must be seriously considered. may be combined w/ ultraviolet B (UVB) phototherapy.inhibits the proliferation of fibroblasts and keratinocytes. cyclosporine. one injection will usually produce clearing at the injection site.five clinical patterns exist: asymmetric distal interphalangeal joint involvement with nail damage (16% of cases) Arthritis mutilans with radiographic picture is similar to rheumatoid arthritis but RA is NOT at distal ends. risk of enhanced carcinogenesis is proven particularly SCC and melanoma suspensions of triamincinolone are frequently used (triamcinolone acetonide (Kenalog) suspension 10 mg/ml may be diluted with distilled sterile water to a concentration of 5 mg/ml.suppresses the immune system resulting in a slower turnover of skin cells . fluorinated corticosteroids.effective in Intralesional therapy Systemic Therapy 27 . the "Goeckerman" treatment combines application of coal tar ointment and UVB phototherapy. passive motion. Won’t work as quickly as glucocorticoids but have fewer S/E. "cupping" of proximal ends of the phalanges that results in telescoping metacarpals (5%) of the involved digit • bony anklysosis. good results are obtained with psoriatic nails by injecting traimcinolone into the region. Foot lesions . X-ray: bamboo spine Treatment PT and NSAIDS 3. it is acceptable to initiate treatment with high dose. MOA . lower back pain. Dapsone 100mg bid for 3 wks may be effective. intramuscular corticosteroid injection may be useful and utilization of NSAIDS Management of Psoriasis short doses that do not create a sunburn may be effective in clearing plaques topical steroids are usually applied directly on the skin bid if less than 10% of the skin is involved. wrists. maintenance treatment must be continued indefinitely. with claw hands (15%) of DIPJ involvement and a tendency for ankylosis of the DIPJ & PIPJ digital involvement will lead to a "claw" or "paddle" deformity of he hands Oligoarthritis with swelling and this is the most common initial manifestation and is present in two-thirds of the patients: affects one large tenosynovitis of one or a few hand jts joint (such as the hip or knee) and 1 or 2 IPJs with concomitant edema . +HLA-B27. may be irritating to the skin and should not be used in skin folds. Of 8-methoxypsoralen (Oxsoralen-Ultra) followed by the use of high-intensity long wave UV 2 hours earlier. treatments are usually given three times a week for two to three months and in conjunction w/ other treatments such as coal tar or anthralin and salicylic acid. M>F 10:1. and erythema. peeling. Signs and symptoms Onset -15-35yrs.of the hand or foot. Xerostemia (dry mouth) and dry vagina. psychosis. relapsing inflammatory and often febrile multisystem disorder of connective tissue. often associated with RA Keratoconjunctivitis sicca (dry eyes). interstial lung disease. Primary OA occurs without any type of injury or identifiable cause. leukopenia. Scleroderma Description Onset/incidence Signs and symptoms Diagnosis Treatment A systemic disorder of the c/t characterized by induration. Commonly involes the knee. Esophogeal motility abnormalities. avoid sunlight Diagnosis Treatment 2. muscle biopsy. It can also cause bone spurs. naeck and pharynx usually associated with pain and tenderness. WBC and platelets Systemic steroids. COX 2 inhibitors. Before the age of 55 it occurs equally in both sexes. Gradual/ subtle onset of deep aching jt pain worse after exercise or WB. glucoamine chondroitin. Skin thickening/induration. elevated creatine (>200mg) in a 24hy urine specimen 28 . arthralgias of small/large jts. The most common causes of secondary OA are metabolic conditions. interstial nephritis and interstial lung disease Mild anemia.82% responded. autoimmune thyroiditis. 18 4. antimalarials (chloroquine). elbows and wrists. Flares tend to parallel flares of the underlying bowel disease. corneal ulcerations salivary or lacrimal gland enlargement. 15-35yrs. preicarditis/pleuritis. glomerulonephritis. seizures and cognitive impairment. alopecia. Epidemiology Clinical Presentation Treatment Collagen vascular diseases that lead to arthritis 1. W>M (4:1) CREST syndrome: Calcinosis. anti-dsDNA. Bouchard nodes-at the PIPJ and Heberden nodes at the DIPJ. or inflammatory disorders like septic arthritis. +ANA Symptomatic 3. +ANA. Phenytoin and Procanamide Increased ESR. decreased Hgb. thrombocytopenia and leucopenia +ANA. arthritis of predominantly small joints. 5 mg/kg 3xd . INH. non-specific Ab against ANA. placebo control. often relieved by rest. The chronic disease causes the cartilage to wear away. Dermatomyositis/polymyositis Description Onset/incidence Signs and symptoms Diagnosis Chronic progressive inflammatory disease of skeletal muscle characterized by symmetrical weakness of the limb girdles. leading to pain and stiffness. Raynaud’s Phenomenon. W>M (10:1) Joint pain in 90% is an early manisfestation. nondestructive and transient arthritis. causes Libman Sacks Endocarditis (valvular vegetations found on both sides of the mitral valveà MS) Drugs that can causes SLE: HIPPà Hydralazine. anemia. Telengectasiasß HTN. Scelerodactyly. after 55 it is more common in women. dysphagia. Enteropathic arthritis (UC and Crohn’s disease) Description Rheumatologic manifestations seen in about 15-20% of pts with IBD Signs and symptoms Asymmetric. arthralgia seizures. Leads to migratory polyarthritis 30-40’s. 10 mg/kg 3xd. myopathies. Autoimmunityà Type III. immunosuppressants. ages 40-60’s. problems with anatomy. dysarthria and pulmonary fibrosis and proximal nail fold telegectasias Elevated CPK and aldolase. ankles. Description Onset/incidence Signs and symptoms Systemic Lupus Erythematosus A chronic remitting. However. Photosensitivity.clearing psoriatic plaques . 91% responded. steroids. injury. flexion contractures. Malar rash. +RA Symptomatic 4. Symptoms of osteoarthritis usually appear in middle age and are present in almost everyone by the age of 70. Polyarthritis occurs in 1/3 of pts. X-ray: assymetrical joint space narrowing NSAIDS. thickening and tightness of skin. Secondary OA is osteroarthritis due to another disease or underlying condition. such as acromegaly.Heliotrope of purple discoloration on upper eyelids is characteristic) and Gottron’s sign (flat toped violaceus papules over the dorsal aspect of the knuckles) Symmetrical m weakness and arthritis and arthralgias. anti-Smith and anti-histone antobodies with the formation of LE bodies. W>M 2:1 If associated with skin lesions (Dermatomyositi. abnormal EMG. GI abnormalities Treatment PT and NSAIDS Osteoarthritis Description Osteoarthritis is the most common joint disorder. Sjogren’s syndrome or Sytemic Sclerosis Description Onset/incidence Signs and symptoms Diagnosis Treatment Chronic Immune mediated disorder of exocrine glands resulting in decreased secretions W>M (9:1). OA is classified as primary or secondary. to grow around the joints. swelling of hands and feet. Etharcept Immunomodulator. tender. Ig 1st line DMARD syn and IL-2 prod Cyclosporine Immunosuppressive response to Il-2 Prophylaxis of organ Increases susceptibility to infection. Symmetrical sweeling of the wrists. morning stiffness (>1 hr). Chloroquine retinopathyà irreversible damage. risk of 20mg qd two in decreasing s/s. interferes with APC action.5% release enteric-coated and of IgM RA factor have agranulocytosis tabs) Methotrexate Inhibits the dihydrofolate reductase #1 for refractory RA. uses with Levels will increase: erythromycins. and stiff. when the joint is not used for a while. ameliorate s/s of choriocarcinoma. as effective as the previous Warning: hepatotoxic. use with Pruritic rash. dilantin Penicillamine Lowers IgM level. pregnancy/lactation. anklosing pediatrics and porphyria in divided doses (delayed the prod of inflam cytokines (IL-1 and TNF) spondylitis Adverse reactions: nausea. decreases synovial inflam. 500mg bid to 2. neoangenesis and joint destruction. the body attacks itself. Oral 2. Suppresses T-cell fx. depresses T-cells. kidney. immnosupressive. heart. Lymphadenopathy and weight loss Diagnostic testing Rh Factor RF is an antibody that attaches to a substance in the body called immunoglobulin G (IgG). rejection. renal disease. agranulocytosis and 15mg once weekly joint inflam but no (-) of bone erosion. a pyrimidine RA only!!! Pregancy and lactation 100mg tab qd x 3d then (Arava) synthesis inhibitor. decrease: rifampin. + in 70-80% Description Etiology DMARDS Drug Hydroxychloroquine sulfate (Plaquenil) Contraindication Dose Allergy to any 4-aminoquinoline 200mg po bid for compound. 50mg tabs and 100mg IV (Imuran) as cytotoxic agent. MCPJs. 2nd line tx for RA Immunosuppressive. not a causes pancytopenia. pancytopernia single daily dose MOA/ Description Anti-malarial drug. very toxic. UC. The body's immune system normally fights off foreign substances.5%. lysozomal enzyme activity. loss of appetite. the immune system confuses healthy tissue for foreign substances. 10mg test 1st week then weekly factor] and Igs. It is considered autoimmune disease. alopecia. and malaria 29 . increased liver enzymes TNF (-) Cytokine. and proteinuria are Gold Na thiomalate NSAIDS and cortisone the most common 50% gold Leflunomide Oral isoxazole. and weakness. epidermoid pancytopenia (bone marrow suppression). As a result. Women> Men Clinical Presentation Onset gradually w/fatigue. But in an autoimmune disease.Treatment Steroids Rheumatoid Arthritis Chronic (long-term) disease that causes inflammation of the joints and surrounding tissues. it can become warm. marrow hypoplasia. CHF. (+) chemotaxis. 3-6weeks carcinomas. The cause of rheumatoid arthritis (RA) is unknown. may take up to 6 mo for a clinical response Indication Mod to severe RA DOC for discoid SLE. stomatitis. Adverse reaction: very toxic. blood dyscrasias. neoplastic diseases: liver dysfunction. widespread muscle aches. Not a Chelating agent (Cu Very toxic. Given IV Asathioprine Metabolized to 6-mercaptopurine analog used Prevent refection of Pregnancy. Leuckocytompenia. bone injection usually 10Rheumatrex) decreases IL-2 prod. diarrhea (50%) run CBC. prolonged period. agranulocytosis. of the hands but spares the DIPJs Swan neck deformity. Etohism. chorioadenoma. immunosuppressive.5mg tabs or (MTXenzyme. BLACK BOX: fatal. decreases leukocyte chemotaxis. Eventually. jt pain appears. like viruses. need eye exams q 6-12 mo and baseline visual fields Sulfasalazine Prodrugà ASA (stays). 1. present in high [ ] in RA in the synovium. It usually occurs in people between 25 and 55. diarrhea. hypersent. renal/liver and lung disease (hypersentivity psoriasis pneumonitis) Gold Gold suppresses/prevents arthritis and RA and Uncontrolled DM. often Adverse reactions: mainly GI 8. Epidemiology RA can occur at any age. causes inflam and immune responseà IL-1/ IL-6. very often causes 125mg or 250mg in a (Cupramine) first line DMARD. anemia or caps bid Crysotherapy by macs resulting in (-) of phagocytosis and hemorrhage disorders. and often combined with liver. allergy Guanine synthesis. juvenile RA Allergy to sulfa drugs. nausea. N ab combined with slow dose pain and diarrhea cortisone and NSAIDS Warnings: psoriasis. inhibits the binding of TNF to cell surface RA 2. Decreases psoriasis. Children. MTX CCS. HTN. nephrotoxic. hydatiform anemia. 1. salycilates. radiographic progression lymphoproliferative disorders is increased ANTI-VIRAL Adverse reactions: Diarrhea. is taken up psoriasis hepatic disease. (Sandimmune) Not a first line DMARD. forming a molecule known as an immune complex. Infliximab (Remicade) Human recombinant IgG Ab which binds to TNFa and inhibits its binding. binds histocombatibility Ags and alter their recognition by T-cells. and sulfapyrazine RA. check for Response is quicker than Plaquenil à moles. hemolysis in G6PD.(hyperextension of the PIPJ w/flexion of the DIPJ) Boutoneirre deformity (flexion of the PIPJ w/hyperextension of the DIPJ) Felty’s Syndrome RA. IM: Aurothioglucose. nephrotoxic. pregnancy. risk of lymphomas. 50% gold. extensively metabolized by P450 use oral solution. rash. reevaluate in 3-6mo slowly within 6-8wks to 3-8mo. run LFTs. UA injections 25-50mg up to 1g retards cartilage and bone destruction. recombinant DNA protein (TNF Methotrexate refractory Infection and sepsis 25mg SQ 2x week (Embrel) recptor+ IgG). porphyria. fluconazole.0g daily (Azulfidine) (absorbed) in the colon by bacteria. decreases the [RA Warnings: Pancytopenia. renal or Oral: Aurofin 29% gold 3mg compounds synovitis (not a cure or remission). occurs prior to and q 2wks. when used for RA removal) in Wilson’s rash. Splenomegaly . RA barbiturates. PIPJs. stablize lysozomal membranes. interferes with Adenine and renal transplant. lung cancer. when used for RA. This immune complex can activate various inflammatory processes in the body. Infection can play a role in pathogenesis of the dz. Most common affected jointà Lis Franc joint. subluxation. Still’s disease (20%)-assoc w/ systemic manifestations (splenomegaly and generalized adenopathy Negative RA.Disease Juvenile Rheumatoid arthritis Description Onset/incidence Signs and symptoms Classification Diagnosis Treatment Not a life long disorder. PE of the foot may reveal a “rocker bottom” subluxation of the midtarsal region w/subluxations of the MPJs. and neuropathic ulcers may overlie or appear directly adjacent to the affected joint Prevention of further trauma by the cessation of WB: cast immobilization may be indicated but should not be applied until the edema assoc w/ the development phase has subsided (this is usually one to two weeks after the onset of the disease process)àuse of total contact cast w/ or w/o WB. these occur secondary to reabsorption of the metatarsal heads and shafts àpencilling or “hourglass” resorption of the phalangeal diaphyses will also occur Neuropathic Absent or diminished: DTR’s Pain and vibration proprioception Skeletal Rocker bottom foot. bone tumor. digital Cutaneous neurotrophic ulcer hyperkeratosis Vascular Bounding pulse Erythema Swelling Warmth 30 . DVT cellulites Clinical s/s Red. quite warm to the touch and demonstrates anhidrosis. and loose body formationà these changes occur primarily in the solid bones of the tarsus and can mimic osteomyelitis. subchondral sclerosis and the formation of marginal osteophytes Stage of Reconstruction this is an attempt to restore the joint architectureàrevascularization and remodeling of bone and bony fragments occur Stage of Reconstruction greater definition of bony contours adequate reconstruction of affected bones or ankylosis of involved bonesà a poor prognosis is indicated by the continuation of joint destruction Eikenholtz Stages X-ray dDx: OM. it is believed that the sudomotor and vasomotor dysfunction may contribute to the characteristic diabetic osteolysis seen in the metatarsals and phalanges (often referred to as a pencil-cup deformity or mortar-pestle type deformity)à the bones are literally “washed away” and present with characteristic “pencilling” of the metatarsals tubular bones of the metatarsals and toes will develop atrophic bone changes. osteochondral fragmentation & debris formationàa synovial biopsy at this stage reveals osseous debris embedded in a thickened synovium (pathognomonic for initial Charcot jt changes) Stage of Development Bony changes consistent w/exaggerated form of osteoarthritis.Charcot Restraint Orthotic Walker use of a bone stimulator may aid in the reparative process. jt disruption. including ASA and other anti-inflammatory modalities Neurotrophic Joint Disease or Charcot Joint Changes French theoryà Neurovascular theory: concomitant loss of SNS results in bounding pulses & increased osseous circulation resulting in demineralization of involved bones German theoryà Neurotraumatic theory à secondary to the loss of jt “proprioception” allowing jts to undergo extremes in ROM and pressures resulting in pathologic fractures Common etiologies: 1) tabes dorsalis 2) DM 3) Syringomyelia Hypertrophic Charcot Joint Disease Types Description VIP to recognize early The most common variant. & are in a state of poor gl-control. cartilaginous fibrillation. however +ANA Same as RA. usually fades with age <16yrs. although about 10% of patients may complain of some discomfortà pulses will be bounding. Polyarticular (40%) 2. collapsed arches (“rocker” bottom foot) and shortened digits Stage of Development initial destructive phase characterized by jt laxity. elbows. swollen foot Treatment Acute charcot NWB! Description X-ray changes Clinical Features of the Acute There is a higher incidence in pts who have had DM on an average of 12 to 18 yrsà important to correlate the probability of Charcot joint disease w/the duration of the diabetesà realize that in order for Charcot to occur. this occurs. hot. generalized demineralization. CROW boot . Patients will present w/markedly swollen footà it is erythematous. It is usually painless. It is important to identify and rule out any potential nidus of bone infection Stage of Coalescence characterized by marked absorption of the fine debris and subsequent fusion of the larger joints Stage of Coalescence presence of sclerotic bone (AVN) periosteal new bone formation. adequate circulation is necessary the majority of the patients have unilateral foot involvement. theoretically. long term tx should include appropriate use of accommodative footwearà molded shoes will support the misshapen foot as well as prevent ulceration but should not be fabricated until the end stage of the process has occurred Diabetic Osteolysis or Atrophic joint disease a less common variety is that of bone resorption. ankles or neck-often associated with fever and skin rash 1. fractures assoc w/ this process are healed by an exuberant repair process (reconstruction and coalescence)àsequelae include bone deposits. characterized by deformity and proliferative or dense osseous rx w/ subsequent coalescence or “healing” concomitant loss of SNS results in bounding pulses & increased osseous circulation resulting in demineralization of involved bones. and palpation of the jt will reveal some degree of hypermobility and crepitus. crepitus midtarsal subluxation. Pauciarticular (40%-accociated with iridocyolitis 3. W>M (4:1) Usually involves the knees. are in their 5/6th decades. in response to a hyperemia of the maximally dilated b/v to the foot. (PPP) weight loss. B-blockers. Ankle joint. pregnant women. FBS>126mg/dl and OGTT>200mg/dl Ø HbA1c (test semi-annually) < 7. hypermobility infection Frykberg-Anatomic Classification of Charcot I. peripheral and autonomic neuropathy. estrogen and catecholamines will inhibit insulin action Lispro: rDNA origin. Peak: 4-10 hrs.c Ø Generalized Granuloma Annulare: a ring of small. . insulin secretion or both Epidemiology >16 million are afflicted w/ DMà6th leading cause of death in US. marker for heterogenous group of endocrine disorders . retinopathy. Hispanics. serum lipid levels. abrupt onset. Ø Family hx of DM. or Native Americans) FBS-no food or beverage 8 hrs before testing >115mg/dl (non-pregnant adult) >130mg/dl (child) ► Confirmatory testing: RPG >200mg/dl with classic s/s. use of dipyridamole (Persantine®) 225 mg and aspirin one gram per day and pentoxyfylline (Trental®) 400 mg. blurred vision and fatigue ß prone to ketoacidotic coma Treatment-requires insulin Characteristics-insulin resistance/action and secretion Pathogenesis-genetic defect in insulin action resulting from abnormalities in glycogen synthesis or in glucose transport Clinical Presentation-aging. Diabetes Mellitus Etiology Description Screening tests It is a disease of insulin action (insulin resistance). Insulin can be utilized Gestational diabetes treated with insulin b/c it does not cross placenta and Impaired glucose tolerance Micro/macroangiopathy.has also been shown to be effective Ø Diabetic Dermopathy (Shin spots) m. obesity and genetics can lead to insulin resistance and s/s similar to Type 1 DM ß not prone to ketoacidotic coma because endogenous insulin is usually sufficient to prevent it Treatment-reverse underlying insulin resistance and impaired cell function ► Normalizing body wt. increased atherosclerosis. Tarsal/ Chopart’s joint IV. onset at any age (75% before age 18). HTN or hyperlipidemia. waxy. t. lente. human insulin analog: Soluble & rapidly absorbed ► Onset: 15 min. achrocordonsß will alleviate with tx of underlying malady Ø Candida infections Treatment Thiazides.0% Blood pressure : <130/80 Ø LDL cholesterol (q 2yrs) < 100 Foot exam (annually) Types Characteristics: 10% of all DM pts. histologically demonstrate red blood cell extravasion and capillary basement membrane thickening Ø Diabetic Bullosis or Bullous Diabeticorum: a spontaneous non-inflammatory blistering which occurs spontaneously in a fairly symmetrical distribution Ø Acanthosis Nigricans: warty pigmentation of the skin. polyuria. amputations and blindness.i. occurs most commonly in the axillae. polyphagia.d. obesity >120% of desired body weight. loop diuretics. possible association with malignant neoplasm.Charcot anhidrosis or hyperhidrosis subluxation. Calcaneus E. duration: 3-6hrs NPH: Neutral Protamine Hagedorn: Onset: <2-4hr. members of a high-risk group (African Americans. flesh-colored or red papules ß tx with intralesional triamcinolone acetonide Ø Periungual Erythema: dilation of the capillary loops secondary to microvascular disease Ø Diabetic Dermopathy (Shin Spots): hyperpigmentation and retracted atrophic scars present on the shins. Peak: 2-3hrs. growth hormone. abnormalities in lipoprotein metabolism and periodontal dz Ø Nicrobiosis Lipoidica Diabeticorum: well circumscribed. Metabolic and endocrine disorders 1. and phenytoin will inhibit secretion and corticosteroids. Peak: 30-70 min. atrophic patches ß tx with intralesional triamcinolone acetonide. Forefoot/MTP II. leading cause of ESRD. firm. HTN.insulin resistance is the marker. nephropathy. duration 2-3hrs Regular: crystalline zinc (pump use) Soluble & rapidly absorbed ► Onset: <1hr. impaired immunity. hard. prevent micro and macrovascular complications. and HLA associations Pathogenesis-progressive autoimmune destruction of beta islet cells in the pancreas Clinical Presentation-diagnosis is made after 90% of the cells have been destroyed ► Polydyspsia. Midfoot/LisFranc’s joint III. STJ V. depressed. ultralente Treatmetn goals Type 1 Type 2 Other types Complications Cutaneous manifestations of DM Insulin 31 . duration: 10-16 hrs Lente: semilente. yellow-brown. Also the increase in the concentration of hypoxanthine precursors(over-producers) à increased urate o Lack of uricase enzyme (urateà allantoin) high purine foodsà urate that is normally degraded in the gut by bacteria and excreted renally. Most common cause of inflammatory arthritis in men over 30 in women estrogen is protective to promote urate excretion. most commonly the 1st MPJ (Podagra)à hallux limitus. uric acid analysis in the synovial fluid for (-) birefringent crystals. PRPP excess.e. G6Pdef Epidemiology: Disease of adult me w/a peak incidence of the 5th decade. Thiazolidinediones-decreases insulin resistance at peripheral receptor site. shifts the pathway to acute gout GI toxicity =4 take with food Acute gout: 50 mg tid twice the the lipoxygenase side anti-inflammatory dose Uricosurics MOA: block Potentiate the urinary excretion of urateß acidifies the Chronic DON”T USE WITH ASA the renal tubular absorption urine and can cause urolithiasis and renal colic which gout with recurrent May accentuate the action of the of urate can be buffered by Na bicarb po. decreases plasma acute attacks anticoagulants i. therefore no chance of hypoglycemia: Contraindicationsà monitor LFTS and CHF risk Ø Rosiglitazone (Avandia) 4-8mg qd Ø Pioglitazone (Actos) 15-45 mg qd Gout A familial metabolic disease characterized by hyperurecemia. Reiter’s syndrome Treatment: Drug MOA/ Description Indication Contraindication Dose Colchicine Inhibits poly chemotaxis Rapidly Pregnancy. may enhance glucose utilization in muscle and adipose. can lead to lactic acidosis and gastric upset. blood dycrasias and agranulocytosis Dose: 100-200 mg bid 1st week and then 200-400mg bid maintenance 2. Causes recurrent episodes of acute monoarticular arthritis Etiology: Ø Primary: Failure to excrete the urate is the number one cause (under-secretors).production and intestinal absorption. Will lower HgbA1c by 1-2% o 1st generation (chlorpropamide) Highly bound to serum albumin therefore may compete for binding sites with other drugs and may potentiate ADH which will lead to water retention. does not lead to hypoglycemia o Discontinue use with contrast. OA. leukemia/polycythemia. N/V. XR findings include rat bite erosions and Martel’s signà overhanging margins. 24 urine test to check for overproduction >700mg/24h or undersecretion <700mg/24h. Ø Secondary: Renal failure. Goal of tx: Reduce serum urate level below the level of the saturation point to suppress precipitation of urate. charcot. psoriasis. and size of old tophi (undersecretors) 1. mono. peak and duration similar to NPH. RA. diuretics (particularly the thiazides). Probenicid Benzoic acid deriv. and resultant deposition of monosodium urate monohydrate in the tissues. start with ½ tab (250mg) bid the 32 . ankle edema and hyponatremiaà cardiovascular events o 2nd generation restores sensitivity of Beta cell to glucose. o Normal UA = 2. metabolic acidosis. new tophi formation. UC or bowel obstruction and pts with liver dz Ø Ascarbose (Precose) 50-100mg tid ac Ø Miglitol (Glyset) 50-100mg tid ac 4. in the presence of renal dz will cause binguanide induced lactic acidosis (50%) mortality rate 3. diarrhea in toxic an additional 1 mg in 6hrs if needed. with no increase in insulin secretion o Contraindicated in patients with IBD. renal tubular blocking agent. red hot. less likely to interact with other drugs b/c of non-ionic nature Ø Glypizide-Glucotrol 5mg qd Ø Glimeperide-Amaryl 1-4 mg qd Ø Glyburide-Diabeta 5-20mg qd o Complications: hypoglycemia 2. joint and the kidneys. only active in the presence of insulin “insulin sensitizers”à will not increase insulin secretion.► Altered to slow absorption. Alpha-glucosidase inhibitors-delays digestion of CHOs and delays glucose absorption via competitive reversible inhibition of pancreatic amylase and intestinal alpha glucoside hydrolase enzymesà intestinal gas. Clinical Presentation: acute. Pb poisoning. allergy. Meglitinides:+[insulin] blocks K+ channelà Repaglinide (Prandin) 5. Sulfinpyrazone Active peptic ulcer disease.. Biguanides-decreases hepatic Gl. coumadin urate. DVT. Sulfonylureas: stimulates production of endogenous insulin by blocking K+ channels. and works synergistically with the sulfonylureas Ø Metformin (Glucophage) 500mg bid o contraindicated in pts with renal dz. hypothyroid.6mg po qh x 5 doses BEST DRUG (-) histamine release from great for motility. Max 10 tabs Fatal as low as 7.0mg and phagocytosis of urate absorbed increases intestinal Dose: 0. renal disease. Lesch-Nyhan Syndrome.5-7 female and 3-8mg% male ([UA] should not be the end all and be all of the diagnosis) dDx: cellulitis. swollen joint. inhibits the 500mg tabs. no more synthesis ß pain doses than 4mg in 24hrs Indomethacin Most potent inhibitors of Anti-inflam and Not for chronic administration PO drug only COX. abdominal Inpatient: 2mg IV in 20ml NS over 10 min then FOR GOUT mast cells. available in long-acting form(Ultralente) Premixed insulin ► 70/30 70% NPH w/ 30% Regular & 75/25 75% NPL with 25% Lispro premixed (humalog insulin pen) Oral hypoglycemic agents 1. Inhibits LTB4 acute gout pain. Dx: [uric]p. leaving us with high [hypoxanthine and xanthine] ßok because excreted 10x greater than urate w/o urolithiasis problems and low [urate] Indications (over-producers) Best for patients with renal gouty nephropathy. It increases heart rate and elevates the blood sugar level by increasing glycogenolysis in the liver. Clinical features include: central obesity. ColBenemid tubular secretion of PCN Combination of colchicines 0. aldosterone and the sex hormoes. Disorder of mineral metabolism resulting from def secretion of PTHà hypocalcemia. 10% are bilateral. moon facies. and on X-ray will see calcifications of the articular cartilage or meniscus Immobilization. and lower levels in the evening. hirsutism.(Benemid) 3. and osteoporosis. decrease in renal function Contraindications Shellfish and alcoholics Dose 100mg and 300mg tablets Dose: titrate 100-300 mg/day in single dose MOA Description Symptoms Onset Diagnosis Treatment Pseudogout (Calcium pyrophosphate dehydrate. Other s/s include persistent HTN (due to the aldosterone-like effects) and insulin resistance. catecholamines . Untreated Cushing's syndromeà heart dz and increased mortality. wrist and shoulder. Adrenal Disorders of hyperfunction Cushing’s Causes increased cortosol. Conn’s Disease Primary hyperaldosteronism Addison’s Autoimmune primary chronic adrenocortical deficiency leading to decreased production and secretion of the adrenal hormones Disease (hypocortisol) secondary to deficiency in ACTHà hyperkalemia and nyponatremia Pheochromocytoma Tumor of the adrenal medulla causing excessive production of catecholamines which induces hypertension. 1) The adrenal cortex which is responsible for the production of the glucocorticoids (cortisol). mineralcorticoids (aldosterone) and the androgens (estrogen and testosterone) 2) The adrenal medulla which is responsible for the production of the catecholamines Function of hormones Aldosterone Regulated by RAAS to increases Na resorbtion and K excretion in the Kidney to balance BP. In a calcium depleted state. end product of purine metabolism. hyperkalemia and many develop frank DM.5mg and probenecid 0. HyperTH. it may present with s/s of tetany and/or a seizure disorder however. leading to hyperglycemia. This is a disorder of mineral metabolism in which the end-organ is unresponsive to the action of PTH. Acts on the kidney and the bone to maintain Calicium balance A disorder of mineral and/or bone metabolism caused by increased and incompletely regulated secretion of PTH. A rise in plasma calcium decreases PTH release. This Description Clinical Presentation Type: Small cell carcinoma of the lung Description Hyperparathyoidism Hypoparathyoidism Pseudo- 33 . Assoc w/ high grade fever Risk increases with age. PTH will cause a decrease in osseous mineralization and an increase in bone remodeling. it increases BP and [glucose] and suppresses the immune system. several hours after the onset of sleep. Clinically.5g 1st week. CPPD) Associated with chronic or acute inflammatory arthritis. pts hospitalized for other medical condition and those with metabolic dz (HypoTH. abdominal Syndrome striae. They are composed of 2 parts. Cortisol (anti-insulin) This hormone is involved in the response to stress. then 500mg bid for maintenance Chronic gout only One tablet bid Chronic gout maintenance: Allopurinol (Zyloprim) Xanthine oxidase inhibitor (hypoxanthineà xanthineà urate. it leads to hypercalcemia and osteitis fibrosa cystica. weakness. it has no consequence for bone density. The knees are often involved (50%) followed by the ankles. NSAIDS and analgesis Adrenal Disease Adrenal glands are triangular-shaped glands located on top of the kidneys. can be caused by deposition of Calcium pyrophosphate dehydrate crystals in the joint Similar to gout but tends to run longer course (reaches maximum severity at 1-3 days and resolves in 1 week or longer. trauma.which stimulate PTH to release vitD which decreases PTH release. urate stones. with the highest levels present in the early morning. Follow’s rule of 10: 10% are located outside of the adrenal gland. gout and amyloidosis) Microscopic examination of the joint aspiration reveals rhomboid crystals that are positively birefringent. Description Paraneoplastic Syndrome Benign neuroendocrine tumor that can secrete hormones/proteins that directly affect the fx of specific organ systems Indirect effect of proteins and hormones over secreted wreaking havoc on the body’s homeostasis This tumor is neuroendocrine in origin (carcinoid tumor) Tumors that secrete PTHà High [Ca2+] Tumors that secrete ACTHà Cushing’s Syndrome Parathyroid disease PTH has 2 major fx: to maintain plasma [calcium] and to regulate the rate of bone remodeling. and 10% are malignant. The amount of cortisol present in the serum undergoes diurnal variation. and lipolysis in fat cells. Epinephrine Plays a central role in the short-term stress reaction—the fight-or-flight response. kidney stones and bone fractures. a drop is plasma calcium stimulates PTH releaseà regulatory system is dependent upon two additional factors. 5g/dà hypoalbuminemia. glomerulomephritis. a crush injury. Nephritic syndrome Proteinuria in excess of 3. Increased appetite. Chronic renal disease Can develop fom ARF but more often a complication of a chronic systemic disease. The cysts may cause the kidneys to increase production of erythropoietin resulting in too many red blood cells. joint elevated TSH). Polycystic kidney Inherited disorder (autosomal dominant). uremic frost. alcohol abuse . Most commonly occurs in the setting of AGN but can be glomerulonephritis drug-induced. acute tubular necrosis (ischemic insult. interstitial nephritis or tubular obstruction Myoglobinuria -. rather than the expected anemia of chronic kidney disease. Good pasture’s Autoimmune disorder against collagen is present in the alveoli (tiny air sacs in the lungs) and in the glomeruli (the filtering syndrome units of the kidney). Chronic glomerulonephritis Slowly progressive glomerular dz that leads to ESRD over a pd of mo-yrs. hyperPTH. polycytic kidney disease Uremia.c.hypoparathyroidism “unresponsiveness” generally stems from the kidney and not the boneà alteration in mineral metabolism will result in skeletal abnormalities and pts will present with hypocalcemia and if long standing will be of short statue. DM. Restlessness . GI disturbances. VitD def. Occassionally long term dialysis in needed. interstial nephritis. diet modification. or seizures or Infections such as acute pyelonephritis or septicemia. in pts with comorbid conditions. and is m. sepsis and shock Postrenal-ureteral obstruction. In early stages of the disease. disease resulting in chronic high blood pressure and kidney infections. fluid and NA restriction Complications Diagnosis Treatment Description Etiology Etiology s/s Treatment Glomerular disease Description Heterogeneous dzàinjury to the glomerulus. ARF s/s do not appear until renal fx has fallen to 10% of normal. hyperpigmentation. TH acts throughout the body to regulate metabolism and increase iodine. renovascular disease. Fatigue. and altered mental status Determine underlying cause. HTN and edema Rapidly progressive Glomerulonephritis that advances to ESRD in days to weeks. mildly obese with shortening of one or more of the metacarpal or metatarsal bones. The key pathologic finding is th extensive formation of extracapillary crescents over ½ of the glomeruli. Hair loss. Goiter. radioactive iodine or sx Graves’s disease of TH s/s: Weight loss. urethral obstruction (secondary to enlarged prostate) Intravascular overload. Constipation . at 30-40% of the normal GFR. osteomalacia. to remove the thyroidà replacement TH opthalmapathy and dermopathy. VitD metabolism and BP regulation via RAAS Renal failure Description Etiology Acute renal failure Arises over 2hrs to weeks. anemia. endocrine/metabolic disorders (hypertriglyceridemia. hyperK+. or impotence) CV disorders. asterixis. Nervousness. dermatological disorders. Small or atrophic thyroid gland (late in the disease). Prerenal: hypovolemia. biochemical evidence can be seen. proteinuria. impairment of RAAS via angiotensinconverting enzyme inhibitors or NSAIDS Intrinsic renal-glomerulomephritis. the cysts enlarges the kidney and interfere with fx. Hashimoto (increased [mucoploysaccharides] in ct à to edematous thickening of the Replacement therapy with thyroid hormone Thyroiditis skin)à Most common cause of goitrous hypothyrodism. edema and hyperlipidemia. and acute renal failureà salt and water retention. PN. Menstrual irregularities life. These antibodies are called anti-glomerular basement membrane antibodies (or anti-GBM antibodies). renal vasoconstriction. Distal neuropathy w/ brisk DTRs Hypothyroidism Autoimmine disorder. Dry skin. Heat intolerance . fluid and electrolyte imbalance. results in cretinism in infancy and myxedma in adults A def of TH may develop at a later time. Fatigue . Enlarged neck or presence of deficient or may be given if there is goiter. S/s do not appear until renal function has fallen to 10-15% of normal function. Difficulty concentrating or thinking. Nephritis (inflam) or nephrosis (abnormal permeability of the glomerular membrane Acute glomerulonephritis Abrupt onset of hermaturia. uremic fetor Elevated BUN and Creatinine Correction of the fluid and electrolyte imbalance and the underlying cause. uremic syndrome (pericarditis.this condition may be caused by rhabdomyolysis . s/s: Intolerance to (levothyroxine) is given if the hormone is cold . hematologic/immunologic abnormalities and neurologic disorders (peripheral neuropathy) Fatigue. decreased CO. also known as subclinical stiffness. metabolic acidosis. Thyroid disease Description The thyroid gland is located in the base of the neck on both sides of the lower part of the larynx and upper part of the trachea. nephrotoxic drugs ie aminoglycoside/antibiotic). Other Wegener’s granulomatosis and SLE nepritis 34 . Distal neuropathy w/ delayed DTRs hypothyroidism Renal Function: Maintanence of water and electrolyte balance. peripheral edema. Increased must be taken for the rest of the person's sweating . hemnodialysis or peritoneal dialysis renal transplant. removal of metabolic wastes. Most important cause in US is DM. Frequent bowel movements . wt gain .esential trace element used in the production of thyroxine/ triiodothyronine Diagnostic screening à Thyroid function tests: Elevation of Free T4 test Serum TSH T3 Hyperthyroidism Autoimmune activation of TSH receptors which leads to a feedback inhibition Antithyroid meds. HTN. post-surgical.mild . bladder neck problems. evidence of mild thyroid failure (such as Heavy and irregular menses. protein. GFR falls to 10-20% of normal. HTN. The gland produces TH in response to stimulation by TSH from the pituitary gland. asterixis. lactoferrin. Thyroid acropachy Rareß sequelae of thyroid gland ablationàperiostitis of feet bones occurà a rectangular appearance to these bones Macrodactyly this is a congenital problem and is present from birth Vit C deficiency “Scurvy” Most of the bone is reactive and woven with vascular fragilityà hemorrhage VitD deficiency Loss of mineralization (calcified component only) with hyperplasia of the cartilage in the growth plate Osteitis fibrosis cystica Secondary hyperPTH. dressing. Combination OM (I-IV) 6.c. *infection violates the outer cortex. disease of the growth plate.only membranous ossification Pyogenic OM Staph aureus is responsible for 80-90% Tuberculous OM Pott’s disease-spread of TB from the lung to the vertebral medulary cavity Osteonecrosis Osteoporosis Osteomalacia Acromegaly Osteomyelitis Description Pathophysiology Areas of dead bone (sequestrum) surrounded by reactive bone (involucrum) denotes inflammation and infection metabolic by-products of the bacteria and associated anti-inflammatory rxs → decrease local pH → breakdown of trabeculae and removal of bone matrix & Ca+ salts [demineralization seen radiographically as radiolucency] *infection spreads via Haversian and Volkmann channels → vascular channels occluded → bone necrosis and osteocyte death → pus formation within channels → raises the intraosseous pressure & compromises blood flow → sequestra localized segments of necrotic bone become fragmented. pathogen & Staphylococcus epidermidis is associated with joint implants and bone hardware. alcohol abuse and corticosteroids Overall loss of bone itself-loss of the calcified and osteoid component ß assoc with Vit D def and inactivity Loss of mineralization (calcified component only) with pitting of the calcified portion of the bone Acquired condition secondary to excessive production of growth hormone after skeletal maturity. when excess growth hormones are produced before skeletal maturity.Bone Metabolic Bone diseases Avascular or aseptic necrosis à associated with Gaucher disease. the condition is referred to as gigantism and it is associated with a high incidence of impaired sugar tolerance. good for neoplasms/ inflam. if intracapsular → septic arthritis. drugs Conservative IV antibiotic tx is indicated and continued for 4-6weeks and can often be accomplished on an out-patient basisà osteomyelitis of the phalanges usually responds well to anti-microbial therapy alone Surgical imperative when necessary and is indicated in acute osteomyelitis when there has been no response after 48 to 72 hours of antimicrobial tx à radical debridement and removal of fixation devices is recommended Classification schemes of OM Waldvolgel Hemotogenous OM – 19%most common in pediatrics/ pts over 50.Loss of mineralization as well as destruction of the bone ß overactive osteoclastic activity caused by prolonged hypoCa produces the characteristic brown tumors (Von recklinghausen dx of bone) Paget’s disease Triphasic polyaustotic collage of matrix madness with concurrent/simultaneous activation of osteoblastic and Osteitis deformans osteoclastic activity may have a metastatic malignancy and can lead to osteosarcoma that occurs in the elderly Achondroplasia M. Pseudomonas aeruginosa is the most common pathogen in osteomyelitis secondary to a puncture wound Diagnostic screening Radiographic Sequestra. Ceretec HMPAO.c. tri-phasic (non-specific for OM) Blood flow-immediate Blood pool. Chronic OM Cierny and Anatomic type Physiologic type Mader 1. Wound induced OM 2. Mechagenistic OM 3. cloaca ß not seen for 10-14 days Bone Scans Technitium-99: uptake by osteoblast and hydroxyapatite crystals. Physeal OM 4.highly suggestive of chronic OM *bone infection may →s/t involvement as infection "breaks through" the periosteum or jt capsule →drainage & spread AKA cloaca Infectious pathogens Staphylococcus aureus is the m. involucrum. Ischemic limb disease 5. Good immune system and delivery 35 . Osteitis with septic arthritis 7. . image for infxn taken 6-24hrs. drainage.after 4 hours Gallium-67: binds WBCs. affects metaphyseal bone because in the pediatric pt: OM secondary to contiguous focusà47% (the most common type seen in podiatry) • Post-op contiguous OM (w/in 1 month of surgery) OM associated with vascular insufficiency à 34% Chronic or Recurrent Buckholtz: Based on mechanism of action (7 types) 1. best for infection dx. Meduallary OM 5. if subperiosteal suppurative material → abscess formation & proliferative periosteal reaction/lifting known as involucrum. image for tumors taken 24-72 hrs Indium-111: binds to WBCs cytoplasm components. plasma proteins. debridement. About 18% of all patients with acromegaly have insulin resistant diabetes mellitus. (autosomal dominant) w loss of endochondral ossification.binds WBCs-4th phase Definitive dx Definite-bone biopsy and bone culture Treatment: Decompression.after 5 minutes Delayed. Superficial OM Localized OM Diffuse OM 6. pyrexia PAINFUL Pathognomonic XR findings: sunburst periosteal reactionà delicate rays of periosteal bone formation and Codman's triangle periosteal reactionàtriangular elevation of periosteum dDX: OM tumor possesses lytic foci: MRI and CT useful to delineate bounds of tumor cortical bone destruction (may actually be disrupted)“motheaten" bone destruction may possess dense sclerosis in center of lesion.3% peak incidence .c in 3rd decadeà may undergo malignant transformation 36 .primarily 2nd0 yrs predilection for long tubular bones.c primary malignant bone tumor.c. pain and tenderness expansile lesion àblood filled sinusoidal cavities w/in cyst may appear quite aggressive and difficult to distinguish from a sarcomatous lesion of bone.c in blacks M>F Incidence/ Location Characteristic findings Benign Cartilaginous Tumors Enchondroma usually occur in 3-4th decade chondrosarcoma VIP may develop. Incidence and Location generally affects long bones m. soft tissue ( Increase in ALP and leuckocyosis) Pre-op chemo to shrink the tumor.predilection for epiphysis usually seen when growth plate is open. Rare tumor Chondromyxoid Fibroma Can affect any age group but m. Syndromes Associated with Enchondroma: multiple enchondromatosis à Ollier's dx or multiple enchondromatosis whemangiotosis à Marfucci's syndrome Chondroblastoma (Codman's tumor) Occurs: 10-20 years . BKA .if fat pad of heel and dorsal skin are uninvolved.c cystic lesion of foot m. Compromised locally or systemically 7. No treatment b/c treatment is worse than the disease Benign bone lesion Solitary Bone Cyst (Unicameral Bone Cyst) PAINLESS Aneurysmal Bone Cyst Description Bone tumors Characteristics and Radiographic Findings m. 4. IgA and rarely IgD) or light chains only) plasma cellsàdisplays diffuse. May arise from pre-existing Paget's lesionß 50-70 y/o bimodal peak.c occurs àcalcaneus usually asymptomatic unless pathologic fx occurrence possibly secondary to low grade inflammatory process 1st and 2nd decade (young adults) M:Fà2:1 occurs in distal ends of long bones "fallen fragment" signà no appreciative periosteal rx initial tx may include aspiration and introduction of acetated glucocorticoids (case study reports resolution of cyst with introduction of 240 mg of DepoMedrol) Painful lesion assoc w/focal swelling. whereas the primary tumor is m.c in whites than blacks/ Asians due to unregulated growth and proliferation of clonal (overproduction of either intact monoclonal Igs (IgG. but radical excision imperativeà5 yr survival rate w/ resection only and not amputation essentially zero Symes amputation . generally solitary Description PAINFUL classically associated w/ relief of pain by aspirin ß tumors produce PGs & night pain PAINFUL uncommon accounts for 1% of all primary bone tumors to trauma locally aggressive. m.c malignant bone tumor in footàsurpassed by: multiple myeloma osteosarcoma Chondrosarcoma 2nd m. neural arches of vertebral column peak incidence – 2nd decade Benign Bone tumors Osteoid Osteoma Osteoblastoma Description Characteristics and XR Findings Treatment Malignant bone forming tumor: Osteosarcoma Primary malignant tumor of the foot. in the younger population (3rd mc malignancy in pediatrics) may arise from soft tissueà s/s pain. primary malignant bone tumor in foot incidence in foot is 5. limitation of ROM. 1 of the most common if not the most common malignant bone tumors in foot mortality w/in 3 yr onset in middle and old ageà90% pts > 45yrs. swelling.2.c in the calcaneus and talus affects young adults . infections.if tarsal bones are involved. m.5 to 15 yr m. may possess an extraosseous mass. Rotationoplasty Marrow Tumors Affecting Bone Incidence and Location Tumors Ewings Sarcoma Description Characteristics arises from medullary cavities possessing a diaphyseal location motheaten or mottled XR appearance onion skin periosteal reaction generalized osteoporosis scattered “punched-out" osteolytic hypercalcemia may be present secondary to bone destructionà poorer prognosis Multiple Myeloma malignant primary bone tumor PAINFUL w/ palpable swelling constitutional s/sà dramatic: anemia of chronic disease fever and leukocytosis highly metastatic& expansile radical amputation 4th m. anemia and renal insufficiency predominates w/skeletal destruction. plasma cell infiltration in bone marrow (check for Jone’s protein in the urine*UA) constitutional signs such as bone pain.c. Incidence: 20-30 but may appear in younger and older people. 3. bloody and satellite lesions are incredibly common may involve the tendon sheaths appearing fluid dDx: hematoma radiation tx ànot recommended in the foot Insidious onset. First. M>F. frequently. Progressive.S/s: Weakness. and other organs. Treatment: Chemotherapy.5 cases (26. may involve soft tissue extensive size may result in pathologic fracture. the rapid proliferation of these cells. neutropenia. must recurrence rate-implantations of the AKA benign fibrous of the foot about aspirate-but will not tumor into surrounding soft tissue histiocytomas fibroxanthomas the ankle suck out anything may be difficult to treat may arise from any synovial acute synovitis aspiration of the joint synovectomy is the ideal TOC. Lymphadenopathy and Fever Treatment: Chemotherapy.c around jts but will produce a brownish. Malignant disease of the bone marrow in which hematopoietic precursors are arrested in an early stage of developmentà 2 disease processes. The person becomes susceptible to bleeding. Nada-Curretage with bone lobulated. kidney. These cells can be found in the blood. rarely arise in foot. Average age of onset is 55. Swollen lymph glands. Epiphysis is never affected. the lymph nodes. Consequently.c along dDX: Ganglionic cyst marginal excision.2%) in calcaneus Characteristic XR Findings thick radiolucent defect. which results in varying degrees of anemia. recurrence tissue occurs m. lytic lesions àcommon occurs w/i metaphyseal bone or at the metaphysealpacking if >50% of self healing lesion epiphyseal junction. Symptoms:. Hematologic disorders-including anemias and leukemias Leukemias-neoplasms of the bone marrow Hairy cell Leukemia m>w 2% of the leukemias Acute myelogenous leukemia m>w Chronic myelogenous leukemia 7% to 20% cases Acute lymphocytic anemia Uncommon caused by the abnormal growth of B cells. packing with bone chips-bone multinucleated type metatarsals and tarsal bones “soap grafting may be required à radiation tx must giant cells PAINFUL (especially calcaneus) bubble" monitor these as tumor growth is unpredictable arises from the synovial lining occur m. may correlate w/ trauma. bone marrow transplant and radiation tx. very low of tendon sheaths PAINLESS the dorsal surface and PVNS. present w/i the medullary cavity of bone.Osteochondroma Subungual Exostosis 2nd-3rd decade cartilaginous capàpotential to transform 50% of bone tumors. In fact. excision bone these are geographic. Treatment: Chemotherapy. the spleen. Bone and joint pain or tenderness . the production of RBCs markedly decreases. Weight loss . peripheral blood. usually seen in 3rd to 6th decade. Excessive sweating (especially at night). the spleen and liver. the dz progresses to a "blast crisis. Easy brusing or bleeding . DIC and infection. brain) Miscellaneous Tumors Tumors Description Incidence and Location Characteristics and XR Findings Treatment Giant Cell Tumors rare tumors typified by foot lesions seen in lytic lesionà surgical curettage. The disease can occur in adults (usually middle-aged) and children. and. results in their accumulation in the bone marrow. and neutropenia. the bone marrow. multiGenerally btwn 5-20 yrs often an incidental finding. thrombocytopenia. Recurrent infections and fevers. diameter is involved Giant Cell Tumor of Tendon Sheath Pigmented Villonodular Synovitis Synovial Sarcoma Non-Ossifying Fibroma F. Symptoms: W/i 5 yrs. and thrombocytopenia. MI may be the 1st presenting symptom of acute leukemia in an older patient. . Fatigue. Most commonly found in the tarsal bones 42.c. s/t sarcomas usually present as PAINFUL tri-phasic scan will often radical excision of lesions on the plantar aspect of the footàdDX: frequently demonstrate increased uptake involved area with mistaken for a fibroma when occurring on the bottom of the in all 3 phases secondary to wide margins foot and a ganglionic cyst when occurring on the dorsum if periosteal inflam of adjacent needle aspiration fails to produce fluid. the peripheral blood cell profile shows an increased number of granulocytes and their immature precursors. It causes rapid growth of myeloid precursors in the bone marrow. bone marrow transplant and radiation therapy. and body tissues. along with a reduction in their ability to undergo apoptosis.1% . malignant dz characterized by large numbers of immature WBCs that resemble lymphoblasts. in adolescent females fibrocartilaginous cap Malignant Cartilagenous tumor: Chondrosarcoma Description malignant tumor usually arising from within the boneàmay arise from a preexisting cartilaginous tumor metastasize (primarily to the lungs) six subgroups of chondrosarcoma exist: Incidence and Location predilection for long tubular bones.c on halluxà frequently located underneath nail plateà m. bone marrow transplant and radiation therapy. In ALL. in most people. Treatment chemotherapy. and Early satiety. blood. Foot pain over small joints of the foot . Symptoms of bone marrow failure are related to anemia. Second. m. the malignant cell loses its ability to mature and differentiate. Interferon and the removal of the spleen may improve blood counts may be required. possesses speckled calcification Treatment radical excision with adequate resection of uninvolved osseous margins and surrounding soft tissue amputationà monitor for metastasis (generally pulmonary but may involve liver. Fatigue. 80% of childhood leukemia ages 3 -7 and 20% of adult 37 . A myeloproliferative disorder characterized by increased proliferation of the granulocytic cell line without the loss of their capacity to differentiate. patients with hypothyroidism and liver disease usually have a normocytic anemia G. G6P def -disorder asymptomatic unless pts subjected to oxidant injury-X-linked transmission. eosinophils and killer cells platelets. malaise. ATOPY this is an antibody-dependent cytotoxic hypersensitivity. IgE urticaria. sulfonamides.spleen 3x its normal sizeà this occurs m. diarrhea)à the neurologic disease points to a B12 def and not a folate deficiency alone (degeneration of the dorsal and lateral columns .generally obvious 2. phenacetin.concomitant findings generally include an elevated indirect (unconjugated) bilirubin levels • -congenital hemolytic anemias include disorders of the cell membrane. and pyrimethamine • Clinical Features: glossitis and a variety of GI s/s (indigestion. fifth disease in children is the m. primarily d/t defective RBC prodand secondarily d/t abnormal incorporation of Fe into the RBC’s (> 50% of normal) Chronic Disease Hemolytic anemia is a condition of an inadequate number of circulating RBCs caused by premature destruction of red blood Hemolytic cells.the marrow is primarily hypocellular Decreased RBC 2. increase vascular permeabilityà serum sickness Protective cell (s) Mast cells macrophage.a def of folate manifests more quickly as body stores only last a few monthsà drug tx may cause folate defà phenytoin.c inciting factor is and is augmented in persons with a pre-existing hemolytic anemia 4.5 g/dlà these include iron-deficiency. Characterization of Anemias Folate or B12 deficiency • B12 binds with IF and subsequently absorbed in the ileum à a decrease in B12 is referred to as “pernicious anemia” and is often caused from atrophy of the gastric musosa which results in decreased intrinsic factor secretion it may occur possibly secondary to an Ab reaction against the gastric parietal cellsà this more commonly occurs in the elderly (>60 year of age) • folate is absorbed in the upper small intestine . acute blood loss . anorexia. progressive increase of these cells in the blood and the bone marrow. autoimmune disorders 38 . which are: anemias 1.c w/ portal HTN. neutrophils.Chronic lymphocytic anemia Megaloblastic anemias M of the WBCs characterized by a slow. and invasion of malignant cells into organs. Immunologic disorders-allergic and sensitivity reactions and immunosuppressive states Allergic and sensitivity reactions Types I-Immediate II-Cytotoxic III-Immune complex Description of s/s Anti-body Anaphylactoidà allergy. hemoglobin abnormalities. myalgia and a Production fleeting rash 3. tachycardia (d/t increase in resting CO). (90% of cases are found in people over 50).Causes. and risk factors CLL affects the B lymphocytes and causes immunosuppression. Hemoglobin SC. asthma. Hemoglobin C -a mild to moderate anemia à do not use a tourniquet on these patients 5. hemoglobinopathies and red cell enzyme deficiencies • -hereditary spherocytosis .c. affects 2-3% of the world population antimalarials. hemolytic disorders . splenomegaly . basophils and N/A mast cells which cause endothelial cell retraction and therefore. rhinitis and food.Hemoglobin SS (sickle cell anemia). vertigo and HA • Lab Findings: Low serum Fe and ferritin and high Total iron binding concentration (TIBC) Anemia of chronic inflammatory process of greater >1 mo will often cause an anemia and typically. There are a number of specific types of hemolytic anemia.and hereditary elliptocytosis 4. a wide PP. aplastic anemia . IgG where the IgG binds to an antigen in the BMà phagocytosis. porphyrin and globin Physical findingsà pallor is the m. eczema.Usually the symptoms develop gradually. hay fever. the Hgb will be between 9-11 g/dl. incidence. Human parvo virus B19 (HPV) pt’s present with a nonspecific viral syndrome (fever. failure of the bone marrow. bloating. feature.the legs affected more severely than the arms) Microcytic Severely anemic patients will often be symptomatic at rest and unable to tolerate significant exertion when the [Hgb] falls below Anemias 7. Hemoglobin S-beta thalassemia. killer cell activity or complement mediated lysis vasoactive amines are released by platelets. anti-metabolites (especially MTX). Bactrim. aspirin drug causes occurs when the corrected reticulocyte count is not elevated and is a primary marrow disease affecting all cell lines Anemia of 1. the anemia of chronic renal failureà declining erythropoietin production by the kidneys with shortened red cell survival 5. basophils and mast cells and continuous C3a and C5a activity Examples Anaphylaxis Immune Hemolytic Anemia leukocytoclastic vasculitis. headache. nitrofurantoin. chronic infections and myeloproliferative disorders 3. siderolastic and the thalassemias • they collectively represent a decrease in the availability or synthesis of one of the3 major constituents of the Hgb molecule iron. muscle strain. herpes zoster Intra-abdominal Disorders – that irritate the diaphragm or stimulate breathing Hyperventilation syndrome . CHF. Curshman spirals in the epithelium caused by the plugs shedding epithelium and charcot layden crystals Bronchiectasis Caused by tumors. and diaphoresis. costochondritis. chronic bronchitis. foreign bodies. & results of specific diagnostic studies. rib fracture. congenital. previous hx of PE.fast & shallow w/ rales. P. Major conditions of COPD –syndromes characterized by dyspnea (difficulty breathing) and are accompanied by chronic or recurrent obstruction to airflow. influenzae. Strep. Over distended lungs Marked goblet cell hyperplasia. COPD. apprehension/anxiety cough.Def in adenine deaminase enzyme. surgery in the last 3 months. marked obesity. sarcoidosis Sarcoidosis Disease of unknown causeà Lung fibrosis and cor pulmonale. macrophages and T cells then elaborate cytokines which mediate a vast range of inflammatory responses Immunosuppressive states allergic contact dermatitis and graft rejection Primary Immunodeficiencies X-linked agammaglobulinemia Defect of Btk protein kinase for B-cell maturationà recurrent pyogenic infections Digeorge Syndrome Thymic hypoplasiaà poor defense against viral and fungal infections SCIDS Purine salvage deficinecy. Asthma Hyperactive airways leading to reversible bronchco-constriction. CXR.mimic PE. Pneumoconicosis. à Dx confirmed or excluded only with more sophisticated tests. pleura + lymph nodes. Ml. bilateral hilar lymph nodes.tachypnea >16 breaths/min . such as ventilation-perfusion lung scan (V-Q scan) or pulmonary angiography. and syncope. spontaneous pneumothorax. Histologically: Noncasseating granulomas. cystic fibrosis and necrotizing pneumonia Permanent abnormal dilation of the bronchi and bronchioles Restrictive lung disease Decreased expansion of lung parenchyma with decreased total lung capacity. pericarditis Musculoskeletal disorders . phlebitis or DVT. Diagnosis of PE is based on . and pleurisy Cardiac disorders – Ml.. aeruginosa. B/T cells Tx: BM transplant s/s: thrombocytopenia and eczema H. 39 . or lung rx to inhalation of mineral dust i. Supplementary Tests that show PE: Arterial blood gases. in patients with H/O heart or lung disease – S & S are mistaken for manifestations of the underlying disease. estrogen use. pregnancy. Presence or absence of any s/s does not confirm or exclude the diagnosis of PE. Assoc with bronchogenic carcinoma and mesotheliomas (malignant neoplasms of mesothelial origin Pneumonia Bacterial: Staph.Ghon complex àparenchymal subpleural lesion and enlarged casseous lymph nodes. COPD. and coliform bacteriaàBronchopneumonia (patchy consolidation) Pneumococcus *most common* Lobar pneumonia (entire lobe) Tuberculosis (M. Common Signs .IV-Delayed Antigens are trapped in macrophage and cannot be cleared. Bronchi are occluded by thick mucinous plugs. T0 >100. characterized by Langerhan giant cells Definition Risk factors Clinical features Nonspecific & INCONCLUSIVE! (DDx) PULMONARY EMBOLISM Defined as a thrombotic occlusion of the pulmonary arteries. Emphysema Smoking Destruction of bronchiole wall w/o obvious fibrosis Pink puffers-tend to over ventilate to remain well oxygenated Chronic Smoking Exhibits squamous metaplasia Persistent cough w/sputum production for at least 3mo in 2 consecutive years bronchitis Slight increase in goblet cells Blue bloaters.dyspnea. N/A CTLs.barrel chested and dyspneic. COPD. H. Involves. Respiratory disorders. due to an increase in resistance to air flow owing to partial or complete obstruction of the respiratory tubes. Can be caused by infections. In patients without prior cardiopulmonary disease – S & S are clues to the diagnosis.clinical suspicion.asthma. history of DVT. malignancy. Tuberculosis) Primary TB. sweats. tachycardia or pulse rate >100b/min. or lower extremity trauma. pleural pain.4. asbestosis. ancillary laboratory data. laminated calcium secretions Asbestos Silicates that form fibers. prolonged immobilization. Respiratory disorders . Dx of exclusion. 12-lead EKG. and emphysema Diseases of the lungs Obstructive Disease of the airway. including asthma. pneumonia.e. cardiac gallop. Secondary TBàcollection of granulomas in tissue. Common symptoms . pneumonitis. hemoptysis. breathe through pursed lips with Severe cases may cause obliteration of slow forced expirationsà Thick mucinous plugs and submucosal gland bronchiole lumina-bronchiolitis obliterans hypertrophy occlude bronchi (same as in asthma). combined B and T cell deficiency -recurrent infections Wisckott Aldrich Syndrome X-linked instability of the platelet and lymphocyte associated glycoprotein which reduces their life spanà low platelet. miserable. • Administer O2 pulse oximetry à 95% • Rx hypotension (500-1. MOAIs Consider in children with recurrent fractures Abuse Mania Bipolar disorder : Anger disordersß tx with anti-depressants and Lithium Constant exicitability Characterized by pds of (mania) alternating w/ pds of depression-"mood swings" btwn mania and depression can be very abrupt. Heparin Rx ß prevents embolization of existing clots so further embolization of formed clots & shock most commonly occur w/in 4h of initial symptoms. or down in the dumps True clinical depression is a mood disorder in which feelings of sadness. • Systemic thrombolytic therapy with streptokinase. * KNOW* Venography Helpful. In PE . 5% of patients with PE . o Patients undergoing pulmonary embolectomy.HyperLDH. moderate. hemodynamically unstable & are not prone to bleeding § (2) Pt w/ persistent hypotension despite medical Mx • Pulmonary embolectomy should be considered in the rare patients w/ confirmed emboli (angiographically) & remain in shock despite thrombolytic therapy and supportive care. recent stroke. o Massive pulmonary emboli with hemodynamic compromise. intracranial or intraspinal neoplasm. severe diastolic hypertension).90% pts.33% pts. noninvasive blood pressure device. including depression. clinical signs . in elderly patients. aiming for 1. The presence of an increased alveolar-arterial (A-a) gradient is more sensitive (95%) for PE pH: 7. self-hate. Stable pts may be admitted to telemetry bed. IV line. o Thrombolytic therapy considered in Rx of pts w/ § (1) Acute massive embolism. or severe. uncontrolled severe hypertension.radiographic evidence of DVT . Pts w/ clinical instability should be admitted to ICU. PT. o Contraindications to anticoagulation . Westermark – large lung lucency in area of embolus. and inappropriate guilt • Extreme difficulty concentrating Agitation. and thrombolytic therapy in extreme cases.000 to 20. Pa02 < 80 mm Hg in 80% of patients with PE. Enzymes Triad associated with PE . influences how you are treated. needed to confirm the Dx if the V-Q scan demonstrates equivocal results medium or low probability for PE in at risk pts. abuse.000 ml of NS IV bolus)à Crystalloid IV fluids • Unresponsive hypotension & absence of hypovolemia à Rx w/ Dopamine 2 . Pts stable & beyond this initial highrisk period are admited to a non-monitored bed. including common medical emergencies and CPR 40 . pulse oximetry monitor.Pa02 >90 mm Hg. and anger disorders and their effects on patient compliance Depression Depression may be described as feeling sad. or in whom thrombolytic therapy is contraindicated. TREATMENT of PE Initial stabilizationà anticoagulation with heparin. unhappy. Symptoms of depression include • Trouble sleeping or excessive sleeping A dramatic change in appetite. anger.e. • Inferior vena cava interruption considered in: Green field filter o Pts in whom anticoagulation is absolutely contra indicated (i.45 PaCO2: 35-45mmhg PaO2: 80-100mmhg HCO3: 22-28meq O2 sat: 95-98%(a) 60-85% (v) BE: 0+/-2mmol 12-lead EKG S1Q3T3 Prominent S in Lead 1 ST depression in Lead 2 Prominent Q in lead 3 Inverted T in lead 3 Pattern on the EKG is highly suggestive of PE present only in 12% CXR Hamptons Hump –wedge shaped pleural-based infiltrates. Emergency Medicine. V-Q scan Very sensitive test for PE & completely normal study rules out the disorder. hyperbilirubinemia and hyper SGOT. blue.ABGs. TCAs.mild. PTT (anticoagulation). J. recent trauma.000 U/h adjusted using PTT. w/ evidence of residual thrombus in a lower extremity.35-7.active internal bleeding. Behavioral disorders. which your doctor can determine.52x control. • Full heparinization: Heparin IV bolus of 10. The degree of your depression. often with weight gain or loss • Fatigue and lack of energy Feelings of worthlessness. esp. urokinase or t-PA hastens the resolution of the clot but has not yet been shown to reduce mortality. active bleeding. Other supplemental tests – CBC (R/O DDx). recent surgery. immediate postoperative period. esp. loss. I. restlessness. Angiogiaphy ß more potential complications. and irritability • Inactivity and withdrawal from usual activities Feelings of hopelessness and helplessness • Recurring thoughts of death or suicide Treatment: SSRIs. Depression is generally ranked in terms of severity -. or frustration interfere with everyday life for an extended time. o Pts w/ compromised cardiac or pulmonary function who may not survive a recurrent event. neurosurgical patients. continuous drip of 1. cardiac monitoring.000 U.5 mcg/Kg/min & titrated to maintain a systolic blood pressure of 90 mm Hg. Pulmonary angiography Is gold standard for diagnosing PE and is a much more specific test than the V-Q scan. or ephedrine – 15 – 25 mg IM or IV Insulin Shock or Hypoglycemic Crisis Description Recognition Management low BS – it is defined by a serum glucose level of < 50 mg/dl in men.finger stick to ascertain [Gl-]p and if conscious. hunger and belching and ANS effects manifest as diaphoresis.4 mg IM or IV. bradycardia. an overall feeling of weakness or fatigue and dehydration: Kussmaul breathing . deep breathing immediate transport to the hospital so that glucose levels must be brought under control and for patient to be hydrated Insuilin Sliding scale 150-200 201-250 251-300 301-350 351-400 401-450 >450 O units 2 Units 4 Units 6 units 8 units 10 units 12 Units Description Management Description Recognition Management Seizures seizures are generally associated with epilepsy or drug toxicity and are characterized by an abrupt. monitor BP and PP. nervousness Late s/s: headache.0. Inject slowly until the patient begins to react.heavy. polydipsia. keep the head turned to the side. altered mental status such as confusion and mood changes. polyuriaà patient may have N/V and abdominal pain. administer IV gl.Description Etiology and Pathology Incidence Recognition Management Office Emergencies Vasovagal Reaction (Syncope) Syncopy is defined as the “sudden loss of postural tone and consciousness with subsequent spontaneous recovery” Transient LOC stimulated by fear/ pain. ammonium salts. administer diazepam. < 45 mg/dl in women and < 40 mg/dl in children • the CNS or ANS are most sensitive to low {Gl-]p insulin shock – hypoglycemia secondary to either too much insulin or insulin taken without food. with eventual coma or seizure Treatment should not be withheld while awaiting laboratory studies! • do a Gl. the peripheral vasculature vasoconstricts to help maintain perfusion of the brain patients < 40 y/o are at ↑ risk for a vasovagal reaction. patient may become pale or cyanotic w/possible loss of bladder or bowel control Following a seizure.20 to 50 cc of 50% dextrose solution.atropine (if bradycardia does not respond to palliative care). feeling hot or cold. where the heart b/c overstimulated by the vagus n – an ANS mediated responseà ↓HR and therefore. too much exercise may also precipitate hypoglycemia and alcoholic beverages may cause blood sugar to drop early symptoms: • CNS changes are observes w/ weakness. blurry vision. Protect the patient and allow the seizure to run its course: following the seizure check for vomitus in the airway.5 to 25 mg IV Description Management 41 . poor concentration and personality changes • CVS manifestations: palpitations. sensory. where pt may c/o chest pain and have an elevated BP toxic effects can be managed with an alpha and a beta blocker.12. supine with legs elevated) to reassure patient. candy or a gl pill (three gl tablets are recommended) • If demonstrating altered mental status or the patient is unconscious. autonomic and/or psychic changes. "pounding" HA. a period of sleep (post ictal depression frequently occurs). if a patient goes into status epilepticus. Transport pt to ER ASAP. faintness. oxygen (flow rate – 2 to 4 liters/minute) as a very last resort . transient change in the general nervous system – may demonstrate motor. ↓CO to compensate for the ↓CO. labored. give orange juice or soda (6 oz). ↑HR and palpitations. and most patients will experience an aura prior to a seizure grand mal seizure – signs of CNS over stimulation and muscle spasms LOC with profound muscle spasms. hypotension. 5 to 10 mg IV slow push or dilantin 300 mg IV and administer oxygen. Hyperglycemia or Diabetic Coma Characterized by increased thirst and urination usually for one to several days: polyphagia. diaphoresis. rapid. a rapid acting vasodilator such as the nitrates may also be effected and demerol . Epinephrine Reaction recognized by restlessness. GI s/s: nausea. may shiver or even vomit PE include: pallor. mydriasis (full-pupil dilation) LOC and reflex tachycardia Supportive – Trendelenberg position (recumbent. patients who are sitting – rather than lying down – are more susceptible multiple “needle sticks” may increase the risk and lengthier procedures increase risk patients may complain of nausea.must observe for status epilepticus which is a dangerous condition with seizures will occur one right after the other preventing the patient from recovering in between. agitation. Airway. may repeat in 30 minutesà in the hospital setting.5 mg of epinephrine 1:1000 every 5 to 15 minutes for up to 3 doses (use the epinephrine with some caution as it will increased both the blood pressure and heart rate) administer an albuterol inhaler (a bronchodilator) deliver oxygen (2 to 4 liters/minute) Asthma and Bronchospasm – Acute Asthma Attack bronchospasm with concomitant angioedema wheezing – this may occur secondary to a hypersensitivy reaction or may be triggered by anxiety or other emotional stress a patient with a history of asthma will generally recognize the oncoming symptoms and know to loosen their clothing and use their inhaler this should be a reversible process and with proper management will not lead to respiratory arrest Description Manifestations of Lidocaine toxicity Incidence Recognition Treatment Lidocaine Toxicity toxicity will occur 2 ˚ to an overdose of the local anesthetic and high [local anesthetic] manifest with CNS and CVS effects • Rxs occur 2˚ to direct IV administration or by use of excessive ats of local anesthetic resulting in rapid systemic absorption • other factors precluding to toxicity include the speed of injection. it is likely that many rxs are not reportedà with an epidural. however. and idiosyncratic sensitivity and the presence of concomitant disease processes • A true toxic reaction must differentiate this from a hypersensitivity reaction to the localà patients are most often sensitive to the PABA preservative in local anesthetics. and the patient may also complain of dizziness and numbness • as toxicity further takes hold. visual changes.8 mg sympatholytic maximum onset of action is between half an hour and 2 hours and lasts for 6-8 hours. the prevalence is 0. numbness. utilize 0.1 mg qh up to a 0. Evaluate circulation and Drugs • give O2 via nasal cannula or a face mask at 100%. with a dose of 0. HA. onset in 10-30 minutes and lasts for 2-4 hour . If this is not possible. CO.possible arrest and CVS depression in patients premedicated with benzodiazapenes or barbiturates. the patient may develop a “metallic” taste in his mouth. give a vasopressor (for example. ASA for headache Clonidine central it is administered p.2% CNS signs may range from a feeling of sleepiness to a feeling of being intoxicated. mental status changes and biphasic ultimately. intubation may be required. vascular necrosis and impaired renal fx w/hematuria acute↑in BP. whereas. nitroprusside may be administered Management if asthma is occurring secondary to an extrinsic trigger.2 mg initially followed by 0. cardiovascular depression may be the first sign of toxicity a useful mnemonic for treating these reactions is SAVED – Stop injection.o. pCO2. ringing in the ears and coma supportive – the goal is to drop the patient’s DBP to between 90-100 mm/Hg: elevate head.Description Recognition Management Hypertensive Crisis Serious consequences if DBP elevates >140 mm/Hgà retinal hemorrhage. seizures second considered a depressive state and is recognized by the onset of LOC with respiratory depression phase or arrest CVS effectsà at high levels can lead to re-entrant arrhythmiasà licodaine is commonly associated with sinus higher [lidocaine] tachycardia where as bupivacaine may lead to ventricular tachycardia and fibrillation as local anesthetics are used in many settings. methoxamine (which affects α-receptors only) will elevate only BP w/ no effect on HR • ventricular arrhythmias should be managed with bretylium • initiate EMS prn ® 42 . have difficulty focusing and become agitated and anxious • as levels continue to ↑. seizures give way to respiratory depression . monitor vitals • anticonvulsant drugs should be administered as indicated – one may use either a benzodiazapene or a barbiturate • diazepam 5 mg -10 mg or thiopental 50-100 mg can be administered • succinylcholine may also be effective. will require a controlled airway and controlled ventilation • if BP remains depressed for > 30 minutes. papilledema. Ampules of local can be purchased that do not contain preservative CNS due to the initial inhibition of the excitatory neurons à inhibition of both inhibitory and excitatory neurons as the effects process develops first phase considered to be “excitatory” and include tingling. Ventilate. the pt may c/o of tinnitus.importantly this vasodilator medication may cause myocardial infarction or anginaà The main use of this medication is for pregnant individuals nifedipine 10 mg sublingually.3 to 0. sedation with clonidine is prominent and rebound hypertension may occur Hydralazine arteriolar dosage is 10-20 mg IV. demonstrating an ↑ RR and begin twitching in the extremities – this “twitching” may evolve to seizures • if levels remain high. 20 mg methoxamine .Vasoxyl IM or ephedrine 20 mg IM) • ephedrine will elevate both BP & HR. larger and more rapid blood volume losses result in least 20ml/kg more severe shock symptoms. diphenhydramine 25 to 50 mg IM. Loss of contractility occurs whenever the heart is unable to pump enough blood for the Restore strength of pump +beta needs of the body.min Loss of afterload or a decrease in • 0. stimulation causes § decreased vagal tone. internal. mg IM or Solu-cortef (hydrocortisone) 200 mg IV • aminophylline 250-500 mg IV given slowly.3 to 0. also helps compensate for acidosis Cerebral Ischemia (Cerebral Perfusion Pressure<40mmHg) o activates sympathoadrenal system (more potent than Chemo or Baroreceptor mechanisms) o increased catecholamine release from both adrenal gland and sympathetic nerves (can also get vagal stimulation which is counterproductive) Reabsorption of tissue fluids o decreased mean arterial pressure o arteriolar constriction } All lead to decreased hydrostatic pressure o decreased venous pressure in capillaries. o up to 1l/hour can be reabsorbed in adult sized patients When Hydrostatic Pressure > Osmotic Pressure: Filtration Osmotic Pressure>Hydrostatic: Reabsorption Endogenous Vasoconstrictors o Epinephrine and Norepinephrine § released from adrenal medulla and sympathetic nerves § cause vasoconstriction and increased cardiac output o Vasopressin (Antidiuretic hormone) § released from posterior pituitary § potent vasoconstrictor o Renin (from decreased renal perfusion) § leads to angiotensinogen production eventually yielding angiotensin .dogs § skin and skeletal muscle . Caused by disorders of the heart muscle.Types and compensatory mechanisms for shock Etiology and treatment Tx: ABCs Loss of preload. dexamethasone 8 mechanisms from histamine. monitor vitals and activate EMS prn Fnctional loss of preload.improves venous return (pump model). or from hypovolemia. the valves. or the heart's receptors electrical conduction system.5 mg of epinephrine 1:1000 subQ q15min prn. which causes § venoconstriction § constriction of blood reservoirs (increasing circulating blood volume) § spleen . afterload and contractility Identify and correct etiology Hypovolemic Cardiogenic Anaphylactic Septic and Neurogenic Compensatory Mechanisms of shock • • • • aroreceptor reflexes (respond to small changes in vascular tone/pressure) o located in the carotid sinus and aortic arch. and cause § further vasoconstriction § respiratory stimulation . leading to increased resorption of fluid15ml/kg/hour.humans Chemoreceptor o located locally in tissue beds o sense hypoxia (due to inadequate blood flow in peripheral tissues). In general. and establish IV access and systemic vasculature resistance run normal saline wide open owing to increase in vasodilatory • deliver oxygen 6 liters/minute. Dopamine 2-20mcg/kg/min or dobutamine 3-20 mcg/kg. Fluid replacement-at bleeding.a very potent vasoconstrictor Renal Conservation of Water o o • • 43 . which § increases heart rate § decreases coronary resistance (improves myocardial oxygen supply) § increased sympathetic tone. Loss of approximately >1/5 the normal blood volume Cause: bleeding external. b. At the same time. See if the person moves or makes a noise. 2. 11. 6. b. c." 10. Give 15 chest compressions. Count the 5 compressions quickly: "a. 14. and feel for breathing. begin chest compressions. d. Press down on the child's chest so that it compresses about 1/3 to 1/2 the depth of the chest. Shout. listen. let the chest rise completely. full breaths If the chest does NOT rise. coughing. off. For an infnat (>1yr) steps 9-13 are different 9. 11. Lift up the chin with 2 fingers. Place the heel of your other hand on top of the first hand. 12. try the chin lift-head tilt again. d. or any movement. d. Check for responsiveness. 7. d. Perform chest compressions: a. These compressions should be FAST with no pausing. Open the airway. Give the person 2 slow. push down on the forehead with the other hand. Each time. DO NOT lean back or forward. c. Repeat steps 11 and 12 until the person recovers or help arrives. (head-tilt chin lift or jaw thrust if suspected neck injury) Look. full breath." Give the child 1 slow.just below the nipples. off. Keep the chin lifted and head tilted d. keeping the head tilted back. Watch for chest movement. c. keeping the head tilted back. d. Position your body directly over your hands. b. 10. 13. press down about 2 inches into the chest. Place the heel of one hand on the breastbone -. As you gaze down. 5.normal breathing. 13. full breaths. If there is a chance the person has a spinal injury. Perform chest compressions: a. leave the child to call 911. Repeat steps 11 and 12 until the child recovers or help arrives. call 911 even if you have to leave the person. full breaths. you should be looking directly down on your hands. Place the heel of one hand on the breastbone -. 4. Keep your other hand on the child's forehead. Make sure your heel is not at the very end of the breastbone. Place 2-3 fingers on the breastbone -. Place your ear close to the person's mouth and nose. 11. Give the infant 1 slow. c. Each time. Keep your other hand on the infant's forehead. Perform chest compressions: a. If the person is not breathing: a. If you are alone. b. Continue cycles of 15 chest compressions followed by 2 slow. Press down on the infant's chest so that it compresses about 1/3 to 1/2 the depth of the chest. After about 1 minute (four cycles of 15 compressions and 2 breaths). The chest should rise. After about 1 minute. coughing. At this time. 44 . If these signs are absent. These compressions should be FAST with no pausing. Pinch the nose closed c. 9. Cover the person's mouth tightly with your mouth b. full breath. Shake or tap the person gently. re-check for signs of circulation. b. let the chest rise completely.just below the nipples. Count the 5 compressions quickly: "a. The chest should rise. Shout for help and send someone to call 911. Each time. Your shoulders should be in line with your hands. full breath. Give 2 slow. 8. Give 5 chest compressions. Look for signs of circulation -. The chest should rise. check again for signs of circulation. Continue cycles of 5 chest compressions followed by 1 slow. Carefully place the person on his or her back. if the child still does not have normal breathing. full breath. Feel for breath on your cheek.o Aldosterone release § stimulated by vasopressin § Causes Na reabsorption in distal tubules § Water follows the sodium Facts about CPR 1. 12. "Are you OK?" Call 911 if there is no response. 10. For a child (1-8yrs) steps 9-13 are different 9. Give 5 chest compressions.right between the nipples. 3. Make sure not to press at the very end of the breastbone. or movement. and give 2 more breaths. Continue cycles of 5 chest compressions followed by 1 slow. c. two people are needed to move the person without twisting the head and neck. T versicolorà yellow gold 2. leave the infant to call 911. At this time. GNà Red Ø Aerobic and anaerobics should be ordered for clinically infected wounds Ø Fungal.12. KOH dissolves kertin so that the skin. check again for signs of circulation. Repeat steps 11 and 12 until the infant recovers or help arrives.000-400. After about 1 minute.5-11. Examine under the microscope for the presence of fungal hyphae which can be seen growing in epithelial cells Cultures require 10d to grow. T capitus (M. Histopathological studies Gram staim KOH test Dermatophyte test medium Gram-positive bacteria (purple) and gram-negative bacteria (pink) Scrape the scale from a lesion and apply 10-20% KOH. 14. Dermatophytes have powdery white colonies. 3 cx should be taken from 3 sites. Erythrasma-C. black or green in color.0 ·103 /µ l o Neutrophils 57-67%: Segs 54-62% and Bands 3-5% Lymphocytes 23-33% o Monocytes 3-7% Eosinophils 1-3% Basophils 0-1% RBC’s 3. sinus tracts or ulcers that do not appear to be infected. P aeruginosaà green. so the colonies must be examined. Dialated capillaries (erythema) will blanch.000 per cubic millimeter Non specific indicator for inflammation. Medium will turn red if dermatophytes are present.0-18. nail or hair shaft becomes clear. or any movement.8 Normal values: ESR CRP 45 .5 million cells/mcL Hemoglobin Male 14. coughing. K. white.0-16. Deep cultures are best and should be taken before the patient is places on antibiotics Must be done if bacteremia is suspected or hematogenous OM is diagnosed. a false positive may be seen w/saprophtes. Minutissimumà coral red. B. saprophytes have shiny colonies that may be brown. Auspitz sign Diascopy Nikolsky sign Shave biopsy Punch biopsy Excisonal biopsy Tzank test Wood’s Light Lab studies and special procedures Pin point bleeding that occurs when the scales of a psoriatic lesion are removed Glass slide test-press a clear glass against the lesion and look for blanching. canisà bright green. VSV and molluscum contagiosum A black light w/ 360nm of wavelength filtered through glass used to dx certain infections by causinf different colors to fluoresce. GPà Blue.0 Hematocrit Male 40-54% Hematocrit Female 36-48% Platelets 150. 13. and acid fast culturesà Ziehl Neelson is widely used for acid fast staining DO NOT culture superficial wounds. may be useful to follow course of infectious dz w/ Ab use Normal value: Male: 15 Female: 16 Non specific indicator for inflammation (more sensitive than ESR) Normal value: <0. Diagnosis A. Gram Stain Culture and Sensitivity Blood cultures CBC w/diff Infections Laboratory testing in the diagnosis of Infection Extremely important for initial antibiotic choice. may represent cytoplasm’s cytoplasmic failure to mature d/t infection Ø Toxic granulation-stress response to infection: dark blue-black granules in a neutrophil’s cytoplasm Ø “Shift to the left”-release of immature band cells from bone marrow in response to infection Ø Eosiniphilia may be observed when parasites are observed in infectious process or may be 2o to allergy WBC 4.5-5. hemorrhagic lesions (pupura) will not An epidermal detachment produced by lack of skin cohesion.000-11. Dermatology 1.0 Hemoglobin Female 12. 10-30min apart Ø Leukocytosis occurs in the presence if acute infection Ø Dohle bodies-remnants of RER RNA which appears as a light blue stained area in a neutrophil’s cytoplasm. however.000/ cubic millimeters Leukocytes 5. if the infant still does not have normal breathing. seen in bullous diabeticorum Suited to lesions confined to the dermis ie seborrheic keratosis or molluscum contagiousum Method of choice for most inflammatory or infiltrative disease-produces a full thickness specimen Method of choice for diagnosis and removal of dermal and subQ cysts and tumors (epidermal cysts and lipomas) use for melanoma and lesions too big to punch out >8mm in diameter Used to diagnose viral disease-HSV. a protein produced by staph with phage group II in its genomeà staphylococcal scalded skin syndrome resistant to common staph antibiotics: meticillin. p. dry.e. Other drugs that may be used include: The skin may become grayish. most frequently ulcers.all topical. warm moist compresses and topical anti-microbial agents systemic anti-microbial tx is boil.Ancylostoma braziliense Toxoplasmosis Gondii Leshmenia (mucocutaneous leishmaniasis).Infectious organisms Parasitic Cutaneous larvae migrans . p. imipenem and beta lactams and it is no more virulent than other forms of Staph. Rare regional 100 mg/kg/day q6h and widespread cases bullous presentation lymphadenopathy and these lesions heal amoxacillin 100-200 mg/kg/day q6h IM -occurs in neonate. p. A visible path that marks the migratory stages and oral thiabendazole is available in trail of the larvae is often seen. oral ivermectin (15-200mg/kg) has also and scaling plaques been shown to be effective against scabies Bacterial Description virulence factors MRSA Type Bullous Impetigo Impetigo Neonatorum Scalded Skin Syndrome (Ritter's Dz.c in grape-like clusters-staphylococcal pyodermas are characterized by purulence.this protein will bind to the Fc portion of IgG preventing it from binding to complement teichoic acids present in the staphylococcal cell wall may stimulate antibody production against them à SBE exfoliatin . 50exfoliatin. which is PCNresistant à fragile bullaeà ruptured bullae resembles mupirocin. and mental retardation. rapidly evolving. 0.o.50-100 mg/kg/d q6h A furuncle is a tender. Protein A in its cell wall . p. and fungus. sulfonamide and spleen. nafcillin. 100-200 mg/kg/day q6h generalized desquamative off in large sheetsà there is in equal doses IM or IV and tx must be maintained for 10 days disease a positive Nikolsky's sign cloxacillin. enlargement of the liver Medications: include: pyrimethamine. it is a FT slough occurring w/i the it is a febrile. 250-500 mg q6h furuncles with and may develop a • nafcillin. large flaccid and is a result of a rxn to meds which cause sub-epidermal adults bullae appear and then the blister formation (i. if it is the pathogen causing an infection. skin cancer (basal cell carcinoma). It can affect the mucous membranes with a wide range of Antimony-containing compounds are the principal appearances. and bactrim. it is more difficult to treatà vancomycin is the DOC Description/Location Clinical Presentation Treatment usually caused by Group II coagulase-+ a vesicle that rapidly evolved into a large warm compresses.o.young kids without scar formation or IV infants and young children and after >12 hrs of generalized Must r/o TEN that generally occurs in older kids/ adults rarely in immuno-compromised erythema. folate clindamycin. tender and painful swelling PNA with appropriate wound care w/ appropriate soaking about the nail plate that occurs secondary to a separation of instructions could be: 2 tablespoons of epsom salts in 1 qt of the eponychial fold from the nail plate and usually caused lukewarm or tepid water or 1/4 vinegar in one qt of (acetic acid soak) by trauma.o. a staphylococcus. Protozoans Infants may show signs of CNS disorders. leprosy. Dermatitis Exfoliative Neonatorum) Deep Folliculitis or Furunculosis Connective Tissue Infections Secondary to Staphylococcal Infections Catalase + GPC occurring m. Sarcoptes scabeii “Scabies” Hookworms “Creeping eruption”à the larvae burrow into the skin. neomycin or staph. p. appropriate for individuals with chronic or extensive problems or in areas which are very is simply two or nodule sensitive more confluent à after few days • cloxacillin. crotamiton (not rec for children or pregnant women) papules.o. epidermis will begin to peel basement membrane zone) oxacillin. dark. and flaky. 250-500 separate "heads" "yellow head" mg q6h • systemicAb tx should be continued for months in individuals with chronic disease Is an acute or chronic purulent. cepahlosporins.5-2g 16h IM or IV PCNallergy: erythromycin. infants.o. blindness. meglumine antimonate /sodium stibogluconate syphilis. but also Pseudomonas aeruginosa and penicillinase-resistant antibiotic such as dicloxacillin would be streptococcus appropriate Nikolsky's sign involves the superficial layers of skin slipping free from the lower layers with slight pressure and is seen in TEN and pehphigus vulgaris as well Pyogenic Paronychia 46 . bullae. oxacillin. mild àcloxacillin. chewable 500 mg tables. These include: resemble those of other diseases including cutaneous tuberculosis. nodules and sulfur ointment 5% . 3 major tx: lindane 1%. vesicles. It may cause skin lesions that medications used to treat leishmaniasis. causing an Tx: with topical thiabendazole 10% aqueous intense inflammatory response that follows their progress beneath the solution is good for mild infection in the early skin and leads to severe itching. moisture and causative agents not only include occasionally oral antibiotic therapy may be necessary. drugs. pustules and bullae with a narrow rim of erythema Coagulase. pentamidine /amphotericin B Arthropoda Characteristic variety of lesions: macules. a carbuncle erythematous. ß is responsible vesicular or a second degree burn. the condition is exacerbated by increased covering the wound with a telfa-type bandaid is generally adequate. 0.5-1 g q6h dicloxacillin. pustules. Edematous.this is an anti-phagocytic factorà Causes a type 3 hypersensitivity reaction (acute glomerulitis and RF) and because of this. qid and cellulitis ß Staphy aureus. working its way appropriate anti-microbial towards the regional lymph nodes that often tender and enlarged. wound or cellulitis. red cell casts and protein onset in more than 95% of pts occurs more commonly it presents with fever. warm. occurs secondary to a tx involves identification of GAS infection. Always associated inflam. migratory polyarthritis and carditis which results in myocardial and valvular following a strep pharyngitis damageàlaboratory findings include elevated ASO titres and erythrocyte sedimentation rate is an erythematous skin eruption an erythemogenic toxin causes a mild inflammatory response that leads to dilation of blood most commonly associated with vessels. A prodrome is secondary finding therapy and wound care rapidly spreading necrosis of subQ m.c GAS and strains that cause NF produce streptococcal pyrogenic exotoxins which are directly toxic . purulence is not a common feature Type Nonbullous Impetigo Ecthyma Erysipelas Cellulitis Ascending Lymphangitis Necrotizing Fasciitis 50 and 60 y/o Cutaneous infections caused by group A Streptococcus Description/Location Clinical Presentation Treatment Impetigoß bacterial infection in the upper Honey colored stuck-on crusts lymphtopical Bactroban (mupirocin) epidermis beta-hemolytic streptococcus are adenopathy is often a secondary findingß these For lymphadenopathyà PCN generally the etiologic factor although staph will usually heal w/o scar formation if pedal 250-500mg qid X 7d for adults may also be recovered à highly contagious edema and/or hypertension occur in this 56mg/kg/day divided for children individualsà suspect glomerulohephritis Erythromycin in PCN allergy A bacterial infection lower epidermis. and high fever that often begin before the onset of the skin lesions.this is a hemolysin that is antigenic and inactivated by oxidation. we must be aggressive with antibiotics for at least 10d to achieve the lag phase kill of this organism. Diagnostic Criteria no muscle involvement/destruction or major vascular occlusion w/mod to severe systemic toxicity and mental disorientation.M-protein in the cell wall .PCN still remains the DOC for strep mediated disease HBO provides adjuvant tx with appropriate wound care regiment w/ or w/o delayed primary closure 47 . tender streaks (follows course the etiologic factor ad the of GSV) developing on the skin and extending proximally from the break in the skin. perfringens system w/ lymphedema and even elephantiasis necessary to prevent septicemia Secondary to an acute inflammation of the subcutaneous lymphatic channels. streptolysin O . Prodrome: fever. warm limb Upon necrosis of the subQà deep fasciectomy is indicated with radical debridement • parenteral Ab tx . punctate eruption that appears within 1-4 days following the onset of the illnessà this rash usually lasts for four to five days. and w/ strep super-antigen (SSAà release of cytokines that produce clinical signs such as LBPà mortality rate is very high 75% the LE is among the areas most commonly involved and is found commonly in individuals with preexisting or co-morbid conditions Types of NF based of the etiologic organisms I the presence of both anaerobic (the most common being Bacterioides fragilis) and facultative anaerobic bacteria II the presence of GAS alone or w/ Staph III clostridial myonecrosis or gas gangrene à skeletal muscle infection assoc w/ recent sx or trauma. chills. therefore the “scarlet” rash à the presence of a strawberry tongue. lymphadenopathy and fever. Vesicleà pustule with a wide rim of induration deep lesions require PCN and must r/o with a slightly elevated advancing margin on erythemaà ulcer. burning. and tenderness are typical complaints. including warmth. edema of the face & ankle and "smoky" urine. the [C3]is normal by 6-8 wks after are the presence of red bleed cells.prodrome of malaise. and contribute to the development of cellulitis. and tenderness and there may be lymphatic involvement manifested by overlying skin streaking and regional lymphadenopathyà tx leaves a permanent hyperpigmentation involves the deep dermis Sudden tender and warm erythematous and hard plaque that enlarges Penicillin and erythromycin from a break in the skin peripherally. irregular. edema. with skin group A strep It generally follows changes that resemble sand paper. tachycardia malaise. regional 250-500 mg p. In children. edema infection and may occur in up to 10% of patients with a usually resolves within 5-10 days and blood pressure usually streptococcal pyoderma. flu. the characteristic exanthem consists of a fine an episode of strep pharyngitis erythematous. with high bilirubin levels Clinical Recognition(dDx: cellulites) Prodrome: Febrile and tachycardia. lymphadenopathy pseudomonas in those lesions occurring generally occurs and healing results in a depressed atrophic scar in the groin or axilla Involves superficial dermis and is always caused by strep. Characteristic Lesion Well-defined vesicular and bullous clindamycin in penicillinP aeruginosa.. the antibody to it – ASO (antibody steptolysin O) . Red. an abrasion.Type Streptococcus pyogenes GAS Description GPC typically arranged in chainsà betahemolysis Streptococcus agalactiae GBS Types of relevant pathogenic Streptococcus (obligate aerobic GPC) Virulence Factors collagenase and hyaluronidase à facilitates spread of the bacteria through the connective tissue stroma of the skin. necrosis. It is common on the legs/face and very frequently will arise from venous stasis dermatitisà pruritus. chills. lymphangitis. adenopathy. gross hematuria usually urine findings consistent with sub-acute glomerulonephritis disappears within 1-3 weeks. lesions with plaques that may show petechiaeàlong range sequelae allergic patientsß parenteral Cryptococcus neoformans from erysipelas & cellulitis à damage to the lymphatic drainage administration may be C. then resolves with fine scaling Generally: streptococcal infections à wide rim of spreading erythema.can be measured and is an important diagnostic indicator of RF normal found in the female genital tract but may cause an opportunistic infection Type acute glomerulonephritis rheumatic fever scarlet fever Connective Tissue Infections Secondary to Streptococcal Pyogenes Infections Description/Incidence Clinical Presentation Occurs more commonly in children 2-3 weeks after a skin S/S: HTN.o. H. must do a UC&Sà returns to normal within 2-3 weeks. as vesicles ruptureà round ulcers w/ necrotic black centers. gentamycin pool Prodrome: sore throat. As the Tetracycline. is the most common cause of serious anaerobic infections • infections usually require a break in the mucosal membrane and are generally not communicable • both bacteroides and facultative anaerobes are often present in skin infections . this is generally a self-limiting process Folliculitis . 500 mg by mouth. eval w/ wood’s lamp Povidone/iodine soaks and oral erythromycin Muscle fails to display the 4 C’sà Prompt debridement is necessary. the spread of infection. antimicrobial should include coverage for both these bugs and the most likely facultative anaerobe they are co-habitating with • The polysaccharide capsule of Bacteriodes sp. Erythromycin. associated with hyperhydrosis and bromhydrosisà result of keratolytic enzymes from Corynebacterium or Micrococcus sedentarius Antibody inducing pseudomembranous collitis Metronidazole 500mg po tid or vancomtcin po 500 mg qid ® Pitted keratolysis C. anorexia and generally antibiotic therapy is not necessary headacheàrarely serious sequelae develop resulting in bacillary angiomatosis. is its most powerful virulence factor • members of Bacteroides fragilis are resistant to penicillins. it has better staph coverage and will kill Pseudomonas Type Ecthyma Gangrenosum Description Bacteroides fragilis non-spore forming. malaise. ie tap water virulence factors approximately 10% of the pop carry it as a normal part of the colon floraà likes to infect burns . clindamycin and chloramphenicol as alternative • infection Virulence Factors and Treatment Other miscellaneous soft tissue infections Erythrasma Myonecrosis “Gas gangrene” Cat scratch disease Chlamydia Lymphopathia venereum Interdigital infection caused by C. whirlpool or public swimming symptoms include marked swelling. The skin above the lymph node is often swollen (edematous) and red. the inguinal (groin) lymph nodes swell. gram-negative rods. Doxycycline.Gram-Negative Skin Infections Pseudomonas Aeuriginosa P. Bacteroides sp. 500 mg by mouth. fever & redness and a sensation of heat cream and if widespreadà malaise Flouroquinolone Cipro is a great drug. Amputation of an arm or leg may be indicated to control contractility. b/c tender. the and Zithromax is the agent of choice when skin and lymph nodes are often infectedà subcutaneous skin-colored or red antibiotic intervention is indicated papules The dz starts as a painless ulcer on the male genitalia or in the female genital tract. first-generation cephalosporins and aminoglycosides metronidazole is the drug of choice with cefoxitin. maculo-papular. The vesiclesb/c aminoglycoside with an hemorrhagic and violaceous. aeurginosa these organisms are GNR which are strict aerobes and they are able to grow in water containing only trace nutrients. HA. Antibiotics. should be given. Bacillus Henselai : systemic symptoms include fever. preferably penicillin-type. à blue purulence and pyoverdin (fluorescein) . and may rupture and 4xd for 3 weeks drain through the skin. 100 mg by mouth. These enlarged nodes are called buboes. The pt may also have systemic s/s including fever. capillary bleeding. patients w/ an absolute neutrophil count of >500 and pts who are immuno-compromised are at riskà produces an exotoxin A . These areas may 2xd for 3 weeks appear to heal. and malaise. and although these organisms are part of the normal skin flora. color and consistency. but the pt will have repeated episodes of lymph node swelling and drainage. 4xd for 3 weeks Superficial pitting in the sratum corneum on the soles of the feet giving rise to a mothTopical or oral erythromycin eaten appearance. Difficile 48 . Hyperbaric oxygen has been Clostridium tried with varying degrees of success.The anti-pseudomonal PCN presence of these vesicles along suggest the diagnosisà DIC. Caused by this is given IV Analgesics may be required to control pain. 1% acetic acid compresses. decreased appetite.Hot generally occurs 1 to 4 days after bathing in vesicular or pustular lesions that will leave w/in seven to 14 days Tub Folliculitis a hot tub. frequently assoc acid (vinegar) soaks 2-4x a day in water or wet areas w/ paronychia of the adjacent nail folds G(-) don’t like vinegar G(+)don’t like salty Pseudomonal most lesions occur on the trunk and LE pruritic follicular.which (-) eukaryotic protein synthesis pigments pyocyanin. grouped vesicles surrounded by pink to violaceous halos. minitissimus. tense.a yellow-green pigment that fluoresces under UV light Types of cutanuous Pseudomonas infections Description/Location Clinical Presentation Treatment Opalescent.the facultative bacteria utilizing the available oxygen and enhancing the living conditions for the strict anaerobesà likes to co-habitate • when bacteroides is present as a pathogen. Initially. organism spreads. altered mentationß GNR sepsis Green Nail frequently seen in patients distal onycholysis with a striking PNA & I&D Topical polymixin B and 1% acetic Syndrome whose hands and feet are often greenish discoloration. Pallidum Description Clinical Presentation Treatment eruptions are symmetrical. single intramuscular injection of 2. Other Cutaneous viral infections 49 . painful.drug of choice for all stages of syphilis.tetracycline or erythromycin orally 500 mg every six hours for 15 days a Jarisch-Herxheimer reaction may occur following the initial dose of penicillin. lesions on soles are highly suggestive of syphilis Characteristic Lesions • macular lesions < one centimeter in diameter. occasionally appear on the trunk. headache. symptoms absent or eruptions self-limiting and disappear spontaneously after a few days or weeks. distributed over face."shoddy" nodes Viral HPV Type 1 Type 2 Type 4 Description Characteristic lesions Treatment the lesions are very deep palmoplantar lesions and may causes an endophytic.000 units of penicillin G procaine penicillin allergic pts . slightly palpable with a deep firm infiltration distributed on the face and flexor surfaces of the arms and lower legs. most common presentation of all syphilids. the earliest eruptions is macular penicillin . flanks. develops rapidly and is pronounced a few days after onset. occurs six to eight hours after injection Podiatric Manifestations of syphilis Macular eruptions early macular eruption are "ham-colored macules" on the soles of the foot. sore throat. nodes are enlarged not tender possessing a hard. if positive followed by VDRL tests • generalized lymphadenopathy may be present. about 10% of immunodeficient patients.4 million units of penicillin G benzathine and 8 daily intramuscular injections of 600. punctate.Syphilis ß T. shoulders.6 wks after infection. may last a few hours or several months. fever. VDRL is most commonly utilized test • VDRL becomes positive w/i 5 . pruritis in 10 . early lesions are teeming with spirochetes.strongly reactive. • a positive STS (Serologic Tests for Syphilis) . papules range from 25 mm. consists of shaking chills. transient macular roseolas develop into maculopapular or papular eruptions. papular lesions .c of the papular eruptions is the lenticular papular syphilid. malaise. occurs in 90% of patients with seropositive primary or secondary syphilis. rubbery feel . light pink color ⇒ becomes brownish red depending on amt of pigmentation in the patient's skin.raw ham color or a coppery tint Development of Lesions • develops insidiously appearing six to eight weeks after the chancre. tachycardia and exacerbation of the inflammatory reaction at involved sites. fully developed lesions are ham or coppery colored or dusky yellowish red on the soles. palms and soles and anogenital regions. superficial. occasionally sequelae of macular syphilid ⇒ livedo reticularis Papular eruptions m. hyperkeratotic "pinpoint" bleeding seen upon debridement plantar warts. solitary wart which appears as a be described as “myrmecia” or type I or inclusion warts hyperplastic hyperkeratotic epidermal lesions extremely resistant to therapy.exquisite tenderness to direct touch or a papule with a blunt probe Diagnosis of Syphillis • Treponema pallidum isolated from skin lesions. remains highly positive throughout the secondary phase although HIV+ infection patients may be seronegative • RPR may be utilized. mosaic warts or Topical acid preparations papillomaviridiae can transform from a benign wart into a SCC "group" warts Effudex-mosiac flat wartts capillary loops of the dermis elongate and are responsible for the causes smaller. multiple. myalgia.20%. Ollendorf's sign present . affecting the muscles of the involved neurotome (for example.an enterovirus tongue. onset is rapid with fever and neuralgic pain after an Lesion incubation period of 7-12 days. it occurs substrate for and secondary to skin to skin contact during the sport inhibitor of herpes • HSV-2 is frequently more severe and assoc w/ venereal transmission virus DNA polymerase • 80 -90% of pts presenting with a first episode of genital herpes present with the HSV-2 strain Dose: 200 mg orally 5x daily for 7-10d HSV and herpes zoster are vesiculating skin lesion associated with cutaneous nerves and 1/3 of Erythema Multiforme is caused by herpes virus Fungal Hand-FootAnd-Mouth Disease 50 .1 is frequently a less severe and less frequency recurring stain of the virusà “cold sore” txà it is a selective a presentation of herpes simplex. Tx: acyclovir Herpes it occurs most frequently at or near a mucocutaneous jx and in the course of a few days. some become hemorrhagic or necrotic and may ulcerate and sloughà bullous lesions have been reported in pts wi/ CA Lab Findings in all Tzanck smear shows multinucleated giant cells and within and at the sides of the vesicles in a herpes eruption are large. the lesions oral or IV acyclovir Simplex rupture and the dried serum forms a flaky crust w/ Concomitant Regional Lymphadenopathy (Zovirax®) appears to Virus the lesions tend to heal w/oscarring and two recognized strains: HSV-1 and HSV-2 be the only effective HSV. may occur often in young wrestlers. and prodromal symptoms are typically reported as anorexia. which is available in 0. every eight hours for seven days local applications of heat are recommended. and acidophilic inclusion bodies are seen within the nucleus of the cells Mitigating Factors in Stress/emotional factors may induce the virus to proceed along the course of a peripheral nerve to terminate. soft palate and gingivaeà lesions on the hands and feet begins as red papules. the prickle cell layer (stratum spinosum) contains cytoplasmic inclusions referred to as molluscum bodies (or HendersonPaterson bodies) Treament Best accomplished by curettage and if the lesion rests on a particularly sensitive location.o. it is more convenient Dosage: 1000 mg p.oral prednisone is acceptable but anti-viral medications are more likely to shorten the severity and duration of the attack acyclovir Valtrex ® Valacyclovir Other Name Molluscum Contagiosum Description Characteristic Lesion caused by members of the pox virus group and the papules range from 2 to 5 mm in diameter. local infiltration of an anesthetic agent may precede curettage. and administration of analgesics may be useful to diminish the symptoms associated with the acute neuralgia • systemic administration is extremely useful in patients over 50 . herpes gladiatorum. and patients should also experience less painà dosage: 800 mg by mouth five times daily for seven to ten days Undergoes biotransformation to acyclovir. tenderness or pain.75% strengths available OTC and utilized in the initial txt of post herpetic painà2nd line tx would include gabapentin followed by amytriptylline Characteristic several groups of vesicles on an erythematous and edematous base. resolving in 5 to 7 by Coxsackie virus type A days Characteristic Lesions: small vesicles that are surrounded by a red halo on the buccal mucosa. epidermal All Herpes Attacks manifestations. and it is more commonly referred to as "shingles" Physical Findings hyperesthesia to light touch typically occurs throughout the involved neurotome. and the eruption usually appears in patches of various sizes as the vesicles dry up. Its advantage over acyclovir is that although it is more expensive. are unbilicated and contain a caseous plug histologically. malaise and low grade fever Treatment Famvir Undergoes biotransformation into the active metabolite penciclovir which is believed to inhibit viral DNA synthesis and ® Famciclovir Zovirax – ® replication: dosage: 500 mg by mouth every eight hours for seven days treatment should be initiated within the first forty-eight hours of the onset of the attackà reduces the duration of the lesions.25 and 0. Other forms of tx may use of topical salicylic acid and CO2 laser ablation this disease process is Course of Disease: viral implantation generally occurs on buccal mucosa spreading to the regional produced most frequently lymph node w/i 24 hrsà incubation is generally 3. 16 .Varicella Zoster HSV-III Description Eruption is generally accompanied by local itching. HHV Attacks swollen cells called "balloon cells"à these balloon cells represent degenerated and swollen prickle cells.6 d. and abdominal m weakness with T10 to L1 involvement) attacks vary widely in their intensity but rarely last for more 3-4wks post zoster neuralgia may occur in 10 to 25% of older individuals when they do not receive systemic corticosteroid therapy : capsaicin cream. muscle weakness may accompany an outbreak. prodrome is self-limited. quadriceps weakness may occur w/ L2 involvement. Persistent Miconazole infection may be deforming Sulfones and sulfonamides 51 . mentagrophytes Discolored nail plates. May be hypo or hyperpigmented stratum corneum Topical cyclosporix olamine Brown to black macular lesions Mold visible in stratum corneum Surgical or chemical intervention on palmar and plantar Hard. Mentagraph. Wart-like growths and cauliflower-like nodules. verucosum Pruritic follicultis KOH DTM/ Sabouraud and Dextrose T. white. minutissimum Ketoconazole an dHygiene T. SabouraudTopical allyamines T. Pedis Chronic KOH prep of skin scrapings Topical imidazoles. Involves the hair shaft only Griseofulvin Topical antifungals Soft. Systemic itraconazole Amphitericin B for disseminated cases Pathology Tinea or Pityriasis versicolor Tinea Nigra Black Piedra White Piedra Chromoblastomycosis Eumycotic mycetoma Differential dx: Bacterial derived eubacterial actimycotic mycetoma Small isolated lesions: Cryosurgery Larger lesions: Itraconazole. Capitis T. scaly inflamed. Lymphadenopathy and abscesses along lymphatic tracts Psedallescherichia boydii (USA) Philaphora species Madurella mycetomatis and M. Hair breakage or loss . eyes. Corporis Microsporum Round to oval erythrematous rings KOH prep of skin scrapings Focal: Topical imadizoles and “Ringworm” canis (zoophilic) or patches. lungs. firbrosis and pedrosoicongestion. common foot/Dry Ti. AmphitericinB Ketoconazole. Unguim T. Albicans Yeast Maceration does not spare the folds KOH DTM/ Sabouraud and Dextrose Superficial mycosis Etiology Epidemiology Malassezia Furfur Exophiala wernicki Piedraia hortae Trichosporon beigelii Clinical Manifestation Treatment Prevention Most lesions are on the upper body: Limited to the Selenium sulfide shampoos and scrubs Fine scaly lesions with or w/o pruritis superficial Oral ketokonazole. Maceration Dextrose Agar Steroids. mentagrophytes T. Tolfnaftate. can extend to the underlying KOH “onychomycosis” E. Topical imidazole. floccosum C. grisea. floccosum and discolored. Large tumor No established protocollike lesionsà draining sinuses Surgery + itroconazole yielding granular pus. Tonserans May be vesicular or pustular And oral ketoconazole + topicals T. rubrum –most hyperkaratoticà moccasin Culture: DTM. Albicans nail matrix and nail bedà onycholysis DTM/ Sabouraud and Dextrose Diaper rash “Napkin rash” C. Sabouraudand allyamines Dextrose Agar T. autoinoculated from feet KOH prep of skin scrapings Topical imadizoles Tonserans Kerion-boggy lesion that may b/c superinfected Culture: DTM. Pedis Trichophyton Interdigital T. Barbae “Beard itch” T. 5Flourocytosine Madura foot or hands. scalp hair shaft. black nodules. Pedis Vesicularà Dextrose Agar Bacteriologic Oral Griseofulvin Dermatophytid media to r/o C. Painless papule developing after the traumatic inoculationà ulceration. Albicans when moist severe disease T. and CNS Fonsecaea May result in elephantiasisß lymphadenitis. Rubrum T. borders with quiescent centers Dextrose Agar Disseminated: Oral griseofulvin T. Cruris T. SabouraudTolnafate. Undecylenic acid E. mentagrophytes Centrally papular or pustalar Culture: DTM. Oral fluconazole for C. SabouraudTopical &oral allylamines.Cutaneous mycosis Etiology Clinical Manifestation Laboratory Diagnosis Treatment Epidemiology Prevention T. Rubrum T. Rubrum Pruritic rash KOH prep of skin scrapings Topical: imadizoles allyamines “Jock itch” T. Culture: DTM. Acremonium spp Treatment Prevention Potassium iodide for simple cases. Ciclosporix Olamine. Lymphangitis and lymphadenopathy Lymphotogenous spread to the joints. Raised.brown nodules on hair shafts Involves the hair shaft only Griseofulvin Topical antifungals Subcutaneous mycosis Sporotrichosis Spororothrix Schenkii Saprophyte Etiology Epidemiology Clinical Manifestation Asymptomatic infections. malaise.N. Eczematous Type Urticaria Eczema Specific Allergic and Reactive Dermatoses Description/Cause Characteristic Lesion Clinical Presentation Treatment acute urticaria is generally a type I hypersensitivity it is characterized by the most cases of urticaria are self-limited systematic treatment reaction histamine-mediated by mast cell activation – classic wheal and flare and of short duration in extremely can include fixed IgE common offending agents include latex gloves. Meningitis liver and other viscera Amphitericin B + (synergistic) Flucytosine (granulomatous)Benign pulmonary Thermally dimorphic Itraconazole. Clinical Manifestation Systemic Mycosis Laboratory Diagnosis Cryptococcosis “Cryptococcus Neoformans” Histoplasmosis Histoplasma Capsulatum Paracoccidiomycosis S American Blastomycosis Asso: bat and starling guano. Pentamidine + prednisone. tobacco and alcohol 52 . itraconazole/ Ketoconazole Pnemocytosis Only in neonates & Interstitial pneumonia Trimethoprim + sulfamethoxazole + prednisone “Pnemocystic immunosuppressed IN AIDS-pneumonitis Clindamycin & Primaquine. Sulfamethoxazole & Pentamidine 3. Blood Nephritis. American Blastomycosis dermatiditis” Coccidioidomycosis “Coccidiomycosis” Etiology Epidemiology Transmitted via inhalation and contaminated soil SW: Valley fever or Dessert rheumatism Transmitted via dust particles Pigeons are the vectors. IV Ketoconazole and pleuritic pain. Amphitericin B with lymphatogenous spread to Thick walled yeasts with via IV Ketoconazole mucosa of the mouth. Itraconzaole. Amphitericin B via Delayed hypersen. Severe infection: Fluconazole. Molds and dust are inhaled. Can disseminate to the skin. Amphitericin B + (synergistic) Flucytosine Disseminated: Granulomas. fatigue or glucoeczematous eruptions is parched or burned àmoderate pruritis overexertion. skins. Fluconazole joints. blastoconidia –ship’s wheel GI. liver and other viscera. Amd Erythema multiforme or Severe Menigitis: nodosum. Amphitericin B via IV Ketoconazole *Amphotericin B is widely used for fungal meningitis and is synergistic along with Flucytosine Etiology Epidemiology Clinical Manifestation Treatment Prevention Aspergillosis 2nd most common Mycotoxicosis: Hepatotoxicity. walled blastoconidia mucosa testis. HIV prophylaxis: disease foamy exudate Trimethoprim + Dapsone. Latin America Dust and inhalation M>F Granulomatous and suppurative Yeast-broad based thick disease in the lungs. nose and multiple narrow-based URT. bones. Amphitericin B Lymphadenopathy and Splenopathy Spiny microconidia via IV Ketoconazole Disseminated RES diseaseà infects Knobby macroconidia macrophages: CNS fatal meningitis Granulomatous lesions in the lungs. adrenals. reaction. and CNS. Pneumonitis proceeds dissemination Non-meningeal to CNS-meningitis. Sequelaeà persistent arthralgia. epididymis and prostate glands Incub: 1-3 weeks Symptomatic: Fever. Disseminated pulmonary or meningitis: myalgis. blisters. no pus Itraconazole Can disseminates also to skin. Treatment Prevention Itraconazole. endocarditis. Children: Metronidazole. Thermally dimorphic Itraconazole. dry cough (10-21 days) Fluconazole. tea. the acute stage: vesicles. very pruritic symptomatic areas of cutaneous administration of oral animal proteins and food stuffs edema antihistamines histologically. bullaeàsevere pruritis emotional factors and psychologic stress Topical hallmark of all subacute stage:redness. fumigasà Nosocomial infections hepatocellular CA rifampin: itraconazole. and aflavotoxins Cases predisposed by Allergic Aspergillosis Voiconazole underlying respiratory disease Aspergilloma-Fungus ball Aspergilloma: Itraconazole and/or surgery or persons on anti-neoplastic Invasive Aspergillosis-necrotizing Allergic aspergillosis:Corticosteroids and antibitotic agents pneumonia Candidiasis Most common fungal human Superficialà Oropharangeal/Esophageal: Clotrimazole troches (lozenges) or “Monaliasis” pathogen and the most Disseminated Nystatin washes. of the skin. hepatitis. Caspofungin. frequently isolated. bone. skin is exacerbate all forms of eczema. and Invasive Aspergillosis Amphitericin B + “A. Dermatoses A. and Caspofungin for refractory monoliasisSystemicAmphitericin B and surgery DIC Oral thrush + Flucytosine. bones. gustatory factors such as corticoids spongiosis chronic stage: lichenification àmoderately pruritic caffeine. Ketoconazole. scaling and fissuring. Itraconazole. cystic gelatinous Amphitericin B + (synergistic) Flucytosine masses in the gray matter. Trimethoprim & Dapsone carinii” AIDS defining and pneumonia with Atavaquone. Amphitericin B “swish and swallow” borne-IV devices or drug use pneumonia. warty nodules may be dome-shaped in appearance An inflammatory Assoc w/ streptococci (a URT infection in the onset of an attack is typically heralded by (PRODROME) Tx underlying process resulting in children).Type Pompholyx Dyshidrotic Eczema – Dyshidrosis Lamellar dyshidrosis – Keratolysis Exfoliativa Dyshidroses derived from the fact that the skin may be red and wet with perspiration Description Characteristic Lesion Clinical Presentation Treatment it may occur in intraepidermal vesicular the symptoms characteristically consist of pruritis and corticosteroids.to 48-hour occlusion with polyethylene or Unna-boot are the treatment of choice for the rapid resolution of asteatotic dermatitis. hyperhidrotic eruption localized to the palms. Syphilis. Dermatophytoses Psoriasis Description chronic. during the 1st week. scaly and flexure) as well as the back exacerbate the elevated with accentuated and sides of the neck symptoms skin lines this is considered a variant of it may present as a single or multiple lesion frequently on the LE on the extensor surfaces of the cryosurgery or intralesional lichen simplex chronicus that are generally “pea-sized” but may be as feet. having a PCN of the larger cutaneous tender. very fine superficial cracking and fine flakes (ddx: Keratolysis Exfoliativa) that it is very pruriticàmost commonly it is occurs on the anteriolateral aspects of the shins in the elderly population LE are the most common siteà the symptoms include severe paroxysma pruritis à lesions tend to become colonized with Staphylococcus aureusà the lesions may become lichenified Treatment topical corticosteroid ointments with 24. smooth nodules that are usually tendency to occur in “crops” they undergo slow involution erythromycin to tx b/v symmetrically distributed on the anterior shins and resemble an ecchymotic bruise the Strep B. circumscribed. firm. 1 – 2% of the population have psoriasis. it will appear as tiny pinhead-sized white spots that 5% tar in gel (Estar Gel absence of any exfoliative tend to spread peripherally and as the spots spread. lesions generally persist for 1-2wks. affect all races – uncommon in blacks. albicans 53 . Woronoff ring – concentric blanching of the erythematous skin at the periphery of a healing psoriatic plaque. tendency to persist. Papulosquamous Common papulosquamous diseases that occur in the foot: Psoriasis. the mild emollient usually inflammatory changes dermatosis central horny layer ruptures and peels off and the skin may suffices and small dose it may represent a affecting the take on a flaky appearance from desquamation of the – 20 to 30 mg – of disorder of adhesion of palms and superficial stratum corneum à must r/o Tenia triamcinolone acetonide the stratum corneum soles (Kenalog ) IM  Dishydroses continued Type Asteototic Dermatitis xerotic eczema. Pityriasis rosea. seborrehic dermatitis and superficial BCC Incidence M=F onset – 27yrs. scaling grayish white or silvery white. and cat-scratch fever. erythematous. drug rx and sarcoidosis may also be constitutional symptoms . etc. indurated and when the disease process becomes chronic. antibiotics may be indicated if bacterial infection ensues with empiric initiation of antistaphylocccal spectrum drugs • mild sedatives and tranquilizers may help with the itching (ddx: lichen simplex chronicus) intralesional injection of triamcinolone suspension is quite effective done only if there are no signs of secondary infection. it can be associated with the Koebner phenomenon resulting in multiple linear streaks the onset of this linear excoriations may be predilection for the extremities process is generally present with time. There is one variation – inverse psoriasis. elbows. ulcerations. stress and lesion becomes thickly ankles (particularly the ankle anxiety may tend to lichenified. Presentation Koebner reaction commonly seen as well with Nummular Eczema Auspitz sign occurs because of the severe thinning of the epidermis over the tips of the dermal papillae. knees and sacral region. western European or Scandinavian descent – 3%. the pruritis is the most prominent feature cortisone injections may be characterized by discrete or large as 2 cm in diameter and the nodules and is commonly paroxysmal effective multiple hyperkeratotic are erythematous or brownish. symmetrical bilaterally. occurs on the intertriginous areas. burning Transient: à lesions are often on the sides of the drying agent feet the process soles and interdigital spaces fingers and may be arranged in clusters or coalesce to form K+ does not involve à it is exacerbated by both bullae that are filled with clear to straw-colored permanganate the sweat ducts psychological/physical stress: fluidànegative for the presence of any fungi or bacteria erythromycin characterized by the superficial initially. AIDS. 25% of pts have arthritis involvement. Clinical The chief features of psoriasis include: tendency to recur. Characteristic rounded. extensor surfaces. inflame dz predilection for the scalp (50%). No bacteria present. high potency topicals such as betamethasone dipropionate (Diprolene ) or clobetasol propionate (Temovate ) may be used but only for initial therapy • occlusion is occasionally helpful with Cordran Tape   Lichen Simplex Chronicus neurodermatitis circumscripta F>M Prurigo Nodularis Erythema Nodosum young women during the spring or autumn clinical appearance of the coinshaped lesions often occurs concomitantly with venous stasis dermatitis. winter itch and eczema craquele Nummular Eczema M>F 60-70y/o discoid eczema Description dehydration of the skin secondary to a decrease in the surface lipids of the skin which leads to an increase in skin water loss generally presents with dry skin and is more common in the winter Characteristic Lesion xerotic eczema will show a condensed or “packed” appearance to the outer keratin layer Clinical Presentation skin will present with redness. Systemic causes (thyroid.) for dry skin should be ruled out in persistent cases potent topical steroid creams applied once or twice daily may be useful. imbricated and lamellar scales. Severe psoriasis linked with an increased risk of lymphoma and non-melanoma skin malignancies Characteristic nail numerous >10 pits 1 mm tan oval spots 2 to 4 mm in diameter – “oil spots” onycholysis heaped-up crusts accumulated beneath changes the free nail edges may become secondarily infected with C. malignancies. nails. indurated and tender –do not suppurate or develop into àsalicylates reaction w/involvement The generally fairly mild lesions begin as red. the – especially the wrists and insidious. Lesion nummular or coin-sized lesions common (dDx: nummular eczema. lesions are abnormality delayed hypersensitivity assoc w/Trichophytin ssp. Munro abscesses – (small collections of neutrophils in the stratum corneum). aggravated by – emotional stress (40%)of pts. if may often follow an infection of ulceration – eating and drinking becomes difficult if there is a poor response to herpes simplex or M. scaly erythroderma which may have an orange hue and on the several months in a dose of 1 scalp first. evolves to orangy-red or salmon-colored (yellowish-pink) scaly Carmol 40 isotretinoin is the affects all races plaques with well defined borders that are frequently interrupted by characteristic treatment of choice in adultislands of normal skin. and abrupt onset of a fever of 39 to 40° C. HA arthralgias: secondary lesions appear 2lotions. ulceration of the leg – not common. rough. lesions are helpful.000 to 300. alcoholism. 54 . vesicles on dose has been recommended RR and jt pains. etiologic factors – chronic alcohol abuse and contraceptive steroids are common. rapid erythematous. pinhead sized lesion topped by a central horny plug w/I which Hydrin 12% cream or male = female incidence there is a hair. on the palms and soles and w/I the psoriatic plaquesà S/s: pruritis and intense burning causes extreme discomfort. Rx – etritinate is the drug of choice Uncommon papulosquamous diseases that occur in the foot: Description Clinical Presentation Treatment Lichen Planus an inflammatory pruritic disease of the skin & mucous membranes hypertriamcinolone – 60 mg IM is safer and pigmentation . knees and elbows. 20% of pts have h/o diabetes mellitus and chronic occur secondary to blister formation. “pseudoporphyria” drug induced or acquired as ↑ accumulation -porphyrinogens PCT fairly common. – vitamin E – 400 units 2mild pruritis brittle and striated. Erythema Multiforme Bullosum – StevensJohnson Syndrome – Erythema Multiforme Major M> F 2:1 Porphyria Cutanea Tarda Bullous Epidermolysis Bullosa Acquisita occurs on the Large bullae on an erythematous base. pruritic cutaneous eruption. for erythema pruritis is generally often a prodrome of URT s/s and most mild cases are multiforme minor. pneumoniae not impossible prednisone after 3 to 4 days an acquired abnormality of include fragility of the skin and bulla formation of the sun-exposed areas(esp. fever. however may the hands & feet progress gradually to purpuric papules and petchiae – young adults children occur secondary to coxsackie B6 taking on the appearance of a bruise dermatologic signs may be it occurs most frequently in the and CMV. creams à pruritius . skin lesions sporadic. referred to as “keratodermic sandal Nail Changes dull. acquired immunobullous disorder → presence of antibodies against the type VII collagen within the basement membrane. which form a network over the surface of the papule mg retinoids DMARDS. appearance of palms and soles lesions coalesce to form a confluent hyperkeratotic – 2 mg/kg/day. there is measures and systemic corticosteroids. longitudinal grooving. i triamcinolone – 20 to 40 mg distribution virus 6 or HH7 pruritic 25% of the time is generally self-limited IM Papular Purpuric Gloves and most commonly associated with a symmetrical. inflammatory bowel disease and diabetes mellitus. Wickham’s striae.Generalized Pustular Psoriasis (von Zumbusch): severe & sometimes fatal. Target .5 to 2 constitutional s/s develop – these malaise and a sore throat: presence of an mg/kg/day of prednisone in one daily include a rapid. Location –the dorsum of the foot and shins Location – lesions occur in areas with a proclivity for minor trauma – hands. feet. generally 125 mg twice daily for three to six occur more commonly located on the buccal mucosa. rarely pitted (in contrast to psoriatic nails) C. 60 . vitamin A in exaggerated gooseflesh erythroderma and hyperkeratosis of the palms and soles has a tendency to doses of 150. characteristic lesion is a sub-epidermal bullae which develops at the dermal-epidermal junction. sharply demarcated. Follicular lesions eventually disappear and leave onset. a peculiar reported to be efficacious calcipotriene midline fissure may occur may also be effective Pityriasis rosea pruritic exanthema acute round or oval salmon-colored lesions which arrange rash irritated by soap so wash with onset lesions rapidly spread in a Christmas tree pattern on the trunk (Chevron water only B/c skin eruption makes the 15 and 40 and then spontaneously appearance. has also been associated accompanied by systemic symptoms of anorexia. subungual hyperkeratosis 3xd IM Kenalog. ridging and splitting. pterygium formation is dapsone and metronidazole (500 mg/day distinctive of LP. hands) porphyrin metabolism. sudden onset w/ formation of “lakes of pus” periungually. griseofulvin Lesions in the foot Characteristic Lesion – Oral Mucosa occurs in about 25% of patients.women on oral contraceptives. a short 2 to 3 week course of absent associated with an underlying infection of HSV prednisone – starting as 30-40 mg/day is often given a more severe and fatal version it is a mucocutaneous disorder heralded by the acute • remove the underlying etiology of erythema multiforme bullosa. distal yellow-brown discoloration. Characteristic Nail Changes: occurs in 10% of patients. Concommitant systemic therapy is generally limited to supportive palms and soles subungual hemorrhage has been reported. like Zoster)begins with a single herald skin extremely dry. and CA rarely develops. tongue and buccal mucosa and these lesions • triamcinolone acetonide may be early and conspicuous symptom progress to pseudomembranous exudation and administered 1 mg/kg IM dose. amyloidosis.80mg • splinter hemorrhages. May lead to obliteration of the entire nail and is generally a for up to 60 days) have both been permanent change. non-familial form mc b/t 40. weak pulse. thickened. early ccs tx is the DOC – up to 1.000 only subjective symptoms may be fissure. fever and arthralgias will spring and summer months with HHS-6 usually resolve spontaneously in 1-2 weeks no treatment Pityriasis Rubra Pilaris small follicular papules – most important diagnostic feature -emollient creams Lac reddish brown. episode triggered by iodides and salicylates. purpuric or tender edema & erythema of Socks Syndrome parvovirus B19. type I given over chronic skin disease often affect the widespread dry. this is more likely if the oral lesions ulcerate w/ oral mucosa erosion and ulceration.corticosteroid lesions affect the etiology unknown although malaise. headache. creams or sprays and trunk more often in a a viral origin is suspected 21d after the appearance of the herald patch and the antihistamines UVB sunlight may be dermatomal secondary to human herpes palms and soles are rarely affected. fetid odor exists. Koebner’s Elementary Lesion: possesses grayish puncta or more effective oral prednisone 40 to 50 Adults streaks.60. stomatitis in as the lips. associated with multiple myeloma. silvery-white pinhead sized papulesà mistaken for months has also been reported in cases on the sides leukoplakia . use emollient F>M disappear in 3-8wks à or mother patch and mild prodrome of fever. Vascular disorders (nodose. Hypersensitivity vasculitis. infarctive ulcers of varying sizes and livedo reticularis *TQ Multiple mononeuropathies are the most typical neurologic manifestation and occur in more than 50% of all patients. etc) Systemic Necrotizing Vasculitis Vasculitis PAN Periarteritis Nodosa Churg Strauss Clinical Manifestation Laboratory features Unknown etiology.Symptoms – pruritis. Erythema EKG may reveal ischemic pattern (high ST nodusum-like vasculitis lesion may occur over the legs. purpura. normochromic anemia young women of Asian descent. may remain stationary for associated with chronic renal failure and dialysis. (30% Hep B). BP difference segment. HTN in 40% and bruits.A allergic granulomatosis and angiitis.occurs secondary to a genetic keratoses mutation that affects desmosomal assembly and cell to . the palms and soles are generally phosphorus level. that is required to transport calcium within the cell . Liverà increased LFTs. mutation codes for the SERCA enzyme or develop into persistent yellowish brown.4. pulmonary infiltrates and CXRà patchy nodular vasculitis. abdomin and face -high dose ela tx of Retin-A and vitamin E may be benefit generally palliative with the use of emollient creams -dermabrasive creams (Retin-A) may sometimes be utilized Pyoderma Gangrenosum Systemic steroids B. Kyrle’s Disease hyperkeratosis follicularis M=F onset is b/t 30 – 60 yrs of age Darier’s Disease – Keratosis Follicularis M=F 4 y/oà heat and stress Systemic relationships A. Eosinophilia Involvement of the lungs Decerased renal funct Triad of: asthma. 2X common in men. Heart. Genodermatoses Central keratin plugs or cones that may develop in a widespread small papule with a central silvery scale distribution pattern. elevated years. particularly on the lower legs. infiltrates Cutaneous findings include petichiae. Elevated ESR and thrombocytosis idiopathic medial aortopathyà involving the AA. eosinophilia.Also known as pulseless disease.5 cm –often surrounded by a zone of comprised of necrotic cellular debris and degenerated c. disease breakdown of desmosome keratin intermediate filament -nail changes –very specific there are white attachment and perinuclear aggregates of keratin and red longitudinal bands & nail ridges. eosinophilic infiltrates Eosinophilic granulomatous involvement of the GI and urinary ddX: TB tracts or prostate is diagnostic Diagnosis Progressive & fatal asthma dDX: Loffler’s syndrome.characterized by hyperkeratotic papules in seborrheic also involve the feet. livedo reticularis. Constitutional s/s are present Characterized by focal inflammatory lesions. coronary arteritis à CHF A.desmosomes cannot assemble properly without adequate lesions is the major “morbidity” in the amounts of calcium. vasculitides.first appear as skin-colored papules which cell adhesion. pigmented purpuric eruptions. DM alcoholic cirrhosis. The cone projects into dermis & is >1. pump (SarcoEndoplasmic Reticulum Calcium-ATPase) greasy papules. T wave inversion **TQ) of >10 mmHG are found in most patients. Nervesà diffuse polyneuropathy Kidney àRenovascular HTN GIà acute abdomen with hematemesis or melena. or ulcerations Biopsyà extramural Peripheral multiple mononeuropathies microgranulomas with Abdominal involvement. congestive heart spared failure.t m.The lungs are usually spared Cutaneous manifestations are palpable purpura. but appearance of . lesions.c erythema. or may occur alone dominantly inherited genodermatosis -lesions on hands are most common but may . Giant cell (medium to large vessels) arteritis Clinical Manifestation Laboratory features Diagnosis Treatment Chronic vasculitis of the aorta and its branches and is most common in Normocytic. resultant loss of desmosomes.K. CXR-widening of the thoracic aorta Temporal arteritis It is associated with polymyalgia rheumatica (PMR) The ESR tends to be +biopsiesà mural CorticoVasculitis Takayasu’s arteritis 55 . Wegener’s Treatment Prednisone 1mg/kg day Prednisone and cyclophosp hamide (2mg/kg/da y) More responsive to corticosteroids than PAN Eosinophilic All vasculitides weaken the vessel wall and can cause dilatation and aneurysm formation. a Vintermediate filaments shaped nick at the free margin of the nail is the most pathognomonic nail finding A rare disease frequently assoc w/ GI Consists of large ulcers with characteristic purple diseases (ulcerative colitis or Crohn’s overhanging edges which develop rapidly from pustules and dz) Exact etiology is unknown tender nodules. small pits on the palms regions & various nail abnormalities and soles are very characteristic – punctuate -chronic disease process. GI.K. Proteinnuria. Prednisone and Granulo-matosis It is often considered a triad of necrotizing granulomatous Biopsies show leukocytoclastic vasculitis cyclophosphamide vasculitis of the tissues of the lower respiratory tract + ANCA à pneumocystis carinii *TQ. diplopia. and rifampin) indolent Other Vasospastic diseases Description Unknown etiology-caused by dysfunction of the SNS causing excessive vasomotor tone of the arterioles at ordinary room temp Painless but peripheral coldness and cyanosis/blueness of the distal extremities. Conservative tx: protect from cold. polyarteritis of varying severity and multi-system involvement. visual loss. lymphangitis. Palpable purpura is the m. cough.dDx: Wegener’s Takayasu’s >50yrs. occasionally the face and ears Diagnosis/ Treatment Female patient with persistent coldness and cyanosis of the fingers and hands of many years duration. biopsies may be helpful 25% of cases have been associated with lympholenticular neoplasms. Wegener’s Granulomatosis HSP Know causes of palpable purpura 1. fever. Hypersensitivity vasculitis Vasculitis CNS Vasculitis – Clinical Features Other Vasculitides Clinical Manifestation Laboratory features Diagnosis Treatment Uncommon Men > women. condition) Patients present with abdominal pain due to GI ischemia. Joints. red or violacous flat-topped papuples. SLE. clinical manifestation. across the buttocks. Iodine Rx to medication that occurs in 7-14 days à serum sickness reactions It is a hypersensitivity vasculitis. tenderness over the temples. fragmented IEL. sore throat. lymphoma dDX: Behcet’s syndrome. and Kidneys. strep) therapy Erythema Acute immune-mediated vasculitis characterized by cutaneous and subcutaneous Ddx: Associated with TB antimicrobial Induratum lesion on the posterior aspect of the lower legs. (Given a case about a boy…had previous. PAN 2. Erythema It’s an acute vasculitis of immune-mediated origin characterized by inflammatory nonAntecedent exposures to a NSAIDS. Purpura. Nasal pr oral mucosal ulcerations are early (Classic) of Wegener’s findings. High LFTs loss usually results from retinal ischemia secondary to Elevated factor 8 and involvement of the ophthalmic or posterior ciliary arteries. (westergren and the aortic arch syndrome (pulseless disease). sulfa or infections (fungi. lymphadenopathy. PAN . topical nodosum suppurative cutaneous and subcutaneous lesion of the extremities. Affects young pts (18y/o or younger) It affects small vessels of the skin. and extensor surfaces of the UE. toxin or drug (ASA.c. or illicit drug use Hodgkin’s lymphoma Wegener’s Uncommon More common in young men. painful. lymphadenopathy. and TB. and HIV especially if they have a infection. Classically lesions are located on the anterior lower leg. and cranial nerve HSV infection. Visual High CRP. Focal neurological deficits. Very little is known of the pathogenesis. Treatment: eliminate the cause prednisone. Resp. optionally sympathectoy when severe. W>M Common manifestations of vascular high averaging about involvement include HA. tender. but Dx is difficult Angiograms. and rash involving the extensor surface of the lower legs and feet. (“Classic”).A musculocutaneous lymph node syndrome (MLNS)*TQ It affects infants and children < 5y/o and consists of skin and mucous membrane eruptions. 80-100 mm/ hr jaw claudication. May need to biopsy contralateral side steriods à most effective Prednisone Vasculitis Hyper-sensitivity Vasculitis HSP Clinical Manifestation Treatment Reactions to drugs are the most common Onset is usually abrupt and occurs after exposure to the etiologic agent. glomerulonephritis. therapgy (isonizid The lesions are often ulcerated deep-seated nodules red to bluish. Churgnodules. involvement are common. Pain over the sinus areas and purulent episodes generally a complication of bloody nasal discharge are typical upper respiratory CXR reveals cavitations and infiltrates immunosuppresion. venules) lymphocyes macrophages. giant cells. IL-6 *Do not use ccs for very longà (-) HPA axis and can cause Cushionoid syndrome Hypersensitivity Vasculitides (small vessels-aterioles. purpura and ulcerations Strauss. hematuria. Consider a pt w/CNS s/s but are difficult to perform also been associated with herpes zoster. Kawasaki Disease A. antimicrobial They are pink. small doses of reserpine and guanethedine Acrocyanosis 56 . Males> females. HTLV-III virus. Destructive changes lead to a saddle-nose dDx: fungal (histoblasma and deformity. Cutaneous findings include blastomycosis) /bacterial infections. and focal and segmental glomerulonephritis. RCB casts. Transient arthralgias frequent. PCN systemic steroids. and systemic symptoms may be present Rx to medication that occurs quickly à PCN. It is preceded by an acute respiratory condition *TQ. seizures. not just symptoms. Usually associated with thromboembolic pneumonia (PCP) *TQ arthritis also. Mobility. Pts respond to discomfort when the feet are covered while sleeping. blister or shallow crater Stage III: a FT skin loss.abnormal gl tolerance curve is Treatment àintralesional injection of triamcinoloneà be careful of fat pat atropy àdipyridamole (Persantine®) 225 mg and ASA1g/dà tx must be continued for up to 3-4 mo b/f results are achieved 57 . moisture or friction forces 1989. and Infection The combination of this triad leads to gangrene and amputation. Peripheral arterial disease (PAD). a superficial ulcer presents as abrasion. loss of adequate muscular calf-pump action and obstruction ie DVT Large pitting edematous exteremities with variable hemosidderin deposition and leakage of serosanguinous fluid. atrophic patches. gangrene. Tx: ASA and methlysergide C. reddish blue discoloration More intense mottling. 1st s/sà dilated long saphenous vein +varicosities and telengectasias Compression therapy.5 Arterial None Prominent veins without Edema (very light) 8 – 15mmHg (light) 16 – 20mmHg Lipodermatosclerosis (Class 2) 30 – 40mmHg Venous edema Lymphedema (Class 1) 20 – 30mmHg (Class 3) Over 40mmHg Arterial ulcers Description Secondary to ischemia via P VD. CVI. pre-ulcerative state Stage II:a partial thickness skin loss of epidermis and/or dermis. lesions must be revascularized to heal. occurrence of lesions in DM does not depressed.Livedo Reticularis Intensified by the cold and relieved by warming Types: Cutis memorata Livedo reticularis idiopathica Livedo reticularis sympathomatica Amantidine HCL à a benign form and is not a reason for stopping the medication (anti-viral) Skin of the hands and feet become very red and warm. prophylactic surgery D. waxy. DM. disappears 2-6 weeks after the drug is discontinued Primary or seconday: Primary occurs in myeloproliferative disorders. HTN. Off-loading. persists despite change in temperature Mottling persists.c in young (2-3 hemosidderin deposition decade) F> ½ the pts with this dz have DM. yellow. there is evidence of some other disease that affects the vascular system (SLE) It usually affects both sexes and pedal edema may occur. Daignosis Norton Scale Assess 5 parametersàPhysical condition.correlate w/ [gl] control in DMàoccurs in brown. ContinenceßScored 1 – 4 High Risk 16 Treatment Off loading Venous Ulcers Description Clinical history Treatment Dysfunction of the valves in the superficial to deep venous systems. Clinical Presentation Punched out lytic lesion. ABI Diagnosis Therapy 8. SLE and RA.5-0.0-1. Mental condition. (However. Necrobiosis Lipoidica Diabeticorum (NLD) Metabolic disorders Diabetic Dermopathies Description/Classic Lesion Occurrence well circumscribed.8 Mixed Low compression <0. cyanotic surrounding with no evidence of inflammation. the National Pressure Ulcer Advisory Panel – NPUAP Stage I: non-blanchable erythema of intact skin. and arterial ulcer are the three situations when you MUST refer the patient to a vascular surgeon. Activity. assoc w/ burning and itching. tendons or jt capsules. Smoking is the worse preventable risk factor associated with occlusive arterial disease. damage or necrosis of subQ tissue. 3/1000 DM patients. 3x m. Rest pain. hard. Erythromelalgia Mottling and blotching of the skin of the extremities. bone. tissue necrosis or damage to muscle. Intermittent claudication and rest pain are the two classic symptoms of arterial peripheral vascular disease (PVD).0 Venous High compression 0. Description Classification Ulcers Pressure ulcers Ulcers from excess pressure or shear. Treatment Revascularization Diabetic ulcers Description Treatment Major Risk Factors: Peripheral neuropathy (PN). extend down to – but not through underlying fascia Stage IV: Ft skin loss with extensive destruction. remaining ½. both PN and PAD can be independent factors that lead to amputation). lower extremely painful part of the leg. characteristic lesions – firm. Hb must be maintained above 10g/100ml for ulcers to heal associated ulcers occur both in the homozygous and heterozygous forms w/ varying degrees of severity. t. red. excess GF. histologically demonstrate RBC become eroded.. polycythemia vera . in intermediate forms of thalassemia. variety. Location . they can occur anywhere on the body Sickle Cell Anemia Thalassemia hereditary spherocytosis Pretibial Myxedema Pyoderma Gangrenosa 58 . occurs most commonly in the axillae.c cause. ↑ risk for thrombophlebitis & ulcers act like a characteristic of venous stasis and venous insufficiency occurs in 3% of patients with toxic diffuse goiters. m. simply defined edges and bases with granulation tissue. scaly papules to macules retracted atrophic scars present on the shins. empyema and chronic debilitating infections. . and the interdigital areas and particularly susceptible 2o to the glysosylation are also useful as well and nail folds are frequently affectedàthese of collagen à brittle skin Nail Changes: a Naftin gel. nodules or plaques. non-pitting. where the pidermis assumes a serrated and papillomatoses configuration will usually resolve with identification and treatment of the underlying systemic malady Lac-ydrin 12% lactic creams may help soften lesions.c cutaneous manifestation of diabetes M>F 2:1à hyperpigmentation and topped.c. halo gastric ulcer. imidazole AB susceptible . axillae and groinsà occurs in obesity with or without endocrine disorders.due secondary to regional enteritis. 25 – appear to be of the venous stasis type ulcer. and the bullae are from several mm to several cm in diameter. hydroxychlorogquine and niacinamide early lesions are round to oval.*TQ insulin resistance is the marker possible association with malignant neoplasm Type I • may precede (18%) or accompany (60%) or follow (22%) internal cancer • most commonly associated with adenocarcinoma – especially of the GI tract – most Malignant Acanthosis Nigricans often stomach Type II Familial Acanthosis nigricans extremely rare and is present at birth. active border deeply chronic ulcerative colitis. the lesions eventually extravasion and capillary basement membrane thickening and lesions clear or heal with epidermal atrophy demonstrate impaired healing and post-inflammatory hyperpigmentation. papules lateral/ dorsal surfaces of the hands and feet isotretinoin. Carmol-40% urea cream may also be effective. Tx: after rupturing or de-roofing. ring of small. in thalassemia minor. central portion – necrotic/ulcerated. Marker for heterogenous group of endocrine disorders . ulcers never occur. IM triamcinolone eruptions may be quite pruriticàthe candidal paronychia results in chronic fourth interdigital space is most inflammation of the nail fold and is common in commonly involved the diabetic Acanthosis Nigricans warty pigmentation of the skin. presents as a grayish.c on the àthe disseminated variety may respond to dapsone. borders begin as erythematous papulonodules → enlarge rapidly rolled and vegetating.d. thalassemia major (Cooley’s anemia) leg ulcers uncommon. there is little or no pain or discomfort.leg ulcers in 5% of patients. daily application of Retin-A cream of gelàoral isotretinoin or Accutane can be used but the lesions recur when treatment is stopped chronic leg ulcers are very common. considered pathognomonic for underlying into plaques 2 -12cm diameter. irregular swellings.c • intra-lesional injections of triamcinolone acetonide colored or red in young adult females and it is m.Generalized Granuloma Annulare similar to NLD Diabetic Dermopathy (Shin Spots) Diabetic Bullosis or Bullous Diabeticorum Periungual Erythema Carotenodermia Candida Infections often present àpentoxyfylline (Trental®) 400 mg. Location – lower leg.side of the neck. presence of acrochordons (skin tags) • histologically resembles verruca.b/c no hemolytic process. lesions of erythema surrounds the lesions. fleshpast middle age. can occur in the flexural creases of the lower extremity.i. hyperkeratosis and acanthosis together with a mild degree of hyperpigmentation • dermal papillae are evenly elongated. occurs in various well-recognized insulin resistant states Description Types Characteristics Clinical Appearance Treatment greater insulin resistance and raised insulin levels type A syndrome – linked w/genetic defects in receptor fxor post receptor pathwaysàmay show hyperandrogenism type B syndrome – circulating antibodies to the insulin receptor. may or may not be assoc w/ DM. the DM pt Castellani . flatthe m.< 2% develop ulcers. usually around the 75% of patients have leg ulcers.↑ blood viscosity & ↑ platelet numbers. inherited in an autosomal dominant manner Type III M. symmetrically distributedà hyperpigmented (2o to hyperkeratosis and not actual melanin deposition) elevated lesions. and or hyperpigmentation these lesions are present in up to 50% of all DM pts a non-inflammatory blistering which occurs spontaneously in a fairly symmetrical distributionà lesions usually develop overnight abruptly is DM pts – usually on the feet and lower legs. flesh colored & waxy. ulcers heal very quickly following a splenectomy.c in F lesions are generally asymptomatic firm. ulcerations – uncommon characterized by acute necrotizing ulcerations of the leg. these bullae leave a deep painless ulcer àlesions will resolve spontaneously over a period of 2-5 weeks usually without scarring dilation of the capillary loops secondary to microvascular disease yellowish discoloration of the skin w of the 50% Dm who have if the yellow pigmentation persists after greatest intensity on the palms and carotenemia-10% have carotenosis carrots are omitted. velvety thickening of the skin. highly resistant to common in older age group healing. . more malleoli. localized form occurs m. violaceous red. the patient should also solesàthe sclera remain ddx: commonly associated with refrain from ingesting excessive amounts of jaundice there may be night blindness hypoTH other colored fruits and vegetables intertriginous areas are particularly Candida albicans is the m. associated with burn scars and other pre-existing lesions. dysplasia of skin cells in the chronic wound develops resulting in the development of squamous cell carcinoma 59 .Marjolin’s Ulcer refers to malignant changes that occur in a chronic non-healing wound. pimecrolimus (Elidel ) lesions with secondary changes such as this does not affect the sensitization phase of the lichenification. appear • there are some reports that plasmaphoresis may be useful – plasma exchange within the blister fluid should prove beneficial by removing the offending drug or its metabolites 60 . penicillamine. circinate the skin but in the or iris-shaped symptoms may include urticaria. pruritis may be persistent mucous membranes evolution is rapid. As many as 1/3 of all cases may be associated with lower elanoma it the herpes virus. annular. use the rule of nines it is differentiated from scalded skin syndrome • treatment of the skin lesions is essentially that of treatment of second degree (which is caused by staphylococcus) by the burns. arms. a fullpetrolatum thickness slough of the epidermis • silvadene (silver sulfadiazine) cream is also however.collagen vascular diseases . excoriation and fissuring effector phase Non“housewife’s eczema typically the patient clinical consequences are desiccation. photosensitivity. hydralazine. drug-induced reaction advantages of steroid are outweigh be its drawbacks to rapidly calculate the amount of total body surface involved. mumps. vesicular or bullous. coxsackie viruses. vasculitis and questionable efficacy s eruption vesticulobullous reactions ® ® a type IV delayed hypersensitivity reaction allergic dermatitis is mediated by the Langerhans cells. the supposed independent. barbiturates. viral infections – may be the most predilection in the frequently encountered etiology. involvement. elanom. dose• once a large area of the dermis is uncovered (greater than 20%). but may also occur with the Asian flu. Histopathology demonstrates acute inflammation in the outer Irritant in individuals who do itching papules or most layers of the dermis with extension into the epidermis and healing generally occurs Dermatitis “wet work vesicles with no residual scars. an underlying antigenic stimulus should be suspected 20% of all cases occur in children Toxic Epidermal Necrolysis – “Lyell’s Syndrome” Characteristic Lesion Clinical Presentation Treatment manifests not only in the lesions may be macular. steroids should be withheld because it is generally skin that is so severe that it has the appearance agreed that the benefits of steroid therapy – if any – would be observed only in of a widespread scalding burn the early stages of a slowly evolving case of TEN it is believed to be an idiosyncratic. include the legs and measles. preservatives. iodides. dilantin. gold salts. topical medications and occupational agents identification of the offending agent!!! Patch testing acuteà short course of systemic corticosteroids (Medrol) and oral anti-histamine chronicà the above including topical NSAIDS acute eruption generally results about 48 that work by inhibiting calcineurin which is an hours after exposureà lesions may have important mediator of interleukin induction and characteristic linear or geographic forms T-cell activation ie lesions are edematous red papules 1. legs and feet – the offending drug drugs presentation palms and soles are generally spared systemic corticosteroids is the eruptions may last for one to two weeks may be used in severe morbilliform less common presentations include: acenform (dDX: acne vulgaris). pigment changes. scaling and fissuring which then progresses to an Allergic or ccurs most frequently will present with eczematous dermatitis. when it is applied over large open areas. tacrolimus (Protopic )ß inhibit expression developing into vesicles and bullae when the of high-affinity receptors for igE and dendritic reaction is severe enough cell presentation of antigens to lymphocytes chronic exposure results in minimal primary 2. mercurials. phenytoin. bromides. cases but are of exanthematou lichenoid (dDx: Lichen Planus). silver nitrate solution level of skin cleavage with in TEN is at the followed by a dexamethasone-neomycin erosol spray and cholesterinized dermal-epidermal junction – i. SPARES THE INTERDIGITAL SPACES Photo-contact reaction to substance in the skin and UB light (longwave UV – 320 – 400 ng UVA) psoralens – present in many types of plants Dermatitis always occurs on sun-exposed skin. pattern morphology (papules. . perfumed. common causes are aftershave and PABA àit is fairly rare Drug Eruption Cutaneous the most antibiotics are the most common cause of these eruptions in the hospital discontinuation of the reaction to common setting common locations include the trunk.E. if the 20-50% of the body can be involved sulfonamides are suspected in the etiology of TEN. digitalis. It may include: analgesias for pain relief. arsenic. Also. it should obviously be the pathogenesis is unclear but T-cell avoided. penicillin. nodose. plants (Rhus genus). vesicles or bullae) of the lesions presence of spongiosis this can be confirmed via a biopsy or Tzanck smear the most common contact allergens include nickel. generally over 12 to 24 hours main sites of Some etiologies include: bacterial infections . there is a risk of granulocytopenia. antipyrine. T lymphocytes and Tcell lymphokines be non-protein in nature solubleàhaptens Type Erythema Multiforme M> F 40 y/o Description viewed as a hypersensitivity syndrome – therefore. tetracycline. echoviruses and poliomyelitis dorsum of the feet mycotic infections . elanoma ital. dapsone. sulfonamides. tolbutamide and trimethoprim-sulfamethoxazole characterized by widespread blistering of the • in full blown cases of TEN.may occur in association with erythema iris. salicylates. is found systemic lupus erythematosis chiefly on the ankles drugs – include allopurinol. specifically the CD8+. but it does inhibit the hyperkeratosis. infectious mononucleosis. Allergic Contact Dermatitis Drug reactions locale.e. allergic contact process. whitish ring develops around a compound nevus resulting n the so-called “halo nevus” rarely transform into malignant melanoma Intradermal Nevus this nevus is usually small – less than one centimeter with regular edges. tumor be quite deadly radiation and chemotherapy glomus tumors are benign neuroarterial lesions appear skin-colored to slightly dusky surgical excision àwhen complete. color is frequently dark where nests of melanocytes are located in the basal layer of the epidermis • it is very common in children. progressive nevus or “bathing trunk nevus” is a variation of either the intradermal or compound nevus. biomechanical control of subcutaneous tissue through in patients who are extremely obese. generally <1cm and ranges in color from brown to black and hairs may be presentàoccasionally a depigmented. but are freely from venous or arterial smooth muscle moveable over subcutaneous fat smooth muscle are often • recurrence rate with resection is angiomyomas are usually solitary nodules referred to as they occur most commonly on the lower extremity it rare angioleiomyoma usually presents as a skin colored nodule (dDX above) malignant variant of a smooth muscle they have a tendency to metastasize and can they are managed with excision. lesions composed of vascular channels blue. smooth muscleà symptoms chief complaint surgical excision is indicated are exacerbated by cold • this pain is believed to be secondary to impingement • they possess numerous blood weather . lesions arising composed of poorly defined • lesions are “fixed” within the skin.for biopsy and potential malignancy NOT EASY TO REMOVE 61 ."goose bump" of vascular or neural structures vessels with thick muscular walls conditions. red or pink coloration with typically have ill-defined bordersàatypical mole syndrome is an autosomal dominant familial disorder placing an individual at greater risk to develop malignant melanoma at an early age Blue Nevi Regarded as a failure of melanocytes from the neural crest to arrive at the dermo-epidermal junction.F. pigment and other disorders (see other derm lesions) Nevi Description 20 to 50% of melanomas develop in a pre-existing nevus • development of a new nevus over the age of 35 should alert the physician to possible melanoma Types Junctional Nevus it is evenly pigmented flat and well-circumscribed. and it is the most likely pre-existing nevus to develop into a melanoma Compound Nevus common in adolescents and comprise the majority of pigmented lesions in children. Hair. brown. familial tendinous xanthomas are smooth and attached to tendons. The melanocytes are found in the lower dermis giving refraction of light at this level gives rise to the blue color 5. it is generally constantly tender and relief of symptoms is immediate surrounded by nerve fibers they arise exquisitely painful with paroxysmal recurrence if possible with from the AV shunts exacerbation incomplete resection these are herniations or they typically occur on the medial and lateral NONEà the patient should be counseled protrusions of the plantar aspects of the heel pad abd these lesions occur regarding weight loss. elbows and extensor surfaces of the extremities hyperlipidemia. ligaments and deep fasciaà hypercholesterolemia and familial these occur frequently on the tendo achilles hyperbetalipoproteinemia subcutaneous tumors composes lesions are generally asymptomatic unless impinging surgical excision is generally curative of fat tissue skin overlying a on surrounding vascular or neural elements intralesion triamcinolone acetonide lipoma may exhibit dimpling if the lesions is larger than 10 cm in diameter. Type Leiomyoma more common in adults than children angioleiomyomas are F>M. 2:1 Tumors A. and transformation of these giant progressive lesions may be as high as 10% Atypical or macular lesions that may be present in great numbers and may be up to 6 mm or greater in diameter. very rare in adults over 50 years old. primary lesions are indurated and have a tendency to coalesce hyperlipoproteinemias. they possess a cobblestone Dysplastic Nevi surface with a variable mixture of tan.particularly when occurring liposuction cellulite) on the thigh . Benign Leiomyosarcoma Glomus Tumor Piezogenic Papules Xanthoma Lipoma Soft Tissue Tumors Description Characteristic Lesion Clinical Presentation Treatment benign tumor arising from quite painful – such that pain is generally the presenting when lesions become symptomatic. more common in adults • the giant. it injectionsà atrophy of fat when pulled (resembling should be resected . foot function and/or change in weight bearing the connective tissue striae of excessively pronate or stand for long activities the plantar fat pad periods of time these occur secondary to the deposition of tuberous xanthomas may develop as slow growing yellow papules or nodules lipids in the skin as a result of • they affect the knees. papillomatouc lesions patient is gaining weight without the need for anesthesia and on the plantar surface of the foot.most Peyrone's disease. the nail folds and lesions bleed easily when irritated: are believed to be secondary to underlying nail bed and matrix is friable previous nail surgery or minor recommended these lesions have a symptoms may occur secondary to shoe gear irritation trauma high recurrence rate classically. 4% alcohol sclerosing agent may be successful following exostosesà most foot and along the aspiration of the fluid contents. May • recurrence rate of these lesions is >65% with recurrence. the entire lining of the cyst should be removed along with joint structures tendon sheathes identification and ligation of the stalk 62 . principle locations include the lower NONE. albeit common of all lesions dorsal of the foot often frustrating due to the high recurrence rate and when surgical resection affecting the joint and from the extensor required.5 cm) with partial fasciectomy may be necessary on the sole . aspiration and subsequent injection the joint linings and the wrists and ankle. sclerosing sides of the trunk. the nail plate to 10 punctures with aspiration of the contents DIPJ may arise from herniation may demonstrate longitudinal • surgical excision may be effective with careful of tendon sheath grooving or ridging dissection of both the cyst and its stalk they are herniations of occur commonly over NONE unless the lesions are symptomatic. concomitant injection of a small amount of aspect of the distal phalangeal noninflamed. wide surgical excision (margins slowly enlarging nodules palmer fascia or > 0. and • some recommend "multiple" needlings consisting of up PSEUDO-CYSTS.Type Acrochordons M=F common in 60% population Periungual Fibroma Kernan's Tumor occurs in the early teens Acral Fibrokeratoma average age 40 years Dermatofibroma occur adults rarely in children Plantar Fibromatosis Ledderhose's Syndrome Synovial or Myxoid Cysts Digital Mucin Cysts F>M Ganglionic Cyst Soft Tissue Cysts Description Characteristic Lesion Clinical Presentation Treatment small. dome-shaped papule that is due to their predominantly acral simple shave excision and asymptomatic and is usually solitary. surgical excision of symptomatic nodulus cutaneous. subepidermal extremities. above the elbow. it is reddish brown and hyperpigmentation . it presents as a smooth. they increase in number when the most lesions can be "clipped" away pedunculated. these lesions tend are frequently associated with cauterization of the base can be to become flattened and are generally seborrheic keratosis and possess a accomplished with electrodesiccation or asymptomatic unless there is some type of external statistical correlation with colonic other techniques recurrence following irritation and lesions may be up to 1 cm polyps this treatment is unusual these lesions may become large enough to disrupt the these are associated with tuberous generally no treatment is required entire nail bed sclerosis 50% of the time unless lesions become symptomatic firm. sessile. skincorticosteroid may be helpful or proximal nail folds: colored or bluish nodules. intralesional injection of over the lesion. slerosing solution. solitary. smooth flesh-colored growth protruding from the remainder of the time. acquired pinkish. surgical excision or benign. make sure you put in a band of the plantar plantar fasciaà drain to prevent hematoma fascia àfocal accumulations of OA is sometimes present in the high rate of recurrence lesions will often refill within eight mucin in the dermis or dorsal adjacent joint . it is characterized as a location. lesions radical excision of the tumor. they are usually weeks. of a corticosteroid may be effective and aspiration may need to be repeated often associated with heel or dorsum of the several times. hornlike projection a precipitating factor àit usually ablation • this projection usually emerges from a collarette of slightly occurs on the digits but may occur • recurrence rate is rare on the palms and soles elevated skin – which an important differentiating factor also known as fibroma durum. it • may occur secondary to an 4 mg/cc) may be helpful may actually involute insect bite or minor trauma the skin is freely moveable Assoc w/ Dupuytren's accommodative directed towards off-loading . hyperkeratotic. lesions over with DIPJ lesions. trauma is thought to be cautery. surgical excision may be necessary.patient's often find this hemangiomait is a common nodular occasionally has a slightly scaly or velvety more objectionable than the original lesion skin lesion and these lesions are appearing surfaceàdiagnosis may be aided • intralesional injection of short acting occasionally pruritic by the "dimple sign" . flesh to dark brown colored. or on the lesions may result in residual nodular fibrosis. occurs as a contracture of the triamcinolone acetonide. resection of common on the medial infiltrate through the the entire plantar fascia is recommended.that is upon corticosteroid (dexamethasone phosphate squeezing the lesion from side to side. zinc eyebrows. often more cosmetic than allowing the • middle age to Incidence: Occurs very common on the shins and calves in women (ddx of AD lesion to resolve on its own and lesions on elderly and EN) Clinical Appearance: the initial lesion is a smooth. sprue (Selsun). lesions develop quickly and it possesses a central • injection therapy of 5-fluorouracil :1 keratin-filled plug which may become crusted and the initial rapid growth period is injection/week for 3 consecutive wks then followed by a stationary period for two to six weeks where the lesion will then is spontaneously regress over 2 to 8 months . cone-shaped red papule the plantar surface of the foot should always and may resemble molluscum contagiosum or verruca vulgaris and telangiectasias be excised may course through the lesion. discrete. tar. Type Eccrine Poroma F>M >40 y/o Pyogenic Granuloma granuloma telangiectaticum most frequently in children Seborrheic Dermatitis Premalignant Lesions Mimicking Malignancies/Pre-Malignant Lesions Description and Characteristic Lesion Clinical Presentation Treatment a benign tumor of the intraepidermal portion of he eccrine sweat duct most commonly on the enbloc surgical excision a slightly protruding fleshy mass generally pink to reddish in color that sides and soles of the when present on the plantar may appear to be slightly lobulated or verrucous foot surface of the foot “proud flesh”. sternal of a lipophilic. ears. on very rare occasions.even those which prognosis Characteristic Lesion: lesions are commonly 1 to 2 cm. scalp umbilicus. horny excrescences Keratoacanthoma Arise from a viral origin and histologically they may be quite difficult to the scar from careful surgical excision is differentiate from SCC though they have the potential to transform into SCC. Characteristic Lesions: skin colored. malignant insure adequate removal and appropriate Peak: 60 and 70 lesions at the base of the horn occur 16 to 20% of the timeà frequently SCC although therapy for underlying malignant lesion years of age occasionally it will be a basal cell carcinoma if present. eyelids. flat or elevated. pleomorphic fungus Malassezia ovalis responded well to 2% for cases of the area. axillae. and it is dull red in color ioccurs on exposed nail fold lesions often respond well to isotretinoin “shelling and “friable” àmay form at the site of an surfaces of the body and out” the lesions with a curette. in the presence Keratosis individuals àmay give rise to SCC and mimics Bowens disease and of broad.Cornu are generally precipitated by UV radiation exposure and have a superimposed lesion examination of the underlying lesion to Cutaneum (often seborrheic keratosis. AIDS . the disease process is generally not seen before pubertyà hormonal influence Presence resorcin shampoos lips. extensive lesions -à topical 5-fluorouracil and it SCC that arise are well recognized as non-aggressive w/ an excellent has a tendency to "seek out" the lesions . smaller lesions may respond to injury.B. ketoconazole cream. fibrous scar 63 . • CO2 laser ablation and oral etritinate is quite effective red. (celiac disease) and epilepsy may also increase the incidence of seborrhea. pigmented or skin colored patches or scales prophylaxis against future lesions is imperative with the use of total UV blocking sunscreens Cutaneous occur most frequently on the face and scalpà generally a benign condition. usually generally undergoes remissions and exacerbations and may be accentuated/ aggravated selenium sulfide targets the scalp. have a high hemangiomas” sites and the lesion incidence of recurrence common. by:Parkinson's disease. verruca vulgaris. are are not clinically apparent generally multiple. these lesions excisional biopsy with histopathologic Horn . Incidence: pyrithionate and nasolabial creases. especially in obese patients. groins • lesions are also exacerbated by emotional stress and have been induced by the and gluteal crease neuroleptic drugs (haloperidol and droperidol) Actinic or Solar occur on sun exposed areas of the body and frequently in fair elderly Liquid N2à lesions on the back and face.diabetes mellitus. angioma) 50 to 60% of the time.spontaneous involution that will heal • intralesional injection of triamcinolone with a residual. verrucous or keratotic. these the sole of the foot or cauterization following removal with silver nitrate sticksà lesions may occur subcutaneously and nail fold of a toe or may stain the skin and mask erythemaàlesions when are then referred to as “lobular capillary finger are also common simply resected without ablation of the base. cell proliferation apply bid x 3 wks. only 5% strength is recommended. this method is probably superior to all others High cure rate. and it is generally reserved for in situ and actinic keratotic lesions desiccation radiation Is mainly used as a treatment in patients who cannot tolerate or wish to avoid surgery Treatment 64 . and the most difficult to treat. and it is actually the most common tumor of light-complexioned people Pigmented Basal Cell Carcinoma Sclerosing or Morpheaform BCC Superficial Basal Cell Carcinoma Biopsy and Excision Electrosurgery curettage Mohs Surgery Cryosurgery 5-Fluoro-uracil (Efudex ) ® 6 Types Characteristic Lesion Incidence Variants TX Topical agents Imiquimod (Aldara ) ® Slow developing light to dark brown lesions that mimic melanoma (d/t considerable pigmentation). has a threadlike raised border and appears as a flat growth with no tendency to ulcerate. the five-year cure rate is 34% and recurrence and metastasis typically occurs w/in a three year time from of initial treatment Prevention excisional removal emphasized with the use of sunscreens most ideal with good margin control in well differentiated lesions less than 2 cm in diameter may be treated with surgical excision with a cure rate of approximately 99% elliptical excision is acceptable with margins of 4 to 6 mm Moh's sx technique it is used when tissue to be removed must be kept an a minimum cyrotherapy curettage are reserved only for the most superficial lesions.c skin cancer comprising 20% of all nonmelanotic skin cancers. however. followed by the hands and forearms. Most common in the elderly d/t cumulative solar insult. elliptical excision with margins of 2 to 5 mm is the procedure of choice and with excision of tumors less than 2 cm . 20% m. and 60% arise from the site of a previous actinic keratosis.a 4 mm margin of surrounding. Most common type of squamous cell carcinoma is sun induced. but it is a blind approach and it is difficult to tell if all the lesion has been removed (-) DNA syn by blocking methylation of deoxyuridylic acid and inhibiting thymidylate synthetase and therefore.5 cm in diameter.5 cm in diameter. crusted and gradually enlarging ulceration. and in the hands of an experienced operator. specifically from hair follicle stem cells residing just below the sebaceous gland duct. and pigmentation persists post-treatment Waxy sclerotic plauque mimicking scleroderma or a scar with no rolled in borders. > 1. and the disease process is characterized by chronicity . discrete (freely moveable unlike the fixed BCC) lesion that is flesh-colored containing telengectasias These lesions may be painful and tender even when small and 50-60% occur on the head and neck. M>F 3:1 incidence is far less than that of basal cell carcinoma Superficial. If the lesion is 5 to 7 mm in diameter. more common in darker skinned. therefore the lesions are characteristically asymptomaticß PAINLESS nodular . chronic ulcers and radio-dermatitis (Marjolin’s ulcers) are generally more aggressive with incidences of metastases reported between 20 to 30% à when metastasis occurs. The tumors usually arise from the epidermis and occasionally arise from the outer root sheath of a hair follicle. May correlate with the ingestion of arsenic over a long period of time. upper trunk and lower legsà presentation in the foot is often atypical a history of blistering sunburns and frequent sun exposure is routinely elicited and may grow rapidly over 6mo -1 year skin biopsy specimens must reach at least the depth of the mid-dermis to allow for determination of the presence or the absence of invasive disease • SCC arising from burn scars. Treatment of choice for sclerosing BCC.which is most common pigmented cystic sclerosing or morpheaform superficial nevoid Central Depression:à bleeding. Why is metastasis so rare in basal cell carcinoma? growth of the lesion is dependent upon the underlying connective tissue stroma and most reported metastases are secondary to incomplete excision or because of long-term neglect Squamous Cell Carcinoma Description Etiology Incidence Characteristic Course Metastasis It is the second m. Translucent Borders may actually appear to be "rolled" in w/ telangectasias: Diagnostic highest among middle-aged and elderly people who have a history of considerable sun exposure. BCC is considered the most common of all malignancies in man. it is very uncommon in the foot. tx is repeated 5 d/ week for 6 wks 1. therapy might be required for up to 10-12 wk the cream is applied to the lesion and an additional 1 cm of surrounding skin and left on 8hrs.C.growing slowly. DisAdv: It is time consuming. ulceration or crusting.c in blacks than BCCs. Telengectasias are the most prominent Erythematous scaly plaques with telengectasias that mimics psoriasis. > 1. Malignant Basal Cell Carcinoma Description Lesions often arise historically from actinic activity and it is believed by many to arise from pluripotential cells within the basal layer of the epidermis or follicular structures. healthy appearing tissue is acceptable. 49 mm 87 to 94% five year survival Stage 3 1. correlation btwn levodopa tx in Parkinson's disease and the onset of melanoma A .50 to 3.surface . and the CNS metastasis is the most common cause of death Incidence melanoma is not common in darker races. overlap of the two systems is defined below: • Breslow Group I (0.Squamous Cell Carcinoma in Situ Description Characteristic Lesion Incidence Treatment an intraepidermal SCC à in a classic windblown appearance àin situ disease connotes a superior prognosis regardless of treatment modality. therefore it is imperative to examine local lymph nodes when there is an index of suspicion as lymphadenopathy may be an early sign of metastasis where lymph nodes become hard and discreteà skin is the primary metastatic site. warty. these lesions may become invasive podiatric appearance may be atypical (examples of such cases include a presentation initially mistaken for a "wet" fungal infection and another mistaken for a heloma durum) • highest incidence in older white men and arise on primarily sun-exposed surfaces • simple elliptical excision. there is no invasion of the reticular dermis .76 mm > 98% five year survival Stage 2 0.asymmetry . the DE junction and the papillary dermis not visible to the naked eye.border irregularity .99 mm 66 to 83% five year survival Stage 4 > 4.71% five year survival Level V melanoma penetrates into the subcutaneous tissue .76 to 1. unevenness in general one should consider biopsy if one or more of these signs are present Classification Systems for Melanoma Breslow's Classification this system is a measure of invasion in mm of thickness and thickness of the lesion correlates with prognosis • Stage 1 tumors (localized disease without regional node involvement) are listed as follows*: Stage 1 < 0.0 mm < 50% five year survival Clark's Levels of Cutaneous Invasion this micro-staging classification is based on the level of penetration of the melanoma through the skin • Clark's levels do not impose a prognostic outcome Level I melanoma located above the basement membrane of the epidermis. but a small number arise de novoàsurvival rate can be as high as 90% for early detected lesions Metastases Early lymphatic spread with late hemotogenous spread.this is present when.color variation .95% five year survival Level III melanoma characterized by widening of the melanoma cells which fill the papillary dermis down to its interface with the reticular dermis.greater than 6 mm may suggest malignance S . Mohs surgery is recommended for larger lesions Malignant Melanoma Discussion Description most arise from a pre-existing nevus.more than one shade of pigment warrants concern D .this is suggestive if the border is scalloped.65 and 1. and untreated. if the lesion is bisected.5 mm correlates with Clark's levels IV and V • Clark's level III falls between 0.5 mm of thickness Prognosis of these lesions may also relate to other factors: • survival rate is more favorable in women. often present from birth.Bowen's Disease .surface irregularities . 65 . What is Dermoscopyà a noninvasive method that allows the in vivo evaluation of colors and microstructures of the epidermis.diameter .nodular.65mm) compares to Clark's level II (down to but not penetrating the papillary dermis) • Breslow penetration greater than 1. raggedy or uneven Recognition C . these lesions are essentially in situ and present no re year survival Level II melanoma invades through the basement membrane into the papillary dermis . the halves are not identical Clinical B . individuals less than 50 years of age and for tumors on the extremity (excluding those on the hands and feet) • the five year survival rate inpatients with nodal spread is about 36% and drops to 5% with distant metastasis 1.82% five year survival Level IV melanoma penetrates into the reticular dermis .49% five year survival Because each classification system has its followers. Streptococcus. digital pulp or nail fold or where the thumb and bed is virtually diagnostic. and anthrax. the palms and soles m/c site of melanoma • notable predilection exists for the distal phalanges of the toes and fingersàsubungual lesions foot . Use of probenicid will increase levels of PCN. brown. sole. pneumococcal or meningococcal meningitis PCN G is difficult to get drug to site of infection. red. Lyme. neisseria.20 years. Klebsiella. rat-bite fever. listeria.Type Lentigo Maligna Melanoma in Situ Amelanotic Melanoma Superficial Spreading Melanoma Acral Lentiginous Melanoma Nodular Melanoma Types of Melanomas Clinical features Incidence % Treatment begins as a tan macule that becomes irregular M=F 60's. 7. enlarging black macule on the palm. endocarditis. Legionella) Plasmid mediated B-lactamase resistance (Staph. Variable activity against gram (-) Staphylococcus aeurus and epidermis produce B-lactamases which destroy both PCNs B-lactamase (eg S. and some bacteroides species. pharyngitis. erosion or ulceration is a sign of sun-exposed skin growth only 1-5 years malignancy and heralds metastases female shins M=F 50's 10% • involves not only acral but volar structures. syphilis. erythematous or flesh-colored but otherwise mimic exactly the biopsy. oxacillin. recurrences of rheumatic fever.Coli. pasturella. anaerobic streptococcus. Klebsiella. the possesses no those of lentigo border is often notched. Increasing resistance to both: MRSA Gram +: Streptococcus pneumoniae. Actinomyces. Exception is nafcillin. metastasis to the axillary and epi-trochlear nodes develop in late stages of the disease and are common lesions may arise without any clinically apparent preceding radial growth M>F 2:1 15% Clinical Features these lesions tend to invade the dermis very rapidly. Special disorders of nails and appendages of skin (see other derm lesions) Treatment (see each lesion) Organisms that cause gas in tissues: BECKSSà Bacteroides. Clostridium. neisseria. aureus) Chromosomal mediated resistance (Baceroides. endopeptidase) responsible for the peptide linkages involved in peptidoglycan synàinhibit cross-linking of the peptidoglycan. a black discoloration of the proximal nail fold at the end of a pigmented hallux are more streak (melanonychia striata) is ominous and may signal a melanoma of the nail matrix frequently involved early lesions may appear simply as a light brown. Staph Pharmocology revisitied Antibiotics The Natural PCNs: PCN G and PCN V MOA Metabolism Indications Disadvantages Most susceptible pathogens PCN resistance methods Drug rx Bind regulatory enzymes (PBPs-transpeptidase. coli) DO NOT USE GRISEOFULVIN with PCN allergy 66 . that is.The lesion will exhibit a radial predilection for sunthen thorough surgical growth over a period of 5. diphtheria. hemophilus.70's 5% photographed first and w/variegated coloring . E.60% have generally fall into this category plantar or subungual • radial growth rate lies midway between lentigo maligna and superficially spreading melanoma "Hutchinson's most common type in blacks. Works on actively dividing cellsà bacteriacidal Minimally. Seratia. Pts with renal impairment dosages do not have to be adjusted Simple and inexpensive drug that has a broad spectrum of bactericidal activity DOC for strep viridans. clostridia.they maybe frequently predilection for sun30% misdiagnosed . cloxicillin and ureidopenicillins which have biliary excretion. excreted primarily unchanged by the kidneys. primary syphilis.resembling blood blisters or hemangiomas and older lesions become papillary. characterized by areas of focal regression and/or predilection for maligna with radial asymmetric growthàeasy bleeding. exposed areas fungoid or ulceratedàbleeding is generally a late sign and coal black or sepia colored spots may appear as if sprayed about the lesion Description Hutchinson as a melanotic freckle 6. GBS. Decarbazine (DTIC) pigmented lesion and is often mistaken for a pyogenic granuloma Temozolomide and vaccines lesions grow much characterized by radial growth: it is multicolored and possesses not only M=F 50's 70% more rapidly than different shades of tan but variegated black. streptococcus viridans. dark brown or black discoloration & late lesions than the other digits may become nodular & ulcerative. E. carbaxypeptidase. Hispanics and American Indians melanotic whitlow" Clinical Features: an irregular. because it is very unstable in the acidic PH. blue and white. and then may become damaged skin excised vertically invasive that is nodular melanoma as the Moh's surgery is again hallmark of its vertical invasion is demonstrated useful in this situation this differs from all the other melanomas only in its lack of pigment Surgical excision with sentinel node the lesions may be pink. 5g po 6hrs after the procedure All of PCN G. well Dicloxà higher blood Cloxacillin 250-500mg qid absorbed. degraded in gastric acid. Coli. E. indanyl carbenicillin. expensive Penicillinase-Resistant Penicillins: Methicillin.gentamicin • All are highly protein bound.600. not acid stable. faecalis (enterococcus) or gram negatives (both anaerobic or aerobic) o The DOC for these species is Vancomycin +/. can give IV or IM Amoxicillin DOC for endocarditis prophylaxis. T1/2 (1wk). Amoxicillin. excellent tissue penetration and therapeutic index Absorbed 60% after po dose on an empty stomach. NO coverage against pseudomonas. Also has no coverage for S. N. Enterobacter and Bacteroides Clindamycin 600mg: 2-300mg tabs Aminopenicillin preparations Drug Description Contraindications. including. imipenem.Drug PCN G Benzylpenicillin PCN V MOA/ Description Not acid stable.0g = 1. readily destroyed by staph penicillinase Aminopenicillins Indications Gram-positive susceptible pathogens Gram-negative susceptible pathogens PCN allergy? Otitis media. longer T1/2 IM use 1-3xd Benzathine PCN G-IM long acting. and Becampicillin Presence of an amino group on the beta side chain of the pcn nucleus extends coverage to include some gram-negative bacteria. 90% strep than methecillin which are also protein bound. most have hepatic metabolism and biliary excretionà contraindicated in patients with liver failure Drug Description Indications Contraindications.Coli. and meopenem.000units 500mg = 800. short lactamase stable pcn nephritisà 1-2g IV PB q4h T1/2. Klebsiella. T1/2 30min Procaine/PCN G-equimolar conc of both. (long surgical procedures). NO advantages. Proteus. Nafcillin/oxacillin Indicated for the DOC for penicillinase producing staph and also effective for penicillin-sensitive pneumococci and streptococci S Aureus/epidermidis are resistant to these penicillins and are called methicillin-resitant staphylococci or MRSAà theses are also resistant to cephalosporins. the most preferred [ ] PO ONLY Diclox: 250mg q6h Nafcillin and Oxacillin Oral absorption is erratic. AHA recommended for the prophylaxis of bacterial endocarditis: 2. absorption is 2x greater than 250mg po q8h or tid (Amoxil) ampicillin.0g po 1 hour before procedure and 1. Providencia and all are destroyed by b-lactamases Act synergistically with aminoglycosides against pseudomonas TnT (Ticarcillin and Tobramycin) generally used IV. Better for staph and Pts on coumadin 4-12g/day or 2g q4h must be given IV. streptococcus faecalis and non penicillinase producing staphylococcus H. and absorption levels about the same. Influenza. T1/2 (2hrs) Disadvantages 10% hypersensitivity.7meq of salt and full doses of 12-30g qd may precipitate CHF à fluid retention (CHF) and hypokalemia o Both bind ADP receptor sites on platelets preventing normal aggregation and therefore prolong bleeding time o Hypokalemia 67 . DOC for syphilis Potassium salt: Pen VK or V-cillin K Dose: 250mg-1g 4-6x day Indicated: throat. lasts 34wks. shigella. Neisseria. E. salmonella (about the same as ampicillin) NO coverage for enterobacter. no blactamase resistance must give IM/IV only Acid stable. Units of PCN V: 125mg = 200. Psedomonas. osteomyletis (pseudomonas) Susceptible organisms: excellent for gram-negative bacillary eg Haemophilus. no blactamase resistance PO ONLY. morganella. cloxacillin/Doxacillin. poorly absorbed Better than PCN G Forms Crystalline (aq) rapid for continuous IV. shigella.000 Advantages Low cost.000 • The Aminopenicillins: Ampicillin. gonorrhea. respiratory and soft tissue infections 1. sinusistis. IM/po doses fail to achieve adequate serum/tissue levels adequate for pseudomonas Know the 3 Problems with carbenicillin and indanyl carbenicillin o Contain 4. need higher PARENTERAL ONLY highly protein plasma levels bound • • The Carboxypenicillins: Carbenicillin. po levels approach IM levels Bacampicillin (Spectrobid) Prodrug-hydrolyxed to amoxixillin. Proteus mirablis. ticarcillin o o o o o o Indicated for the treatment of UTIs. salmonella. low plasma binding PARENTERAL ONLY NEPHROTOXIC Cloxacillin Doxacillin Isoxazole pcns. Warnings Dose Methicillin No longer marketed in the 1st anti-staph bCauses interstitial Must give q4h (Staphcillin) US. bronchitis. Warnings Dose Ampicillin First 80% develop rash 500mg q6h.rifampin+/.000 units 250mg = 400. 3g/375mg. some enterococci. E.0g in total 2. Enterobacter. bacteriocidal. Haemophilus.1 or 3. Fragilis and Klebsiella Pneumonia Contraindications. III. active against many gram + cocci (not enterococcus or MRSA). aureus and some anaerobes.1-0. o Does not inhibit class I or chromosome mediated resistance as in Pseudomonas. Piperacillin o o o o o Derivatives of ampicillin. K. hypokalemia and CHF Dose 1-2g dose q4h generally given with 1g of probenicid The Uerido penicillins: Azlocillin. Warnings Platelet aggregation. hypokalemia and CHF Not for systemic infection Platelet aggregation.2g IV/IM Ampicillin 1-2g and Sulbactam 0. Warnings Not absorbed orally. Mezlocillin.2q q4h IV 3. must be given IV Dose 8-16 g IV 8-16 g IV 8-16 g IV Drug Azlocillin (Azlin) Description 8-16 X more active than carbenicillin for pseudomonas.Drug Carbenicillin (Geopen) Indanyl carbenicillin (Geocillin) Ticarcillin (Ticar) Description The first with antipseudomonal activity Is acid stable and well absorbed 4x more active than carbenicillin Indications IV/IM PO ONLY UTI and prostatitis Pseudomas Contraindications. fragilis and gram negative bacteria B-lactamase inhibitors o Clavulonateà PO ONLY (-) Richmond Sykes class II. H. Proteus mirabilis (20% of hospital acquired are resistant) 68 . and Citrobacter species o Drug Augmentin Timentin Unasym #1 Zosyn Combinationß KNOW Amoxillin 250-500mg and Clavulonate 125 mg PO Indicated for Otitis media. excellent (Piperacil) against strept. Timentin. IV and V as well as S. Unasym and Zosyn Due to the structural similarity to the basic PCN molecule. None are highly protein bound probenecid will prolong activity Cephalosporin generations-as we go from the 1st – 4th generation the more effective the gram-negative killing First Generationà PEK o All have similar spectrum of activity. influenza. UTI Ticarcillin 3gms and Clavulonate 0. soft tissue infections. Pneumoniae. must be given IV Not absorbed orally. o Does not inhibit class I or chromosome mediated resistance as in Pseudomonas. and Citrobacter species o Sulbactamà IV only. and IV penicillinases/cephalosporins. broader spectrum than PCNs but is variable in each generation Liver à inactive lactone. B. parental only Similar in spectrum as ticarcillin. 4g/500mg Dose: q6h slow IV over 30 min The Cephalosporin Family Introduction MOA Spectrum of activity Metabolism Produced by the fungus Cephalosporium acremoium Differs from pcns by the presence of the dihyrothiazine ring that gives more resistance to b-lactamases rather than the 5membered thiazolidine ring of pcn Inhibits cell wall synthesis. must be given IV Not absorbed orally. Enterobacter. the DOC for pseudomonas. do not have to adjust for renal disease Indications: For serious gram negative infection.25mg. and are destroyed by b-lactamases All dosed at 8-16g IV and are usually combined with an aminoglycoside Excreted differently than PCN o 20-3-% via hepatobiliary system. then excreted by the kidney. B. structure. (-) Richmond Sykes class II. Can be used for penicillin sensitive patients.5-10g Piperacillin 2. peptidoglycan coss-linking. Neissria.coli. an inhibitor of Class I or chromosome mediated b-lacatamases: Is synergistic with penicillin o Tazobactamà Low level of binding to PBP. but is less active for Proteus Mezlocillin (Mezlin) Comparable to Ticar against Psedomonas Piperacilin The most active pcn. but better coverage for streptococcus. Seratia. Cefoperazone is metabolized and excreted exclusively by the liver ß can use in patients with renal disease. Seratia. these inhibitors have a high affinity and irreversible binding to many bacterial b-lactamases and thus prevent the enzyme hydrolysis of the co-administered penicillin Indications: o For the empirical treatment of infections causes by penicillinase producers o Extends the spectrum of activity to include Staph aureus. and Bacteroides B-lactamase inhibitors combinations: Augmentin. III.25g and Tazobactam 250mg Dose 875 mg po q8h Hospital order: 3. Neisseria Absorption: PO-best absorption on empty stomach. better for staph except MRSA). Moraxella. best taken with food. H flu. can give qd. staph osteo (good (Ancef. long T ½ (4hrs) IM/IV. Better for proteus. good absorption. serratia.coli. Proteus vulgaris.coli. not as good as 1st or 2nd gen • Excellent for GBS. even better for gr. enterococcus.0g IV PB stat then q6h X 6 doses First Generation cephalosporins: Drug Description Cephalexin (Keflex) PO.coli and proteus spp Cephradine (Velosef) PO/IM/IV Cephalothin (Keflin) IM/IV short T1/2 (30-40min) Cephapirin (Cefadyl) IM/IV short T1/2 (30-40min) Cephalozin IM/IV DOC for surgical prophylaxis. peak blood level 15-20 ug/ml. preffered due to long T1/2 Second Generation Cephalosporinsà HENPEK • Generally indicated for lower or URI. fragilis and Neisseria IM/IV Dose 250-500mg po BID 200mg po bid 200mg po bid 250mg q12 tabs/susp 1. skin and skin structure. STDs. proteus (indole postive). Problem: short T ½ IM/IV.coli. than other 1st gen or 2nd gen. Serratia. GI. UTI. poor absorption. strep. decreased gr+ activity. good for staph. increased serum binding and decreased tubular secretion. Neisseria. Neisseria • Good b-lactamase resistance • May not be as good for aneraobes eg Bacteroides as Mefoxin 69 . proteus. has a methoxy group attached to b-lactam ringà b-lactamase resistance. S. not active against bacteroides. good synovial fluid level but does not pass BBB (not good for meningitis) Better for strep and staph than subsequent generations Dose 250-500mg qid 500mg po bid 250mg q8h 250mg po q 6h Give q4h Give q4h Hospital order: start IV with D5Q to KVO then give Ancef 1. and some Klebsiella.o o o No activity against serratia. longer T1/2 Cefaclor (Ceclor) PO. indicated for joint or bone infections Not indicated for joint or bone infections IM/IV. enterobacter. otitis. E. influenza. but may not be better than 1st generations Drug Cefuroxime axetil (Ceftin) Cefpodoxime (Vantin) Loracarbef (Lorabid) Cefprozil (Cefzil) Cefuroxime (Zinacef) Cefmetazole (Zefazone) Cefotetan (Cefotan) Cefamandole (Mandole) Cefonicid (Monocid) Cefoxitin (Mefoxin) Ceforanide (Precef) Description PO. not as effective for gr+ than 1st gen. may be used for surgical prophylaxis IM/IV. long T ½. better for Klebsiella. E. and strep. higher GI upset.e. give VitK. coli. pseudomonas. Problem: contains the MTT side chain but rarely causes decreased PT levels. also good for B.0g IV PB 30 min b/f surgery Post op. GU. Kefzol) bone penetration). Contains the MTT side chain. Bacteroides fragilis. run PT.Ancef 1. more active against gram negatives i. longer T ½ Not indicated for joint or bone infections PO. flu.aerobes eg Enterobacter. indicated for pharyngitis. Enterobacter and Klebsiella • Excellent for strep and staph. broad spectrum Gr+/Cefadroxil (Duricef) PO. or pseudomonas. pneumoniae. proteus.0g q8h 2. bronchitis.E. and bone joint infections • Better than 1st gen at gram negative (E. H. Morganella • Wide range of anaerobic and aerobic activity • Generall 3rd gen are not the DOC for gr+. decreased gr+.0g q8h IV 1-2g q12 h 1g q4-8 h 1-2g q6-8h Third Generation Cephalosorins • IM/IV/PO • Good for gr+ and gr-. enterobacter. (tabs and suspension) Not indicated for joint or bone infections PO. Not indicated for joint or bone infections PO. better for gr. flu. skin/sin structure IM/IV enters CSF. E.. H. H. bacteroides frgalis Fourth generation cephalosporin: à Cefepime (Maxipine) • • • 1. coli. Neisseria. dyspnea and sweating Prolonged bleedingà due to hypothrombinemia ie interferes with VitK production of depended factors 1st generation 2nd generation 3rd generation Moxlactam Cefotetan Cefmandole Cefmetazole Cefoperazone Miscellaneous Beta Lactam Antibiotics for very severe infections 1. excellent resistance to cephalosporinases. even exceeding aminioglycosides. same as cefixime but with poor staph activity Cefotaxime (Claforan) IM/IV. anaerobes. good for gr-. More active against gr.coli. serratia. enterococcus. monobactam.0g q8h q12h qd MTT side chain: Nmethylthiotertraszole group at position 3 causes Dusulfam reaction with ethanol ingestion à increased plasma levels of acetaldehyde causes N/V. Klebsiella. nephrotoxicity.bacilli IM/IV only Clostridium difficile may be resistant Dose 400 mg qd 400 mg qd 1-2g IV q8h 2gm q8h 1. not good for gr+. DOC for OM o Contraindications: do not uses in children and in pregnancy (articular cartilage damage). good conc in CSF. one of the best for Pseudomonas. epidermidis.cell wall Absorption and Metabolism Shirt T ½ (1hr) Spectrum of action Largest of any other antibiotic. poor activity against gr+ and anaerobes. mycoplasma Contraindications May cause seizures Dose IM/IV only combined with Cilastatin* Dose: 500mg –2g q6-8h IV • Cilastatin is a dihydropeptidase inhibitor with no antimicrobial activity to prevent to rapid hydrolysis by the kidney Description 2. clamydia. longest T ½. enerobacter. enterobacter. morganella adjust dose w/ Must combine with Clindamycin (good for gr+ anaerobes) and an antistaph PCN Creatinine Indicated for UTI. faecalis. do not use in pts on Theophylline. tachycardia. against all gr+/. isolated from Chromobacterium violaceum. activity against all GP/GN ex: Pseudomonas and anaerobes 1g IV q24h Quinolones: The Carboxy flouroquinolones o Action: Bacteriocidal: inhibit protein synthesis by binding DNA gyrase which is necessary for DNA replication o Pharm: Good bone penetration. not indicated for skin/ skin structure or bone/joint infections. Imipenem and cilastin (Primaxin) Prototype of the thienamycin class of antibitotics cllaed the carbapenems. no coverage against MRSA. Pseudomonas. excellent for pseudomonas. Pseudomonas (including the species resistant to PCN and aminoglycosides) Also good for anaerobes eg B. not good for Pseudomonas. S.bacteria: Susceptible organisms GBS. CNS disturbances. longer T ½ not for enterococcus Ceftriaxone (Rocephin) IM/IV. better for gr+ and best of group for the staph/strep. can be used with Vancomycin Ceftizoxime (Cefizox) IM/IV. long T ½ Ceftibuten (Cedax) PO. S. good conc in the CSF. bronchitis. antibiotic produced by streptomyces cattleya (soil organism) Owing to its small size the molecule can gain access to the periplasmic space of a gr. blocks bacterial CW synthesis (inhibits 500-1/2q q8h. otitis. LRI. proteus.Third generation cephalosporins Drug Description Cefixime (Suprax) PO. E. long T ½ Ceftazidine (Fortaz) IM/IV. PBP-3) Good for gram negative aerobes only eg E. (qd) excellent spectrum for gr+/-. pneumoniae. and enterobacter. S. Fragilis. photosensitivity rx o 2nd-4th generation with increasing activity in GP organisms. useful as intermediary agents in polymicrobial involvement such as DM foot infections o Preparations: 70 . skin/skin structure infections. Others Drug Azotreonam (Azactam) Description Dose Parenteral only. will increase its [ ] and toxicity o Adverse rx: Tendon ruptures. good for pseudomonas. 2. Enterobacter. intraabdominal and gyn infection clearance Meropenem (Merrem) A perenteral carbapenem similar to imipenem with the same spectrum of activity Ertapenem (Ivanz) Reserved for life threatening infection. least nephrotoxic Cefoperazone (Cefobid) IM/IV-has MTT side chain. not indicated for bone infectins. po good and near complete absorption (95%) requires dosage adjustment for renal failure o Susceptible organisms: Predominantly GN aerobes. indicated for pharyngitis. will cross the placenta anc cause depression and abnormalities in bone growth or with pts with renal dz o Adverse rx: Skin rashes. does not pass CEF or eye. peak serum levels normally 30-90min after IM injection. typhus. well absorbed after IM injection. digoxin( can cause sudden cardiac death). N/V/D. Strept.5-2mgkg IV/IM in q12h 8. Sumycin) 250-500mg po bid-qid o o 71 . less nephrotoxic Neomycin Most toxic of the group.o o o o 2nd gen: Ciprofloxacin (Cipro-po/IV) 3rd gen: Levofloxacin (Levaquin) 4th gen: Gatifloxacin (Tequin) 4th gen : Moxifloxacin (Avelox) 500-750mg q12 or 400mg slow IV over 60 min q12h 750mg po qd or 500mg slow IV over 60 min qd 400mg slow IV over 60 min qd 400mg slow IV over 60 min qd Aminoglycosides: Good for gram negative aerobes. overgrowth of yeast and brownish/black discoloration of tongue o Susceptible organisms: Broad spectrum of g-/g+ DOC for rocky mountain spotted fever. cervicitis. indicated for the tx of urethritis. statins 1-2g IM only in divided doses 3 Ivmg/kg/day in 3 equal doses IM/IV q8h 1. blocks protein synthesis by blocking at the 50s ribosomal unit Toxicity: relative safe. triazolam.5g IV/IM in 2-3 equal doses 3 Ivmg/kg/day in 3 equal doses IM/IV q8h 1. Tuleremia. faecalis or viridans Gentamicin (Garamycin) Peak: 4-10 ug/ml Trough: 2 ug/ml DOC for GN infections. theophyllines. plague. otitis. Pylori in stomach ulcerations. Mg mycobacterium avian infection antacids. only s/e is epigastric distress and nausea. all contain an amino and a iltrate group o Action: Bacteriocidal-block protein synthesis by interrupting the transmission of genetic information at the 30s ribosomeà effective against enteroccoccus + a PCN o Toxicity: Ototoxic d/t damamge to the 8th cranial nerve (irreversible) nephrotoxic (if mild is reversible and occurs 5 days to several weeks after initiating antibiotic) and NM blockade 3-5% of the symptoms. 95% success with Pseudomonas Kanamycin (Kantrex) Used for TB. IV only Chemical: Defined by the presence of amino sugars linked by a glycoside bond to an aminocyclitol ring. photosensitivity. Fewer s/e than erythromycin Dirithromycin (Dynabec) Enteric coated Once daily dosing 500mg po qd Troleandomycin (TAO) Really good for Strep. less against GN. active against Z-pack Tabs 2 (500mg) day 1 then 1 qd days mycoplasma. Initiate therapy w/loading dose to attain a therapeutic serum levels o Synergistic with PCN antipseudomonal pcnase resistant pcns o Not metabolized and excreted via the glom iltrate. less active against Pseudomonas Amikacin (Amikin) Peak: 20-30 ug/ml Trough: 10ug/ml Used when other organisms are resistant to other aminoglycosides Spectinomycin Long t1/2. brucellosis. pharyngitis. also for strep endocarditis if due to S. Chlamydia and mycoplasma. Staph is resistant Preparations: o Tetracycline (Achromycin. permits a one time dose Streptomycin Discussion of peaks and troughs: o o o Macrolides Action: Primarily bacteriostatic or cidal dependent on condentration. similar to azithromycin with shorter T1/2. replaced by gentamycin Tobromycin Peak: 8-10 Trough: 2-4 More active than gentamycin for pseudomonas. increasinf their toxicity: anticoagulants (increased bleeding). can cause cholestatic jaundice 250-500mg po qid Erythromycin Should not be used in pts with hepatic dysfunction since it accumulates in the liver 250mg q6h Can cause prolonged cardiac repolarization associated with torsade de pointes Tetracyclines Action: bacteriostatic.15mg/kg/day IV/IM in 2-3 equal doses Azithromycin (Zithromax) PO. Legionella. Transient ototoxicity can occur at high doses of erythromycin along with hepatic toxicity Drug interaction: Erythromycin is known to decrease hepatic cytP450 and thus decrease the metabolism of other drugs. dairy and Ca interfere w/absorption Clarithromycin PO. excrete proportional to renal function o Aminoglycoside Preparations Streptomyces griseus used in selected cases of TB. less nephro and ototoxic. Chlamydia pneumonias. inhibits 30s ribosome Contraindications: DO not give to children or pregnant females. used for intestinal and intraoperative irrigation Netilmicin A derivative of gentamycin. and AIDS associated 2-5à take on an empty stomach: Al. 250-500mg po q12h (Biaxin) used for MAC and H. high pitch iltrate dose and duration related Pharmacology: Not normally absorbed from the GI tract. more active against GP. good absorption po with long t1/2 Metronidazole (Flagyl) Description Extensive use in the tx of anaerobes including Bacteroides and Clostridial infections MOA Binds to nitroreductase enzyme. diff induced pseudomembranous colitisà500mg po q6h slow infusion over 60 minutes Dose 1.0g IV then 500mg q8h Pharm Disulfram-like rx with alcohold/t accumulation of acetylaldehyde Miscellaneus Antibiotics Sulfonamides (-) folate metabolismà Bactrim Description Trimethoprim + Sulfamethoxazole MOA Competitive antagonist of PABA Adverse rx High risk of thrombocytopenia. MRSE. MRSA and some GN organisms Dose 300mg po q8-12h or 600-900mg IV q12h Pharm PO/IV/IM. coli. 150mg quinopristin/500mg dalfopristin MRSA. oral alternative to Vancomycin 100mg po bid *used in combination Topical Antibacterials (Sulfonamides) Drug MOA Indications Contra-indications Side Effects Warnings Silvadene Bacteriostatic-gr+/gr-. vomiting. Ototoxic and nephrotoxic(reversible) Spectrum MRSA. MRSE. High risk of thrombocytopenia. Diff Pharm Peak: 30-40ug/mol Trough: 5-10ug/mol Anti-anerobes: Metronidazole and Clindamycin Clindamycin (Cleocin) Description Usually used in allergies to PCN MOA Inhibits the protein synthesis of bacteria by binding to 50s ribosomal subunità bacteriastatic.0g IV q12h or 500 mg po qid for C. leukopenia Spectrum PEEK: Proteus. IV=po absorption Minocycline A tetracycline. Vancomycin Hospital acquired MRSA 1g IV q12h slow infusion over 60 min Bactrim* Community acquired MRSA Synercid Quinupristin/Dalfopristin 500mg IV q8h IV only Synergistically block bacterial ribosomal protein synthesis. E. MRSA. indicated for VREF. take care with epi. diff PMMA beads can be impregnanted with Vancomycin Dose 500 mg IV q6h or 1.1st. inhibiting creation of bacterial DNA Adverse rx Nausea. yeast Burns. pseudomembrenous colitis d/t C. fulminant group of GAS infections such as NF. The hospital acquired MRSA is significantly more resistant than the community acquired strain. cramps and epigastric distress Spectrum DOC for C. Klebsiella Trimethoprim Inhibits dihydrofolate reductase Sulfamethoxazole Inhibits dyhydropterate synthetase Drugs used in the treatment of MRSA Let’s talk a little bit about MRSA. ephedrine and 400-600 mg po q12h SSRIs. must double cover Adverse rx Diarrhea. Enterococcus. Adverse rx: thrombophlebitis. weak 600mg q12h IV infused over 40-120 min inhibitor of MAOIà increase BP w/ tyramine. there is a community acquired and hospital acquired MRSA. and potent inhibitor of cytP34A Daptomycin Rifampin* Community acquired MRSA Zyvox Inhibits 30s ribosome (bacteriostatic) Indicated for VRE. alters baterial membrane permeability and DNA synthesis Toxicity Red man syndrome: too rapid infusion will cause histamine release characteriszed by generalized erythema. drop in BP ß antidote: Benedryl.o o Doxycycline (Vibramycin) Minocycline (Minocin) 100 mg po qd-bidà DOC for Lyme dz 100mg po bid Vancomycinà GP anaerobic organisms only! Parenteral use po for enterocolitis only Description Glycopeptide derived from Streptomyces Nocardia MOA Bacteriocidal and inhibits cell wall synthesis. GP organisms. chills. 2nd and 3rd Allergy to Pts with G6Pase defà severe Silver sulfadiazine cr & pseudomonas degree sulfonamides fatal hemolytic anemia 20g and 50g 72 . fever. pseudomembranous colitis Spectrum GP and anerobes. versicolor) and systemic Blastomycosis. DO NOT GIVE IF PTS HAVE ANY LIVER OR RENAL IMPAIRMENà Fatal hepatotoxicity and Jaundice Systemic oral antifungal. Cream Chloromycetin. oint. Histoplasmosis. TB or other fungal lesions of the skin and allergy paronychia. thalamus. Contraindicated in pts with CHF and pregnant women Oral allylamineàBlocks squalene epoxidase which is necessary for the synthesis of ergosterol. elimination 2 and 8hrs Sytemic fungal infections: Hepatotoxic 1:10. flushing. fever. Aerobactor nephrotoxicity Neosporin ointment Topical triple antibiotic First Aid. and Strep. ie with BCPs. S. N/V. Tolwin. moraxella. affects rate of respiration. chills. inhibits fungal mitosis. medulla. Cryptococcus (septicemia. lotion. Onycomycosis due to Trichophyton rubrum/ mentagrophytes ONLY. neisseria Heomophilus. Itraconazole (Sporanox) Terbinifine HCL (Lamisil) 250 mg po qd x 12 weeks Fluconazole (Diflucan) 100-400mg po qd Tolfinate (Tinactin) Ketoconazole 200-400mg po qd Imidazole. vertigo fatigue. Coli. DOC: Salmonella aplastic anemia 1% cr. staph/Strep. RNA synthetase itch Griseofulvin Non-HIV antifungals Causes shrinkage and stunting of fungal hyphae. Adverse rx: Severe skin reactions (Steven-Johnson Syndrome) and Cholestatic hepatitis 12% headache. Not for ophthalmic use Bactroban 2% oint/cr by binding to isoleucyl transferepidermiditis. renal and hematologic values. hippocampus. Inhibits fungal cytoP450 sterol C-14 alpha demthylation. anoxia Metabolism: Liver to undergo glucoronide conjugationà tachyphylaxis 73 . mood changes. pyogenes Burning. euphoria. N/V. respiratory depression. pain. and sigma) in the limbic system. dysphoria). obesity. headache. Coli. impairs the synthesis of ergosterol*. Gram neg: Klebsiella. blurred vison SKIN à photosensitivity and urticaria GI à N/V/D heartburn and gas LIVER à hepatotoxic HEMATOLOGIC à neutropenia. S. Vericonazole IV triazole related to Difulcanà Invasive aspergillosis and refractory Scedosporium apiospermum Nystatin Alters PM permeability in bacteriaàOral and intestinal candidiasis. or solution (oral) Corticosporin Steroid responsive dermatoses with secondary bacterial infections. Bone marrow suppression. but basophilia and monocytosis RENAL à albumenemia Activates cytP450 decreases activity. meningitis. ointment) and IV only NOT GIVEN PO causes increased permeability of the fungal PM due to binding and interfering with the ergosterol Large number of side effects-anaphylaxis. a protein Ophthalmic us. binds keratin and has a greater affinity for diseased tissueT ½ (24hr) metaobolized in the liver to 6-methylgriseofulvin and excreted in the kidney CNSà occasional headaches. confusion. impetigo. kappa. DOCfor cryptococcal meningitis. anemia. smokers (>2ppd). Tx of skin infections caused by E. candidal and P. hypothalamus. Aspergillosis fungal infections(refractory to Amphotericin B) DON’T USE THIS DRUGà too hepatotoxic CHF and ventricular dysfunction (-) cytochrome P450. Binds the 30s ribosomeà inhibiting protein synthesis. E. can cause bone marrow suppression Serious infections of Candida. Good for Gram negative Ototoxicity and 1% cr/oint infections: Ecthyma.000 may be irreversible. and spinal cord. lethargy. indicated for superficial (dermatophytosis. phlebitis. anorexia. jaundice or simple transient elevations in liver enzymes (-) cytochrome P450. mid-brain. head injuries. thrombopenia. Not active against bacteria Amphotericin B Topical (cr. Ultram. intrathecal or bladder infusion NOT USED PO Severe systemic fungal infections. or UTI *Ergosterol is necessary for the stability of the fungal cell membrane Opiates Allergy to codeine: STUDà Stadol. cuts. convulsion. o Produces sedaton (lethargy. burs. can be fatal and blocks adrenal steroid synthesis with possible gynecomastia Miconazole IV. abrasions Allergy to aminoglycosides (neomycin) Mupirocin Inhibits bacterial protein synthesis Infections caused by MRSA (b-lactamase producers). Darvon General characteristics of all Opiates Action: o Centrally acting opiod receptor blockade (mu. not depth: use with caution with pts with emphysema. fatigue. miosis (except Demerolà mydriasis). stinging. hepatic. constipation. decreased renal function Flucytosine Synergistic with amphotericin B Monitor blood levels. do not use in PCN allergies Blocks ergosterol synthesis via inhibting fungal cytP450. pulmonary. saprophyticus. deep sleep.disseminated candidal infections (usually seen in pts with DM and renal disease) Has been associated with severe fatal hepatic toxicity.Topical Antibiotics Drug MOA Indications Contra-indications Side Effects Chloramphenicol Bacteriostaic. asthma.synthesis inhibitor Stap/Strep. Must run baseline LFTs** Biphasic T ½ . bacterial pyoderma to aminoglycosides (neomycin) Gentamycin sulfate Amionoglycoside. pain. Indications: Moderate to severe pain Contraindications: acute EtOhism. GBS. 7. hyperkalemia.30mg Generally added to drug combinations Synthetic Derivatives of the Opiates Drug Hydromorphone (Dilaudid) CIII Hydrocodone CII (Vicodin. dizziness. 4. po-MS-contin 15-200mg (less potent due to 1st pass effect) 12-64mg Preparations MSIR (immediate relief) 15 and 30mg tabs and caps q 4-6h MSIR oral solution 10 & 20mg/5ml MSContin (continuous relief) 15. asthma and allergic rhinitis Inhibition of PGE2à reduce renal Q. Lortab) Oxymorphone CII (Numorphan) Oxycodone HCL Percodan/Percocet/Tylox Oxycontin/ Oxyfast CII As compared to others 10x as potent. skin rash. 5Ht uptakeà antidepressant 30 min APAP 3-4 hrs Ibuprofen-OTC Ketorolac Tro-methamine Tramadol HCL Ultram Rx 50mg tabs 2 hrs Nonsteroidal Anti-inflamamtory Drugs (NSAIDS) 1.200mg q12h (bid) RMS (rectal MS) 5. Less nausea. ASA demonstratesà Zero order kinetics ASA Anti-inflammatory. pain. caution with barbiturates. Causes an (-) of NE. An important MOA of NSAIDS isà(-) histamine release from mast cells (-) COX 1à (-) Thromboxane formation in platelets to interfere with aggregation (-) COX 2à (-) Prostacyclin production in the endothelium Ketoprophen (Orudis)and Diclophenac (Voltaren) also block the LIPO pathway à (-) LTB4 –pain mediator NSAIDS are all à weak acids Classic triad of NSAID ODà nasal polyps. and phys dependency than MS or dilaudid 1st codeine derivative that is equal to MS and Dilaudid in analgesia. respiratory depression. and analgesia than Dilaudid Parenteral only in 1. bleeding. antipyretic. mild to mod pain. Toxicity: Hepatotoxic at 10-15g (30-46 tabs) Antidoteà Mucomist (acetylcystine) Fatal dose: 25g Allergy to ASA.60.5mg IM/ rectal suppository q4 prn pain = 10mg MS OxyIR oral caps 5mg (i cap q6h) Non-narcotic analgesics 1. perforation.Natural Opiate Alkaloids Drug Morphine Sulfate IM/subQ/po/ suppository Codeine (methylmorphine) Description Origin: Papaver Somniferum Pure mu receptor blocker Analgesic and anti-tussive (1015mg) Less tachyphylaxis Dose 10 mg IM q4-6h sub Q. renal failure. 3. rifampin. OA (-) PG synthetase Short term tx (<5d) of mod to severe pain (-) PG synthetase and platelet aggregation Centrally acting. NSAIDS Dose Moderate to severe pain: 1-2mg IM/subQ or 3. N/V. Allergy to sulfites. whereas high doses are uricosuric S/S of toxicity à tinnitus. porphyria. depression.0mg suppost 1-4mg po q 4-6h prn Mild-moderate pain Max dose: 10 mg IM q4-6h Moderate to severe pain 1-1. death. and resp. muscle aches. resp.20. binds to opiate receptors. allergy Adverse reactions: glossitis. 6. fever. contipation. headache. resp depression. 5. depression pregnancy. allergy. and cataract toxemia prophylaxis Analgesic dose: (650mg-2 tablets) 9-18 tablets Fatal dose: 10-30g Headache. N/V. alcoholics.10. 2. gastric ulcers Uricosuric effects: low doses increase plasma acid urateà gout.antiplatelet: TIA and MI. pain. sulfinpyrazone that slow down APAP metabolism Fever >6mo. & w/ CNS depressants or respiratory depression.100. hyperventilation. and sedation as MS Less euphoria.30. diarrhea. à Renal toxicity (papillary necrosis) Can use with coumadin May combine w/ narcotics not with other NSAIDSà interstitial nephritis. greater constipation. nausea and phys dependence than dilaudid or vicodin. MAOI’s Seizure risk increased. acid-base imbalance due to salicylate stimulation of the respiratory center of the medullaà hyperventilationà respiratory alkalosis. hydantoins. not an NSAID/Opiate NO RENAL TOXICITY Asthma. Greater nausea. same analgesia. and anti-tussive effects. increase renin release interstitial nephritis. drug interaction as ASA is highly protein bound Ethoism. 74 .5mg/ml. constipation. gastritis. dysmenorrhea RA. decr GFR and Na and H2O excretion. GI toxicity = 0. not active until metabolized in the liver into sulfide Renal sparing drug Little GI (=1) Tometin sulfate (Tolectin) potency > ASA. Choline Magnesium Trisalate Allergy. absorbed in SI safer w/PUD patients. agranulocytosis Oxyphenbutazone(Tandearil) Metabolite of phenylbutazone. highly protein bound GI toxicity = 3.) 75 . immediate release with peak plasma levels in 1 hour. NaphthylkanoneProdrug. anti-inflammatory. SGPT/ALT and GGT as well as the prothrombin time (production of the vitamin K dependent clotting factors. about as OA. Ruffin) T 1/2 (2-3hr) Alopecia. does not affect platelet aggregation. long T 1/2 (several days) Phenyl Methanthrine Deriv Meclofenemate Na (Meclomen) GI toxicity = 3. Which NSAID causes peripheral neuropathyà Fenoprophen Acetylsalicylic Acid Description Salsalate (Disalclid) long T1/2 insoluble in stomach. good for OA and long term tx Indole derivatives –KNOW Indomethacin(Indocin) Renal sparing drug Potency is 10-40x ASA Sulindac (Clinoril) Prodrug. moderate potent anti-inflammatory. do not combine with ASA Fenoprophen Ca(Nalfon) Anti-inflammatory potency = ASA. do effective as ibuprophen and naproxen dysmenorrhea not give to children Meloxicam (Mobic) Similar to above OA Asthma. T1/2 (2hrs). use w/caution in renal failure. long serum half-life (20-30hrs). however. preferential COXNabumetone (Relafen) 2 inhibitor.8. less GI upset = 2. 99% protein boundà not for acute inflammation b/c a very high dose is needed COX-2 inhibitors MOA Indications Contraindications Celecoxib (Celbrex) Blocks the inducible COX 2 enzyme at sites OA and RA Not a good analgesic. DTJ. Oruvail) Not a good NSAID. blocks the LTB4 pathwayà good analgesic and mod potent anti-inflam. Oxicams Piroxicam (Feldene) Long T1/2 (30-86hrs). 15-20% will develop symptoms of gastric upset 9. well absorbed with half-life of 3-4 hours. (Trilisate) can be used on someone taking coumadin Diflunisal (Dolobid) (not indic for gout) Propionic Acid derivatives little antiplatelet effect. urticaria Rofecoxib (Vioxx) Better analgesic than celebrex. acute pain Asthma. More nephrotixic tid. urticaria. excretion entirely renal Phenylacetic Acid Derivatives Diclofenac Na (Voltaren) Peak plasma levels in 2-3 hrs (long onset). 10. OA BEST effective as naproxen dysmenorrhea Laboratory Medicine Revisited Serological Evaluation Clinical Laboratory Medicine is important to appreciate that a diagnosis is not made on labs alone. Sulindac (clinoril) are à the least GI toxic. shortest T1/2 (2 hrs)Can cause peripheral neuropathy Ibuprophen(Motrin.5x phenylbutazone. Etodolac (Lodine) Nabumetone (Relafen). better for pain Oxaprozin (Daypro) Slow onset but longer duration: T ½ (42-50hrs)Weakest. erosive gastritis or PUD. less than indomethacin and Butazolidin Children with RA Pyrazolon Derivatives-Most hepatotoxic. Gastric mucosa injury is due toà (-) of the cytoprotective PGE1. Indicated for acute trauma Etodolac (Lodine) intermediate half-life (7hrs) Analgesic equivalence: GI ulceration with toxicity = 0-1 Renal: papillary necrosis. SGOT/AST. urticaria. takes three weeks to see therapeutic results Mefanamic Acid (Ponstel) Anti-inflammatory potency = 1. Renal Sparing Drug GI toxicity = 2 Hepatotoxic. as RA. most common SE are nausea. of inflammationdoes not affect platelet dyspepsia and diarrhea Allergy to sulfonamides.LDH. A general understanding of the processes unique to different cell types and organ systems is essential. or allergy to ASA. Naproxen induced hepatitis can give to pts w/ renal failure Ketoprophen(Orudis. GI toxicity = 1. A vast majority of these studies have their basis deeply routed in many of the major biochemical pathways in the body. 3-5 hrs to reach peak plasma level. run LFTs Diclofenac Na/ misoprostol (Athrotec) Combination of Voltaren and misoprostol (PGE1) Cytotec will cause spontaneous abortions Diclofenac K (Cataflam) Fast-acting. or allergy to ASA Vadecoxib (Bextra) Good analgesic. Liver Function Tests • intrahepatic disease or damage to hepatocytes can be evaluated with serum enzymes . GI toxicity = 4 Phenylbutazone(Azolid) long T 1/2 50-100 hours Aplastic anemia. I. or aggregation or bleeding time ASA asthma. labs often act to confirm or rule out a specific disease entity. once a differential impression is made. Lowest mucosal injury Naproxen (Anaprox) Longer T 1/2 (12-15 hrs) can be used in kids juvenile RA. red blood cells.very high is skeletal muscle aid in the diagnosis of MI. Other causesà & liver. following a MI → damaged tissues anaerobic glycolysis) to pyruvic acid * LDH1 release ↑LDH1 into the bloodstream disrupting the normal ratio between the two. Kidney pulmonary infarction.SGPT elevate w/ acute myocardial infarction. carcniomatosis Hepatitis.hemolytic anemia. skeletal muscle HHHM 27-37% LDH3 LDH4 LDH5 HHMM 18-25% HMMM 3-8% MMMM 0-5% Summary of Reference Values of Liver Enzymes LDH 38 . ALT rises earlier & higher than AST *Cardiac . congestive heart failure. congestive heart failure (CHF) → Ubiquitous enzyme may have ↑LDH1 & LDH5 d/t the right heart “back-up" into the liver→ hepatomegaly d/t increased venous pressure SGOT .SGOT levels may be elevated 2-fold > SGPT b/c alcohol is more directly toxic to mitochondria. lung Liver.62 U/L SGOT/AST 11-32 IU/L SGPT/ALT 3-30 IU/L 76 .almost exclusively in liver hepatocytes.GGT Liver .5 to 0. LDH1 > LDH2. cirrhotic liver. facilitates the intrahepatic damage LDH1/LDH2--Both LDH1 and LDH2 associated w/ the myocardium.is a microsomal enzyme induced by alcohol and particularly sensitive in detection of large amt present in the renal tubular epithelium chronic alcohol consumption. not to the extent of SGOT & is not used as a marker for MI's SGPT -. brain. reaction requires pyridoxine (Vitamin B6) Can be used to determine liver damage d/t ETOH * liver .SGPT parallels SGOT w/ liver damage except in the case of alcoholic liver disease. there is prevalence of ↑LDH2 >LDH1.monitored in urine to assess specific renal tubular damage Name LDH1 LDH2 Composition HHHH 17-27% % Main Location Elevated mainly in: myocardial infarction. cirrhosis Hepatitis. pulmonary infarction Pancreatitis.useful RBCs. elevation of AST in conjunction with other cardiac components (CK. Brain Brain. skeletal muscle. carcinomatosis. congestive heart failure. much more specific for liver damage than AS.liver.Serum Glutamate Oxaloacetate Transaminase/ AST-Aspartate Aminotransaminase enzyme is present primarily within the mitochondria of body tissues esp liver and heart. hemolytic anemia megaloblastic anemia.very high in myocardial tissue and occur.Serum Glutamate Pyruvate Transaminase/ALT . à LDH1/LDH2 ratio normally < 1(~0. muscular dystrophy Myocardial infaraction. megaloblastic anemia.75).SGOT elevates with hepatocyte damage. LDH2 > LDH1 >LDH3 >LDH4 >LDH5 . present primarily in the cytosol of the hepatocytes Gamma-Glytamyl Transpepidase . pernicious anemia (erythrocyte any disease process resulting in cell destruction disorders). a good marker for and is significantly elevated in hepatobiliary and in the liver disease .Tests for Intrahepatic damage Clinical Significance LDH -. renal disorders & some muscular dystrophies. LDH5 . *an SGOT (AST)/ SGPT (ALT) ratio > 2:1 strongly suggests alcholic liver damage *cardiac . kidney. Liver. becomes > 1.obstructive jaundice and infiltrative disease of the liver Renal GGT does not serum or liver.SGOT is released into the plasma with myocardial damage. healthy heart. occurs in over 95% of patients with an acute MI. muscular dystrophy myelocytic leukemia. A “LDH flip” and LDH2 . red blood cells Myocardium. rises in the aminotransferases (ALT) parallel the extent of hepatocellular damage. responsible for the transamination reaction between aspartate and alpha-ketoglutamic acids in amino acid catabolism. myelocytic leukemia. cirrhosis GGT 2-65 IU/L Myocardium. hemolytic anemia. Increaseà myocardium and muscles. Pulmonary infarction. markedly elevated w/ hepatitis and other chronic liver diseases. LDH flip and troponins) are consistent with myocardial injury *Hepatic .Alanine Aminotransferase *enzyme catalyzes the amino group between alanine and alpha-ketoglutamic acid.Lactate Dehydrogenase * normal concentrations of LDH isomers in the blood is in descending order: *found in many tissues mainly . kidney. In the conversion of lactic acid (the end product of normal. total circulating LDH levels increase in “false” flip . notably biliary obstruction. bilirubin is conjugated to bile in liver.t lymphoma (DC: direct:conjugated) the van den Bergh test is used to differentiate b/t direct and *transported in an inactive form bound to albumin.derived primarily from the catabolism of hemoglobin . fenestrated sinusoidal membrane → hepatocytes → conjugated w/ glucuronic acid → hemolysis → benign elevation of unconjugated indirect bile canaliculi → excreted in feces. free bilirubin levels build up in the *normal elevation of bilirubin may occur in patients with body.0 High density lipoproteins low risk >60 mg/dl Cholesterol/ low risk 3.4-7. synthesis is regulated by the intake of exogenous cholesterol→ a biofeedback at the synthetic level of HMG CoA reductase (beta-hydroxy-beta-methyglutaryl reductase) step. catalase and eroxidase *elevation in direct (conjugated) à biliary obstruction.0 high risk >240 mg/dl high risk >6.1 high risk <35 mg/dl moderate risk 7.kernicterus bilirubin d/t a transport defect in the sinusoidal membrane *for excretion.II.evaluate total cholesterol and high-density lipoprotein (HDL) cholesterol levels in all adults 20 yrs & older • increasing importance is assigned to the HDL levels b/c of its protective function • treatment decisions .4. condition exacerbated by fasting. cytochromes.4 moderate risk 35-60 mg/dl HDL ratio average risk 4. but liver function may be a factor Hyperlipidemia: National Cholesterol Education Program (NCEP) proposed treatment goals based on levels of low-density lipoprotein (LDL) cholesterol • recommendations . however. but no(direct) conjugated bilirubin damage or in hemolytic anemiasàd.g. enzyme most often measured to indicate bile duct obstruction *enzyme functions to remove phosphate groups from proteins and other molecules. this is referred to as "conjugated" direct bilirubin bilirubin Summary of Reference Values of Liver Enzymes that determine extrahepatic liver damage ALP bilirubin 0.0.0 high risk >11. hepatitis and breakdown heme → biliverdin → bilirubin Gilbert’s symdrome *in normal healthy individual.1 .1)present in the liver which is heat stabile and 2)the other in bone which is heat labile at 56%C (“bone burns”) and heat fractionation is actually used to help identify the source of AL. free or “indirect” bilirubin →diffuses freely through the blood-brain barrier & Gilbert’s syndrome → elevation in unconjugated. Extrahepatic Liver Damage • Elevation of alkaline phosphatase.& also a disease breakdown product of myoglobin. bilirubin and cholesterol suggest the presence of extrahepatic disease. abnormalities in cholesterol are considered to be nutritional or genetic.2 mg/dL 25-165 Cholesterol: cholesterol is synthesized in all body tissues. HDL levels should be considered in selecting drug therapy • patients with triglyceride levels greater than 1000 mg/dL should generally be treated to prevent the risk of pancreatitis • guidelines used to assess the risk of coronary heart disease include: Serum cholesterol desirable level <200mg/dL LDL/HDL ratio low risk 0.1. Enzymes Clinical Significance Alkaline Phosphatase/ALP *two isoenzymes:.0 borderline risk 200-239 mg/dl moderate risk 3. RBC hemolysis. made from precursor.0 Low density lipoproteins desirable level <130 mg/dl Serum triglycerides 10-190 mg/dl borderline risk 130-159 mg/dl high risk >160 mg/dl 77 .based on the LDL levels. indirect body tissues → (yellow discoloration) jaundiced patient . degree of phosphorylation of biologic compounds responsible for their own inherent activities (as enzymes) or structural interactions with other molecules (as in membranes) * high levels of ALP present in rapidly dividing or otherwise metabolically active cells Bilirubin overall increase in serum bilirubin is indicative of biliary * bilirubin .0-6. When there is a lack of albumin indirect bilirubin or an excess of bilirubin (e. the serum has small amount of (indirect) *elevation of indirect (unconjugated)à hepatocyte unconjugated bilirubin d/t hemolytic processes.5-3. from hemolysis). free bilirubin →through of the hepatocyte. acetyl-CoA and undergoes a lengthy process resulting in the four ring compound. *reaction occurs w/i Kupffer cells of liver utilizing the cytochrome P-450 enzymes to d/t lack of serum albumin.1-11. but in higher amts within the liver. (2) soft tissue & (3) the ankle joint.5 mg%. hot.o to CNS diseases. so if you see red. more reliable marker than serum creatinine which may be glomerulus & almost completely reabsorbed misleading in the obese. renal losses d/t osmotic diuresis.especially in children. degradation of amino acids. high-protein diet.732 2 Mildly decreased GFR 60 .: crystalization occurs more readily. swollen 4th MPJ still think gout or joint sepsis. adrenal failure. normal BUN:creatinine ratio is approximately 10:1 ratio increase >15:1à pre-renal pathology. It is a nearly ideal biologic product to measure glomerular filtration rate.6. infection.89 mL/min/1. remains unchanged by pregnancy. diuretics . IV.37° C . low hematocrit and low BUN. on light microscopy . and neoplastic states. tissue destruction & myopathies drugs in those individuals with renal failure assoc w/ Cushing’s syndrome. 30°C . freely filtered at the glomerulus . released when creatinemedication cleared renally.approx 29°C .732 Clinical Significance or uric acid • Of all arthritides. it is not reabsorbed & only a trace →alter the dose of drugs.Blood Urea Nitrogen Iincreases With severe renal disease the upper limit of acceptable BUN is urea is the end product of “amino group removal" in the very narrow. secreted and resorbed joints. reduced equation is used to determine the dose of muscle mass. elderly & malnourished patients. compartment syndrome Cystatin-C** NEW proposed as an endogenous serum marker for early assessment of changes in glomerular filtration. i. • gout attacks the foot and tophi forms on ear lobes b/c at lower temperatures → uric acid is less soluble. With a color compensator.5 fold increased risk of death from cardiovascular disease.e.732 5 Kidney Failure <15 mL/min/1. levels in upper 1/3 had a >2.Temperature of the normal ankle joint .732 4 Severely Decreased GFR 15 . urea cycle occurs in liver → urea is Elevations of . secretion of inappropriate levels of ADHß2.see a needle-like crystal piercing WBC.29 mL/min/1. Free have direct correlation to precipitation of uric acid crystals in urate is filtered by the kidney. elevated levels correlate with 120 amino acid residues. serum uric acid levels excreted via the kidney and transported in an inert form bound to albumin. time of day . urea is not an ideal index of in BUN synthesize urea. ie diabetes insipidus Ion Sodium (Na) *major cation in extracellular fluid & principal osmotic particle outside the cell. glomerular filtration and dosing of creatine-PO3 is the “storehouse" w/in muscle which provides additional ATP. 90% of all acute gouty attacks involves (1) first metatarsal phalangeal joint (aka podagra) & 15% >1 foci of involvement.8 mg%.59 mL/min/1. • gouty arthritis can occur in any joint of the body.70% of the amount filtered is reabsorbed. National Kidney Foundation recommendations for classification of kidney disease based on glomerular filtration rate together S Description GFR tage 1 Normal or Increased GFR >90 mL/min/1. freely filtered by decreased glomerular filtration rate. changes in extracellular sodium concentration → increases or decreases in the osmolality of the extracellular fluid which will affect the distribution of body water 78 . diet or drugs. Solubility of uric acid . sex or acute inflammatory states Uric Acid. trauma and neoplastic conditions. • recent evidence suggest that elevated serum uric acid levels in middle aged men → strong predictor of future cardiovascular death. only 5% in females b/c Estrogens play a protective role preventing the precipitation of gouty crystals. is filtered completely by the kidney. compromised renal function. why uric acid appears to correlate with cardiac death is unclear.732 3 Moderately decreased GFR 30 . its concentration Creatinine Creatinine used as an indicator of Creatinine is most sensitive indicator of glomerular filtrationßend product of creatine metabolism. gout has the highest predilection for the joints of the foot. • Positive joint aspiration yielding uric acid crystals →positive diagnosis of gout. causes: over hydration →triad of low serum Na.serum sodium is 135 mEq/L vary from subtle changes in mentation to profound neurologic abnormalities including generalized convulsions.decreases to 4.end product of purine (adenine and guanine) nucleic acid metabolism Not a useful indicator of any renal function. ↑ Creatinine Cockroft/Gault Equation: Basically this secreted. not affected by body mass. a basic low molecular mass protein with Normally cystatin C is not found in detectable levels in excreted urine. the crystal appears negatively birefringent. aldosterone deficit → prevent reabsorption Na-K and Na-H exchange in the distal convoluted tubules Hypernatremia seen in excessive loss of water to sodium or increase in total Na extra-renal → profuse sweating and diarrhea . primarily a male disease. Visualization of these crystals is required for the absolute diagnosis of gout. Electrolytes Hyper / Hypo Hyponatremia .25 ¼ or (25%) of nephrons are already seriously released into the blood stream → freely filtered by the glomerulus mg/100ml damaged (normal is 8-22mg/dl) and d/t concentration diffuses back into the loop of Henle à intraelevation in serum BUN Classic glomerular injury renal cycling *the ratio of reabsorbed urea:secreted urea (60^% decrease Severe liver damage & reflects the liver's decreased ability to reabsorbed: 40% secreted). This is seen with poisoning and hepatitis (drug glomerular function b/c there are many non-renal events that affect or virally induced). Statistically. it may simply circumstantial or a result of processes leading to cardiovascular injury PO3 is utilized by the muscle.Renal Function Tests Clinical Significance BUN . catabolized by the proximal tubular cells (diurnal variation). increased plasma proteins as with dehydration decreased anion gaps (<10 mmol/l) occur from Li+ intoxication. renal losses most commonly d/t diuretics. prealbumin levels are not disease so can be measured & monitored *hydration status does not affect accurate in acute alcoholics. fatty acids. it takes 14 days to return to normal Prealbumin preferred marker for protein malnutrition *less affected by * prealbumin production decreases after 14 days of consuming a diet that liver disease than other serum proteinsà a distinct marker for protein provides only 60% of required proteins. and sulfa drugs. pH and PCO2 measurements in evaluation acid-base disorders and the management of acute MIs Anion Gap • • normal value . when pool is depleted. plasma volume. 55-60% & H20 soluble). diabetic ketoacidosis.half-life is about 1 wk à respond more quickly to changes in contraceptives and possibly hepatitis nutritional status & is desirable marker. renal the stomach during digestion tubular acidosis and mineral-corticoid deficiency & some hyperPTH Total CO2: formation of CO2 during metabolism → dissociation of H2CO3 → production of HCO3→ reabsorbed by the proximal tubules. and metabolic alkalosis secretion of hydrochloric acid by the parietal cells of hyperchloridemia 2o to losses from the lower gastrointestinal tract in diarrhea. prevent artifactual hyperkalemia by collecting specimen in a heparinized tube → prevent the release of K from the platelets as clotting occurs. renal release of angiotensin → increased aldosterone and ADH → creating additional fluid retention.made up primarily of globular proteins comprised of albumin low serum protein (approx. including inflammation. increased catabolism (secondary to malignancy or starvation). alcoholism or starvation.5 order a prealbumin levels Prealbumin** TQ Serum Transferrin Causes for increase TIBC: iron deficiency anemia.methanol. oral transferrin ..[Cl.indicates decreased ability of the liver to synthesize physiologic reasons as serum protein.+ HCO3-] increased ( >17 mmol/l) .indicates significantly increased concentrations of unmeasured anions. synthesis b/c liver production will be maintained until late-stage liver prealbumin levels should rise 2 g/dl/day. determined by 2 site enzyme serum ferritin . a hemolyzed specimen → elevate the serum potassium secondary to the high concentration of potassium within RBCs d/t hemolysis Peaked T-waves on the ECG Chloride major extracellular anion Hypochloridemia: gastrointestinal losses. albumin has more specific role in the blood filtration apparatus good marker of the severity of chronic liver disease. nephrosis or other measuring the iron in the infiltrate direct loss.the presence of U-wave on EKG is suggestive of hypokalemia .Causes: serum levels become depleted. weakness and paralysis *myocardial effects → serious arrhythmias hypokalemia can occur when total amount of K in the body is normal. the cytochromes and most cellular enzymes. Albumin*plasma albumin concentration is important for the same decreased serum albumin . viral hepatitis and measuring transferring total iron binding capacity (TIBC) is determined malignancy. increase with many immuno-assay. good acting as transport protein for many things “carried" by albumin indicator of a patient’s overall nutritional status but it is relatively insensitive to include. the globulins which include the immunoglobulins.12-18 meq/l in adultsà Normal Anion Gap = [Na+ + K+] . Functional uses: important plasma buffers d/t free carboxyl and amino groups. bilirubin. has a large body pool and a albumin or an increased rate of albumin excretion due to damage of the glomerular half-life of 20 days. • V. *elevated & depressed serum potassium may have profound effects on the neuromuscular system → apathy. Bone Metabolism Tests 79 . VI. loss of gastrointestinal fluid d/t vomiting/diarrhea. a source of stored protein that can be catabolized in starvation. ketotic states e.readily available to synthesize adequate serum proteins. mineral-corticoid excess. serum concentrations affected by the patient's hydration and renal aspirin and many of the sedative hypnotics function. 2o to uremia. multiple myeloma. the penicillins. lipoproteins. including tetracycline. necessary for →proteins filtered through the maintenance of plasma oncotic pressure.this is an upright wave at directly follows the T-wave hyperkalemia occurs in acute and chronic renal failure. provide insight into a patient’s nutritional “leaky" glomerular filtration apparatus status. uric acid. serum proteins . *slight post prandial depletion of serum chloride 2o to salt-losing renal diseases. with nutritional supplementation. pregnancy. polyclonal gammopathy and polymyxin B therapy Serum Proteins: All are synthesized in the LIVER Protein Clinical Significance Total Serum Protein: serum protein . ingestion of toxins . measuring total iron binding capacity is equivalent to chronic disorders.Potassium major intracellular cation . total CO2 determinations. movement from extra → intracellular fluid occurs in akalemia and insulin therapy.g. hyperMgt. removing the unbound iron and then Causes for decreased TIBC: reduced protein synthesis. salicylate.sensitive indicator of iron deficiency. by saturating transferrin with iron. causes: failure of the liver hepatocytes fibrinogen. If serum albumin < 3. diabetes. lactic acidosis. Vit C. infection. compensated respiratory acidosis. depletion of plasma proteins → fluid loss into the interstitium → podocytes or basement membrane. numerous short term changes in nutritionàhypoalbuminemia → ascites and expanded antibiotics.only 2% of total body potassium is extracellular. ethylene glycol. hemoglobin. Tests commonly ordered - alkaline phosphatase, serum calcium and serum phosphorus. Serum calcium and phosphorus levels are dependent greatly on the level of circulating parathyroid hormone. Tests Alkaline Phosphatase * released by osteoblasts when they are actively secreting bone matrix *diffuses freely into the bloodstream and elevated levels may be indicative of osteoblast activity • Clinical Significance pronounced elevation (5x or more) will occur in bile duct obstruction, biliary cirrhosis, osteitis deformans (Paget’s disease), osteogenic sarcoma and hyperparathyroidism • moderate elevation (3-5x normal) can occur in, granulomatous or infiltrative diseases of liver, infectious mononucleosis, metastatic tumors in bone, metabolic bone diseases (rickets, osteomalacia) • slight elevation (up to 3x normal) can occur in viral hepatitis, cirrhosis, healing fractures, pregnancy and normal growth patterns in children hypocalcemia - secondary to aklalosis, renal failure & hypoparathyroidism hypercalcemia - secondary to acidosis and "CHIMPS" - cancer, hyperthyroidism, iatrogenic causes, multiple myeloma, parathyroidism and sarcoidosisTQ • hypercalcemia → depressed neuromuscular excitability with anorexia, drowsiness, nausea, polyuria and polydipsia (increased urine flow & increased thirst) d/t high calcium levels which affect the kidneys ability to concentrate urine (i.e. increased calcium levels lead to increased urinary excretion of more dilute urine). Calcium most abundant cation calcium & phosphorus are considered together in routine clinical evaluations; The kidneys are important in the regulation of calcium; with renal failure- calcium levels fall & phosphate levels rise Phosphorus these ions are in flux continually controlled by the action of parathyroid hormone ( vitamin D and calcitonin) PTH elevates calcium levels by increasing the resorption of calcium from bone and suppressing loss into the urine, vitamin D decreases Calcium levels by promoting its absorption by the intestine and accelerates turnover of the minerals in bone; phosphorus is necessary for every insulin-mediated transfer of gl- into a cell Elevated serum phosphorus - healing bone fracture & some bone tumors. Severe hypophosphatemia - DM d/t renal manifestations o (DM Ketoacidosis) VII. Evaluation of Muscle Clinical Significance cardiac elevations - if the CK-MB band is elevated during the 48 hours post event and the LDH isoenzyme pattern is flipped, a reliable diagnosis may be made rises in the serum within 2 to 8 hours of an acute MI; serial measurements taken q2-4 hrs for 12hrs after the onset of symptoms will be indicative of myocardial injury if values are increasing; Tests Creatine Kinase:elevations occur with any type of muscle damage; CK enzyme levels first to elevate in acute injury; muscular dystrophies - serum CK elevates as much as 200-fold (normal being 0-12 units/ml); three isoenzymes: = BB or CK1 - high percentage in the brain - ~90% MB or CK2 - about 20% in cardiac muscle MM or CK3 80% in cardiac muscle; exists in a high % in skeletal muscle - 99% - the remaining small percentage is in the MB form. Pathological Increases MB isoenzyme - w/ myocardial injury & muscular dystrophy - 15 to 40%;** MM isoenzyme - elevated with skeletal muscle injury BB isoenzyme - seldomly encountered clinically; associated w/ brain necrosis, large intestine necrosis and Reye's syndrome normally, CK will drop within 1 - 3 d following the incident; if values stay elevated - indicative of reinfarction or extensive myocardial damage myoglobin found in both skeletal and cardiac muscle; very sensitive indicator of muscle injury; rise in myoglobin will elevate even before the CKMB circumstantial evidence as skeletal muscle injury will also cause it to elevate troponin I structural components of cardiac muscle; highly specific for myocardial injury; begin to increase w/i 3-12 hrs after injury; troponin I and T remain elevated for 5 - 9 d; troponin T - remains elevated for up to 2 wks; continued elevation make them of little value in reinfarction Aldolase becomes elevated - following skeletal muscle disease or injury; secondary to metastatic carcinoma (especially liver), glycolytic enzyme granulocytic leukemia, megaloblastic anemia and hemolytic anemia; B/c of a wide range elevating factors it is not useful involved in the except for the evaluation of inflammatory disease of the muscle (e.g. dermatomyositisà Groton’s lesions) & are elevated metabolism of glucose early in pts who later develop muscular dystrophy; An increase in CK-MM with aldolase is indicative of skeletal muscle distributed in all tissues injury ** Elevations of the MB band are consistent with cardiac injury even though there is more of the MM band in cardiac tissue. The MB isoenzyme is "more unique" to cardiac muscle than any other tissue type. **characteristic pattern of elevation of serum markers associated with myocardial disease** TIMELINE 0—24 hours 24-days troponin levels peak at 72 hours & remains elevated for 10-14 days; but • high levels of troponin (w/i 24hrs) and CK(w/i 2-8 hrs) are extent of elevation is not as striking as AST- the second enzyme to rise earliest to rise after myocardial injury pattern of normal AST and elevated LDH within 1-2 days after an episode of • the LDH “flip” occurs where levels of LDH1 > LDH2 ; occurs chest pain suggests pulmonary infarction within 12-24 hrs and is present w/i 38 hrs in 80% of patients VIII. Evaluation of Pancreas 80 Amylase 2 major sources & exists as 2 isoenzymes - pancreatic or P-form & salivary or S-form • acute pancreatitis serum level elevate 4-6x above normal; rise within 6 - 24 hrs & return to normal in 48 - 72 hrs despite continued presence of pancreatic disease; à no correlation b/t severity of the disease and the amount of amylase elevation; • elevations in serum amylase - almost always secondary to pancreatic sources; o morphine elevation in serum pancreatic amylase o bone marrow ablation and exposure to nuclear reactor accidents (i.e. 2o to ionizing radiation) elevation of salivary amylase • pronounced elevations in amylase (>5x normal) occur with acute pancreatitis, pancreatic pseudocyst and the administration of morphine • moderate elevations in amylase (3-5x normal) occur with pancreatic carcinoma, mumps or a salivary gland infection and with a perforated peptic ulcer Lipase elevated serum lipase is diagnostic for acute pancreatitisà almost exclusively in the pancreas; technically more difficult measure than amylase which is why amylase is ordered more commonly; not cleared by the urine and remains elevated in the serum much longer than amylase • an elevation in serum lipase may occur following the administration of heparin as a result of the release of lipase from the vascular endothelial membranes IX. Evaluation of Serum Glucose * The ambient temperature at which the blood sample is kept will affect the rate of glycolysis; refrigerator temperatures - glucose will remain stable for several hours; room temperatures - loss of 1-2% of glucose per hour à Use fluoride-containing (gray top) tubes which inhibit glycolysis to circumvent this problem Fasting Plasma Glucose: the best indicator of overall glucose homeostasis and the best screening test for diabetes; fasting - no food or beverage for 8 hrs before testing; diagnose diabetes by fasting plasma glucose - above 126 mg/dl (7mmol)on 2 or more occasions; blood glucose finger stick utilized for easy monitoring; drugs - glucocorticoids and nicotinic acid hyperglycemia Casual Plasma Glucose above 200 mg/dl (11.1 mmol) - pathognomonic of diabetes mellitus; when elevated, a fasting plasma glucose or glucose tolerance test should be performed on a different day Oral Glucose Tolerance Test: Diagnostically useful when fasting or postprandial blood levels are equivocal; useful if there is a good reason to suspect diabetes eg. an obese pt w/ a + family history or in pt w/ unexplained vascular, neurologic or infectious illness; standard glucose load should be 75 grams; baseline blood glucose is taken and the glucose load is administered; 2 hrs after the load, plasma glucose should return to the fasting level ; persistent elevation at two hours is abnormal • mild elevation at two hours but return to normal at three hours suggests slightly impaired glucose metabolism but not overt diabetes; • Advancing age declines in speed of glucose clearance; in a non-diabetic patient with a negative family history, levels routinely elevate 6 mg/dl for every decade over 30 years of age. Glycosylated Hemoglobin - A1C test: test is a valuable tool in monitoring glucose control; the beta chain of Hb acquires & irreversibly binds a glucose moiety when plasma glucose levels are elevated resulting in "glycosylated" Hb; glycosylation does not affect O2 carrying potential but it does reflect periods of glucose control in the preceding two to three months; normal individuals have 3 to 6% of their hemoglobin glycosylated in the form of A1c; HbA1c levels >10 signal necessity for better glucose control; Test should be done q3mths Glycosylated Albumin NEW, check more short term glucose control (window into the past 2 weeks) b/c albumin has a serum T1/2 in the circulation of about 17-20d, amt of glycosylated albumin reflects hypergl- periods w/i the previous few wks; < 8% may be normally glycosylated Diagnosis of Diabetes Mellitus • Fasting glucose: >126 mg/dl - diabetes o 100 - 126 mg/dl - glucose impaired o <100 mg/dl - normal • Casual glucose: >200 mg/dl on two separate occasions plus symptoms of diabetes • Glucose tolerance: >200 mg/dl two hours after a 75 gram glucose challenge - diabetes o 140 – 199 mg/dl two hours post challenge – glucose impaired o <140 mg/dl – normal 81 Other Test C-peptide Measurements: the secretory rate of C-peptide to insulin is 1:1; however, the ratio in serum is 5-15:1; 50% of insulin is removed on its initial passage through the liver, however, hepatic removal of the C-peptide is negligible; measured at times to evaluate the residual function of the islet cells in the pancreas; diabetics w/ no residual islet function are considered to be "brittle" and are frequently quite difficult to control; Brittle diabetics produce no insulinà Normal to high levelsà Type 2 and absent to low levelsà Type 1 C-Reactive Protein – CRP: an acute phase reactant; demonstrates a rapid rise in response to acute disease and a rapid clearance once the stimulus has abated most sensitive indicator of inflammation regardless of etiology and a useful marker in the evaluation of the efficacy of empiric therapy with infectious diseases & recently, CRP levels have been used as a screening tool for future coronary eventsà levels should decrease if risk decreases; currently, there is no definitive guidelines for ascribing risk categories, however, widely acceptable values are: CRP levels < 1 mg/Là low risk CRP levels from 1-3 mg/Là average risk, and CRP levels > 3 mg/L à high risk Homocysteine plasma homocysteine is a useful predictor of CV morbidity and mortality; elevated levels → atherosclerosis via: damage to the arterial intimal cells via direct toxicity; interference with clotting factors & oxidation of low density lipoproteins; blood is drawn after a twelve hr fast Ø normal: 5 – 15 µmol/L Ø moderate risk 16 – 30 µmol/L intermediate risk 31 – 100 µmol/L severe risk > 100 µmol/L Complement Measurements Ø C3 reference values: 75 - 175 mg/dl; C4 reference values: 14 - 45 mg/dl *most abundant complement proteins in the serum; consists of a series of proteins that work to "complement" the work of antibodies in destroying bacteria; circulate in the blood in an inactive form; *levels of serum complement proteins can be related to autoimmune disease activity low levels indicate depletion of complement proteins secondary to their activationà Ag-Ab reactions, colleen vascular and c.t. diseases normal or high levels indicate disease regression or response to therapyà Carcinoma and Ulcerative colitis Synovial Fluid Analysis • Joints normally have small amt of synovial fluid & depends on the size of the joint; normally b/t 0.1 - 3.5 ml unless there is a joint effusion; the fluid collected using aseptic technique and placed in a sterile, heparinized tube (green top) or EDTA; the use of other anticoagulants may cause artifacts during microscopic analysis • examination for the presence of microorganisms and crystals is the most critical part of the analysis; most findings are non-specific and can be found in a number of disease entities; fluid is inspected for viscosity, color and appearance • Microscopic examination - total number of cells present, the type of cells and the presence and type of crystals • Chemical analysis - mucin clot test, synovial fluid glucose and protein determinations, and immunologic studies (rheumatoid factor, antinuclear antibodies, complement,etc.) • Microbiologic studies, including a Gram stain and cultures Surgery, anesthesia, hospital protocol A. General Anesthesia-a reversible state of unconsciousness produced by anesthetic agents, with loss of sensation of pain over the whole body. The order of descending depression of the CNS during anesthesia is: cortical and psychic centersà basal ganglia and cerebellumà medullary centersà spinal cord TRIAD OF ANETHESIA: UNCONSCIOUSNESS, ANALGESIA AND MUSCLE RELAXATION Class 1 Class 2 Class 3 Class 4 Class 5 E ASA classification for Anesthesia Normal Healthy-no known diseases Mild Systemic Disease Severe systemic disease (not incapacitating) Incapacitating Systemic disease (is a threat to life) Moriboud patient who is not expected to live with or without surgery Emergency Operation Anesthesia staging Stage 1-Amnesia and analgesia -plane 1: preanalgesia, memory and sensation intact -plane 2: partial amnesia and analgesia -plane 3: total amnesia and analgesia Stage 2: Delirium 82 non-irritating. increase hypotension bronchodilator. fauces and uvula. depression. non-flammable. and the first signs are: • Tachycardia Tachypnea ventricular arrhythmias Acidosis • Sweating Arrhythmia Cyanosis Hypoxemia • Muscle rigidity fever (late s/s/) Hyperkalemia Causes: Reduction of Ca uptake by the SR necessary for cessation of muscle contraction Tx: Sodium Dantroleneà 2. postoperative asthmatics shivering decreases HR Isoflurane Rapid induction and recovery. respiration. nonemetic.-unconsciousness. fauces. w/increase depth of anesthesia. regular breathing -plane 3: partial intercostal paralysis. Class III visualization of the soft palate and the base of the uvula Class IV soft palate is not visible at all What is trismusà masseter muscle spasm Thyromental distance-thyroid motch to mental protuberance of the mandible should be larger than 6. pleasant smell. irritating to mucous bronchodilator. BP. liver Least potent. No muscular relaxation. Rapid induction and recovery. non-irritating. recovery. Volatile liquids Chloroform Diethyl ether Types of anesthetics: Inhalation agents Advantages Disadvantages rapid induction/ recovery. non-irritating. faint lid reflex. may cause seizures! compatible w/epinephrine Nephrotoxic and hepatotoxic Methoxyflurane Most potent. Increased risk of spontaneous abortion with prolonged use Hydrogenated Flourocarbons Halothane Most potent. respiratory and cardiac failure -plane 1: Reversible respiratory paralysis -plane 2: Irreversible cardiovascular and respiratory arrest Malignant hyperthermia. non-irritating. membranes of upper airway. hypotention. uvula. nonflammable. No longer in use. bronchodilatorà may be safely used in action of catecholamines. dangerous in patients w/full relatively safe stomachs(emetic) flammable and explosive NO Little effect on the HR.a condition characterized by acute idiopathic muscle over activity and increases in oxygen demand. nonarrhythmia-producing drug. involuntary movement Stage 3: Surgical anesthesia -plane 1: Sleeping. lowest MAC. compatible w/epinephrine Enflurane Pleasant smell. maintains ability of CV system. hypotonia -plane 4: complete intercostal paralysis and respiratory arrest. diffusion hypoxia. Class II Visualization of the soft palate. possible non-emetic. Great margin of safety. eyeball centrally fixed. Rapid smooth induction and Myocardial depression. good m relaxation No longer in use. CNS irritant. O2 100% must be given at termination of sx to prevent or kidney metabolism. shivering post-op. Depresses the cardiovascular system. may trigger malignant hyperthermia. requires artificial ventilation Stage 4: Medullary paralysis. smooth muscle relaxant. No longer in use. tachycardia.5cm 1. Halothane hepatitis. non-emetic. The skeletal muscle goes into spasm. rapid induction/ recovery. respiration stimulated. circulation not depressed.5 mg/kg (may repeat up to 10mg/kg) Suspect in pts with Ø Familial hx of MH Muscular dystrophy Pts on halothane and succynylcholine Mallampati classification of the soft palate Class I Visualization of the soft palate. least volatile anesthetic. good muscle relaxant. myocardial contractility. recovery. bronchodilator. Disadvantages: Myocardial depression. irregular breathing. anterior and posterior pillars. Prolonged induction of anesthesia and prolonged good m relaxant. hepatotoxic Reliable signs of anesthesia depth. sensitizes the myocardium to the flammable. intense analgesia compatible w/epi. good muscle relaxation. nonflammable recoveryà Nephrotoxic MAC-minimum alveolar concentration (the higher the value the least potent the agent) 83 . non-emetic. and fatal agranulocytosis from prolonged administration or Very high MAC* exposure. Myocardial depressant. possible bone marrow bronchodilator. mydriasis. maintains acute or delayed liver injury (less likely than Enflurane or Halothane) stable cardiac rhythm. prolonged induction. Ischemic heart disease Smokers Pts with liver disease Alcoholics Pts with renal disease Pts with DM Gouty patient Pts with sickle cell The rheumatoid patient Chronic steroid use Diuretic use Perioperative conditions and medication cessations Wait at least 6 months after before elective surgery Must perform a pulmonary function test. pain controlled w/oral analgesics. arterial blood gases. heart irritability. no liquids for 3 hours. spinal neuraligia. tolerate foods by mouth. Local Anesthesia blocks Saphenous nerve-the only nerve at the ankle that comes from the femoral nerve. peridural Local anesthetic medication is Hypotension caused by sympathectomy hematoma. dizziness. use 5% IV dextrose. injected in the epidural space just Late complications include: postural HA. no bleeding through bandage. LOC evaluated by a simple motor responseà “squeeze my hand” Discharge Vital signs w/in 20% of pre-op for 30 min. lies in the posterior compartment above the lacinate ligament • Sural-made up of branches from the tibial and common peroneal nerves • DPN-les between the EHL and Anterior tibial tendons • Superficial peroneal-becomes superficial 7-8cm above the ankle Ankle Block Deep peroneal Intermediate dorsal cutaneus nerve Medial dorsal cutaneus nerve Sural nerve Saphenous nerve Posterior tibial nerve Digital block 2 dorsal digital proper nerves 2 plantar digital proper nerves Hallux block 1st dorsal digital proper nerve Deep peroneal nerve 1st-2nd plantar digital proper Mayo Deep peroneal Intermediate dorsal cutaneus nerve Medial dorsal cutaneus nerve Medial plantar nerve Saphenous nerve Mini-Mayo LCDN4th common dorsal4th common plantar 3. coumadin is highly plama bound and may potentiate depressant effect Discontinue ASA 2-3 weeks prior to surgery Arrythmia medications Hypertensive medications Psychotropic medications Bronchodialators Anticoagulants Pre-op considerations No solid foods for a minimum of 5 hours. circulation to toes intact. Regional Anesthesia 1. anticoagulant tx. angina and arrythmias if discontinued Continue these medications Continue these medications Discontinue these medications 3-6 days pre-op. DJS.2. afebrile. can ambulate with assistance Re-admit post-op 4 reasons Uncontrollable pain. can not ambulate B. Balance btwn sedation/ amnesia yet patient can maintain own airway. asthma. medical monitoring is needed. untrollable nausea. consider NPO status Discontinue chlorpropramide and oral hypoglycemic medications Sx can precipitate attack. Abnormal K+ will cancel surgery Continue these medications Continue these medications-may produce HBP. spinal compression. Techniques of local anesthesia Ø Local Anesthetics: weak bases that may exist as non-ionized/ionized molecules: Two anesthetic types: 84 . Decrease gastric fluids and acidity with Tagament 300mg IV (histamine 2 antagonist) and Reglan 10mg IV (Dopamine antagonist Conscious Sedation Sedative/hypnotic and opiod are titrated slowly.Should be considered whenever general anesthesia poses greater risk Contraindications Location Side effects and complications Hypovolemia. obesity. BP stable supine/standing. pre-existing NMD. continue 2 days postoperatively otherwise LBP and CV collapse may occur Causes hypokalemia will lead to prolonged non-depolarizing muscle relaxant activityà digitalis toxicity. Spinal Anesthesia. 2. COPD. cauda equine local/systemic sepsis or the debilitated host unlike an epidural which injects syndrome and epidural hemotoma just outside the sac. do cervical spine X-ray Increase steroid therapy. muscle relaxants and halothane are contraindicated in these patients Must perform LFTs and Kidney functions tests muscle relaxants and enflurane are contraindicated in these patients Anesthesia masks hypoglycemia Give ½ morning insulin. able to void. use colchicine 0. sensation and motor return to epidural limbs. weakness. below L2 into the dural sac. lies medial to the greater saphenous vein • PTN-branch of the sciatic. and repiratory depressants-use local anesthetics Reduce MTX therapy. It is meningitis. orientedx3.5mg 3xd for 2-3d prior to surgery and 4-5d post-op Avoid tourniquet use. Post-op complicationsà atelectasis Succinylcholine. arteriosclerotic dz. Clark’s rule: used for children >1 yr Weight of child in lbs x Adult dose 150 C. sufficiently long duration Soluble in saline and water Sterilizable & compatible with vasoconstrictor drugs Non-irritating to tissues Stabilizes the axonal membrane By stabilizing the nerve membrane. low system toxicity Action confined principally to nerve tissue Short onset time. Pediatric a. since halothane sensitizes the myocardium in the presence of exogenously administered catecholamine Conversion 1% solution has 10mg/cc. and PVD o -can create reactive hyperthermic reaction o -should be avoided in pts using Halothane. w/high potential for allergenicity d/t PABA Amide-metabolized by the microsomal enzyme system of the liver (long T1/2)-no cross allergenicity with esters Characteristics Actions Completely reversible. Elderly-1/2 or less of adults dose 2.Ø Ø Ester-ester linkage is metabolized in plasma by Psedocholinesterase (short T1/2).000 solution contains 0. because it inhibits the ability of the nerve membrane to alter its permeability to the larger NA ion.5mg/kg body weight 7mg/kg bw Toxicity Potency Comparable to procaine Comparable to procaine No children <12 yrs Do not use w/ renal dz Advantages: o Reduces the vascularity locally at the site of injection o Reduces the absorption rate of the local anesthetic o Permits higher allowable single dose of local anesthetic to be used o Increases the duration of action of the block o A 1:1000 solution contains 1mg/ml of epinephrine o A 1: 200. This is a NON-DEPOLARIZING block and prevents the generation and conduction of nerve impulses (sensory>motor) The ester group of local anesthetics Drug Cocaine Tetracaine (Pontocaine) Procaine (Novocain) *Chloprocaine (Nesacaine) Vaso-constrictive Topically active Yes Yes Yes Used for spinals Yes NO -NO Max dose w/epi 150kg 100 mg 1000mg 1000 mg Dose 2mg/kg bw 1mg/kg bw 10mg/kg bw Onset Intermediate Slow <15 minutes long acting Rapid 2-5 min Slow 6-12 minutes Duration Short-20-60 min 2-3hours Slow metabolism Short 30-60 min Short 30 min Toxicity Potency High 10x > procaine ¼ coke Low Safest Drug Lidocaine Lidocaine W/ epi Bupivicaine (Marcaine) Mepivicaine (Carbocaine) Max dose 300-350mg 500mg 175mg 225mg w/epi 500mg 400mg w/epi The amide group of local anesthetics Onset Duration Very rapid <2 1-2 hours-shortest minutes Very rapid Duration is doubled 24 hours 2mg/kg/ body Intermediate 4 hours-longest.5mcg/ml of epinephrine Contraindications/cautions: o -should not be used for infiltration around end arteries. 24h as analgesic 7mg/kg bw Rapid Short (max 400) 2-5 minutes 1-2hour Epinephrine Dose 4. Intravenous sedation = 20mg/cc xylocaine plain = 15cc’s 300mg = Adjusted dose 85 . The threshold for excitation increases with the [ ] of the anesthetic. HBP. 12h weight 5 minutes w/epi. CVD. these drugs prevent Na and K exchange in action potential depolarization. can lead to tissue necrosis-should use diluted epi at the digits o -use cautiously in pts with hyperPTH. the maximum dose can be calculated in cc’s as % solution converted mg/cc = max cc’s or Max mg Special cases for lowering maximum dose 1. Most commonly used to acytelcholinesterase inhibitors (pyrodistimine.5mg droperidol Chloral hydrate One of the oldest and best hypnotics and a very good alternative to barbiturates Induction dose 0. No myocardial depression.05 Fentanyl citrate + 2. mild Diazepam (Valium) immediate 15-30min anti-anxiety. but is Vistaril used primarily for sedative properties. IM or IV) Hallmark of ODà pinpoint pupil constriction.5-1g po in the elderly or children Ketamine Hypnotic. reduces the number of Short acting(<30min) Mivacurium receptors accessible to acetylcholine molecules by the motor nerve. antiemetic and bronchodilating properties. Dose 10mg q3h (subQ. Inhalation agents are not used first because will cause muscle relaxation and cessation of breathing.5-25 to counteract nausea absorption Produces depression of ventilation which is short in duration. molecular size and state of ionization (only non-ionized molecules freely diffuse) determine effectiveness Induction Agent Most commonly used induction agent Induction dose 3-6mg/kg Ultra-short acting Barbiturate Onset <30s Extravascular injection can produce tissue necrosis/arterial injectionà spasm Duration 5-10min Neuroleptic drugs Propofol Cleared by liver at a much faster rate. Surgical Principles 86 . sedative. Only effects skeletal muscle and not the smooth muscle of the gut. anti-convulsant. May produce post-op LBP and lethargy Compazine 5-10mg Phenergan 25-50mg D. pipecuronium Agents combine reversibly with postjunctional cholinergic-nicotinic agents Inter acting (30-60min)-atracurium. amnesia sedation during local anesthesia. neostigmine. pancuronium. Analgesia. used for Midazolam (Verded) 1-5 10-30min sedative.5mg/kg Diprivan following induction dose is more rapid than any other IV anesthetic agent Onset 15-45s Duration 5-10min Inovar Analgesia. will promptly reverse respiratory depression of opiods 100ug acts in 2 min Tranquilizers Preanesthetic medication b/c of their sedative. sedative Anti-emetic. Hallucinogen produces Induction dose 1-2mg/kg body wt IV over 1 min dissociative amnesia. duration 4-7hrsß antagonists hepatic disease increases duration Will not decrease HR. anti-histamine) 25 or Phenergan 12. or anesthesia Analgesia only. antiemetic. time from emergence to full recovery Induction dose 1. Major site of action at the cholinergic and nicotinic receptors at the motor end plate. No CV side effects. antihistaminic and Thorazine 15-25mg temperature regulating effects. Other s/e: Nausea. amnesia respiratory depression. gallmaine. Antidiarrheal=constipation.SubQor PO) Poor po sedative. sedative Opiod antagonist. therefore side effects can be unexpectedly severe • All are highly protein bound by albumin (over 80% except ketamine 12% which causes it to be so nephrotoxic because only nonprotein bound molecules are renally cleared) • Lipid solubility.2mg/kg Lorazepam 60-120 min Morphine Meperidine (Demorol) Fentanyl (Sublimaze) Narcan Phenothiazines Opiods-analgesic but no amnesia.4-2. vecuronium receptors without opening Na channels. itching and urinary retention.C orthopedic med: Combine with Vistaril (antianxiety. M.5-2. antihistamine. facilitate tracheal intubation-May cause malignant hypothermia edrophonium) recovery depends on washout of agent from MEP Non-depolarizing Long acting (>60min). -useful in traumas where BP drops of analgesia per single dose Etomidate Can be used safely in cardiac and asthmatic patients Muscle relaxants Description Compounds that block the transmission of neural impulses between motor nerve endings and muscle fibers. hynoptic.0mgq2-3min) Short acting (30 min). Depolarizing Succinylcholine-2 acetylcholine molecules chemically linked. Excellent in pts w/hx of bronchial asthma Onset(min ) Duration (min) Long duration of action. Dose 100mg q3h (IM. Preanesthesia sedation.2mg Thiopental Pentathol What is the MOA of death by narcoticà respiratory depression What anesthetic can be safely used in cardiac and asthmatic patientsà Etomidate Sedatives Hydroxazine Benzodiazepines Sedatives and hypnotic agents Used before surgery to relieve anxiety and tension-cerebral cortex depression Pentobarbitol and Secobarbital Has sedative.Intravenous anestheticsà used first b/c of rapid distribution. short duration (3-5min)-must be broken down by agents combines reversibly with postjunctional cholinergic-nicotinic plasma pseudocholinesterases. will not respond to receptors opening Na channels. Antidoteà Flumazenil 0. Asthmatics react poorly to MSO4 b/c of PCA pump DOC (5mg/hr) histamine releaseà smooth m constriction which will lead to LBP. 40min bad dreams. intraoperative Onset-almost immediate Reversed by narcotic antagonist (Naloxone 0. Onset in 60s. • A disadvantage of IV anesthetics is the inability to quickly reverse the effects. and anti-adrenergic effects-LBP 0. Reversed by aceytlcholnesterase inhibitors and atropine 1. analgesia and amnestic effects. Then intubation is used to control breathing and then inhalation agents are used. counteracted by histamine onset: 20 min. Common in Kidsà emergence frequently assoc w/ Duration: 10-15min of sleep and anesthesia.curare. interfere with capillary budding. Phases of Wound Healing Phases Inflammatory Phase Duration Immediate to 2-5 days Principal cell Platelets and Neutrophils Hemostasis Vasoconstriction Platelet aggregation-d/y exposure of the platelets to vWF8 in the damaged endothelium End result is activation of X which activates thrombin which converts fibrinogen to fibrin which forms the clot. subsequent tissue destruction and abnormal immune response Venous Stasis disease Collagen Vascular Diseases Treatment of Non-healing wounds-see wound management Bone healing Primary bone healing Secondary bone healing (Callus) Fracture reduction Six phases Fracture stabilization-bypasses the Hematoma formation-btwn fx fragments (1-3d) Hematoma organization-inflammation (3-10d) formation of a fibrocartilagenous fibrocartilagenous callus-osteoclastic phagocytosis of necrotic bone with osteoblastic differentiation into callus with simultaneous bone cartilage –low O2 tension or fracture instability or bone-high O2 tension/stability (10d-6wks) formation via Haversian remodeling primary bone callus-condensation of fibrocartilagenous callus into bone that bridges fracture interface (6Preservation of fragment vascularity 10weeks) primary bone callus absorption-new bone remodeling into 2o bone callus (2. Inflammation ß Histamine Vasodilation and Phagocytosis via Neutrophils and macrophages(which appear in the 1st 5 days and have a long 7-10 d life span) Proliferative Phase Remodeling Phase 2 days to 3 weeks 3 weeks to 2 years Macrophages Fibroblasts Contraction (0. Wound Healing: The entire wound healing process is a complex series of events that begins at the moment of injury and can continue for months to years.occurs at the same time as original tissue migration and is under the influence of GFs released by MACs.1. slow fibroblast proliferation as well as the rate of epitheliazation Cytotoxic drugs inhibit cellular proliferation and slows wound healing but not prevent it Radiation-leads to reduction in perfusion b/c of cellular destruction Diseases which are associated with or predispose to chronic wounds Diabetes Mellitus Deposits in the arteries interfere with tissue perfusion DM neuropathy leads to reduced sensation and gait abnormality Metabolic problems lead to a reduction in nutrients available for wound healing Impaired phagocytosis seen as part of the disease spectrum on diabetics lead to increase in bacterial infections.7mm/day) New collagen forms Wound edges pull together to reduce defect-tension on the which increases tensile wound stimulates production of collagen.6-0.5-4mo) remodeling-alteration of bone in response to applied forcesà Wolf’s Law 87 . and may reverse wound healing problems assoc w/ steroid use VitCà scurvy-is important in the enzymatic synthesis of collagen Steroid and Cytotoxic medications Steroids slow protein synthesis when given exogenously. percent as strong as fills defect. such as collagenases Ingestion of bacteria and cell debris Capillary formation-neovascularization Antigen processing/presentation Ø Ø Factors that influence wound healing Age Inadequate perfusion Infection Edema Poor nutrition Vitamin and Trace Element deficiencs VitA def can interfere w/wound healing. the tensile Epithelialization Crosses moist surface strength has increased by Cell travel about 3 cm from point of origin in all directions -rate 40-50% of migratory movement is proportional to the pO2 which is highest under HBO condition Scar tissue is only 80 Neoangenesis -Granulation Fibroblasts lay bed of collagen. therefore excessive strength to wounds tension should be avoided and steristrips used whenever possible By 1 month. produces new capillaries. The wound reaches 35% of it’s original strength by day 14Scar revision should be at this time the tensile strength of the wound=tensile postponed for 1 yr strength of the suture that can now be removed Activated macrophages are the most important inflammatory cells involved in wound healing because they are the only cells able to tolerate the low oxygen levels at the wound edge and thay are involved in: Wound debridement-accomplished by the production and secretion of proteinases. Gradual progressive strengthening will occur w/subsequent passive/active exercises Nerve Healing Seddon’s classification Neuropraxia Axonotmesis Neurotmesis slight damage to nerve with transient loss of conductivity. Alloimplant. torn or destroyed. No structural stability.non-living tissue Less effective incorporation Absence of osteoconduction transfer. few viable cells. immunologically compatible. . radioopaque when healing. many viable cells. but the structural framework of the nerve remains intact. immobilization. overly employment of Electrical bone Hypertrophic (Vascular/reactive) Elephant’s foot. 88 . fills defects. OM. reduction and/or bone grafting. and local collagen. highest success Second surgical site Limited quantity rate Stress fracture at the surgical site Allogenic Allograft. Gap. Wallerian degeneration occurs in the distal segmentà bulbous neuroma.promotes bone growth Without gap. Description Advantages Disadvantages Cortical Dense. can determine vascular compromise differences with bone scan Pseudoarthosis When a non-union involves a fibrocartilagenous interface between the fracture fragments creating a type of new jointàpainful Bone grafting Main functions: promote osteogenesis. less does not facilitate osteogenesis. incorporated via osteoconduction Immunogenic risk Increased expense Osteoconduction-bone graft functions as a scaffold or conduit for migration of new bone as it replaces necrotic old bone Osteoinduction-BMP acts as an inducer substance. refixation or Non-union Weber and Cech classificationà 8-9months of non-healing: immobilization. more stable fixation osteogenesis. bone replacement Advantages Disadvantages autogenous Material of choice. Oligotrophic (periosteal stripping) or electronegative polarity in the Atrophic (Avascular/Nonreactive)-no bone callus injury. Wallerian degeneration (breakdown of the myelin sheathes around nerve) into lipid material does not occur axons are damaged. used w/ fixation devices. Butterfly fragment. position. Horse’s aggressive surgery growth stimulation-creates an hoof.living tissue transfer. facilitates osteogenesis via osteoconduction increased osteogenesis. cancellous bone grafting. has no osteoconduction fixate Haversion systems Techniques of bone grafting Onlay Uses a large autogenous cortical bone graft to brideg a non-union Inlay Formation of a slot or window in which bone graft is placed Sliding Create a graft from the shaft of the long bone. esp in motor fibers. axons distal to injury undergo Wallerian degeneration Tinel's sign can be utilized to monitor regeneration of the damaged nerve Internal structural framework of the enclosed axons are divided. difficult to and osteoinduction.Inadequate fracture healing Cause Description Treatment Delayed union Inadequate fracture Fx has not healed in a reasonable period immobilization. NWB. however it requires synovial fluid for nutrition on it’s superficial/articular surface Healing Ø Limited chondrocyte mitosis and for the most part. skin coverageà designed for rapid revascularization Cartilage healing Description of cartilage Ø Hyaline cartilage consists of chondrocytes within a glycosominoglycan matrix along with type II collagen fibersà responsible for maintaining compressive forces Ø Fibrocartilage contains type I cartilageà responsible for maintaining tensile forces Ø No direct blood supply. metaplasia of mesenchymal stem cells into fibrocartilage or near hyaline cartilage Tendon healing Substrate/Inflammatory Phase 0-14 days Inflammation and fibroblastic splinting with the majority of the tissue strength due to the sutures. No 2nd surgical site. distorted position. Fibroproliferative Phase Week 3 Marked increase in fibroplasia and at this time moderate collagenation strength can sustain gentle passive motion or isometric exercises (brace or cast) Maturation Phase >4 weeks Collagen realignment and remodeling yielding moderate (not full) strength. gives stability. revisional sx. has Haversion systems possible osteoconduction Cancellous Less Dense. slide across non-union or arthrodesis site Papineau Described for the tx of OM: excise necrotic bone. less fenestration of graft is helpful. Stronger to maintain Slower graft incorporation. never revascularizes. immediate Faster incorporation. easilt revascularization. 012 0. finally liquefy hard eschar and slough. and how Split-thickness (dermal) grafts Thin Intermediate Thick Thickness (inches) 0. versatile and May promote anaerobic growth if achieved with hydrocolloids. healthy tissue. Axonal attempts at repair → neuroma-in-continuity develops 5th Complete transection of the nerve. Can be painful to patient form of non-selective debridement. especially if an operating debridement under anesthesia are the fastest Can be extremely room is required. Maggots Selective to necrotic tissue Unsightly.Sunderland’s degrees of nerve injury: 1st Conduction deficit with some demyelination. hydrogels easy to perform. waterborne Hydrotherapy is also a type of mechanical pathogens may cause contamination or infection. using the body's own defense Wound must be monitored closely Autolytic debridement is selective.Normal nerve function is eventually restored. Requires a prescription. Tinel's sign b/c advancing edge of regenerating nerve fiber is initially unmyelinated & very sensitive to minimal mechanical stimuli. 2. Low Non-selective and may traumatize healthy or healing tissue then manually removing the dressing causes a cost Time consuming. axon is completely intact. with no damage to Not as rapid as surgical and moisture to re-hydrate. patient Chemical enzymes are fast acting products that produce Fast acting w/ Expensive. Also.012-0. Sharp surgical debridement and laser Fast and Selective Painful to patient. effective operating room is required. Wound Management Wound Debridement 1. It describes a combination of nerve injuries within one single nerve. will take readily with high success rate. Papainureas: Panafil damage to to the necrotic tissue. The process is safe.008-0. → deficit is transient repairs itself w/i Overlaps neuropraxia hours. additives may be cytotoxic. Discharge Debridement Autolytic Enzymatic Mechanical Surgical Biological Description Advantages: Disadvantages: Autolysis uses the body's own enzymes Very selective.016 0. Costly. 6th Added by Susan Mackinnon . No regeneration injury is possible. Requires less nourishment initially w/lower success rate.016-0. when. B. Edema or erythema . soften and surrounding skin. 2nd Axon or nerve fiber is severed but the endoneurium remains intact. Graft contraction and durability is a problem. Little to no pain for the an occlusive hydrocolloid is used and transparent films. Goal A. Fluctuance .020 Description Require less nourishment initially. Preserve granulation tissue (pink) Debride fibrin tissue (white. Disinfecting debridement. endoneurium and perineurium are disrupted-complete scarring of the endoneurial tubes. an irritable phase which presents with pain. D. secondary dressing Inflammation/ discomfort may occur Allowing a dressing to proceed from moist to wet. not widely used Plastic surgery Explanation of grafting: who. May require a specific and Accuzyme. 3rd Damage to axon and fascicles with some degree of scarring in the endoneurium. Effective. Some enzymatic debriders are minimal or no Application must be performed carefully only selective. Requires transport of patient if methods of debridement. while some are not. Mechanism of injury compression type of injury . yellow. Will heal in 14-20 days 89 . and can be necrotic debris.1988. Dry eschar does not require debridement 2. or green tissue) Debride necrotic tissue (black wound) Exceptions: Stable healing ulcer with dry eschar 1. Will heal in 7-9 days Similar to thin and thick Graft is more durable w/less contraction. Regeneration occurs Fourth degree nerve The axon. paresthesias and hyperesthesias. and collegenases: Santil. C. Hydrotherapy can cause tissue maceration. Nerve undergoes wallerian degeneration & Overlaps axonotomesis regeneration occurs at 1-2 mm/d& depends on presence of an intact endoneural tube. only mechanisms to clean the wound of for signs of infection necrotic tissue is liquefied. Debridement indications a. where. debridement. slough of necrotic tissue. NaHCO3. Will heal by granulating from the edges Types Punch flaps. cardiac arrhythmias. Scopolamine 1. dermis and sybQ) Advancement flaps Moved directly forward to fill a defect without rotation or lateral movement Transposition flaps Rectangular in shape with rounded edges.Full thickness Flaps All of the dermis and some fat which decreases the survivability of the graft b/c fat will seal the basement membrane which relies on the subdermal capillary plexus to oxygenate the base of the graft. administration of blood. prolonged QT intervals Caused by renal failure. K+ PCNs. salt substitutes Tx: CaCl2. blood cultures Occurs post-operative (hours-days) NSAIDs. glucose and insulin Reduction in O2 supply to a tissue below physiologic levels. ST segment depression.63-1. Tx: Warming of patient Prevent with : Metoclopramide 10-20mg IV. airway patency may be maintained and lung atelectasis prevented or reversed.an optimal 60o orientation will theoretically gain 75% length V-Y or Y-V advancement to lengthen or shorten along the scar axis Island flaps where the only link between the cutaneous flaps and it’s bed is the NV bundle Myocutaneous flaps Types Lattisamus dorsi. TA. nourishment diffusion where the stratum basale survives Inosculation 48h-4days Vascular buds from the wound cause new b/v to form. and re-establish or simulate the normal pattern of pulmonary hyperinflation.The objectives of this procedure are to increase transpulmonary pressure and inspiratory volumes. FDBà posterior and plantar heel Description Phases of flap healing Plasmatic Phase 24-48 hours Fibrin anchor layer. QT interval. ABDMà fibula. and patients with full stomachs. Adult/ infants > 6 months must be NPO for 6 hrs (food) à clear liquids may be taken up to 2hrs Fever Wind Water Wound Wonder drug Caused by possible DVT or PE Do a CXR to r/o Prevent with heparin or LMWH Occurs post-operative (days) Atelectasis from the anesthesia. local arterial flaps. Cimetidine 200mg po/IV. pregnant. Vfib/asytole <28C. anorexia. obese.5mg transdermal patch Due to passive regurgitationand seen more in the unconscious. widened QRS complex. 90 . QRS widening.When the procedure is repeated on a regular basis. Prolonged PR intervals. aspiration Use incentive spirometer or intra-op Occurs perioperatively. N/V. during pneumonia. phenytoinß discontinue Occurs post-operative (hours) Walk Walk Wind Water Wound Wonder drug Incentive spirometry is designed to mimic natural sighing or yawning by encouraging the patient to take long. Droperidol 0. slow. Perioperative emergencies Hyponatremia Hypokalemia Hyperkalemia Hypoxemia Hypothermia Post op N/V Pulmonary Aspirations 120-125meq/l <3. This is accomplished by using a device that provides patients with visual or other positive feedback when they inhale at a predetermined flow rate or volume and sustain the inflation for a minimum of 3 seconds. HR/CO decreases as temp falls EKG changes: sinus bradycardia. The graft will pink up and lymphatic drainage will begin Reorganization Months Connective tissue remodeling w/reinnervation in 1-2 years 3. Malignant hyperthermia give NA Dantrolene surgery Dehydration. is the most important regulator of ventilation pulmonary redistribution. constipation Give NS 100 Occurs post-operative (hours) Possible sepsis Do a UA. phenobarbitols. Ranatidine 150mg po or 50mg IV. prolonged PR. PaCO2 Tx by compensatory mechanisms: Hyperventilation.5meq/l Complications of Anesthesia Confusion. rotational flaps. can be rotated 90o Rotation flaps Designed when a pie shaped triangle defect is created to remove a lesion or preexisting defect Z-plasties Type of rotation flap that is used to lengthen an existing scar and to reorient them along lines of minimal tension. increased CO. deep breaths. Dysrhythmias at 28C. lethargy. renal effectsà Give K+ EKG changes: Spiked T waves. coma Give insulin if d/t hyperglycemia and seizures If hypotonic-tx w/isotonic saline Respiratory arrest with <2meq/l. improve inspiratory muscle perfor-mance. increased [Hgb] Decreases VO2 and CO2 production. gastrox. ABH. myocutaneous flaps Local flaps (minimum of epidermis.25 mg IV. indicated to repair low tension lacerations and surgical incisions. the less risk that it may potentiate infection (synthetic. unlike the braided multifilament. ie w/ infection Other closure technique Sterile liquid topical adhesive. most acceptable for vascular synthetic processes Polyester braided Coated with polybutalate Relatively biologically inert. especially in ensuring the eversion of wound edges. thus. Synthetic materials cause less rx. which has been shown to be less reactive than traditional suturing material.e. However. In 3 min provides the same strength as healed tissue at 7 days with traditional closure. and the resultant inflammatory rx around the suture material is minimized. monofilament and non-absorbable sutures are the least reactive i. Resists bacterial contamination and will peel in 5-10 days. In the presence of malignant tumors. Biomaterials and fixation techniques General classification of sutures includes natural and synthetic. tensile strength maintained 7-10d.60% at 7d. Monofilaments have less drag through the tissues but are susceptible to instrumentation damage. resultant scar formation is cosmetically equivalent to that of other techniques. with proper placement. Staples are more expensive than traditional sutures and also require great care in placement. absorption minimal until about 90 dayscompletely absorbed at 6 months KIM-that the less reactive the suture. and. Infection is avoided w/monofilament. prolonging absorption over 90d 75% of original tensile strength remain at day 14 Absorption complete btwn days 56-70 by hydrolysis Tensile strength. painful fx. completely absorbed at 91-119 day 70% of original tensile strength remain at day 14. Staples are composed of stainless steel. Absorbable Suture Plain gut Chromic gut Vicryl (polyglactin 910) Monocryl (Poliglecaprone) PDS (polydiaxone) Type Natural multifilament Natural multifilament Synthetic monofilament Synthetic monofilament Synthetic monofilament Description Digested by body’s own enzymes-more reactive than synthetic Rapidly absorbed. which potentially can sustain bacterial inocula. premature reabsorption) can occur with sutures and lead to an undesirable result. frequent irrigation is recommended to prevent tissue drying. Absorbable sutures are applicable to a wound that heals quickly and needs minimal temporary support. The newer synthetic absorbable sutures retain their strength until the absorption process starts. reactivity.4. fewer microorganisms are carried into the lower skin layers. Use of surgical adhesives can simplify skin closure in that certain problems inherent to suture use can be avoided. prolene) Non-absorbable Suture Ethilon Nurolon Mersilene Ethibond Excel Prolene (polypropelene) Dermabond 2-octylcyanoacrylate Steri-strips Staples Type Description Monofilament Nylon Low tissue reactivity. both cosmetically and functionally. should be long enough to avoid excess traction on the wound margins • Should be parallel to RSTL 5. and monofilament and multifilament. 91 . or infection esmarch exsanguination must not be done. original strength lost 21d. absorbable and nonabsorbable. recommended for use where minimal suture rx is desired. The act of stapling requires minimal skin penetration. Nonabsorbable sutures offer longer mechanical support. Completely absorbed 70 d Chromic Nacl solution to resist body’s enzymes. Hypoallergenic adhesive reinforced with polyester filaments for added strength Staples provide a fast method for wound closure and have been associated with decreased wound infection rates. more pliable when wet 9-0. Problems (eg. only elevate limb for 3-5min is acceptable ankle: 100+SBP thigh 275-350mmhg +SBP max 500mmhg 2 hrs should not be exceeded w/ releasing the tourniquet for 15-20min before reinflating at which time the limb should be elevated about 60 degrees with study pressure on the incision Complications Precautions Pressure setting Inflation time Other types of hemostasis 5. • Open fractures of a leg Post traumatic hand reconstruction • Severe crush injuries Elbow surgery (w/excess swelling) • Severe hypertension Diabetes mellitus • With skin grafts in which all bleeding points must be readily distinguished • Compromised vascular circulation or in the presence or an arterial graft • Sickle cell disease (relative contraindication) as severe pain could result Nerve injury and post-tourniquet increases in Ca Carefully exsanguinate. Surgical Hemostasis Indications Contraindications Tourniquet etiquette To exert enough pressure on the arterial Q to a limb to produce a bloodless field. 30% 14d. Surgical anatomy Incisions • Anatomical landmarks should be identified. Their purpose is to alleviate tension on wound edges. 10-0 for microsurgery Braided nylon Coated to improve handling Multifilament polyester Minimal tissue reaction. Vertical E. inadequate application or not removing enough nail Removal of the nail matrix is integral to prevent recurrence Sodium Hydroxide . Raynauds. cancer. congenital anomaly: ischemia. excessive pronation.1mm deep in nail plate. alopecia. onychogryphosis. gray or black Glomus Tumor Neoplasm of arteriovenous shunts in nail beds (glomus bodies) purplish tumor & pain. cirrhosis of liver Darier White Dz Red & white longitudinal streaks on nail. vascular compromise. incurvated -Contraindications – infection (if longer than 2wks – R/O osteomyelitis). Lichen planus.1980 10% NaOH X 2 15 seconds each Neutralizer – 5% acetic acid Less drainage. Keratocantoma Subungal ulcerated lesion resembles squamous cell carcinoma Leukonychia White Transverse striations. Arsenic Anemia of chronic Dz Addison’s Dz anemia. SBE. Mee’s Lines Horizontal striations on nail d/t arsenic & thallium poisoning Eczema Atopic & contact dermatitis nail colour yellow. green. ACTH. onycholysis. fungal drug toxicity melanoma infection Ingrown Toe nails – Etiology .biomechanics is m. Eti – congenital heart defects. cost less than NaOH 5-10% d/t old phenol. exfoliative dermatitis Clubbed Bulbous deformity of terminal phalanges. melanotic whitlow elevation of nail Periungal Fibroma Acquired or congenital associated w/ tuberous sclerosis. poisoning or Nephrotic Syndrome Junctional nevi. DM. toxic metal poisoning. Darier’s Dz.c.What type of closure technique produces the least amount of tissue reactionà staples Number of days a suture should remain Face and neck 2-5 days Dorsum 7 days Plantar 10-14 days Retention site 3wks-6wks Types of suture techniques • Continuous interlocking stitch • Simple interrupted • Mattress-everts skin edges a. mental retardation. candida. trauma Paronychia Infection + IGTN. respiratory ailments – lung ca. d/t sudden arrest in Fx of nail plate. emphysema. Hodgkin’s Dz. psorias. leprosy.1-. hyperthyroidism.5mm wide x 0. seizures. Horizontal b. vascular Dz. subungal neoplasms fungus. faster healing Low White Spots Injury psoriasis White & pink 92 . Beau’s Lines Transverse ridges 0.Onychocryptosis. scleroderma. Foot Procedures Nail Procedures Nail Pathology – nails can indicate systemic diseases Anonychia No nail. Darier’s DzAtrophy Lichen planus Malignant Melanoma Acral Lentiginous melanoma most dreadful. leprosy. onychomycosis. syphyllis. DM White White Lines Normal Heredity. anemia. cause. trauma >> foreign body rx & infection Surgical Procedures – Chemical Matrixectomies -Indications .typhus. Lovibond’s angle > 160o. onychauxis. nail cutting. spots or total nail psoriasis. toxins.uncontrolled blood glucose Type Applications Neutralizer Advantages Recurrance Rate Phenol 89% Phenol X 3 for 30 seconds each No neutralizerà Flush – Alcohol Phenol longer shelf life. mc – staphylococcus. adenomasebaceum Colour Changes in nail are also disease markers Blue Red Green Black/Brown Cyanosis Cancer Pseudomonas Candida Yellow Addison’s Dz.infection . psoriasis. compression by 2nd digit. convex hard thick nails. Podiatric Surgery 1. Surgical Matrixectomies “cold steel” – indicated for chronic reoccurring IGTN. 3rd plantar interossei. bed & matrix. kinetics.reduces both PIPJ & MPJ. skin flap advanced & sutured to distal nail bed. Overpowers gain mechanical advantage (FDL tight) & over muscles gain a mechanical advantage → the interossei.arthroplasty at DIPJ w resection 2 jts DF of MPJ & PF of PIPJ. PF of DIPJ.(med to lat in transverse plane) Abductor hallucis. Snip nail fold wedge to the bone. scar. cut lateral & anterior borders and removed proximal attachment Complications Reccurence. soft tissue migration upward to dorsum of toe Frost 1950 Whitney 1968 Winograd 1929 Zadik Digital Surgery Functions of the digits: stabilization. 3rd dorsal interossei. Goal of surgery – relieve symptoms & preserve function. sutured Removes nail plate & matrix w/o shortening. 4th dorsal interossei. lat & dorsal) →(Kelekian push-up test) Plantar Plate release w/ maglammery elevator. nail spicules Kaplan Stressed removal of both nail matrix and nail bed. 1st dorsal interossei. 93 . sloughing of flap. 1st plantar interossei. exuberant granulation tissue. Abnormal STJ Tight gastrocnemius or soleus ankle joint Rarest. epidermoid cyst. suture Bilateral frost. resect ½ of distal phalanx & close defect with plantar skin flap. balance. 2nd plantar interossei. equinus or pes cavus foot → Equinus gait. as digits contracts contraction ø adductovarus Flexors> interossei Extensors>lumbricales **Gastrocnemius equinus mc cause → Abnormal STJ pronation & hypermobile flat feet mc cause → Flexor stabilization mc cause → Hammer toe Ø Ø Ø Pathology: Normally when FDL pulls tight → flexes the PIPJ & extends MPJ. Tx . plantar calluses and pain Surgical consideration – Kelikian push-up Test helps determine if deformity is flexible →soft tissue procedure or rigid →osseous procedure. excised skin over base of nail bed proximal to lunula & matrix.shortens digit. derotational skin plasty • flexor tenotomy for mallet toe and flexor tendon transfer for claw toe & hammer toe are ONLY soft tissue procedures used by themselves for completely reducible deformities. Tx 3jts DF MPJ. poor technique destruction. Disadv.arthroplasty at both PIPJ and DIPJ Soft Tissue Indication – reducible/flexible deformity Procedures • Extensor Tenotomy at PIPJ or DIPJ (flexor not often done b/c it sacrifices digital purchase) reduces MPJ • Tenectomy – cutting tendon and taking out a section • extensor hood release – cutting the extensor expansion medially and laterally from MPJ to PIPJ reduces MPJ • capulotomy – cutting MPJ capsule and plantar plate • Extensor tendon lengthening and transfer reduces MPJ • Flexor tenotomy reduces IPJ • Flexor tendon transfers FDL or FDB to proximal phalanx . abductor digiti minimi Etiology of Digital deformities Flexor Stabilization – 90% Extensor Substitution – 10% Flexor Substitution MC cause of hammer toe.045 K-wire for 6-8 wks. 2nd dorsal interossei. Mendelsohn & smith suturing ¼ of nail edge is removed along with matrix and bed. excessive drainage. Anatomy Review: Intrinsic Muscles . PF of PIPJ & DIPJ. To correct this PIPJ arthrodesis is performed to change the function of the FDL → when FDL pulls tight it now flexes the MPJ & correct the contracted deformity. Terminal Symes Removal of entire nail plate. insufficient amount removed. d/t weak gastroc & soleus → pronation → foot pronated in stance phase. deceleration of foot & assist with propulsion. calcaneus gait. bulbous terminal stub. Flexors substitute to Calcaneal eversion →FF abduction →Flexors Assist ankle DF in swing phase. Indications for surgery: digital contractures →impaired function. matrix & hypertrophic ungual labia. Supinated foot in late powers weaker interossei → hammer toe & over powers lumbricales → PF metatarsals stance phase → straight toe adductovarus 5th digit. Anterior assist ankle PF @ heel off. Focused on nail matrix. H incision carried out at two tissue depths Suppan Frees the eponychial fold & removes the nail.→ (Kelekian push-up test) PIPJ arthrodesis fixated w/ 0. of the head of middle phalanx resection of the head of proximal phalanx Tx . Sequential Hammer Extensor Tendon & hood release→(Kelekian push-up test) PIPJ arthroplasty →(Kelekian push-up test) MPJ toe Release capsular release (med. Mallet Toe Hammer Toe Claw Tow 1jt. allow visualization of proximal nail matrix. Adductor hallucis. infection. excessive bleeding. failed chemical procedures and nail pathology Inverse L shaped incision excision of nail. Bishop opening base wedge osteotomy w/bone graft. inclusion cysts. shortens. congenital long 5th met or short 4th met. hammer toe repair. floating or flail toe. cannot act as rigid lever. numbness. B. Lesser Metatarsal Surgery Anatomy: mets 1 & 5 have individual axis of motion. incision proximal lateral to distal medial. short EDL. converts toeà rigid lever Types: End to End fusion(Selig) – resect cartilage on both bones down to sub-chondral bone & fused with K-wire. Resection of base of proximal phalanx – NEVER do this! b/c creates instability & lose attachments of intrinsic muscles. Modified Mitchell – step down osteotomy Reverse Austin – 2mm medial transposition. pathologic 8. must fixate. sagittal plane misalignment plantar flexed 5th met & cuboid.derotational skin V-Y or Z plasty. indicated in bone tumors.05 structural. decreased sensation.2/3 & syndactylization of 4th & 5th toes. rigid FF varus/valgus. NSAIDS. must fixate. residual pain. medial contracted EDl. Chevron double V osteotomy. Abnormal bone shape & size – prominent lateral plantar condyle. Kelikian – partial met head resection ½ . not 2-4. V-arthrodesis – fashion bone into V & fixate. Kelikian – syndactyly of 4th & 5th after capsule release and arthroplasty of 5th toe. Peg in hole fusion – proximal phanlanx fashioned into peg w/ dorsal cortex intact & base of middle phalanx fashioned into hole to accept peg. Lambrinudi – fusion of PIPJ & DIPJ for claw toe. claw or mallet toe. Normal IM High Lat Deviation >> Neck procedures Mercado – medially based closing wedge osteotomy at met neck. Shortens or lengthens. hammer. 5th MET . blue toe. saw or osteotome. flail toe. Abnormal STJ pronation. OM. Weil – transverse osteotomy from surgical neck dorsal distal to plantar proximal phalanx to condyles. OA. Davis – removal of lateral eminance (reverse silver). Floppy digit. adduction. arthritic changes >> exostosis at 5th MPJ. Devries – removal of lateral plantar chondyle. steroid injections reduce callus Surgical Normal IM & Lat Deviation >> resect lateral prominence. plantar plate release. Tx skin incision . apex distal. long or short met. Combination of the above. cut is dorsal distal to plantar proximal. Reverse Hohmann – transverse osteotomy at the level of the metatarsal treatment neck w/ medial displacement of capital fragment.64. vascular impairment. Bone arthroplasty – resection of 5th met head + lateral middle hemi-phalangectomy. wider shoes. Hammer Toe >> retrograde plantar flexin. increased IM angle.. floating toe with metatarsalgia. plantar condylectomy Transfer lesions.sausage digit. fixation not required. implant failure . tailor’s bunion procedure . shortens toe and relaxes soft tissue.persistent edema .transverse base wedge resection using hinge axis guide. Tx reduction/excision of hypertrophied bone from mostly 4th interdigital space Etiology – base of proximal phalanx removed. Ø normal declination of met – 15o IPK Etiology 2-4th met – Biomech – RF varus. metarphyseal and periosteal. Wilson – dorsal V-Y skin plasty & extensor Tenotomy and lengthening Mc .71 positional. less common . edema. malunion/nonunion. cicatrix.Arthroplasty Post 1897 Arthrodesis Interdigital Corn Helloma molle Adductovarus 5th Digit = overlapping 5th toe Complications of digital surgery Osseous Procedure Indication – semi-flexible deformity. FF varus/valgus. Lateral deviation angle normal – 2.Biomech – RF varus. large round dumbbell shaped 5th met head. Tendon re-approximated with absorbable sutures – vicryl & skin closure with nylon. transverse tenotomy at level of PIPJ medial & lateral collateral ligaments severed.soft tissue – release and lengthening of EDL. k-wire fixation. **excessive 5th met head resectionà laxity of internal cubic content of the jt àfurther varus or adductovarus malalignment of 5th toe & more retrograde pressure on the 5th met head** 94 . intrauterine position. dislocation of met head. hypermobility . Fixated with screw/pin. Percutaneous Metaphyseal sliding osteotomy – capital fragment dorsiflexed. infection. Dx – Xrays w/ lesion marker & lateral to evaluate metatarsl parabola – problem if 2mm longer or shorter Reduce the lesion at time of surgery. capsulotomy. Dorsal linear incision. contracture of capsule. Met head resection. transverse IPJ deformity. shortens met. or absorbable pin. foreign body. abductory wedge resection of phalanx. fat pad atrophy Verucca plantaris (pinpt bleeding w/ skin lines around lesion occur in non wt bearing areas. plantar flexed position. pathologic 8. dorsal bump/met elevation.. oblique osteotomy from distal lateral to proximal & medial w/ displacement of capital fragment proximally & medially (reverse wilson). idiopathic. Congenital PF or DF 5th ray. McKeever partial met head resection. lateral deviation or wide 5th met. recurrence of the deformity. McKeever peg in hole . subluxed MPJ. DFWO @ anatomical neck or base. or salvage Boney exostosis on the base of 4th proximal phalanx and head of proximal phalanx of 5th rub together. Rappaport . Presents with keratotic lesion over PIPJ. excessive stiffness. Conservative Treatment padding. V osteotomy @ anatomical neck >> transversae and frontal plane correction& stability. Eg Lapidus – extensor Tenotomy w transfer of distal stump under proximal phalanx and attach to abductor digiti quint. nonunion. DF 4th met. atrophy or displacement of plantar fat pad. very stable w/ high union rate. Allows proximal sliding to increase joint space & improve digital alignment. sagittal plane mis alignment Abnormal declination angle. RA. recurrence DDx Treatment Complications Taylor’s Bunion Etiology Description Uncompensated FF/RF varus.47. increased lateral deviation angle. Blood supply to mets – Nutrient artery. abnormal shaped plantar condyle. Yancy – mid shaft medially based closing wedge osteotomy. XR evaul IM angle normal 6. Indication: failed arthroplasty. Rotation of lateral plantar tubercle into lateral position. screw. High IM >> Base procedures Gerbert -proximal diaphyseal closing wedge osteotomy. shortens. bone resected at the flare of the bone using rongeur. for 1st & 2nd + 4th & 5th. longus & DF of met with transverse head of adductor pulling on hallux and enhancing its lateral migration Sesamoids migrate laterally to change abductory forces on the hallux causes boney adaptation on plantar met head and wearing down of the cristae.prevent further damage & displacement of MPJ. limited ROM. ischemia. deep transverse intermetatarsal ligament. This is accomplished via closing base wedge osteotomy of the 1st metatarsal with AO fixation & distal oblique osteotomy of 5th metatarsal with k-wire fixation. Blood Supply – 1st dorsal and plantar met arteries & the superficial branch of medial plantar artery. loose spicules of bone. cartilaje intact II. hypertrophy of the dorso-medial metatarsal head. greatest deformity >>> High IM. Hallux over-riding the 2nd toe Structural Positional Combined Types of Deformity PASA or DASA Abnormal PASA & DASA Normal PASA OR DASA Abnormal PASA + DASA = HAA PASA + DASA < HAA PASA + DASA > HAA Congruent – lines parallel Deviated/Subluxed {lines intersect outside the jt – deviated} Deviated/Subluxed {lines intersect within the jt – subluxed} 95 . sclerotic rim. Stages of HAV I subclinical subluxation of 1st MPJ. intersesamoidal ligament. Tibial & fibular sesamoidal lig. Rotational osteotomies rotates the lower aspect of the met head dorsally after a section of damaged cartilage has been excised allowing the plantar cartilage to articulate w/proximal phalanx. No DJD. cartilage intact III. Periarticular spurs. nutrient artery is 2. Infraction Dx – Xrays – jt space widening 3-6 wks after symptoms. soft tissue swelling. Adductor Hallucis. Later – implant arthroplasty. metatarsal head remodelling (splinting 3 mths). seen on XR only w/normal IM & HA II Abduction of hallux → force on 2nd met and clinical s/s of HAV III Met primus adductus deformity. cancellous bone grafts to restore contour of met head. Collapse of central met head with non-intact plantar cartilage forming loose bodies V. Smile Classification (JBJS 1957) I. Epiphyseal dysplasia with severe met head involvement C.mc women 3x >men. surgical repair focused on the IM angles. Epi . flatteningof met head w/ osteophytic lipping .Other lesser metatarsal surgery Splay Foot Widening of the entire foot in transverse plane. initially casting & cortisone shots. Complete flattening of the metatarsal head Frieberg’s Infraction I. edema. FHL FHB. prominent plantar metatarsal head w/ excessive loading. jt space narrowing. advanced – inc IM clinically. Severe DJD.7cm from MPJ on lateral side. Abductor Hallucis Ligaments – Medial & lateral collateral ligs. Lateral subluxation of hallux d/t weakening of p. increased density of subchondral bone. Hallux Surgery HAV Anatomy review: Muscles – EHL. Fx of the subchondral plate III. adjacent MPJ hyperkeratosis. EHB. Fx of ischemic epiphysis II. bone margins sclerotic Tx g oal . Collapse of central met head with plantar cartilage intact IV. 1 finger breath separation b/t jt IV Clinical subluxation /dislocation of 1st MPJ. Biomechanics: Flexible FF valgus & gastrocnemius equinus bilateral → abnormal pronation in adducted foot during propulsive phase of gait 1st MPJ DF d/t hypermobility & inverts. Freiberg’s Etiology trauma. loss of cartilage IV. palpable jt irregularities dorsally. age 13 S & S – pain in MPJ. early increase in IMAon xray. Medial Capsuloraphy Hinge Axis Guide used for base procedures. DF – plantar medial to dorsal lateral. laxity of 1st MC joint. proximal corrects DASA. normal TSP. Dissection and Hemostasis 2. line bisecting 1st met is below or above talar 15 – 20 position bisection → plantarflexed or dorsiflexed 1st met. Bunion location – dorsomedial mc. and transfer AHB from plantar to medial to 1st met sutured into hole at base of proximal phalanx to strengthen the medial capsule. orthoses and steroid injection. crepitus or other degenerative jt changes. Lengthen – perpendicular to long axis of bone. distal – HIA. K-wire placement to assist with manipulation of the sagittal plane correction & length of 1st met. Maintain length – perpendicular to2nd met. K-wire or screws. Deep tranverse intermetatarsal ligament b. Conjoined Adductor Hallucis tendon c. Shorten – slightly proximal Soft Tissue Procedures Positional procedure indicated for correction of soft tissue component of HAV.Important Radiological Angles in the evaluation of HAV Angle MA How to Measure Normal Value (degrees) Line perpendicular to bisection of lesser tarsus. • 1st Ray ROM normally 10mm 5mm up & 5mm down . Clinical Evaluation Non-Weight bearing • H/o of recalcitrant pain. This is d/t lateral soft tissue adaptation and joint axis deviation. IM < 10o. + or -.? increase or decrease. To determine the effect of met adductus on Normal in a rectus foot 8-12 the deformity subtract 15 from MA and add it to the IMA. structural deformity adducted foot 8-10 HA Lines bisecting met & proximal phalanx. rarely used alone. lateral capusulotomy. Medial dissection and exostectomy 4.another measurement of MA 24 IM Bisection of lines of 1st & 2nd mets. Structural adaption of shaft of proximal phalanx to base MPD Measure of relative length b/t 1st & 2nd mets. Akin Osteotomy – medial closing wedge osteotomy done in the proximal phalanx. • 1st MPJ DF normal > 65-75o & PF >15o. Axis of motion of 1st ray – Tracking – hallux can be placed in corrected position but drifts back into abnormal position during motion. Plicae attaching proximal surface of sesmoid apparatus to met neck and joint capsule f. Lateral head of the FHB e. central – shortens and derotates the hallux. Usually modified & sesamoid ø removed Hiss – resection of eminence. erythema. >15 in a median point b/t 1st cuneioform & navicular and median point of cuboid connect lines and draw a perpendicular rectus foot line which forms and angle with bisection of 2nd met Engles Angle formed b/t bisection of medial cuneiform & 2nd met shaft. represents transverse plane deviation of hallux 0-15 HAI Line bisecting proximal & distal phalanges of hallux 0-10 PASA Angle between a line perpendicular to shaft bisection and line depicting effective cartilage. Track bound – inability to place hallux into corrected position d/t severe jt axis deviation and articular/boney adaptation. Increase of this angle represents articular cartilage adaptation to deviated hallux in transverse plane DASA Line drawn perpendicular to effective articular cartilage of base of proximal phalanx & line representing dorsal 0-8 longitudinal bisection of the shaft of proximal phalanx. lateral capusulotomy with removal of fibular sesamoid. For PF – dorsal medial to plantar lateral. Fibular sesmoidectomy 3. transfer of the conjoined tendon of the ADH and lateral head of FHB to dorsolateral aspect of 1st met head. Neutral – straight toward 2nd toe.2mm TSP Position measured by relating the position of tibial sesamoid to 1st met bisection 4 or greater is abnormal & 1-3 dictates removal of the fibular sesamoid position Lateral 1st Met Sagittal plane relationship of 1st met to bisection of talus. age of onset of symptoms and failure of conservative tx – shoe modification. Adductor tendon transfer to derotate the sesmoids 6. good ROM. as adjunct. Osteotomy 5. medial or dorsal (hallux limitus or met primus elevatus) Skin lesions – keratoma. Represents the 0-8 articular cartilage deviation on the head of the 1st met. Fibular Sesmoidal Ligament d. Maintain lateral cortex and fixate w/ staple. acceptable cosmetic results Anatomic Dissection 1. bursa Goals of Sx – pain free congruent joint. Proximal Phalanx 96 . lateral capusulotomy and medial capsulorraphy McBride1928 – resection of dorso-medial eminence. Plantar lateral release a. Sliver 1923 – resect the medial eminence of 1st met head. representing bisection of of 2nd met. Bisection of lesser tarsus by 15. except osteotomy is done at the anatomical neck Trapezoidal osteotomy (base oriented medially) to correct sagittal and transverse plane abnormalities. and long/short 1st met Type C1-sagital correction only Type C2-sagitall and long/short 1st met Loison-balacesu Osteotomy described as being at a point just distal to the insertion of the PL. Type B1-corrects transverse/sagittal Type B2. 2 screw fixation NWB 6-8 wks Juvara Oblique closing base wedge osteotomy from proximal-medial to distal-lateral. adductor release & medial capsule reefing. dislocation of capital fragment. screws. Distal cut dorsal. Indicated for long 1 st met. Corrects met edema. will significantly shorten PASA only IM and PASA IM PASA. proximal cut IM up to 18o. IM. easier to fixate *Trough effect* shortens. IM.5cm distal to 1st MC jointà no shortening. Modified Cotton opening medial cuneiform procedure indicated for short 1st ray in juveniles. Able to Bicorrectional-normalizes IM lengthen/shorten. PASA Of distal met shaft. ANV of 1st met head. Potential complications: sesmoidal damage and OA Horizontal L osteotomy. Horizontal “V” chevron osteotomy performed in the center of the met head Unicorrectrionalà IM. hallux varus. both capital fragment can be elevated. IM opposite to ludloff. Osteopenic bone/osteoporosis can be fixated with crossed K-wires Crescentic 97 . with a 60o between arms for transverse plane correction only. shorten. depressed. Plantar shelf-Lateral cortex intact Lateral cortex is cut allowing lateral translation of the capital fragment Potential complications: PASA and mild IM transfer metatarsalgia. post-op NWB Peabody Hohmann 1921 Wilson DRATO 1971 Mitchell 1958 Neck Procedures for HAV Same as Reverdin. Fixation & NWB required. 1st cut-1cm PASA only proximal and parallel to the articular surface. through and through. Distal segment is rotated until the desired correction is IM attained may be fixated slightly dorsi or plantarflexed. 2nd cut-perpendicular to the shaft à Lateral cortex is intact. Type A-corrects transverse plane. performed in the center of the met head. transverse. PASA Sagittal Logroscino Waterman and Labrinudi IM. graft >> lateral & plantar migration IM. Disadv – done in diaphyseal bone → poor healing Labrinudi PF wedge osteotomy IM Kalish *Trough effect-dorsal segment of the cortical bone of the Z wants to sink into the plantar segment when screws are tightened down Base Procedures for HAV Created approx 1. PASA Ludloff 1918 Oblique osteotomy oriented dorsal proximal to plantar distal & exit distal to met head. lengthen. Long dorsal arm allows for 2 screw fixation IM Unable to swivel capital fragment to correct PASA Scarf 3 cuts from medial-lateral through met shaft /diaphyseal resembling a Z. Base lateral w/medial cortical apex intact IM Logroscino Short 1st metàReverdin &Trethowan. Fixation & NWB Derotational Angulated Transpositional Osteotomy: Distal osteotomy-perpendicular to the long axis of the 1st met.Through and through. medial wedge of bone removed. proximal cortical apex is IM 1919 maintained. same as Reverdin Laird with penetration of the plantar cortex to PASA mild IM allow sagittal place correction. can swivel for IM and PASA PASA correction Mau 1926 Oblique osteotomy oriented plantar proximal to dorsal distal exiting proximal to met cuneioform joint. Head shifted laterally. Long 1st metà Reverdin & Loison-balacesu Fixated w/kwire.corrects sagittal. through and through. abnormal met declination IM <15o Shaft Procedures for HAV Modified Austin. PASA Trethowan opening base wedge osteotomy with medial wedge of bone allograft or auto graft pushes over the met to correct IM. adducted or abducted. lack of toe purchase. PASA soft tissue correction. chronic & PASA. DF capital fragment to place the articular cartilage in a more functional position attempting to increase DF at the 1st MPJ Oblique angulated osteotomy to PF met head. elevatus. screw staples IM. fixation & NWB Shortens and lateral displaces head. Capital fragment is shifted proximally & laterally or dorsally & plantarly along osteotomy.Procedure Austin 1962 Reverdin 1881 ReverdinGreen 1977 Reverdin-Laird 1988 ReverdinTodd Capital Procedures for HAV Description/osteotomy Correction Indicated for IM <15o. fixation – staples or K-wire Fixation: Normally bunions are fixated with 2. proximal osteotomy-angled proximal medial to distal lateral. excessive shorteningàtransfer metatarsalgia. avoids sesamoid trauma. shortens/lengthens Angulated osteotomy performed in metaphyseal bone. Adv. Initial cut is made parallel to the WB surface being PASA only sure to exit just proximal to articular surface of the 1st MTJ. lengthens or 1983 plantar. NWB – 10-12 wks. intrarticular fracture correction Allows plantarflexion of the met head. capital fragment slid laterally. fixation with 1 or more screws. avoids possibility of scoring sesmoids. – versatile. Oblique osteomy oriented distal-medial to proximal-lateral. Potential complications: limited 1st MPJ ROM. shortens 1st met and decompress MPJ.7 cortical screw using OA lag screw technique. Fixation – k-wire. is similar to Hohmann ex: lateral cortex strut intact with step off after a medial wedge of boneis removed. adductus. Unstable osteotomies. Complcations of HAV Sx – staking of met head & producing hallux varus, hammered hallux, longitudinal fractures , sesamoiditis, fracture of articular cartilage, unstable osteotomy, non or delayed union, damage to neurovasculator structures, elevatus and hallux rigidus, over or under correction, disuse osteoporosis, infection, AVN, fixation problems Joint Replacement Indications – inflammator arthritidies (RA; Degenerative arthrosis d/t osteochondral fractures, osteochondritis dissecans, congenital deformity, trauma, brachymetatarsi, fkail toe, floating toe, revisional Sx Types Swanson flexible hinge , Sgarlato double stem cup implant, swanson condylar implant Technique – Lazy S incision over MPJ Linear or U capsulotom ; extensor/flexor tenotomies & or plantar plate release, boney resection of met head, ream medulary canals , ck fit, flush & assemble implant; wound closure Contraindications Severe osoteoporosis, prior jt infection w/I 6mths, allergic reaction to implant material, medically compromised Complications – implant instability & pistoning , implant failure, foreign body reaction, osteochondritis dissecans, dendritic synovitis, infection, chronic edema, Fx of proximal phalanx Hallux Limitus • Definition of hallux limitus/rigidus - Limitus-dimished ROM at 1st MPJ; Rigidus-absent ROM at 1st MPJ, less than 10 o of DF. DF is normally 65o-75 o, PF = 40 o • What are the etiologies of Hallux limitus (most common) - . Hypermobile 1st ray, long 1st ray, 1st met elevatus, DJD, neoplasms, trauma, sepsis, iatrogenic, short 1st ray, NMD, systemic arthridities, AVN, foreign bodies (implants) Classification scheme of Hallux Limitus Drago, Olaff and Jacobs Regnauld 1. Functional limitus 1. Development 2. Joint adaptation 2. Established arthosis 3. Joint deterioration 3. Ankylosis 4. Ankylosis 4. Xray findings – nonuniform jt space narrowing; flattening of the met head, osteophytes on 1st met head and base of proximal phalanx (dorsal flag sign), subchondral sclerosis, loose bodies (joint mice), 1st met elevatus Procedure Cheliectomy Kessel&Bonney Regnauld Waterman WatermanGreen Youngwick Van Ness Cotton Labrinudi Procedure Keller Implant arthoplasty Stone Mayo Mckeever Valenti Lapidus Joint preserving surgical procedures for the treatment of Hallux Limitus Description of surgical procedure Removal of osteophytic bone & hypertrophied synovium from the 1st MPJ base, temporary procedure; cleans and remodels jt; stimulates fibrocartilage growth; immediate ambulation & quick recovery; orthotics & PT helps Dorsal DF osteotomy of the base of the proximal phalanx Mexican hat/enclavement procedure; Base of the proximal phalanx is fashioned into a peg and reinserted into the proximal phalanx. Shortens ray DF osteotomy of the 1st met head leaving the plantar articular cartilage intact Similar to bicorrectional Austin; dorsal cut; trapezoidal wedge wider medially; plantar cut-oblique, transverse osteotomy. Angle=45 o , shortens and PF the 1st met head Modified Austin, Rectangular block of bone taken out on dorsal cut. Shortens and PF 1st met. Plantar closing wedge osteotomy at the base of the 1st met Opening base wedge osteotomy of the medial cuneiform PF wedge osteotomy of the 1st met base to correct met primus elevatus Joint Destructive procedures for the treatment of Hallux Limitus/Rigidus Description of surgical procedure Arthroplasty of proximal phalanx with resection of 1/3rd of the proximal phalangeal base; complication – cocked up hallux, lack of purchase, shortening & transfer metatarsalgia Total or hemi, primarily a jt spacer, reduce metatarsalgia, increase stability of jt and length of hallux. Indications: crepitation, pain, good bone stock, no infection or allergy to implant, Advan – early ROM & ambulation. Prophylactic antibiotics Oblique osteotomy Dorsal to plantar wedge resection of 1/4th of the met head & medial eminence leaving plantar condyle and sesamoid apparatus intact Excision of 5mm of the 1st met head including articular surface, interpose capsule and bursa tissue. 1st MPJ arthrodesis; ideal position of fusion DF 5-10o from ground, slight abduction parallel to 2nd digit or 10-15o in transverse plane, NO frontal –valgus or varus rotation Complications – IPJ arthriti, delayed/non union onycryptosis, medial callus, impaired gait, subluxation 2nd & 3rd toe V shaped osteotomy; Metatarsal cut: proximal distal to distal plantar leaves plantar 1/3 of the cartilage intact Phalangeal cut-proximal plantar to distal dorsal Fusion of the 1st MC joint 98 Hallux Varus Juvenile HAV Transverse plane deformity of hallus adductus; Etiology congenital – isolated or congenital abnormalities; Acquired _ iatrogenic from HAV surgery d/t over correction of IM & PASA, excessive resection of sagittal grove, excision of fibular sesamoid, post-op bandage & splinting Presents – shoe gear problems, pain on medial hallux & 1st MPJ Xrays – hallux adducted, staked head of 1st met w/ tibial sesamoid peeping, Negative IM angle and PASA, arthritic changes at jt Tx conservative – strapping & splinting; Sx – soft tissue – medial or lateralcapsulotomy, aBductor hallucis transfer, retransfer aDductor Hallucis laterally; Bone – Reverse Austin to correct negative IM & PASA, Keller arthroplasty w/ or w/o implant, McKeeverarthrodesis; Goal is to correct deformity and prevent DJD. HAV < 20 yrs; F>M Etiology – family H/O, biomechanics – pes planus, limb length discrepancy, STJ pronation, equinus, met primus varus, neuromuscular deformities S & S – less severe valgus rotation, NO DJD, few adaptive changes; congruent jts & hypermobile MC jt, Abnormal PASA Tx – conservative – splinting, toe wedges, bunion shield, wider shoes; Sx No soft tissue; - Lapidus, Akin, Mitchell, Austin, Reverdin, opening wedge of medial cuneioform, closing base wedge osteotomy, cresentic, epiphysiodesis- epiphyseal stapling When to do Sx -< 6 – growth plates 40-50% closed; 6-10 – 60 -80% closed; 10 – 14 ideal time – 90% in boys & 95% in girls D. Rearfoot Pes Planus o Flat Foot deformity >>> collapse of medial longitudinal arch, RF valgus, FF abduction & supinatus, tight heel cord o Planal dominance refers to the primary plane of the flatfoot deformity (transverse, sagittal or frontal) i.e, what plane is most affected. Although procedures are divided into the primary plane of correction, the other planes are affected to a lesser extent; therefore all procedures provide triplanar correction. o A transverse deformity would clinically show abduction of the FF on the RF, (AP x-rays) an increase in TCA, decrease in TN congruity, and increase in cuboid abduction. o Frontal plane deformities clinically show RF eversion, (lateral & calcaneal axial x-rays) increased superimposition of the lesser tarsus, decreased 1st met declination angle, decreased height of the sustentaculum tali o A sagittal plane deformity would show sagging of the midfoot-navicular-cuneiform breech (lateral x-ray) TDA increase, calcaneal inclination angle decrease, TCA increase, talo-1st met angle increaseà negative Meary’s angle Tests to determine flexibility of flatfoot deformity 1. Hubcheur Manuever-a plantarflexed 1st ray will cause the STJ to supinate and recreate the arch if flexible pes planus deformity 2. Trunk twist-STJ flexibility for supination/pronation 3. Jack’s test-pt on heels, heels will invert, checks the PT 4. Supination/inversion stress lateral XR Etiologies of Pes planus Newborn Calcaneovalgus Vertical talus STJ axis Dominant plane Compensation High Transverse plane (higher vertical plane axis) Adduction/abduction Low Frontal plane (higher horizontal plane) Inversion/eversion Correcting pes planus, one must perform a posterior procedure that corrects the equinus (TAL), a medial soft tissue tendon procedure and a lateral calcaneal osteotomy or arthroresis. As a last resort for excessive OA a triple arthrodesis is indicated. Obesity & ligamentous laxity are relative contraindication b/c they >>> poor surgical outcomes. 1. 2. 3. 1. 2. 3. 4. 5. Transverse plane corrections for pes planus Evan’s-calcaneal osteotomy 1.5cm proximal to the C-C joint with insertion of a bone graft to lengthen the lateral column and put the PL on stretch to increase the supinatory motion at the STJ. Indicated for flexible deformities & corrects FF abductus & heel valgus Kidner-removal of the prominent navicular tuberosity or accessory navicular and transposition of PT plantarly into the navicular; increases the mechanical advantage of PT tendon in elevating the medial arch; increase risk for PT tendon rupture in adulthood C-C joint distraction arthrodesis Sagittal plane correction for flatfoot deformity Lowman-TN fusion, TAL, TA under the navicular into the spring ligament Hoke-Naviculo-cuneiform fusion, indicated with hypermobility of the NC joint & stabilizes the medal column Miller- NC-1st met fusion Lapidus- 1st met base-medial cuneiform fusion Cotton-open plantarflexory wedge osteotomy into the medial cuneiform Teenager Adult Equinus, os tibialis externum, met adductus, peroneal spasm TP dysfunction Tarsal coalition TP dysfunction w/OA Flexible Rigid 99 6. 7. Young (Keyhole) Tendosuspension of the TA through a keyhole in the navicular, and rerouted (not detached) PT advancement under the navicular to support the medial arch TAL/GR Frontal plane correction for flatfoot deformity 1. Orthotics 2. Arthroresis: MBA, Sta-peg, to elevate the floor of the sinus tarsi Indications Flexuble flat foot deformity Contraindications No mddle facet coalition, no CN bar, no degernative arthritis Types Vogler Class 1 Self-locking wedge: A device where opposing surfaces of the talus and calcaneus are held apart by an implant inserted between the two segments. Best for adolescents and adultsà MBA and Valenti Vogler Class 2 Axis altering: Will change the axis of motion of the subtalar joint. Best for pediatric patientsà The STA-peg Vogler Class 3 Direct impact blocking : Restricts the forward movement of the lateral talar process where motion is limited by contacting only the talus àThe Sgarlato (Futura) and modified STA-peg l Sgarlato is inserted into the floor of the sinus tarsi w/forward motion of the lateral process of the talus being blocked by the implant which stops the lateral process of the talus from advancing past the posterior facet of the calcaneus 3. Chambers-arthoresis concept with bone graft, to elevate the floor of the sinus tarsi 4. Baker-Hill-osteotomy inferior to the STJ using a bone graft under the sustentaculum tali to elevate the lateral posterior facet 5. Selakovich-opening wedge osteotomy of the sustentaculum tali with bone graft that restricts abnormal STJ motion to medially elevate the sustentaculum tali 6. Calcaneal osteotomies • Dwyer: medial closing wedge to produce varus correcting valgus • Silver: lateral opening wedge with graft • Koutsogiannis-through-through sliding medial displacement transpositional osteotomy • Gleich: oblique osteotomy displaced anteriorly • Lord-displaces the calcaneus anterirly, inferiorly, and medially Pes Cavus Etiology: usually caused by a present neuromuscular diseaseà CMT, polio, Types – anterior – metatarsus, FF, lesser tarsus cavus; Posterior – muscle weakness or spasticity, pseudoequinus>>inadequate ankle DF Clinical evaluation: Coleman Block test-evaluates flexibility/rigidity of the deformity by removing influence of 1st ray. XR evaluation stress inversion XR to evaluate the position of the lateral process of the talus (w/ pronation, will hit the floor of the sinus tarsi) àbullet hole sinus tarsi Signs Normal (degrees) Cavus foot CIA 20-25 >30 Angle of Meary 0 >60 Angle of Hibbs 135-140 >150 Types: Flexible, rigid or digital anterior/global/local flexible with an etiology of extensor substitution Principles of cavus surgery Rearfoot varus in the frontal plane- Dwyer Ankle equinus- TAL or gastrocnemius recession Planterflexed 1st ray - DFWO Rigid anterior cavus - Cole – DF wedge osteotomy through the navicular-cuneiform jt and cuboid bone; or Japas – displacement V-shaped osteotomy through cuboid, navicular and medial cuneiform Digital claw toes or hammertoes - DIPJ or PIPJ arthrodesis, HIbbs Soft tissue release Subcutaneous fasciotomy Steindler Stripping: Plantar fascia, ABH, FDB, Abductor digiti quinti, long plantar ligament. NWB Cast for 3 wks post-op WB for 2-3wks Soft tissue procedures indicated for flexible cavus: pediatric or adjunctive procedure Tendon Transfers >10-11 years 100 PT – ice.Lumbar lordosis.differentiated using Silfverskiold test and charger XR that evaluates anterior osseous structures. E-stim. Be careful of the sural nerve DF calcaneal osteotomy STJ. heat. Using Endotrac system & release 1/3 – ½ of fascia with L-shaped blade. Conservative Tx – Stretching exercises. tongue is distal Baker Inverted McGlammary. tongue is proximal Silfverskiold Release gastrox from origin on the femoral condyles Gastroc soleus Equinus → TAL Z-plasty Frontal or sagittal plane Hasuer Section posterior 2/3 proximally and medial 2/3 distally White Section anterior 2/3 distally and medial 2/3 distally Conrad & Frost Section medial ¾ at distal end and lateral ¾ proximally Hoke Triple hemisection w/ 1st & last medial & 2nd lateral. may be used in conjunction w/ s/t release DFWO through navicular-cuneiform joint Maintains STJ and MTJ motion. Digital contractures (Extensor substitution) Gastrocnemius Equinus → Gastrocnemius Recessions VulpiusV-shaped lengthening of the gastrocnemius Strayer -transverse cut through the gastrocnemius. tibiofibular syndesmosis) and Pseudoequinus anterior cavus foot type Dx . Knee flexion. S & S – pain on palpation of medial Calcaneal tubercle. navicular & medial cuneiform. shifts FF dorsally 1st MC arthrodesis with elevation of the 1st met correcting the anterior cavus Dorsiflexory wedge osteotomy. rerouted down peroneus tertius tendon sheath to it’s insertion at the base of the 5th met using 3 incisions PL transfer -PL is released at the level of cuboid & transferred through the IM septum down the EDL sheath & inserted into the lesser tarsus. Complications: lateral column 101 . but only weakens toe off by 15%) Osseous Equinus (tibiotalar exostosis) → Exostectomy. 42% w/ fasciotomy & exostosis Endoscopic plantar fasciotomy – Barrett & Day 1991. preserves the function of major joints Lateral closing wedge of the body of the calcaneus. Changes action from PL/inversion to DF. ultra sound Surgical – 20% resolution w/ fasciotomy alone. mid foot osteotomy.Muscular: (gastrosoleal –ve silverskoild. tenderness on distal fascia & worsen w/ toe DF. percutaneus o o o Spastic Equinus → Murphy Procedure . No bone removed. percutaneous release of fascia w/o removing spur. increase wt. Hip flexion. metabolic dz. acquired or Osseous (talotibial. NSAIDS. heel lifts.Pes Cavus Soft tissue Procedures -EDL detached from insertion (2-5) and distal stumps are sutured to corresponding EDB tendon (4/5 go to 4th EDB (2-4) -Jones.Tongue and Groove. faster recovery. Def – Inferior Calcaneal spuring & pain d/t microadhesions that form around the plantar fascia especially at night >> worst pain at 1st step in the morning when adhesions tear Post static dyskenesia. Indications: recalcitrant PF Advantages-decreased post-op pin. Hibbs Cole Japas McElvenny-Caldwell DFWO Dwyer Samilson Triple arthodesis Equinus Osseus procedures indicated for rigid deformity/ NMD for foot stabilization. TN and CC joint fusion • • • Def. night splints.less than 10o of DF past 90 degrees is required for normal gait types of equinus . tibiofibular syndesmosis → heel lift accommodations Pseudo-equinus → metarsal osteotomies. suture proximal flap to soleus McGlammary. Spastic Equinus (CP)congenital equinus. Changes action to DF at ankle limiting strong extensor substitution PT transfer -PT is released from insertion on navicular and transffered through interosseous membrane to dorsal midfoot. injections.transfer advancement of Achilles insertion to dorsum of calcaneus just posterior to the posterior facet of STJ anterior to the FHL (weakens triceps surae at the ankle by 50%. à Shorter wider foot Displacement V osteotomy through cuboid. Conservative Tx – 95% successful. foot orthosis. Patho – repetitive traction. triple arthrodesis Heel Spur Syndrome / Plantar Fasciitis • • • • • • o 15% of all adult foot complaints caused by heel pain. long extensor tendons are transferred to intermediate or lateral cuneiform and a TAL Heyman Transfer of all 5 long extensors to respective met heads w/ anastomsis of distal stumps to brevis Jones Split EHL thru drill hole in 1st met (medial-lateral) & sutured back to itself or EHB with IPJ fusion STATT Transfer of lateral ½ Tibialis Anterior tendon. Peroneal anastomosis PL to PB at level of lateral ankle or at the styloid process of the 5th met. pronated or Supinated foot. PF is mc cause of heel pain. gastrocnemius +ve silerskoild). Compensation for equinus Genu recurvatum. malunion. Complications neurovascular injury. neuromuscular disorders. FF varus. passed through the interosseous membrane (Putti) or around medial Tendon Transfer malleolus (Ober) and transferred to lateral 3rd cuneiform. Local PT Surgical Procedures: Keck & Kelly Dorsal wedge osteotomy of calcaneusà BK cast for 6-7 wks. Gives EDL mechanical advantage in DF foot & prevent digital contractures Tibialis Anterior Tendon Flexible FF equinus. Procedure Indications Description Jones Tenosuspension Flexible cavus foot. Usually for flexible conditions or as adjunct to osseous procedure • Muscle must be at least grade 4 to transfer tendon & it loses 1 grade after transfer. ruptured PT. remaining EDL tendon stumps. pes planus. Steroid injections. Medial portion remains intact. w or Transect EDL at its insertion & transfer to 3rd cuneiform or base of 3rd met. NSAIDS. • Indications – Muscle imbalance. PL transected & half anastomosed to TA at its insertion & Peroneus tertius Murphy Procedure Spastic equinus in CP Advancement of Achilles tendon insertion to dorsum of calcaneus just posterior to the posterior facet of STJ anterior to the FHL (weakens triceps surae at the ankle by 50%. split longitudinally to base of proximal phalanx & wrapped around proximal phalanx & sutured 102 . fixed equinus/dorsiflexion Pain relief: tarsal coalitions. lateral ankle instability. hammertoes o Open Plantar Fasciotomy – 3-6cm plantar medial incision >> release band of fascia & resect spur. F 20-30yrs • Etiology: Compensated RF varus. intrinsic myositis. drop foot Released at the level of cuboid & transferred through septum to EDL sheath & inserted into lesser Transfer Pes cavus tarsus or base of 3rd met. Excesssive tertius tendon sheath & attached to tendon and cuboid. severe calcaneal fx. drop foot supination & increase DF Split Tibialis Anterior Weak DF. Tendon Transfer varus. resect cartilage and replace head Haughton-Dunn remove navicular Elmslie indicated for fixed calcaneus foot type Lambrinudi indicated for fixed equinus foot type Grice extraarticularfusion of STJ Tendon Transfers • Goal – active motor power. ankle arthrosis. produce stability. Young Tenosuspension Flat foot TA passed through keyhole in navicular w/o detaching its insertion Tibialis Posterior Equinus. Increases DF & supination. pes cavus Transected at its insertion. Equinus balances lateral forces. Flexible RF TA split at its insertion proximally to the superior extensor retinaculum. EDB is detached/reattached to w/o claw toes. Complications – injury to lateral plantar nerve. Peroneus Longus Tendon Flexible equinus. DDx retrocalcaneal exostosis/bursitis • XR evaluation: Lateral XR Normal (degrees) Pump bump Fowler and Phillip Angle (FPA) 44-69 >75 Total angle of Vega (CIA +FPA) <90 >90 Parallel pitch lines positive Treatment: Conservative – Shoe modification – open heel. but only weakens toe off by 15%) Adductor Tendon Transfer HAV Sx to derotate & Adductor hallucis transected at its attachment to lateral sesamoid & base of proximal phalanx. weakening of calcaneous >> fascia. CC & TN Indications: Correct deformities: Cavus foot. TA transected at its insertion & transferred to 3rd cuneiform through EDL tendon sheathàReduce Transfer clubfoot. medial column destabilization: central arch pain. Post-op – NWB cast 4wks w/ passive ROM & isometric exercises at 3wks >>> active ambulation 4th wk. distal end of EHL sutured to EHB Panmetatarsal Tenosuspension Flexible hammer toes All met heads suspended like Jones procedure Kidner Procedure Flat foot – support medial Resection of accessory navicular & hypertrophied navicular tuberosity and transpose the TP to plantar column navicular Hibbs Tenosuspension Equinus. eliminate deforming force. rerouted realign Sesamoid over met head & attached to medial capsule apparatus Flexor Tendon transfer Digital contractures FDL transected near its insertion on distal phalanx. then NWB for 3-4 wks Duvries Transverse resection of the bump through lateral incision Fowler & Phillip Transverse skin incision through posterior heel w/ a Mercedes incision through AT resect bump through this incision. PF 1st EHL tendon transected & rerouted medial to lateral or dorsal to plantar (Kirk) through head of 1st met & ray sewed back on itself. iatrogenic (HAV). drop foot. FF valgus. recurrence. talipes equinovarus. congenital deformities. scar pain & chronic enthesitis Haglund’s Deformity “Pump Bump” • Description: A painful bony prominence and bursitis of the lateral posterior superior aspect of the calcaneus above the insertion of the Achilles tendon.destabilization: C-C/MTJ pain. peroneal tenosynovitis. nonunion Types: Ryerson simple jt resection Hoke remove talar head. weakens TA Dickenson Curvilinear medial incision from superior-medial to inferior-lateral Triple Arthrodesis Def – fusion of the STJ. lateral portion passed through P. arthritis. or PF 1st ray. sinus tarsi syndrome. hematoma. AES E. plates. Fibula inlay bone graft – eg – Wescott –proximal tibia. infection. Hallock – bone chips. Ramses. 33% of persons w/ unilateral amputation will have contralateral amputation in 3 yrs.7mm Indications – unexplained ankle pain > 6mths. vein. tertius Lat to EHL & med to EDL Lateral to achilles & med to lat malleoli Saphenous nerve & Intermediate dorsal Anterior Tibial artery. no blood no healing. Deep Sural nerve. nutritional. Charcot. Stability requires solid distal syndesmotic fusion assisted by 2 screws across the syndesmosis ß only FDA approved.45 . Drop foot. AP is 5mm larger for more tibial coverage Ø Salto. TA cutaneous nerve Peroneal peronealtendons **Posterior medial gutter NOT used. painful arthritis. 5-15% of diabetics will require and amputation. BV cauterized or tied. < 26 NO healing.90% will heal. neuropathy or infection Pre-op Considerations & indications of Healing Potential – Level is the most distal point capable of healing & can be shaped into functional stump.cortico-cancellous iliac graft. Transcutaneous Partial Pressure of Oxygen-most accurate non-invasive predictor of healing. Trauma. osteomyelitis. functions to DF & invert foot → TA Split Tendon transfer to lateral foot prevent this. What is the position of an ankle/pantalar fusion? • Perpendicular to the tibia in the sagittal plane • Slight valgus in the frontal plane • 12 degrees of abduction in the transverse plane Complications: non-union.0 – 2. chondromalacia. lesser saphenous vein. Portal Location Structures • • • • • • 103 . post polio or club foot procedure fixated w/ screw. Congenital deformities. mignancy. PB inserts into 5th met base & everts & PF the foot. with a long tibial stem.mc Anterior Lateral Anterior Central Posterior Lateral Med to TA Lat to EDL & P. external fixation 2. PVD.0mm. – Sharply section nerves under tension to prevent stump neuromas. arthritis. osteochondral fx. 33% 5 year survival post BKA Indications – End stage vascular Dz. Photoplethysmography.7mm & 4. vascular insufficiency Types Ø STAR (Scadanavian total ankle replacement) Ø Agility total ankle-must resect 50% of medial and lateral mallelous & 20-30% talar dome.Ankle Arthroscopy Directly viewing joint with Ankle scopes – 2. medial to ACH & lat to tarsal canal. 2 many vital structures – Posterior tibial nerve & artery. post-polio. Bleeding of skin at the time of surgery is most accurate clinical sign of healing potential. bone tumor Types of ankle fusions 1.Tibiocalcaneal arthodesisà post AVN talus. Tom Dick. Amputations Def – disarticulation/ removal of a part of a limb Epidemiology – DM is #1 cause of non traumatic LE amputation – 60%. loose chondral /osseous fragments in joint Contraindication . Hallux – 2. little bone is resected o Doets BP Tibial modification-tibia with 2 short stems. Charcot. Anterior Medial . synovitis/capsulitis. Tendon balancing & retain muscle function – preserve TA & PB to prevent equinus with trans met amputations (equinovarus deformity). DJD. ligament repair. Ø Beuchel-Pappas total ankle. >40 will heal Intra-Op Considerations – Eliminate dead space – adequate hemostasis & close in layers and Use most non-reactive skin closure that put least amt tension on wound edges – staples best b/c high tensile strength & non reactive. Life threatening infection. arthritis RA/OA. TA is still attached to plantar medial cuneioform & 1st met base. can use with talar AVN. Blair . severe osteoporosis. AVN of talus. synovitis. Campbell.thick talar component. Trauma Pilon Fx. immune status – Vascular studies: Doppler ultrasound w/ segmental pressures. ABI > 0. Harry** Ankle arthodesis Indications: most commonly due to a short fibula after an ankle fractureà 1mm lateral shift à loss of 40% congruency. Absolute: talar necrosis. misalignment Total ankle replacements Indications – joint pain. complication of DM. Consider also – medical – vascular & glycemic control.Relative: previous deep infection. ind – moderate dz Clayton 1936 Transverse plantar incision. ACH transferred to distal posterior tibia to prevent drop foot. Prevention meticulous surgical technique. Post –op . Tx dx bone scan. plantar condylectomies and removal of the proximal phalanges of 2-5. boutonniere flexion of PIPJ & extension of DIPJ. recurrent plantar lesions. only partial met head resections. hemostasis. nerves & tendons transected under pressure so they retract into stump. for hematoma – strep. healed in undesired position – malunion. tendon balancing & transfer. resect enough bone to relieve all soft tissue tension Hoffman 1911 Transverse plantar incision just behind web of toes . Post op prosthesis & shoe gear necessary Choparts Resection of entire foot except talus and calcaneus with calcaneocuboid + talonavicular jt disarticulation & closed with long platar Amputation flap. brevis if can’t transfer it to cuboid or transfer P. med & lat malleoli resected above articular cartilage. don’t cause termal injury to bone in healing sx with saw & avoid soft tissue interposition b/t bone ends. Prevention – meticulous dissection. Resection of the base of the proximal phalanges and heads of 1-5 mets. use peri-operative antibiotics strict sterility of instruments and hardware Poor Delayed union < 6-9mths. ind severe dz Larmon 1951 3 dorsal longitudinal incisions. decrease operating time. and it does not affect DIPJs. Entire foot except calcaneus which is shifted anteriorly & superiorly & fused to tibia to maintain leg length.General Soft Tissue Infection & Acute: Erythema. deflate tourniquet prior to closing Scar Chronic: contracture and remodeling >> Hypertrohic or keloid scars. Post –op – prosthesis needed. TAL necessary.Types Digital Distal Symes Removal of distal phalanx via dorsal plantar. Pre-op considerations – these pts are usually tx w/ immunosuppressive agents & steroids >> cardiac conditions. Long distal plantar flap brought up to meet proximal dorsal flap for weight bearing surface. joint manipulation under anesthesia Surgical Treatment: Pan met head resectionsà complications – vascular compromise/dry gangrene. resection of met heads 1-5 with consideration of the met parabola (2 1 3 4 5 ) Complication of Foot Surgery . longus in an effort to maintain PF/Ever Transmetatarsal Most common proximal foot amputation. MUST consult rheumatologist to have them alter or stop meds for Sx. debridement of necrotic bone. anemia of chronic disease. AVN Mc in 1st met head and talar neck post bunion surgery and talar neck fx. peripheral neuropathy. Deformities include swan neck – hyperextension of PIPJ & flexion of DIPJ.Resection of met heads 1-5. Keller – arthroplasty proximal phalanx. preserve calcaneal fat pad beneath the tibia for weight bearing surface. ind – minimal deformity Hodor 1983 5 dorsal incisions.Dx biopsy & culture. remove necrotic bone and fuse joint. High rate of complication – distal ulceration. PT & Rehab. least reactive suture and fewest amt of stitches OM Bone infection after Sx. Bone 104 . release joint contractures. TA tendon reattached through drill hole in talar neck to prevent equinus + TAL. BV. surgical excision.> 6-9mths. Posterior tibial artery supplies the flap. Ray Resection Removal of digit & portion or all of the metatarsal. metatarsals transected at distal 1/3 in a smooth parabla w/ 2nd longest. nerve decompression. TA & PB sacrificed >>equinovarus. fibular/ulnar deviation of the digits. excision of rheumatoid nodules. thinnest. tenosynovectomy. Non union . silicone sheeting. peripheral vasculitis. Ind – infection in web-space or met. sutures stay in longer. Tx – po or IV antibiotic for G Hematoma +ve cocci. bursectomy. Complication – TAL & split TA tendon transfer done to prevent equinus. Smooth out uneven Amputation joint and articular surfaces to prevent healing problems.AFO & custom molded shoes w/ plastizote insert & transmet filler. side to side or racquet type incision. bone stimulator. Prevention – adequate fixation and NWN. operative time. fragile non elastic skin. Tx . Tx Sub-Q steroid injections. Post-op prosthesis Boyd Mid tarsal disarticulation and talectomy. mets angled distal Amputation –dorsal to proximal plantar. formation topical Vitamin E. Pirogoff Modified proximal symes that preserves calcaneus for limb length. long plantar flap. Treatment: Soft Tissue – Synovectomy. Prevention – meticulous dissection. fragile vessels. Other effects include pericarditis. Symes Total foot amputated. thin capsules & weak tendons >> poor healing potential. electrical bone bone growth stimulator. drain & compression dressing. Transection in line parallel to the ground. Tx – NWB. length of stay. Skin staples with suture line above anterior margin of distal tibia. non purchasing toes & most common successive transmetatarsl amputation. esophagus contriction and Baker’s cyst in the popliteal fossa. TC arthrodesis Post Op Antibiotics. RA nodules. edema & pain . Inaddition d/t deformities in hands & weakness in shoulders may not be able to manage ambulation aids. lat and plantar surfaces are beveled * with 5th met try to preserve insertion of P. Complication – resection of 1st & 2nd ray → increased pressures under plantar surfaces → more proximal amputations. avoid denuding soft tissue from bone & don’t create thermal burns with power tools. pleuritis ocular. Prevention – screen pts. Prosthesis Rheumatoid Arthritis Presentation: “Pain in the AM” with morning stiffness and constitutional s/s. rocker bottom shoes Lisfranc Resection at level of lisfranc joint. debridement 6 wks antibiotics . ensure medial. etiology for infection – staph. Tetanus Immune Globulin 250 – 500 units IM -Green & Bruno Type I . maybe skin grafting at this time Closed Fractures Rockwood & Green 105 .. backing out.2L Tx irrigation -2L & >5cm dirty. sterile prep.Dx & TX . open packing IV Ab • Complications – soft tissue infections. Immediate sx removal of foreign body. Prev – careful incision planning & dissection Neuroma Sx Chronic numbness & parasthesias in interspace.17%. lavage. tenodese FHL to remaining bone of base 1st MPJ implants Dendritic synovitis. Tetracycline for PCN allergic or Erythromycin **P. excessive shortening (pick procedure & use axis guide). damage to skin. Cefitoxin S. recurrence of plantar keratosis. 57% of pt presenting 1 . weak Flexors. Osteomyelitis resulting from puncture wounds.Rx early I&D &debridement. chronic infection & bone resection for final cure Bite Wounds Dog Bites Cat Bites Mc – 90%. antibiotics. infection. foreign body granuloma. no clinical signs of infection .. Anesthesia. Rx debride ment & antibiotics. septic arthritis. pack open. multicida is resistant to PCN & penicillinase resistant PCN.weekly f/u Sub-q or articular joint involvement. thorns.50% Hands Aerobes: S. hematoma. Eikenella corodens Anerobes: B. antibiotics – Augmentin DOC for animal bites. no clinical signs of infection. 25 . limited ROM > removal of implant. arthroplasty or arthrodesis Hammer Toe Sx chronic edema. peptostreptococcu May need IV antibiotics and wound culture. nonunion. avoid aggressive resection of medial eminence & IM -0 Eleation 1st Met d/t early ambulation following base procedures. hardware failure. Immediate sx removal of foreign body. weaken PF & toe purchase. aureus resistant to PCN & E. etc. wound cultures. TLS drain. 2nd met capsulitis. wound exploration. F/u 3-5 days. infection IR . AVN. probe wound. aureus. Rx TT 0. Complications Prevention . muscular and vascular structures Fx comminuted & 1o closure primary closure Rx Cephalosporin & Aminoglycosies Tx irrigation . Prevention must be NWB – 6 weeks! Keller Lack of toe purchase. heal w/o any bone or joint damage Type II . premature epiphyseal closure. corrodens is resistant to PCNase resistant PCN Open Fracture **Gustilo & Anderson** Golden period – 3 . in water – aeromonas hydophilia. fragilis. open packing IV Ab Secondary osteomyelitis.90% > amputation Tx irrigation . Etiology mc – staphylococcus aureus.4L & delayed primary closure. pin tract infection. for diabetics polymicrobial – Unasyn. Anesthesia. copious irrigation & pulse lavage.150-450lbs/sq inch Etiology – Pasteurella multicida Tx Irrigation w/ NS. Prevention:careful pre-op planning. s/t and osseous debridement. osteomyelitis. cultures.Sterile prep. cultures & pack open Establish s/t infection.7 days after injury develop cellulitis and infection Goals of Tx – convert contaminated wound to clean wound & prevention of tetanus Type I Type II Type IIIA Type IIIB Type IV Resnick & Fallat Superficial penetration of dermis & epidermis. joint degeneration. Infection rate – 25 Infection rate – 0% Infection rate – 3% 50%. sterile prep. jamming of jt.wound exploration. immobilization & elevation. uncomfortable. mycobacterium marinum & vibrio -Tx . NWB. antibiotics – 1st gen ceph. stump neuroma.Augmentin. stress fx. soft tissue and osseous debridement.always countersink for screw Trauma Puncture Wound Nail is common object >>metals.5mls. flail toe.Dx & TX w/i 9 . amputation rate Fx Fx transverse/oblique AR – 5% transverse/oblique Rx Cephalosporin & IIIB insufficient skin for coverage & bone exposed IR 26% AR 5-6% Rx Cephalosporin Aminoglycosies IIIC no skin for coverage & require microvascular surgery to save limb. delayed. Cleanse bid. under correction and Surgery reccurence.5hrs Stage I Stage II Stage III <1cm clean >1cm clean IIIA sufficient skin to cover defect but periosteal stripping.14days. retained foreign body. pack open & antibiotics Foreign body penetration to bone. Keflex poor activity Human bites 3rd mc (2-3%) Highest infection rate .50% Mc in males <20yrs on extremities Females on upper extremities Crush injury . Corynebacterium . heal w residual bone & jt destruction Type III . Timentin or Zosyn + gram negative coverage Tetanus prone – if > 6hrs & larger than 1cm deep.3wks or more. Prevention – meticulous dissection & deflating tourniquette prior to closing Hardware Prominence > soft tissue irritation > removal. pack open. needles. post-op white toe. least infection 2nd mc (5%) Infection rate – 30 . glass. lavage. Hallux varus d/t over correction. mal. Prevention – lengthen EHL tendon to weaken it.Complications of Foot Surgery – specific Procedure Complication Bunion Joint stiffness. moist guaze dressing. lymphocytes. A higher kVp will have a greater penetrating power. use barriers b/t operator/unit – 1. Rapidly growing cell most susceptible – epithelial cell . Fx w/ butterfly fragments o Angulation & Rotation Fx: oblique Fx lines Charnley's principes of closed reduction: Increase deformity > distract > reverse deformity & realign >Maintain correction w/ immobilization Charnley's Classification of Fractures: Based on intrinsic stability of fx to w/stand forces that tend to telescope the fragments.death in 72 hrs. central ray 5cm superiorly from posterior Calcaneal prominence tali Sesamoidal Cassette vertical. 45o to axis of long bones. foot obliqued 45 o laterally. Lower kVp will have less grays. central ray aimed at 4th metatarsal base Forefoot and rearfoot talo-navicular. and the answer equals the total cumulative whole body absorbed dose permitted during the course of 10 yrs. LAT. Calcaneo-cuboid jts LO Angulation – 0 o. Digits extend against the plate and heel elevated. Mortise. Calcaneus – axial. immature RBCs and fetal cells Operator & patient protection • least amt of time in x-ray beam.hair. angulation – 90o. I – Stable II – Unstable III . distance further away from beam > 6ft and use of shields reduce exposure. Photons generated are heterogeneous – some are high energy & pass through pt to contribute to diagnostic image and others are low energy photons that do not contribute to diagnostic image b/c they do not reach the film but are absorbed by pt. crista of 1st met head axial ray aimed at plantar surface of metatarsals Calcaneal axial Horizontal cassette.05 rem) o Maximum Permissable Dose (MPD) 5 (N-18) where N= age in yrs. better soft tissue visualization.Potentially stable RADIOLOGY – 15% Exposure: Rad = rem = Roentgen o Occupational exposure (annual maximum): 5 rem o Public exposure (annual maximum): . Angles – Posterior facet. • Shielding -Lead apron must be > 0. LAT View Positioning Structures viewed AP /DP 15o from vertical. in the xray tube heating tungsten filament >>fast moving electrons >>strike target and 1% is converted to xrays. embryological defects. MO. talar dome.5mm thick. slight flexion in knees. talar 10 o 20 o 30 o 40 o. angulation 90o foot internally rotated 15 -20o Better visualization of lateral malleolus & talar dome 106 .5 mm thickness. plantar aspect on cassette.5 or monthly (0. LAT. higher contrast. and will produce less contrast . central ray aimed at first metatarsal cuneiform joint Best visualization of medial column MO Angulation – 0 o. angle varies 35Medial & posterior sustentaculum Beath 45o. Fx caused by bending forces of long bone Spiral Fx: oblique Fx. tube is angled 45o central ray aimed at posterior heel Post/medial/lateral margins of calcaneus AP Ankle Cassette vertical angulation 90o ray at anterior ankle Ankle joint. increases radiographic density>> increasing darkness of the film. Lethal Dose LD50/30 – kill 50% people in 30 days = 200-300 rems • Long Term–exposure to small amt of radiation over time >> decrease life span d/t carcinogenesis. central ray is aimed at the 4th metatarsal cuboid articulation Broden MO. Other beams scatter in the room >> occupational exposure.5 rem o Fetal exposure (total dose) 0. rotational force Compression Fx: impaction Fx into soft cancellous bone o Angulation & Axial Comp: transv. malleoli Ankle Mortise Cassette vertical. Number of electrons passing from cathode to anode in 1 sec is milliamperes (mA). Sesamoids – axial. the filament is the cathode & target is the anode. genetic mutations • Radiosensitivity – Increase with high rate of division and immature cells. central ray aimed 2cm inferior & 2cm anterior to lateral malleolus sustentaculum tali articulation Harris Pt faces away from tube. central Sesamoids.less soft tissue detail and increased radiographic density. attach o Angulation Fx: transv.• • • • Direct Trauma-closed Fx caused by direct blows of varying velocity o Tapping Fx: low velocity blow w/ no soft tissue damage o Crush Fx: high velocity w/ extensive soft tissue Indirect Trauma . and decreased radiographic density Effects of Radiation • Short term – large doses >> Acute radiation syndrome . The Higher the mA>> increases the QUANTITY of x-ray photons.closed Fx resultin from forces acting at distance to Fx site o Traction Fx: transverse avulsion at site of tendon or lig. xrays are produced when one form of energy is converted to another. Calcaneo-cuboid jts LAT 90o from vertical. skin. Tib/Fib – AP & LAT. Maximum peak potential difference is kVp – quality/penetrating power of xrays produced. Ankle – AP. Standard view: Foot – AP. mA – quantity of xrays. aim central ray at 2nd metatarsal base Forefoot and rearfoot talo-navicular. the potential difference is kilovolts (kV). o ALARA concept: As Low As Reasonably Achievable: use of technical factors which will produce ther lowest dose of radiation yet to achieve a diagnostic study Physics radiation is energy in motion. post calcaneous on cassette front & rotate leg 45o medially invert & DF foot. 7ft high and permanently affixed to floor • -High kVp / low mAs technique reduces patient exposure by 70% Positioning Weight-bearing or non-weight bearing films. gonadal. foot rotated medially 45 o. soft tissue atrophy. bone & soft tissue infections. dislocation. soft tissue calcification. loss of normal osseous architecture w tapering of long bones (licked candy stick) Special Imaging Modalities CT Scan (Computed Axial Tomograghy) Physics: Cross sectional radiography utilizes thinly collimated x-ray beams that circumferentially rotate around the body part. osseous debris. Heavily T2 weighted o o o Contrast material: gadolinium used IV and intra-articular.2-5mm. unexplained pain Contraindications – Pacemakers. soft tissue narrow – 600. arthritis. pregnancy (1st trimester). soft tissue calcification in subcutaneous tissue CVD .Radiographic Pathology AP projection l Normal forefoot will have mild MT convergence at the bases l FF varus will have increased MT convergence l FF valgus will have decreased MT convergence Lateral projection l Normal FF with MT superimposition (but 5th in plantar position). Body parts with lots of water show better because of the high Hydrogen content. Spatial encoding of the signal allows cross-sectional images to be obtained in any plane. Water = 0. tumors of bone & soft tissue. pencil-in –cup deformity of MPJs Reiter’s Disease Exuberant infra-calcaneal spur formation . C shapd erosions with overhanging ridge of bone (Martel’s Sign) . AVN. similar to psoriatic arthritis Ankylosing spondylitis Bilat sacroiliitis with S-1 jt fusion = bamboo spine Rheumatoid Arthritis Periarticular s/t edema jt space widening. x-ray penetration converted to electrical signal and collected by computer and displayed. Air -1000 -black. Pulse sequences Proton Density – balanced T1 T2 (fat suppression) TE <50 TR > 1000 TE <50 TR <1000 TE >50 TR >1000 Fat & Fluid Bright 90 Fat and bone marrow is bright Fluid and blood are bright STIR – fat suppression technique. Bone +1000 . joint abnormalities Diabetes Mockenberg’s medial calcinosis. swan-neck and boutonniere deformities of the hand Gouty arthritis Extra-articular involvement a hallmark. infection. tuftal resorption. osteonecrosis Osteoporosis Ostepenic bone. juxta-articular osteopenia initially. GE – takes shorter time. increased density of bone and s/t. good for evaluating edema in high lipid regions – bone marrow. trauma. cortical bone. post-op clips. The signal from a body tissue depends on the number of mobile protons. w/decrease tensile/compressive trabecular pattern in calcaneal/femoral head. Metallaic implants will create artifact and cause pt harm: iron or sheet metal workers w/ ocular foreign bodies. decrease s/t swelling fragments begin to unite & resorption of fine particulate debris. good for joint imaging. As the protons relax from the pulses.scleroderma Diffuse osteoporosis. ivory phalanx (distal phalanx of hallux) IPJ involvement of hallux. deforming non-erosive arthropathy of hands and feet. normal mineraliation (in contrast to RA). subchondral cysts (intraarticular osseous fragment) jt collapse. cartilage abnormalities. infection. It may also be given intraarticularly for CT arthrograms o Contraindications: ist trimester of pregnancy. erosive changes. arthritis o Contrast material: iodinated contrast may be given IV. general osteoporosis. claustrophobia. will accumulate in the tissues based on vascularity. allergy to contrast agent. Good with fat suppression techniques Indications – tumor. tendon. disorganization. weight limit for gantry MRI à Best imaging modality for soft tissue evaluation. uniform jt space narrowing. destruction. fibro-cartilage and ligaments.white Window width – bone wide – 2000. deformity/malalignment Psoriatic arthritis Acro-osteolysis (distal tuff resorption). they emit radiofrequency signals. There are no known biological effects. Fusion of osseous components. • based on H+ atom (proton) which has highest gyromagnetic ration. retro Calcaneal bursa. l FF varus with ladder configuration of MT and 5th in plantar position l FF valgus with MT superimposition (1st MT in plantar position) Osteoarthritis (DJD) Osteophyte formation. The isotope emits gamma radiation for a brief period of time which are recorded by a gamma camera or scanner Hot – increased uptake of isotopes eg – fx. abnormal widening of joint spaces. but MRI is avoided in the first trimester b/c of lack of data. trauma. claustrophobia Nuclear Medicine Physics: Based on the injection of radioactive isotope into the body and the analysis of its distribution and accumulation. coalitions. osteochondrosis dissecans. Mobile hydrogen ions in the body are first aligned by a stationary magnetic field and then excited by radiofrequency pulses. aneurtsm clips. decrease cortical thickness CRPS Spotty osteoporosis stage 2 = sudeck’s atrophy. and the interactions between the protons in that tissue. Sero-positive Calcaneal erosions. cardiac valve prosthesis. AVN. joint effusion and edema. congenital anomalies. tophi formation: Na urate deposits. ankylosis. Slices . loss of normal osseous configuration & alignment Atrophic – whiskering of articular endresembling licked candy stick CVD . osteochondrosis. low signal from substances with few hydrogen atoms such as air. cold spots Indications – no isotope uptake d/t absence of blood flow eg AVN 107 .SLE Joint space narrowing . calcification of tophi ankylosis Pseudogout Calcification of articular cartilage and peri-articular tissue Charcot Joint Hypertrophic -Jt distention. eburation. Tissue density measured in Hounsfield units: 2000 to +2000. o Indications – Best for pathology of cortical bone. Physics: No ionizing radiation is used. periostitis. gluteal. psoas major and minor muscles. sonogram.àcellulitis § GCTCmd99+ à osteoarthropathy. renal dysfunction Isotopes: Ø Tc99 – most useful measures osteoblastic activity. The Illiac region is composed of the iliac. bone tumors. immunocompromised pt.(2-4 hrs) represents relative osteoblastic activity. stage 1 Charcot & OM is indistinguishable. chronic OM Ø Indium-111 .. not active if pt has porr renal clearance in which case phase 4 is also performedà can differentiate OM from cellulitis. checks for cellulitis. best for acute infections and more conclusive than GC Ø Ceretec HMPAO scan-WBC labeled scan àDifferentiate OM from Charcot using bone scans? On X-ray and clinically.binds to WBC plasma proteins (and lactoferrin) measuring their activity in the soft tissue and is used in conjunction with the TC-99 bone scan to differentiate OM vs cellulitis. (non-specific) ½ lifeà 6hrs § 1st phase-blood pool (1-3s) § 2nd phase-blood flow (3-5 min) represents relative vascularity to a particular area or lesion prior to skeletal uptake § 3rd phase-delayed. neuropathic osteoarthropathy. and muscles of the leg and foot. linea Inferior gluteal gluteal Flexes the hip aspera and medially to 1st perforating branch of L5-S1-S2 Abducts the thigh the STJ profunda femoris Laterally rotates thigh Ossicle 2nd-4th met 5th met 1st met Distal phalanx Proximal phalanx Middle phalanx Calcaneus Talus 108 . Superficial gluteal region Muscle Origin Insertion Blood supply Nerve Action Gluteus medius Illium: between the anterior Lateral surface of the Superior gluteal Superior Abducts thigh and posterior gluteal lines greater trochanter gluteal Medially rotates thigh Gluteus Illium: between the anterior Anterior border of the Superior gluteal Superior Abducts thigh minimus and inferior gluteal lines greater trochanter gluteal Medially rotates thigh Tensor fascia Outer lip of the iliac crest Iliotibial band Superior gluteal Superior Flexes thigh lata ASIS Lateral circumflex gluteal Medially rotates thigh Glueteus Behind posterior gluteal line Gluteal tuberosity. & osteomyelitis § 4th pase-scanned 24hrs post injection to allow for adequate tissue clearance in pts who have poor renal activity Ø Gallium-67 citrate .WBC binding . Best for subacute and chronic infection ½ life-> 79hrs. arthrogram (CT & MRI) § Contraindications – allergy to contrast agent (iodine) unless premedication is given. RSD. Funciton and structure (normal and abnormal) 1. MRI Ø Agents – Iodine contrast – Xrays & CT. CT. cellulitis Orthopedics. biomechanics and sports medicine A. trauma. Air – nontoxic medium. so a ceretec bone scan will be hot in the 4th phase with OM and stay cold in charcot Contrast Studies injection of contrast medium to visualize an area not normally seen on plain radiographs. more reliable in differentiating b/t benign vs malignant tumors § GC+ TCmd99 + à OM GC+ TCmd99. inflammatory arthritis. nonspecific and dependent upon intact vascular supply Ø TC-MD99 bone scan is a triple phase scan. Superior gluteal Inferior Extends thigh maximus iliotibial band. seen in stressed Fx. OA. Gadolinium used with MRI § Indications – Tenogram. tumors. Osseous system Bone ossification dates Primary ossification Secondary ossification Maturance/fusion 9th week IU 3-4 yrs (head) 17-20 yrs 10th week 3-4 yrs (head) 17-20 yrs 10th week 3yrs (base) 17-20 yrs 9-12 week 2-8 yrs (base) 18th year 11-15 week 2-8 yrs (base) 18th year >15 week 2-8 yrs (base) 18th year 3 mo IU 8-12 yrs 6 mo IU If present 8-9 yrs Cuboid 9 mo-at birth Lateral cuneiform 1st yr Medial cuneiform 2nd yr Intermediate cuneiform 3rd yr Navicular 4th yr Sesmoids 8-14 yrs 2. Contraindications – poor vascular supply. femoral.o o o o Indications – osteomyelitis.measuring their activity. stress or occult fx. stress fx. Muscular system The muscle groups of the lower extremity are composed of 5 regions: iliac. non-specific pain. medial femoral circumflex Obdurator. S2 N. internal pudendal Inferior gluteal Inferior gluteal. posterior and medial compartments. floor of femoral triangle Inferior pubic rami Ischiopubic ramus Ischial tuberosity Nerve Femoral Accessory obdurator Anterior obdurator Adductor longus Adductor brevis Adductor pt of the adductor magnus Extensor pt of Adductor magnus Anterior obdurator Anterior obdurator Posterior obdurator Tibial nerve 109 . EXITS the Lesser sciatic foramen Ischial tuberosity Lateral margin of the ischial tuberosity Along the margin of the obdurator foramen Insertion Upper border of the greater trochanter medial surface of the greater trochanter medial surface of the greater trochanter medial surface of the greater trochanter Intertrochanteric crest. obdurator foramen. linea aspera Gluteal tuberosity linea aspera linea aspera. to the obdurator internus L5. L5. S1 N. medial femoral circumflex Obdurator. adductor tubercle Nerve Anterior Femoral Posterior femoral Posterior femoral Posterior femoral Posterior femoral Anterior femoral Blood supply Femoral a Lateral femoral circumflex Lateral femoral circumflex Femoral a. Anterior femoral region “Quads” Origin Insertion ASIS-has it’s own Medial surface of the tibia anterior fascial (covers tendons of pes compartment anserinus) Rectus femoris AIIS Upper border of the patella by ligamentum patellae into tibial tuberosity Vastus lateralis Greater trochanter Lateral border of the patella into Linea aspera the ligamentum patella Vastus medialus Spiral line Medial border of the patella into Linea aspera the ligamentum patella Muscle Sartorius Vastus intermedius Artucularis genus Linea aspera Front and lower pt of femur Tendon of rectus femoris Medial border of the patella into the ligamentum patella Capsule of the knee. the posterior medial septum separates the medial from the posterior compartment and the lateral septum separates the posterior from the anterior compartment. S1. The anterior medial septum separates the anterior from the medial compartment. S2 N. profunda femoris. medial femoral circumflex Obdurator. genicular branches of the popliteal Lateral femoral circumflex Superior genicular branches of popliteal Blood supply Obdurator. just below thetuberosity linea aspera on the femur Pectineal line of the femur. medial femoral circumflex M. suprapatellar bursa Insertion Pectineal line of the femur Medial surface of the tibia. S1. L4) Blood supply Superior & inferior gluteal. to quadratus femoris L4. branches of profunda femoris Obdurator. internal pudendal Inferior gluteal Superior & inferior gluteal. branches of profunda femoris Action Flexes leg Flexes thigh Laterally rotates thigh Flexes thigh Extends leg Extends the leg Draws patella laterally Extends the leg Draws patella medially Extends the leg Tightens the capsule of the knee jt Action Flexes thigh ADDucts thigh Medially rotates thigh Flexes leg ADDucts thigh Medially rotates thigh Flexes thigh ADDucts thigh Flexes thigh ADDucts thigh Flexes leg ADDucts thigh Extends the leg Muscle Pectinues Gracillus Medial femoral region Origin Pectineal line of the pubis Inferior pubic ramus and pubic symphysis Between pubic crest and pubic symphysis. medial femoral circumflex M. below the PIIS Ischial spine Greater sciatic notch. S2 N. to quadratus femoris Posterior branch of the obdurator (L2. medial femoral circumflex Obdurator a Medial femoral circumflex Action Laterally rotates thigh Abducts thigh Latterally rotates thigh Laterally rotates thigh Abducts thigh Latterally rotates thigh Laterally rotates thigh ADDucts thigh Laterally rotates thigh ADDucts thigh Quadratus femoris Obdurator externus The femoral region is divided into anterior. quadrate tubercle Trochanteric fossa passes across hip jt Nerve Ventral rami of S1.Muscle Piriformis Superior Gameullus Obdurator internus Inferior Gamellus Deep muscles of the gluteal region Origin Sacrum SV-4 Sacrotuberous ligament Ilium. L3. to the obdurator internus L5. FibulaTendon descends behind the medial malleolus. Anterior crural muscles “Extensors of the leg” Origin Insertion Tibia Medial plantar surface of the medial cuneiform and medial aspect of the base of the 1st metatarsal w/PL Extensor Hallucis Fibula Superior aspect of the base of the distal phalanx of the big toe Extensor digitorum Tibia and Divides into 4 tendons in front of the AJ.Posterior femoral region “hamstrings” Origin Insertion Linea aspera Combined tendon forms the lateral boundary of the popliteal fossa-can form pt of the fibular collateral ligament at the Ischial tuberosity fibular head Sacrotuberous ligament Semimembranosus Ischial tuberososity Oblique politeal ligament of the knee Ischial ramus and fascia of the popliteal muscle. and distal to the gracilus of the biceps femoris Muscle Short head of biceps femoris Long head of the biceps femoris Nerve Common peroneal Tibial nerve Tibial nerve Blood supply Perforating branches of the profunda femoris. muscular rbranches of the popliteal artery Perforating branches of the profunda femoris. groove on the medial condyle of the tibia Semitendonosis Ischial tuberostiy in Medial surface of the tibia. soleal line med to post-lat and v/v) Lateral to the supracolndyle of Medial side of the posterior part of the the femur calcaneus and Achilles tendon Nerve Deep peroneal Deep peroneal Deep peroneal Blood supply Anterior tibial Aterior tibia Aterior tibia Action Dorsiflexion Inversion of the foot DF/inversion of the foot Extends hallux Extends the 4 toes at the MPJ Everts the foot Deep Peroneal Nerve Tibial nerve Aterior tibia Blood supply DF and everts the foot Action Plantarflexes the foot Soleus Plantaris Tibial nerve Tibial nerve Plantarflexes the foot Plantarflexing the foot Deep posterior crural muscles –separated from the superficial by the deep transverse septum and from the anterior compartment by the interosseus membrane between the tibia and fibula Muscle Origin Insertion Nerve Blood supply Action Popliteus Lateral condyle of the femur. back of soleal line. Principal invertor of the navicular continues onto the plantar surface to insert into all the tarsal foot bones except the talus and base of 2-4 metatarsals FDL Tibia. nerve branches of the Flexes the big toe running obliquely across the sole of the foot above the FDL popliteal between the sesmoids and the belly of FHB (camper’s chiasm) into the base of the distal phalanx 110 . to insert in the base of the distal phalanx nerve Planatarflexion of the in the lateral 4 toes below the FHL foot FHL Fibula Deep to the lacinate ligament along a groove on the Tibial Muscular Plantarflexes the foot posterior surface of the talus and sustentaculum tali. in CKC. medial border of the soleal line. superficial and deep posterior and lateral compartments. bound by Tibial Peroneal a Plantaflexes and inverts sesmoid bone that the lacinate ligament below the FDl under the ABH below nerve Posterior tibial the foot inserts into the the spring ligament inserting into the navicular tuberosity. Triangular area above the Tibial nerve Genicular branches Flexes the leg. and 2 collateral slips to the bast of the distal phalanx Peroneus tertius Fibula Base of the 4th and 5th metatarsal Muscle Tibialis anterior Muscle Gastrocnemius Superficial posterior crural muscles “Plantarflexors of the leg Origin Insertion Lateral head from the lateral (2/3rd) Posterior surface of the calcaneus frmoral condyle ß Fabella via the tendoachilles tendon in conjunction Medial head from medial with the soleal muscle. muscular branches of the popliteal Action Flexes the leg Flexes the leg Extends the thigh Laterally rotates the leg Flexes the leg Extends the thigh Medially rotates the leg Flexes the leg Extends the thigh Laterally rotates the leg Tibial nerve The leg is dicided into 4 compartents contained within the crural fascia of the leg which is continuous with the fascia lata: anterior.antagonist to of popliteal rotates the leg. medially arcuate popliteal ligament. below the Tendon descends behind the medial malleoli. deep to the common w/ long head Sartorius. That expansion divides into 3 slips: at the base of the middle phalanx. below the Tibial Posterior tibial Flexes the lateral 4 toes soleal line sustentaculum tali. the lateral meniscus vastus medialis laterally rotates leg Tibialis posterior Tibia. forms an longus fibula expansion over the dorsum of the MPJ where it fuses with the capsule near the PIPJ. The direction of turn femoral condyle of the fibers of the Achilles tendon from superior to inferiorà AMPL LAMP (antFibula. tibia. muscular branches of the popliteal artery Perforating branches of the profunda femoris. FDL is located between the split on the way to the distal phalanx of the lateral 4 toes Abductor hallucis Medial process of Medial sesmoid at the base of the proximal brevis the tuber calcanei phalanx of the big toe in common with the medial head of the FHB Abductor digiti Lateral/medial Lateral side of the proximal phalanx of the lillte minimi process of the toe-some fibers are inserted in the 5th metatarsal calcaneus Muscle Quadratus plantae 1st lumbrical 2nd lumbrical 3rd lumbrical 4th lumbrical 2nd layer-quadratus plantae. lumbricales Origin Medial head-Medial surface of the calcaneus Lateral head-lateral surface of the calcaneus and long plantar ligament Medial side of the FDL of the 2nd toe Unipennate Medial side oFDL of the 2nd/ 3rd toe Bipennate Medial side of the FDL of the 3rd/4th toe Bipennate Medial side of the FDL of the 4th/5th toe Bipennate Insertion Deep surface of the tendon of the FDL Below the deep transverse intermetatarsal ligament Medial side of the base of the proximal phalanx of the respective 2-5th toes and into the extensor hood apparatus Nerve Medial plantar nerve Medial plantar nerve Lateral plantar nerve “Baxter’s nerve” Nerve Lateral plantar nerve Medial plantar n Lateral plantar nerve Blood supply Medial plantar a Medial plantar a Lateral plantar artery Action Flexes the proximal and middle phalanges at the PIPJ Flexes and ADDucts the hallux Flexes and Abducts the little toe Blood supply Lateral plantar a Plantar metatarsal a Action Flexes the terminal phalanages of the lateral 4 toes Increases the slack of the FDL. proximal phalanx Lateral plantar mets and sheath of PL of the hallux with the lateral head nerve of FHB Transverse head-capsule of Fibrous sheath of the FHL the 2-5 MPJ. ADH. cruciate crural lig. Blood/nerve supply Dorsalis pedis and lateral tarsal a. crosses the sole Plantarflex the 1st met enabling the oblique to inset in the lateral aspect of the medial cuneiform Common foot to remain plantigrade and base of the 1st met peroneal Peroneal brevis Fibula Winds around the back of the lateral malleolus and inserts Superficial Peroneal Plantarflexion and eversion of the into the tuberosity of the 5th met peroneal foot. deep transverse intermetatarsal ligament Flexor digiti minimi Base of the 5th met Lateral side of base of the proximal Lateral plantar phalanx of the 5th toe Blood supply 1st plantar metatarsal artery 1st plantar metatarsal artery Action Flexes hallux ABDucts an dlfexes hallux Lateral plantar Flexes little toe 111 . base of proximal Medial plantar plantar surface of the cuboid. each pt divides brevis the calcaneus into 2 pts and inserts in the middle phalanx. extensor retinaculum over the sinus tarsi Insertion EHB most medial tendon-inserts in the proximal phalanx of the hallux.Master knot of Henry is where FDl/FHL crosses right above the navicular and marks the distal limit of the tarsal tunnel Lateral crural muscle of the leg Muscle Origin Insertion Nerve Blood Action Peroneal longus Tibia and Behind the lateral malleolus in common with the peroneus Superficial Anterior Plantarflexes the foot Fibula brevis through a notch on the lateral side of the cuboid peroneal tibial Everts the foot where a sesmoid bone maybe present. flexor digiti minimi Muscle Origin Insertion Nerve Flexor hallucis brevis Medial head-Medial pt of the Medial sesmoid. Major pronator of the foot Dorsal foot muscle: Extensor Digitorum brevis/ extensor hallucis brevis Origin Upper/lateral surface of the calcaneus. lateral talocalcaneal lig. phalanx of the hallux w/ABH nerve Lateral head-lateral cuneiform. EDB-4 tendons (2-4) into the lateral side of the EDL. Lateral sesmoid in common with and tibialis posterior the oblique head of the ADH Adductor hallucis Oblique head-base of the 2-4 Lateral sesmoid. aids the interossei in the flexion at the MPJ and extension at the IPJ “Wave bye-bye” 3rd layer-FHB. deep peroneal nerve Action Extends the medial 4 toes a t the MPJs and IPJs and abducts There are 4 plantar intrinsic muscle layers of the foot from plantar to dorsal 1ST layer-most plantar layer: FDB. Abductor digiti quinti Muscle Origin Insertion Flexor digitorum medial process of 4 pt tendon for the lateral 4 toes. transverse and oblique head of the adductor hallucis. (DAB) 3 plantar interossei (PAD) Muscle 1st dorsal interossei 2nd dorsal interossei 3rd dorsal interossei 4TH dorsal interossei 1st plantar interossei 2nd plantar ineterossei 3rd plantar interssei Origin Adjacent sides of the 1st/2nd mets (bipennate) 2nd/3rd (bipennate) 3rd/4th (bipennate) 4th/5th (bipennate) Medial side of the 3rd met (unipennate) Medial side of the 4th met (unipennate) Medial side of the 5rd met (unipennate) Insertion Above the deep transverse intermetarsal ligament.4th layer-4 dorsal interossei. the foot begins to pronate to midsupination. Neurological system 4. plays Creeping Walking Developmental history and pertinent milestones 4 months 3-5 months 6 months 7-9 months 9-16 months • • • Femur development: frontal and transverse planes Frontal plane: angle of femoral inclination Infants 135-155 Abnormality Slipped femoral epiphysis Adult (18 months) 120-135 Coxa valga >130 at adulthoodà bow legs with compensatory genu varum Coxa varum <120 at adulthood à knock knees with compensatory genu valgum Common during childbirth. TX: surgical Transverse plane (bony) angle of femoral declination “antetorsion” Infants Adult 30 degrees internal (8-10) degrees internal via external rotation Antetorsion Failure to externally rotate. slight met varus persists therefore a normal MA value at birth is 22-25 degrees. by the end of the 3rd month. aisst the lumbricals with flexion antagonist to the long extensors Lateral plantar nerve Plantar metatarsal arteries Dorsiflexion and ADDuction of the 3rd5th toes. Congenital and developmental Normal development Embryology in biomechanics 4 wks post ovulation Limb buds beginà the plantar surface of the foot is initially cephalid and undergoes internal rotation where it faces the midsagital plane at 6 weeks 7 weeks-primitive shape of the LE is present (1st ossification center in the femur and tibia) 3rd month the foot is in 90 degree equinus with marked adduction and supination. M>F (10-16yrs) where the femoral epiphysis slips inferior and posterior. by the mid month the foot deigns to DF and supinate with the 1st met adducted. Reaches and recognizes Crawls Rolls. the femur is still internally rotated leading to squinting patellaàinternal knees and intoed gait Retrotorsion Externally rotates beyond the normalà frog-eyed patella Reduction pattern Rapid : Adult value in 18 months Gradual : Adult value in 14 yrs Spurt: correlates in growth spurts Transverse plane (soft tissue) “version Infants Adult (18 months) 60 degrees external 12 degrees external via internal rotation Anteversion Too much internal rotationà external knee position Retroversion Overly internally rotatedà internally rotated knee and intoed gait 112 . assist in flexing the PIPJ sans extending the DIPJs 3. medial side or the proximal phalanx of the 2nd toe Lateral side of the proximal phalanx of the 2nd toe Lateral side of the proximal phalanx of the 3rd toe Lateral side of the proximal phalanx of the 4nd toe Medial side of the bases of the proximal phalanx of the 3rd toe Medial side of the bases of the proximal phalanx of the 4th toe Medial side of the bases of the proximal phalanx of the 5th toe Nerve Lateral plantar nerve Deep peroneal Lateral plantar nerve Blood supply Dorsal metatarsal a Action Dorsiflexion and Abduction. secondary to an increased rate of growth of the medial epiphysis of the tibia. Internal genicular position (pseudoloack of malleolar torsion) Age 3 Age 6 (adult) 0 via internal rotation 5° Present when an excess of 20 to 25° transverse plane excursion is present and the internal:external range of motion ratio is 2:1àinternally rotated and intoed gait Frontal Plane Alignment of the knee– Genu Varum and Genu Valgum Birth-2 2 yrs 4 -6yrs 6-8 yrs Girls (12-14) • Genu varum Straight Valgum Straight Secondary valgum From the ages of 4 to 6. • Measurement is taken with the patient in his/her angle and base of gait. alignment should again be straight. Tibial varum/valgum plus subtalar joint neutral position primarily determine the “presentation” of the foot to the ground. At age 6 to 8. Transverse Plane Alignment – Malleolar Position/ External Tibial Torsion child’s age x 3 >6 yrs Infant-1 yrs 2-3 yrs Age 7 (adult) 0 à 11 degrees 17-23 degrees 13-18 degrees via external rotation Failure to externally rotate à intoed gait with the femur in the frontal plane • • This reflects transverse plane alignment of the lower leg. Sagittal Plane Alignment of the knee– Genu Recurvatum • It is important to evaluate the etiology of genu recurvatum – it may be secondary to functional adaption or ligamentous laxityà extension of the knee joint beyond 180° and behind the frontal cardinal plane of the body is significant. Actual tibial torsion is accepted to be malleolar position plus 5°. frontal and sagittal planes Transverse Plane Alignment – “Internal Genicular Position” Infant >15°.) The measure of malleolar position is the clinical method utilized to assess tibial torsion. traumaà occurs almost never • Tibial varum/valgum assesses the relationship between the lower one third of the leg and the ground. Abnormal values can result in deviations from the “normal” angle (approximately 15°) and base of gait (approximately 2 inches when measured between the medial malleoli. • An increase in internal hip joint range of motion or internal knee position is generally significant clinically. TibiA: frontal and transverse planes Frontal Plane – Tibial Varum and Valgum Infants-3 yrs 3yrs + Varum 15 degrees varus <3 degrees of varus Valgum Malunion of the tibial epiphysis.Hip Development: Frontal plant Frontal plane Infants 2-3:1 external to internal ratio Total hip joint ROM 155° Adult (6 yrs) 1:1 ratio 90° Geriatric (>45 yrs) 2:1 along with an increase in the patient’s base of gait >90 degrees Total hip joint ROM is measured with the knee flexed or in full extension. a valgus alignment is normal. A closer look at rotational developmental abnormalities Patellar postion Management Description 113 . Total ROM is 80-120° • Internal rotation> external rotation is NEVER normalà femoral anteversion is most common cause Knee Development: Transverse. The patient should be placed in neutral calcaneal stance position to eliminate the influence of subtalar joint position. 5 ray ROM 35 degrees from sagital. soleus equinus. and functional orthoses Highà out toe gait. the axes of the midtarsal joint become more parallel to each other the more parallel the axes. Functional orthoses Causes outoed gait where ext>internal hip ROM Not enough external rotation of the tibia. By 14 yrs. casting.” <10à equinus. DF with abduction PF with adduction of the 5th ray th triplanar axis (supinatory/pronatory) Motionà 1:1 frontal:transverse Total ROM • triplanar axis (supinatory/pronatory) Motionà sagital < frontal plane 114 . AJ ROMàdifferentiates between a gastroc equinus. gait plates and twister cables Ankle Joint Range of Motion • • Infant >50° Axis Motion • • The anatomy of the triceps surae enables this muscle to influence motion at the level of both knee and ankle joints. transverse and sagittal planes compensating in the same accommodating GRF by pronating and supinating opposite to that of direction of the STJ the STJà total ROM 4-6° 57° from the sagittal plane 9° from the sagittal plane 52° from the transverse plane 15° from the transverse plane • when the subtalar joint is in a pronated position. triplanar axis average axis position to the sagittal plane 16° (supinatory/pronatory) average axis position to the transverse plane 42° Motion Frontal:Transverse plane 1:1 (High) Transverse plane ROM ß increases if STJ has a more vertical axis (more compensation of ad/duuction) (Low) Frontal plane ROM ß increases if STJ has a more horizontal axis (more compensation of inv/eversion) Total ROM Child: 30-45° Adults: 30° in frontal plane. inversion: eversion 2:1 The “normal” range of subtalar joint motion in the adult is approximately 30° and ROM > 10° may suggest a coalition of the tarsal or midtarsal bones. Lowà intoe gait. functional orthoses. bars. Modify the sitting position of the the femur Causes à intoed gait where child. bars and splints. evaluated w/ positive silfverskiold test which elicits more DF with the knee flexed indicating gastrosoleal equinus Subtalar Joint axis and Range of Motion STJ ROM assesses the relationship of the RF to the lower 1/3 of the legà measurement in terms of how the foot is presented to the ground and how the foot will be able to accommodate for any osseous or functional abnormalities. 20 degrees from the transverse plane Dorsiflexion with eversion with promation and plantarflexion with inversion supination CKC. particularly of the oblique midtarsal joint First Ray Range of Motion 45o from the frontal and transverse plane Dorsiflexion with inversion. Plantarflexion with eversion 65-70 degrees of dorsiflexion with 45 degrees of plantarflexion at the MPJ 1 cm total ROMà 5 mm of DF and 5 mm of PF Deviation from a 1:1 dorsiflectory to plantarflectory ratio suggests either osseous deformity (metatarsus primus elevatus or plantarflexed first ray) or soft tissue adaptation (forefoot supinatus). gait plates and twister interal >external hip ROM cables are necessary if fx problem Too much external rotation of the femur Externally rotated Outgrown.Femoral antetorsion Femoral retroversion Pseudolack of malleolar torsion Malleolar position Not enough normal external rotation of Internally rotated Outgrown. serial gastrox and hamstringsà intoed gait casting. the greater the available range of motion. it is angulated 20-30° from the transverse axis of the knee provides the majority of its motion primarily in the sagittal plane A simple guideline to remember is that AJ dorsiflexion “should be 15° by the age of 10 and 10° by the age of 15. gastrosoleus equinus and osseous equinus Age 10 Age 15 (adult) 15° 10° average axis position of the ankle is 83° from the frontal plane. 2o to tight Midline Manipulation via external rotation. the majority of its motion occurs in the • the majority of motion in the LMTJ occurs in the frontal plane. fx medial development of talar head/neck orthoses. Forefoot to Rearfoot Relationship – Midtarsal Joint axes and ROM oblique midtarsal joint longitudinal midtarsal • this joint is comprised of the C-C joint and affects the lateral • this is a functional joint is comprised of the TN articulation affecting the column of the foot medial column of the foot • although it is triplanar. 2o to persistent Midline Not outgrown. Formulaà STJN + Tibial position Measurement of Resting Calcaneal Stance Position • Resting calcaneal stance position is the summation of leg presentation to the ground and the compensation of forefoot to rearfoot relationships (be it compensation at the subtalar or midtarsal joint. the TN joint is dislocated in a supinatory position and the navicular is plantarly dislocated on the talusà EQUINUS + RIGID SUPINATION Causes are idiopathic from intrauterine position or intrinsic (mc) and extrinsic: acquired via neurogenicà spina bifida. so to calculate RCSP in childrenà 7 . Foot completely relaxed with loose skin folds below lateral maleolus & medial skin stretched. anterior & medial facets fused and Navicular Normal shape Hypertrophic tuberosity Plantar medial subluxation May articulate w/ medial Soft Tissue All on MEDIAL side contracted: deltoid ligament. Reassure parents. Autosominal dominant trait w/ 40% penetrance. angle may approach 90 with VT deformity Talocalcaneal 30-50à decreases 20-40 by age 4 Bisection of talar neck to calcaneal bisection. CP. Bone deformity – 1o . It is generally accepted that a limb length of less than ½ inch is not treated. CIA normal. decreases with supination. and forefoot is adducted. Is negative in clubfoot deformity Congenital disorders Def & Symptoms Etiology Xrays DDx Tx CalcaneoValgus Identified at birth.talar neck & head . midfoot & MPJ • • • • • Pathologic Anatomy Talus Smaller head & neck angulated medial plantar. meningitis. 2o –metatarsus adductus Joint deformity – 1o – Talonavicular.) It is measured by evaluating the relationship of the calcaneal bisection to the ground when the patient is standing in their angle and base of gait. interosseus and bifurcate ligamentsà must be released comprehensively 115 . Relaxed TAà NO EQUINUS! Intrauterine position. 2o – STJ. more common in young mothers 1st baby – tight uterus. the hindfoot is in varus. Calcaneus Normal shaped. Dorsum of foot in contact with anterolateral leg. assists in assessment of HAV adductus angle Calcaneal 10-15à arch increases 15-25 by age 6 Angle formed by comparing the proximal and distal aspects of the calcaneal inclination angle inferior surface with WB surface. post-polio or post-traumatic. the foot is flat and everted. Unlike VT. CF. anterior lateral deviation in the ankle mortise. Increases with supination and decreases with pronation and ples planus Talar declination 15-25 à decreases 10-15 Compares bisection of talar neck to the WB surface. That is two-third inversion to one-third eversion.Calculated Subtalar Joint Neutral Position • Subtalar joint neutral position is historically calculated on the belief that the subtalar joint possesses twice as much inversion as it does eversion. Neuromuscular Dz. laterally-posteriorly: PTF. • Limb length is measured from the anterior supierior iliac spine to the tip of the medial malleolus. 33% in identical twins It is a triplanar deformity where the ankle is in equinus. • From 0-5 yrs. Casting Club foot (Talipes Equinovarus) 1/1000 live births w/2:1 M:F and 50% b/l. Formula à Subtalar joint neutral position = Eversion – (Inversion + Eversion / 3) Calculated Neutral Calcaneal Stance Position • Neutral calcaneal stance position reflects the actual degree of neutral inverted or everted presentation of the lower leg to the ground. ankle. under developed sustentaculum tali. spring ligament. MD. Normal angular development Infants (degrees) Adults (degrees Metatarsus 30 (20 at ambulation) 5-15 by age 7 Used to asses adduction of the FF to theh MF. None necessary. Increases with pronation. increases with pronation and (kites) decreases with supination. 1:1. Calcaneus DF Vertical Talus. ATF. Theory – retracing fibrosis during 10th -12th IU week.age Formula in adults à Availabe eversion + STJN Measured Limb Length from ASIS to Medial Malleolus • Limb length discrepancies can influence foot function and compensatory mechanisms.000 F>M. marked DF (calcaneus) at the ankle and eversion of the foot. spina bifida.Rocker bottom. lateral and plantar). BK cast changed wkly – 6mths Lateral Column Ogston 1902 cuboid closing wedge Shortening Evans 1961 Calceneocuboid joint closing wedge & arthrodesis Lichtblau – Anterior calcaneus closing wedge Medial column Fowler 1959 – medial cuneiform opening wedge osteotomy with interposition of bone graft lengthening Ganley – closing abductory cuboid osteotomy Salvage Triple arthrodesis > 12yrs. varus 2nd. 25-50% have repeat Sx for residual deformity Soft Tissue Metatarsus Adductus Description: Transverse plane deformity (C-shaped adduction) in which the mets medially deviates w/ the apex of the deformity at the LisFranc articulation. lateral skin thin & stretched. tight or spastic ABH. tight ABH. Rigid-not manually reducible. bifurcate lig. pes valgus-overcorrection. Ideal age – 3 -12mths & b4 3yrs. Peroneals – elongated lateral side Radiographic evaluation: Normal Clubfoot Kites angle (TCA) 20-40 0-15 (decreased or parallel) CIA 20-25 ~17 Talar head/neck relative Adduction 10-20 80-90ß Transverse plane to the body Plantarflextion 25-30 45-65ß Sagittal plane Simon’s rule of 15 Predicts the presence of TN subluxation using AP x-ray TCA <15 Talo-1st met angle >15 Symptoms –club like foot. forefoot. Cigar sign àwide 1st interspace.TDA – decreased 115 – 135o Talar neck obliquity – 50-60o angulated plantar medial beneath talus malleolus and circumferentially (posterior. FDL. shoes. modify sleeping position. navicular. serial casting before the age of 2 Casting à order: midfoot. Rocker-bottom foot. medial. rearfoot. medial skin creases & furrows. Fixated w K-wire to STJ & TNJ 4-6wks. orthotics. Dynamic-only present upon WB d/t tight ABH Clinical features: adducted ff. sitting/sleeping position Classification: Flexible-deformity is reducible w/manual manipulation <6mo. equinus 3rd (AVE) & AK cast q 1-2wks >> night splints. prominent. splints Surgical procedures are age dependent: 3-8yrs-soft tissue. Turco procedure – posteromedial plantar soft tissue release with internal fixation via hockey stick incision posterior >medial>plantar Lengthen – TP. convex lateral border and concave medially. Calcaneofibular lig. >8yrs-osseous Soft tissue procedure (3-8 yrs) 116 . Commonly associated with tibial or femoral torsion. casting only sometimes effective>2yrs. RF varus Clinical diagnoses: Lichtblau’s testà The ABH is palpated just proximal to the met head. orthotics. small drawn up heel. absent or hypoplastc medial cuneiform. residual deformity. Wheaton Brace. abnormal insertion of AT. adducted foot. Met adductus. Plantar fascia. talus prominent & fibular posterior. arthrogryphosis multiplex congentia XR evaluation: Normal degrees Met Adductus MA Birth (15-35) 1 (20) 4 yrs (15) >21 is considered pathologic Engle’s angle 24 >24o Simon’s angle Talus-1st met bisector 0-20 negative TCA 20-25 increases Cuboid sign Normal cuboid position Medially displaced cuboid Treatment Conservative – passive stretching. Osseous Indication – rigid deformity in children & adults only. FF adducted. prominent styloid process. stretching Complications . 10:1 met adductus to club foot Cause: IUP. wedge shaped naviular Surgical Comprehensive circumferential release – posterior >medial> lateral> plantar via cincinatti incision. equinus and maintained with a Ganley splint. tightness or spasticity can be detected Combined deformities: tibial torsion. PT. ACH. Talectomy – 4-8yrs old Complications Wound dehiscencs. Treatment Conservative Ponsetti method -Manipulation 10 – 15 minutes. Interosseous ligament. FHL. club foot and a windswept deformity (met adductus on 1 foot and calcaneovalgus on the other) Incidence: 1/1000 F>M 4:3 or 3rd child. TN joint & Posterior ankle joint capsulotomy. navicular wedging and subluxation. flat top talus. decreased foot size. flat top talus or talar AVN. manipulation. Deltoids. Spring ligament. overcorrection. calf atrophy. abduct FF. congenital hip dysplasia. evert & distract heel >> Serial casting for FF adductus 1st. Heyman Herndon and Strong 1958 Thompson 1960 Lichtblau 1975 Johnson Lange #1 LaPorta & Sokoloff Peabody & Muro1933 McCormick & Blount 1949 Steytler & Van Der Walt 1966 Berman & Gartland 1971 mc Lepird 1982 Fowler Lange #2 Brink & Levtsky 3 dorsal incisionsà capsulotomies and ligament releases of all met joints at the Lis Franc joint. Bridge – coalition b/t 2 bones that normally share articular surface – intraarticular (TC. 1st & 5 mets fixated w/ steinmen pins. tenotomy and medial capsulotomy. increased difficulty with standard shoe gear. leave plantar structures intact to prevent dorsal dislocation Resection of ABH.TC. Release of ABH tendon with opening wedge osteotomy of the MC with bone graft Osteotmies of 2-4 mets Cuneioform and cuboid wedge osteotomies w/ K-wire fixation 1st met base at least 6mm from physis Untreated Met adductus→ Hammertoes. Met adductus – modified Berman-Gartland or Lepird. Sx Tx Multiple corrections. cuboidnavicular. infection Ca.or congenital – accessory ossicles or genetic Associated abnormalities: synphalangism. division of the ABH tendon. Incidence: M>F 4:1. In order of most common. Equinus – Gastroc recession or TAL.positive Kite’s angle – decreased Talonavicular relationship – navicular medial Met Adductus vs Club Foot (TEV) Metatarsus Adductus FF aDDucted Lateral navicular subluxation RF neutral or valgus NO equinus Radiographic Simon’s angle . painful compensatory symptoms. Cast for 4-6 weeks Modified Thompson. Evans lengthens the lateral column and realigns the midtarsal joint. Types: synostosis. navicularcuneiform Ø TN 3-5 ossification Ø CN bar 6-12 ossification reset bar with interposition of EDB Ø TC bridge 12-16 years ossification fuse a bridge * mc 90%* Bar – coalition b/t 2bones that don’t normally articulate – extraarticular (CN). untreated met adductus compensated w/ excessive STJ pronantion. digits abducted in stance. Midfoot abducts and RF evertsà Z-shaped foot Skew Foot Aka Z foot. S & S mets angled medially. metatarsal. Pes planovalgus – Evans opening calcaneal osteotomy & medial arch tenosuspension. congenital met adductus w/ calcaneovalgus. syndesmosis.positive Kite’s angle – normal or increased Talonavicular relationship – normal or navicular lateral Clinical Tarsal Coalition Description: Bridge that causes restriction or absence of motion between 2 or more tarsal bones. secondary deformities emerging. large space b/t hallux & 2nd toe. TN. Oblique wedges removed from 1-5 mets where the osteotomies are performed at proximal 1/3 of 2-4 mets in an oblique fashion (distal dorsal to proximal plantar) parallel to WB surface. deformity increasing despite conservative Tx. sacroiliac fusions Classification: Tachdjian’s-descriptive classification that suggests the importance of assessing other areas of the foot and the remainder of the body. talar bulging (ptosis) on wt bearing w/ low medial archabducted midfoot w/ internal rotationof malleoli. CN. CC) 117 . vertebral. Pes planus Skew foot: Adducted FF results in severe STJ pronation. Etiology improper manipulation and serial casting for met adductus >> RF pronated. RF equinus Xrays Increased MA angle > 21o Increased Cuboid abduction (calcaneocuboid) angle > 5o Indications for Sx – failed conservative tx or too old. Metatarsus adductus FF. Closing base wedge osteotomy of 1st met base + Resection of cartilages chondrotomies at met bases 2-5 Capsulotomy of the 1st MC joint. can produce dramatic symptom complex ultimately resulting in rigid peroneal spastic flatfoot. prominent base of 5th met. CC. 50% b/l Etiology: Acquired –Trauma. HAV. Serpitine. STJ instability – STJ arthrodesis Club Foot FF ADDucted Navicular subluxed medially RF varus Equinus Simon’s angle . synchondrosis. TN. arthritis. correction by casting Tibialis Posterior lengthening + TN capsulotomy Osseous procedures (>8yrs) Ø Excision of the bases of the 3 central mets (2-4) with closing wedge osteotomy of the 5th Ø Mobilization and luxation of the 1st MC jt with correction of abnormal TA insertion Arthrodesis of the 1st MC joint. tenotomy or partial resection of ABH. normal midfoot w/ pathological RF valgus. compensated met adductus. with Osteotomies of 2-4 and wedge resection of the cuboid Base wedge V-shaped oblique osteotomies of all the mets. apex angled at RF Dome-shaped osteotomies through 1-5 met bases. developmental.Transfer PT into navicular Osseous Resect head and neck of the talus Partial resection of the navicular Fibular arthroeresis • • • • • • Total talectomy Total naviculectomy Description Brachymetatarsia Short. Triceps surae. . superior peroneal retinaculum attenuated >> subluxation. Persian Slipper. PL. Ball and socket ankle adaptive changes. Peroneus Tertius Dislocated TP. With significant arthritis >>>isolated TC or triple arthrodesis.Clincal Findings-insidious onset of pain developing after trauma or unusual activity.5 mm) & interpose EDB is Juvenile CN Tx of choice = CN resection arthroplasty (Badgely Procedure). Complication – recurrent bone growth d/t inadequate resection TN asymptomatic – resection then fusion Calcaneocuboid – fusion . elevated 1st met. gait abnormalities and pain X-ray evaluation: Stress lateral XR to check TN dislocation will see on AP XR Kohler-like changes of the navicular. spina bifida. traumatic. Reverse Clubfoot. sinus tarsi syndrome. sulcus anterior to the fibula. CC diastasis. Post –op BK cast – 4wks. PT. Tibia articulates w/ talus only in posterior 1/3 Calcaneus close to distal tip of fibular & is everted & PF Hypoplastic sustentaculum tali. Contractures to TA. Dorsal dislocation of navicular. TN beaking Calcaneal axial XR: fused middle facet/nonparallel relationship of middle/posterior facetsà lose their normal parallel relationship MO XR: Comma sign-protrusion of calcaneus toward navicualar. PB. relieved with rest. Treatment Restriction of STJ & MTJ motion – shoe modifications. Most common cause of Peroneal spastic flat foot d/t P. Transfer TA into talar neck . Infectious 118 . local steroid injections prn Bridge – intra-articular → fusion/arthrodesis. padding. NO anterior talar facet. evident b/t 4-16. orthoses. recurrent ankle sprain. tight ACH. 4th met most common. Remember navicular ossifies at 3-4yrs so will not see on xray. If small or incomplete . Epidemiology – uni or bilateral. longus is major deforming force. hypoplastic metatarsal d/t premature closure of epiphyseal plate. deep dorsolateral creases. PB Clinical presentation: HPK beneath prominent talar head. 25:1 F:M. 4 clinical patterns: Assoc w/arthrogryphosis. Soft tissue P. CC and AJà EQUINUS AND RIGID PRONATION w/ a contracted Achilles & elongated spring ligament Remember club foot = equinus & rigid supination Epi – 50% B/L R>L. EHL. Talonavicular lig thick & blend w/ deltoids. NSAIDS. and hour-glass talus. brevis attempting to restrict painful STJ motion TC CN bar X-ray Evaluation Lateral XR : Halo sign-absence or diminished visualization of the middle facet with enhancement of the sustenculum tali. Rockerbottom Flatfoot Vertical Talus Description: Primary “button-hole” dislocation of the navicular dorsally on talar neck locking the talus in a vertical positionà 1o joint deformity is the TN jt in a pronatory position & 2o jt deformity at STJ. Norm Vertical talus Club foot Kites angle (TCA) 20-40 >40 (increased) 0-15 (decreased or parallel) CIA 20-25 Negative angle ~17 Surgical procedures – staged FF correction then RF Soft tissue lengthen contracted & dislocated tendons capsulotomy of TN. ankle & CC joints TAL. EDL. cubonavicular – resection Conservative Surgical Vertical Talus § AKA Congenital Convex pes plano valgus. pain aggravated by activity. hallow anterior to lateral malleolus. CN Bar – excise bar (1-1. M=F 50% w/ isolated deformity. Etiology – congenital. iatrogenic. limitation of ROM with spasm. neurofibromatosis & an isolated defect (50%) Pathological Anatomy Osseous Wedge shaped navicular w/ hypoplastic plantar segment PF ankle. FF abducted & pronated.arthroplasty TC Middle facet arthroplasty – resect 4-7 mm of bone. casting. TP tendon attenuated under talar head >> subluxed anteriorlly. flexion contracture of the 1st MPJ. “peg-leg” gait. and the entire foot remains rigid in relationship to the leg. anteater nose sign:anterior facet àtoward navicular on Lat XR Mc osseous coalition in foot = 5th middle & proximal phalanx. Associated Conditions Classification S&S Treatment Down’s Syndrome, Turners, Sickle cell Anemia, Dystrophic dwarfism, Albright’s , pseudohypothyroidism, pseudohypoparathyroidism, poliomyelitis, myositis ossificans I – shortening of 1st met only; Type II shortening of 1-2 lesser mets, Type III shortening of 1st & 1 or more of lesser mets, Type IV shortening of all mets short contracted floating toe, short met, metatarsalgia on adjacent metatarsals, plantar calluses under adjacent met heads, deep sulcus underneath short metatarsal; Xrays: Short underdeveloped metatarsal, osteoporosis of the metatarsal head Conservative – orthotics or accomadative devices, padding Surgical – soft tissue: V-Y skin plasty- relieve tension of contracted skin. Z-plasty lengthening of long extensor tendon, tenotomy of short extensor tendon, MPJ capsulotomy (dorsal,med/lat release) sectioning of deep transverse intermetatarsal ligament Tx Surgical Osseous – osteotomy lengthening frontal plane Z osteotomy; insertion of cortico-cancellous bone graft >>>NWB cast; Callus distraction – 1mm/day, delay 5-7days Complications – lengthening too aggressive >> neurovascular compromise, Lesser MPJ limitus, delayed union / nonunion, resorption/collapse of bone graft, painful pseudoarthrosis, painful limitation of joint motion Other congenital deformities Description/Treatment Classification Macrodactyly Digital gigantism localized NF, fibrolipomatosis of all tissues 1. Macrodactly simplex congenita except blood vessels and tendons. Usually unilateral. Tx: 2. Macrodystrophia lipomatosis progressive epiphysiodesis to arrest growth and de-bulking procedures Syndactyly Congenital webbing of the toes does not interfere with function 1. Davis and Germanà a. Incomplete b Complete c. and is not objectional cosmetically. No tx is needed Simple d. Complicated Polydacyly Supernumary digits. 1. Temtamy and McKusick (post-axial [lateral digits] or pre-axial [hallux]) Autosomal dominant 2. Venn and Watson preaxial polydactyl (wide or short 1st met) à Post-axial polydactyly blacks>whites and F>M. Tx: a. Wide metatarsal b. T-shaped met c. Y-shaped met excision, optimum age for d. Partial polydactyly e. complete duplication surgery between 9-12 mo 3. Central Fibular hemomelia Congenital longitudinal deficiency 1. Achterman and Kalamanchi of the fibulaß interference w/limb 2. Stanitski and Stanistski bud development. Associated a. Fibular morphology (I nearly normal, II small, III absent) b. Tibiotalar joint amd w/tarsal coalitions and absent foot distal tibial epiphyseal morphology (H horizontal, V valgus, S spherical) c. rays Presence of tarsal coalition ( c) d. Number of foot rays, medial to lateral (1-5) Ectrodactyly Autosomal dominant. Assoc w/syndactyly, cleft palate, polydactyly and deafness. Tx: reconstructive surgery is required Congential digital Skin is shortened dorsally; the digit is contracted on the long axis w/nail pointing laterally àb/c hard w/callus. Assoc w/ minimus varus flexor stabilization. Tx: PIPJ arthroplasty and derotational skin arthroplastyà incision is proximal lateral to distal medial Clinodactyly Congenital curly toe-may be d/t to differential epiphyseal growth or overcrowding from adjacent digits. Tx: splints, taping, sx as w/overlapping 5th toe Ainhum Dactylosis spontanea, fibrous bands begins on medial side of the toe (5th esp) encircles and autoamputates Milroy’s dz Hereditary edema of the legs, inherited Achondroplasia Autosomal dominant-defective enchondral bone formation. Associated with ligamentous laxity and trident hand deformity Osyeogenesis imperfecta Inherited disorder of defective collagen maturationà fx in various stages of healing Osteochondroses True AVN • Blount a lateral slippage of the proximal Tibial epiphysis • Freiberg Met heads • Renandier’s dz Tibial sesmoid • Trever’s dz Fibular sesmoid • Diaz or Mouchet Talar body • Legg-Calve-perthes Femoral head • Kohler Navicular Not true AVN • Islen 5th met base • Bushke Cuneiforms • Theiman Phalanges • Lance Cuboid • Sever Calcaneal apophysis • Osgood-Schlatter Tibial tuberosity • Assman’s head of 1st metatarsal 5. Kinesiology and gait analysis 119 B. Trauma 1. Sprains, strains and soft tissue injuries Lateral Ankle Sprains Definition: stretching injury which may involve partial or complete tearing of the lateral ankle ligaments Anatomy 3; Intracapsular – anterior & posterior talofibular ATF/PTF ligaments; extracapular – calcaneofibular CF ligament à Shape of the talar dome: wider anterioly than posteriorly which results in decreased frontal plane stability of the ankle when plantarflexed and increased w/dorsiflexion. (CAM puts ankle in DF) Recall that the ankle is almost parallel to the frontal plane so the STJ provides the inversion that leads to this injury. ATF and CFL make an angle of 70-140 degrees. Mechanism of injury – repetitive inversion injury to ankle Associated injuries: • Ankle fractures • syndesmotic “high” ankle injuryà if suspected tx with NWB cast for 4-6 weeks • Shephard’s fx –of os trigonum Tx: NWB 6-8wks • talar dome fxà evaluate with a talar dome compression test and a AP ankle in max PF • calcaneal anterior process fx via avulsion of bifurcate ligament Tx: NWB 6-8 weeks • 5th met base avulsion fx via avulsion of lateral slip of plantar fascia and to a lesser extent the PB, TX: NWB 6-8wks • sinus tarsi syndrome • Damage to the lateral malleolar artery, CPN à drop foot and acute compartment syndrome Risk factors: • Biomechanical: ankle equinus, gastrox or soleal weakness, FF/RF deformity and transverse plane abnormality • History of previous ankle injury is the #1 risk factor Incidence: 40% of all sports injuries (most common injury in all of sports) Physical examination: 1. evaluate anterior-inferior syndesmosis A. Squeeze test B. Tibio-talar shuck test C. External rotation test 2. Anterior and drawer test (Push-pull test) - evaluates ATF, >2mm anterior displacement as compared to the contralateral side indicates rupture. A positive testà “dimple sign” The ATF is on stretch when the ankle is PFà 1st to be ruptured 3. Posterior drawer test - evaluates PTF clinically. 4. Anterior and posterior ankle palpation 5. Muscle testing especially peroneals and flexorsà possible rupture or tear of these tendons 6. DF and PF of MTPJà shepherd’s fracture or flexor tendon injury X-ray examination: Ottawa ankle rules-who gets an x-rayà someone who canot bear wt or w pain over syndesmosis or posterior malleolus (AP/MP/Lat ankle and MO foot) 1. Stress inversion to evaluate the CFL A. Radiographic grading: 1. Grade 1: + push-pull test and +/- inversion stressà not possible to have a +inversion stress and a – push-pull test because the ATF will rupture b/f the CFL 2. Grade 2: +push-pull and + inversion stress (10 degrees) 3. Grade 3: +push-pull and + inversion stress (15 degrees) 4. Grade 4: +push-pull and + inversion stress (>15 degrees) 2. Talar tilt >5 degrees as compared to contralateral ankle: >5 ATF rupture, >15-30 ATF and CF rupture and >30 ATF, CF and PTF rupture 3. + Anterior drawer (push-pull)à >10mm of anterior displacement or >4mm compared with the contralateral side 4. MO footà checks for C-C crush, fx of anterior process of calcaneus, and fx of 5th met base Classifications 1. Leech: Anatomical Classificationà Grade 1: ATFL sprain, Grade 2: ATFL and CFL sprain, Grade 3: ATFL, CFL and PTFL sprain 2. O’Donoghue Classificationà Grade 1: Partial tear of the ATF, Grade 2: Complete tear of the ATF, and partial tear of the CF, Grade 3: Complete tear of ATF and CF Grade 4: PTF injury 3. Dias - Grade I - partial rupture of CF, Grade II - ATF rupture, Grade III - ATF, PTF and CF rupture Management: Casting vs functionalàCAM (1: allows for early WB 2: ankle DF to reapproximate ATF), begin rehabilitation ASAP vs surgical • Jones compression for 0-5 days, consider bone scan, MRI and CT to r/o other possible injuries • Syndesmotic injury: wb immobilization 4-6weeksà limit pd of NWB Treatment: Acute sprain àProtection, Rest, Ice, Compression, Elevation and NSAIDS (PRICEN) Rehabilitation (4-8 weeks) Phase I: Acute phaseà contrast bathing, interferential stimulation, Jobst compression, CPM Phase II: Rehabilitation phaseàactive ROM, peroneal muscle strengthening, stretching, massage, jt ROM, u/s 120 Phase III: Functional phaseà (Proprioceptive excercises) BAPS, plyometrics, toe raises- balancing exercises Phase IV: Return to activity phaseà sport specific mvmt, preventive strategies, and equipment modifications When should a pt return to activity? • Minimal to no edema, no limp, be able to perform 1 legged toe raise, ability to descend stairs, 1 legged lateral hop test, ankle dorsiflexion test • Ankle braces >>tape: both reduce the incidence of ankle inversion sprains by increasing peroneal reactivity and strength through direct contact with the skin (proprioception). Atheletes should be braced for up to 6 months following an injury. Tapes last 10min (40% reduction) Contraindicated in pts with high tibial varum, may increase lateral instability Chronic lateral ankle instability Definition: #1 complication of inversion ankle sprains Functional vs mechanical (anatomic lateral ankle instability) Functionalà cannot be adequately evaluated via radiography or manipulation • Disabling loss of reliable static/ dynamic support of a jt, deficit in peripheral reflexes, proprioceptive de-afferentiation, proprioceptive nerve ending located primarily in the skin, and pain results increasing stability b/c of the anesthetic blockade of the STJ and AJ. • Loss of: mobility, strength, position sense, stability in single leg stance, detection of passive mvmt, functional ability and reflex speed • 60% of functionally unstable ankles do not show pathologic talar tilt Treatment: • Conservative: more rehab, a better brace, MRI/CT/Bone scans to r/o additional injury or prioltherapy (thermal shrinking of the lateral ligs with a wand and prolotherapy (injectable sugar solution to enhance proprioception • Surgical Procedures ßdo a stress x-ray as a prelude to surgery Delayed primary Brostrum -repair of lateral ligaments/ retinaculum by overlapping (imbrication) & suturing via 00-2-0 ethibond sutures ligamentous repair Gould modification: use of the anterior inferior extensor retinaculum, reinforce over fibula, repair of CFL is unecessary Post-op: NWB 2wks, WB 2wks in CAM and PT for 1 month. Yields better results than stabilization procedures Lateral stabilization procedures Disruptive to normal anatomy, technically difficult, results in overtightening of AJ-Use in pts w/ligamentous laxity or with collagen vascular diseases Repair ATF by passing tendons through Watson-Jones P. brevis sutured to P. longus fibula, talus & calcaneus following Lee – P. brevis to PL anatomic orientation of the ATF Suppan – P brevis Repair ATF & CF Elmslie – fascia Lata Split PB ankle stabilization procedure – Peroneus brevis Christmas and Snook – split P. brevis Repair ATF, CF & PTF Hambly,k Sammarco & DiraimondoRepair without following specific anatomic Nilsonne – P. brevis orientation Keilikian Evans – P. brevis Other ankle conditions 1. Talar dome fractures Description Mechanism of injury Compression fracture of the talar dome via the tibial plafond, most commonly misdiagnosed following an inversion ankle sprain DIAL-A-PIMP AnteroLateral lesions occur with dorsiflexion/inversion injuries - wafer shaped 44%, mortise view; PosteroMedial lesions occur with plantarflexion-inversion injuries - deep cup shaped 56% AP view Classification: Berndt-Hardy ''Osteochondritis dissecans'' is a chronic condition that leads to loose bodies d/t spontaneous necrosis w/o dz, trauma or tumor. • Stage I - subchondral bone compression fx; not seen on radiographs; painless lesion • Stage II - partially avulsed, detached osteochondral fragment; rupture of the lateral collateral ligaments of the ankle Tx – RICE, 4 wks NWB BK cast immobilization or 2-4 wks WB cast • Stage III - completely detached, nondisplaced transchondral fragment Stages III - surgical excision of small fragment, curettage & drill-hole fenestration to subchondral bone to aid revascularization & stimulate fibrocartilage production; 4wks NWB w/CPM machine and then 4wks WB cast • Stage IV - completely detached and displaced transchondral fragment from the talus; Larger fragments - If >2cm fragmentà ORIF via malleolar osteotomy or arthroscopyà fragment excision, saucerize the crater, drill-hole (1.5mm) fenestration of the subchondral bone to enhance fibrocartilage production with early NWB ROM & resumption of WB 2-3 weeks postop. Posterior medial lesions may require malleolar osteotomies for visualization. • Stage V-subchondral cycst is present OATs MosiacPlasty – chondroplasty can be used with large defects from the trochlear talar surface or non-articaular surface of the knee. The size of the graft is usually 5-9mm. This procedure is ideal for focal defects and for young patients. 121 Tx – close by 2o intension using STSG. Medial Ankle Instability Rare. os tibialis externum • Tarsal tunnel syndrome Flexor tendonitis Deltoid ligament injury 4. Fractures and dislocations Nail Trauma Subungual Hematoma “runner’s toe” • Acute tx: PRICEN. on PE pt has weak/painful eversion and pain w/forced inversion or a LLD Tx: RICE. m. general treatment of cleaning. tibiocalcaneal. rupture. Anatomy 5. extracapsular – tibionavicular. • Laceration to nail bed à examine. 5. jt mobilization. acts as tendon graft Schoolfield detached damaged deltoid. invert foot & re-suture into periosteum superior to detachment site. advances the deltoids 2. Anterior ankle impingent exostosis “footballer’s ankle” Descriprion Insidious onset anterior ankle pain with deep anterior ankle palpation and forced DF. debridement and packing Ø Zone I distal to bony phalanx. CFL with bone block Grade II Fibrocartilagenous ridge and retinacular damage Groove deepening procedures Grade III Retinacular damage and Eckert fracture Sx correction is required for all 4 grades d/t ineffectiveness of closed tx. pre-achilles bursitis. V to Y Advancement Ø Zone III proximal to lunula Tx – not amenable to reconstruction 122 . u/s. and a digital aperture may be necessary for athelete to resume activity • If involves >25% or more of nail plate >> remove nail plate and inspect nail bed. dislocation/subluxation Tendonitis Consider with a history of lateral ankle sprains.repair of the lax deltoid ligament Wittberger and Mallory . Tx requires surgery. Eversion ankle sprina/medial ankle pain (5-10% of all ankle injuries) • Medial malleolar contusion/fractureà WB for 4-6wks Medial malleolar stress fracture (insidious) • Posterior tibialis tendonitis Spring ligament injury navicular tuberosity avulsion fx.c. FHL injuryà “dancer’s tendonitis” PE Pain with releve. retrocalcaneal bursitis. Peroneal syndromes: tendonitis. in skiing. surgical debridement 7. tetanus prophylaxis and antibiotic therapy Classifications: Rosenthal . shepard’s fx (medially). repair and suture under LA – use absorbable sutures . massage. Reverse dermal graft Ø Zone II distal to lunula. Prognosis – post traumatic OA 2. Tx – Local pedicle graft .Talar Tilt >10o d/t rupture of TC & superficial TT ligaments Surgical procedures: Duvries . dexamethasone iontophoresis. Dx – stress eversion mortise Xray . 6. injectable phosphate corticosteroid into sheath. Tx requires surgical intervention Peroneal Tendon Dislocation Eckert & Davis Classification Grade Description Treatment Grade I superior peroneal retinaculum is damaged only Primary repair of retinaculum 6 wks short leg cast Fibular rotational osteotomy. arthroscopic debridement 6. the fracture is considered an open fracture • The basic tx for an open distal phalangeal fx is local wound care. Tx is similar to tendonitis however sx debridement is required Dislocation Secondary to rapid ankle DF. Subluxation only significant if painful. intracapsular – deep anterior and posterior tibiotalar ligaments Mechanism of injury .split PT (leave insertion intact) passed through drill hole in medial malleolus & sutured down on itself.5. FTSG. Talar lateral process fracture “Snowboarder’s ankle” Tx: non-displaced: 6-8wks NWB. not common in inversion ankle injury. displaced: ORIF 3. painful Os trigonum. injectable ccs.Autologous Chondrocyte Implantation (ACI) Cultured chondrocytes originally harvested from the pt are re-implanted after 3-4weeks of culturing. superficial tibotalar. Posterior ankle pain Causes Posterior impingment. NWB for 4-6 weeks and PT.goal . hx of chronic tendonitis. tenosynovitis. radiographic spurringß do a stress lateral X-ray or a MRI for soft tissue impingment Management Heel lift. sx debridement if necessary Tenosynovitis Often d/t impingement. ankle PF or with forced PF Management Injectable ccs.try to preserve or restore tip of toe. Post-op – immobilization in short leg NWB cast – 3wks > walking cast.forceful eversion. 7-0 • If the nail bed is lacerated. Diagnosis Equinus. peroneus longus spasm. hallux flexed @ IPJ. bone resection. flexor tendon transfer. recurrent dislocation and up to 50% of turf toe injuries result in long term mobility Pre-dislocation syndrome 2nd digit at MPJ -à pain. FHL.cannot be closed reduced. must be excised Ilfeld’s disease. Metatarsal shft fracture ‘dancer’s fracture” à Tx: 4-6 wks WB immobilization 3. Acute dislocation of lesser mets .pain edema. sesamoids subluxed to each side of met transverse avulsion fx of one sesamoids ISL intact head. They are invested in the tendons of the FHB. and relative rest. sports injuries) S&S – hallux dorsally subluxed @ MPJ w/ met head prominent plantarly. phalanx & the sesamoids.5cm proximal from styloid process at metaphyseal-diaphseal junction Ø Poor blood supply and the tendency to to go onto delayed or non-unions Ø True stress fractureà Tx is aggressive due to high reccurence of injury with 6-8 wks NWB immobilization and bone stimulation or until no evidence of fracture is seen on X-ray. ORIF may be indicated in severe diastasis or non-union Classifications 1. Adductor & abductor hallucis Classification – Jahss – “Turf toe” Mechanism . icing. cavus foot type. swelling. ISL remains intact. MPJ collateral lig. casting. They function as a fulcrum for motion at the MPJ. lateral band of plantar facia and peroneus tertius at the met base Vassal Principle – intact soft tissue structures assist in the reduction of fractures Complications of met Fx – compartment syndrome. eccymosis.1st mpj hyperextension (mva. NSAIDS Type IIA Type IIB dorsal dislocation of the proximal dorsal dislocation of the proximal phalanx. laceration puncture. oblique. slipper or BK cast 6wks open reduced 3 . Tx – PRICE.Digital Injury /Fractures • Can be displaced. plantar to the 1st met head in 85% of population. NSAIDS. inter-sesamoidal lig. chronic dislocation seen in RA. strapping.agenesis of one or both of the sesmoids 5th Metatarsal Fractures • • • Anatomy – Peroneous brevis inserts at styloid process. surgical shoe 3wks NWB. PIPJ arthrodesis. surgical shoe 3wks Complication immediate – compromised circulationà AVN. sesamoid not fx Types of 5th metatarsal fractures: 1. extensor apparatus relaxed & flexor apparatus tightened. spiral. Immobilization . tendonitis. Styloid fracture. When chronic: iontophoresis. Neuropraxia Type I dorsal dislocation of the proximal phalanx & the sesamoids on met head. Most hallux fractures require a rigid surgical shoe for 4-6 weeks. subungual hematoma 1st MPJ Injury/Dislocation • • • • • Anatomy – Tib & Fib sesamoids.rare. Etiology: Crush injury in which the sesamoids are crushed in the sagittal plane between the 1st met and the ground. avulsion. ORIF. transverse fractures and require splinting to the adjacent digits with coban or with a silicon appliance. orthoses. padding. AVN. non-displaced. capsulitis. abduction-adduction • Presentation. stub. arthroplasty. POP & ROM. ISL rupture & sesamoids displaced medial & lateral to met head. must be can be closed reduced.buddy splinting. FHL.common in lateral inversion ankle sprainsà Tx: 4-6 wks WB immobilization 2. FHB tendon. Stewart . • Mechanism of Injury – crush. falls from height. metatarsus primus elevatus Clinical presentation: pain and swelling is sually unsidious. direct trauma. hyperextension. soft tissue release. sesamoids not fractured sesamoid is fractured Tx . neurovascular injury. Treatment: RIICE. ccs injection. jumping or improper shoe gear Biomechamical abnormalities that predispose: Pes Cavus. Jones fractures: Ø 1-1. ABH muscles. is in the appropriate location Treatment.E-I-E-I-Epiphysis 123 . closed reduction & external fixation. slipper or bk cast can be closed reduced. implant or capital osteotomy w/ K-wire fixation Sesmoiditis • • • • • • Anatomy: 2 sesmoid bones are present. If fx of sesmoid is presentß poor prognosis and high non-union rate owing to poor vascularity. metatarsalgia.4 wks. ADH. disruption of entire TMJ/Lisfranc's jt. LAT – flattening of medial column. increased reaction to chronic stress evidence of periosteal new bone formation Tx – Unna boot w NWB BK cast immobilization or ORIF w/medullary curettage + autogenous inlay bone graft 3. avulsion fx must be differentiated from os cuboideum secondarium.ORIF 83% success. displaced or comminuted Type II .disruption of normal continuity bt medial 4 met & cuboid. laterally Tx . Torg .Fx of distal base of 5th metatarsal à classification system for Jones fracture Type I Acute Type II Subacute fx w/ absence of intramedullary sclerosis.Ø Ø Ø Ø Ø Type I . Hx of repetitive trauma & recurrent symptoms. hx of previous injury or fx. 1st met not affected § Type C (divergent) C1 partial divergence with 1st met medially and some of lesser mets. capsule ligamentous strain in cavus foot.bone graft produres 2.intraarticular styloid process fx LISFRANC'S [Tarsometatarsal jt] FRACTURES/DISLOCATIONS • • • Anatomy . fx w delayed union & intramedullary narrow sharp fx margins. direct trauma. 2 & 3 laterally C2 total divergence with 1st met medially and all lesser mets.clinical/radiographic evidence of prior injury non-union type IA or IB Type IIIA . longus tendonitis. MO .extraarticular avulsion fx of epipysis w longitudinal fragment. calcaneocuboid arthritis. in a sagittal or tranverse plane.xrays . crush – arthrodesis Cuboid syndrome “subluxed cuboid” 124 . twisting or equestrian injury occuring as foot is caught in stirrups. periosteal new bone formation Tx . but not 5 B2 lateral displacement of one or more of lesser mets. crush fx. os perineum or os vasalianum DDx . alone or with met's 2 -4.Lisfranc ligament is the strongest interosseous lig attaches med cuneiform to 2nd met base.P. § Type B (partial or isolateral incongruity) dislocation of a portion of TMJ in a sagittal or tranverse plane B1 medial displacement of 1st met. AP – diastasis 2-5mm bt base of 1st & 2nd met base. Chapman • • • • • Type IA . Tx – simple – short leg walking cast 6-8wks.non-articular styloid process fx Type IIIB . consider compartment syndrome Complication – frequently missed injury >> post-traumatic arthritis Trauma to the cuboid Cuboid (Nutcracker) Fractures • • • • 5% of all tarsal fractures. 6wks immobilization. in peds o Can be classified as a Salter Harris I Type III Chronic fx w non-union & complete obliteration of the medullary canal by sclerotic bone. minimal sclerosis. open vs percutaneous sx & fixation. widened fx margins involving both cortical hypertrophy of periosteal cortices. industrial injuries or MVA.Classic extra-articular jones fx Type II – intra-articular fx of 5th met base o This injury is the result of shearing force caused by the internal twisting of the forefoot while the peroneus brevis is contracted Type III – extra-articular avulsion fx of styloid process o Fracture line is usually perpendicular to the long axis of the met base Type IV – intra-articular comminuted fx o The mechanism of injury is similar to Type II.Jones fx. Dx .direct crush injuries to jt or indirect forced abduction of forefoot or loading of plantarflexed foot. k-wires vs screws. avulsion fx of medial 2nd met base & lateral base of 1st met > diastasis = fleck sign in 90%.always do contralateral films. displacement of all mets usually lateral. avulsion fx d/t tension on inferior calcaneocuboid ligament Mechanism of injury – axial & rotatory forces while foot contacts ground in PF assoc w/ 5th met base & calcaneus fx. but in this case the 5th met base gets crushed between the cuboid and the ground/shoe causing the fragmentation Type V . bt lat 3rd met & lateral cuneiform.accute non-displaced fracture at metaphyseal -diaphyseal junction (Jones fx) Type IB . No ligament b/t 1st & 2nd met baseà This ligament is key to the stability of the Mortisse Mechanism . CT is best study for Liscfranc injury Classification Hardcastle (Queno & Kuss) § Type A (total or homolateral) – mc . dropped cuboid. dorsal displacement of mets. mc on lateral aspect at calcaneocuboid jt & base of 5th met base. STJ. assoc w/ lisfranc dislocation. motorcycle & plane accidents – aviators astragalus • Type I (20%) . bone graft or amputation. ORIF via anteromedial approach w screw fixation through head & neck of talus perpendicular to fx line. talectomy.• • An uncommon cause of overuse syndrome or traumatic midfoot pain. AVN . pantalar or triple arthrodesis. widest anteriorly & more secure in mortise in DF.displaced vertical fx of the talar neck w/ dislocation of the STJ.91% • Type IV . Tx short leg cast NWB immobilization x 4-6wks • Type II . skeletal transfixation.dorsal avulsion lip fracture (most common navicular fx . artery of the tarsal canal.transverse fx of the body (29%) w dorsal fragment dislocation vertical or horizontal o mechanism: Fall from height /longitudinal force along the ray when ankle PFed.50%) • Type III .lg dorsal fragment & small plantar fragment. external fix Healing ß Hawkin's sign subchondral radiolucency at fx site 6-8wks after injury.2% of midfoot.tuberosity fracture (24%) mechanism: forceful eversion >> avulsion fx by PT tendon. • • • 125 .displaced vertical fx of talar neck with STJ & Ankle joint dislocation.Posterior tibial > medial calcaneal artery Classifications 1. Orthotics & shoe modification • Type III . Dx CT. bone stimulation. AVN . All of the 3 main blood supply to talar body is disrupted .Posterior tibial àdeltoid artery & artery to tarsal canal. avascular necrosis .artery of the sinus tarsi. body fx & stress fx. short leg walking cast Complication – chronic pain and arthritis Trauma to the navicular Os tibialum externum fracture Commonly a disruption of the existing synchondrosis or synfibrosis of the os and the navicular Tx: Immobilization.artery of the sinus tarsi. 60% covered in articular cartilage. The ability to walk on heels or toes but not heel-toe is pathognomonic. Hawkins Classification .0-15% • Type II (42%) . NWB cast for 6-8 weeks Type II -IV closed reduction.hyperextension & forced dorsiflexion of foot causing the neck of the talus to impact and fracture against the anterior lip of the distal tibia. tx w TN & NC arthrodesis. Blair fusion. 1 of the 3 main blood supplies to the talar neck is disrupted . must r/o os tibiale externum & MTJ subluxation.BK.40%) o mechanism: plantarflexion & forced inversion causes avulsion fx via dorsal talonavicular ligament.displaced vertical fx of the talar neck with dislocation of STJ. Mechanism of injury . bone scan. Tx – NWB short leg cast 4-6wks & resume activity in 3-6 mths Fractures of Talus Anatomy – no muscular or tendinous insertion or origin. indicates revascularization & r/o AVN. Tx . Dx bone scan Tx compression dressing & elevation . Mechanism of injury – direct trauma or rotational forces. horizontal plane fx . Tx short leg cast NWB immobilization 6-8 wks.artery of the sinus tarsi & artery of the tarsal canal. avulsion fx. AJ & TNJ.nondisplaced vertical fx of the neck of the talus.Talar Neck Fx (50%) describes injury + predicts prevalence of AVN. 3-5% of foot fx Blood supply to talus: Head . PT tendon strengthening.42% (15 .artery of the sinus tarsi. 2 of 3 main blood supplies to the talar body is disrupted . Peroneal art à A to sinus tarsi Posterior Tubercle .closed reduction. if not work longer immobilization 3-4 mths. 2nd most common tarsal fx. mva. Short leg cast -12 wks (6NWB & 6 WB • Type IV (as described by Eftekhar) .stress fx of middle 1/3 in young athletes. differentiate from overuse syndrome. All 3 main blood supplies to the talar body is disrupted . icing and orthosis. if crif/orif fails >>> loss of keystone of the medial longitudinal arch >>> flatfoot deformity. deltoid artery. plantarflexion -eversion causes avulsion via dorsal tibionavicular ligament. Cuneiform Fractures 4. Reduction is accomplished with the “black snake heel whip maneuver” and maintained with a strapping and a cuboid pad.100% Treatment Type I .Anterior tibial àdorsalis pedis Body . High rate of recurrence is indicative of early excision Navicular Fracture Ø Anatomy o The blood supply is precarious–anastomsing of the DP & medial plantar artery-where the central portion of the navicular is relatively avascular owing to the large amount of cartilage covering the surface increasing the risk of AVN and pseudoarthrosis o PT tendon attached at navicular tuberosity Ø Mechanism of injury-compression forces that impact the medial column against the head of the talus Classification Watson & Jones • Type I . artery of the tarsal canal & deltoid artery AVN . AJ. non or mal-union.fx of the lateral process of talus. manipulation with knee flexed to 30o ankle at 90o and foot everted and immobilization in short leg cast after 7 days for 6-8 wks.Intra-articular calcaneal fxs BIG ON BOARDS • Type I . Sneppen . Rowe .Extraarticular fx of calcaneus • Type I o IA) Fx of the medial tuberosity o IB) Fx of the sustentaculum tali o IC) Fx of the anterior process – mc (avulsion fx of bificate ligament) • Type II o IIA) Beak Fx no ACH involvemt o IIB) Avulsion Fx of Tendo Achilles • Type III . Essex-Lopresti .AVN Calcaneal Fractures • 60% of all tarsal fx.Talar Body Fx • Group I . 2nd most common talar body fx AKA Snowboarder’s Fx eversion injury w/ lateral process caught b/t fibula and calcaneus. d/t severe plantarflexory force).Mondor's sign (ecchymosis from lat malleolus to sole of foot) & inability to bear wt • Dx . Gissane's angle n 120-140. seen in athletes esp ballet dancers o Sym . males >45yrs. then 6-8 wks BK WB cast if no sign of AVN or delayed or nonunion.Steida's process or sheperd's fx. BK NWB cast x 6-8wks.LAT xray Bohler's angle n 20-40 – Decreased.transchondral or compression fx to talar dome including osteochondritis dessicans • Group II .horizontal fx • IIA – non-displaced fx dividing talar body into superior & inferior halves • Tx . o Tx .displacement of trochlear articular surfaces • IC . associated w falling from a height > 6ft.closed reduction or ORIF.non-displaced • IB .crush fx (comminuted) of talar body o Dx – xrays AP. Increase.tongue type fx d/t vertical fall.comminuted fx of STJ w/ central or severe depression Tx – surgery pull calcaneus tuberosity distally. • Classification 1. AKA "basket-ball" or acquired clubfoot. CT Subtalar Joint Dislocation”basketball foot’ Atraumatic inversion force to the STJ may result in medial dislocation of the clacaneus. DDx .extraarticular oblique Fx of calcaneal body not involving STJ • Type IV . LAT.Medial STJ dislocation. AVN 2.coronal. fill calcaneal defect with bone chips. L1 mc. 2° fx line exists posteriorly • Type II . 1° fx line exists plantarly. 75% intrarticular M:F 5:1 • Mechanism of injury .coronal or sagittal fx • IA . os trigonum syndrome. infection. intra-articular fx. screw fixation from lateral into sustentaculum tali 2.severe dorsiflexion w/ compressive forces when talus is sandwiched b/t tibia & calcaneus o Type I .displacement of trochlear articular surfaces w STJ dislocation • ID . 1° line exists plantarly. • Group III -fx in the posterior tubercle (most common fx of talar body. When complete dislocation occurs the talocalcaneal and cervical ligaments are injured (cruciate ligaments of the foot).degenerative OA of the STJ. • Group V .joint depression fx d/t posterior fall.total dislocation of talar body w displacement of STJ & AJ o Type II . elevate posterior facet. BK WB cast x 6wks or surgical excision of lateral tubercle • Group IV . Etiology .pain in posterior ankle causing limited ROM reproduced on FHL movement.Fx involving STJ w/o jt depression or comminution • Type V . 2° fx line exists posterosuperiorly (dorsally) 126 .Lateral STJ dislocation. Buckingham classification • Type A .axial load drives lateral process of talus into calcaneus. complication .BK NWB cast 6-8 wks • IIB . navicular and distal bones with the talus undisturbed.injection therapy of LA/steroid q3wks. superior portion shifts on inferior portion Tx . calcaneus medial to talus • Type B .Complications.displaced fx. sagittal or horizontal shearing fx of entire talar body. 20% associated w spinal fx b/t T12 & L2. calcaneus lateral to talus • Type C – Anterior & posterior STJ dislocation Tx – RICE. mc. • Symp . posterior tibial. 4 fragments w commminution Tx . Mortise ankle. staged relative to sequential pattern of injury.0cm (2 1/2in) proximal to it’s tip accompanied by a rupture of the syndesmosis and either fracture of the medial malleolus or a tear of the deltoid ligament (PER III) 5. posterior tibial artery. short leg walking cast closed reduction & short leg cast. Dupuytren fracture: a fracture of the fibula about 6. PITF interosseous talotibial. tibia. talus. peroneal artery Nerve supply. ORIF 2. interosseous talofibular ligament and interosseous membrane). 3 fx lines. PER most destructive Supination/ADD Rupture of the lateral collateral ligaments Transverse fibular fracture DWA Oblique fracture of the medial malleolus Lauge Hansen Classification of Ankle fractures Pronation/Abduction Supination/External rotation Rupture of the deltoid ligament Rupture of the AITF/PITF or TC Transverse fracture of the medial or WF fx malleolus Rupture of the AITF/PITF or TC or WF fx Oblique fracture of the fibula DWB Butterfly fragment Spiral fracture of the fibula DWB Posterior spike on lat X-ray Volkman’s fracture or PITF rupture Rupture of the deltoid ligament Transverse fracture of the medial malleolus ORIF Pronation/External rotation Rupture of the deltoid ligament Transverse fracture of the medial malleolus Rupture of the AITF/PITF or TC or WF fx Spiral fracture of the fibula DWC Massoineuve Volkman’s fracture or PITF rupture <30% of the articular surface is not fixated ORIF I II III IV TX closed reduction. MRI.fibular fx line below the tibiofibular syndesmosis Most likely a stable fx and does not need ORIF. SER most common.articular Fx of posterior facet. Wagstaffe-Lefort fracture: avulsion fracture of the fibula 9.lateral. posterior – PT. posterior displacement of the fibula and Volkman’s fracture 4. use CT to determine location & fx lines through posterior facet & # of articular fragments. FDL. fx occur in clockwise pattern for rotational injuries & direct blows pattern across the ankle for abduction & adduction injuries.obj to restore architecture of calcaneus by reestablishing height & width of the heel & relocating the posterior facet to its anatomical position. Lateral – PL.NWB cast x 6wks • Type II . = SAD 1 127 .nondisplaced articular Fx of posterior facet. Frost fracture: pediatric triplane fracture with the combinationof a SH2 lateral view and SH3 (Tillaux fracture on AP view) of the distal tibia Classification 1. guide for surgical repair & prognosis Talus divided into 3 equal columns by 2 fx lines (A & B) & 3 pieces of posterior facet. 3 fragments w depression of central fragment • Type IV .II-IV . C is 3rd fx line separting posterior facet from sustentaculum & creating a 4th fragment. Cotton fracture: Trimalleolar fracture 3. Eponyms: Pott’s fracture: transverse fracture of the fibula 5. talus move relative to the leg & tibia moves relative to talus.lateral • Type III . Arthrography. 1 fx line. second term is direction/motion of pathologic force on the talus. syndesmosis Blood supply – anterior tibial artery. Maissoneuve fracture: Spiral fracture of the fibula that occurs at the anatomical neck 6. CT.articular Fx of posterior facet. ORIF via lateral hockey stick (palmer) incision to maintain peroneal tendons & sural nerve in flap Ankle Fractures • • • • • • Anatomy – Bones . EHL.Fibular Fx in Ankle Injuries • Type A . PTF and CFL) syndesmotic interval (AITF. 2 fragments. Cowtail fracture: an anterioposterior tibiofibular PER frcture involving rupture of th einterosseus membrane with an associated high fibular fracture 7. superficial peroneal Muscles/tendons – anterior – AT. Stress views.Sural saphenous.articular Fx of posterior facet.central. FHL.3. Ligaments – lateral colateral ligaments (ATF. EDL & PT. A. Sanders (CT) CT classification of intraarticular calcaneal fxs. Tillaux fracture: Avulsion fracture of the tubercle of Chaput of the tibia 8. 2 fx line.0-7. C . Bone scanning 1. B . PB Ginglymus (hinge) joint à DF & PF and gliding Dx – Xrays – AP. deltoids. • Type I . Lauge Hanson Ankle Fx Named after a Denmark physician. Danis-Weber . Tx . Bosworth fracture: Bimalleolar fracture.fibula. first term describes position of foot at time of injury.5cm above the distal end and associated with a tear of the delotoid ligament and lateral subluxation of the talus secondary to valgus injury 2. MO. LAT. unstable and needs ORIF. Tension band wiring used for compression or lag screw technique with 4. Type III. 3rd & 4th met. chronic tenosynovitis. if fracture unfixable .Re-establishing the length & position of the lateral malleolus is the most critical element of management.shear force . Soft cancellous metaphysis of distal tibia is compacted >> large void when length is resotred.fibular fracture line begins above the tibiofibular syndesmosis.Midsubstance tear at the level of the medial malleolus. worse when fx crosses physis completely covered by articular cartilage.Longitudnial splits without complete rupture. BK. pes valgus.• Type B . RA. Johnson & Strom Stage I Stage II Stage III Tendon length normal. Indirect trauma is the mc mechanism of ankle injury in children. PF. corticosteroid injection.4% tibia\fibula. longitudinal arch collapses. spring ligament stretched.5mm cortical screws and Tubular plate w two screws both above & below the fx. does not involve physis Periosteal avulsion POSTERIOR TIBIAL TENDON RUPTURE Anatomy PT originates from fibular and tibia inserts into navicular + medial & intermediate cuneioform. 1986 Group 1 .Tenosynovitits without visible disruption Treatment – conservative: .reconstruct the fibular 2. Watershed area behind medial malleolus. Jack’s test – lose heel inversion with heel lift.bone graft the metaphyseal deficit 4 – buttress plate the tibia medially. = PER 3 Tx . Tx – FDL side to side anastomosis Group 3 .9% foot) are more common during the first year average age 10-12 & puberty.Tx debridement & synovectomy and sheath decompression Group 4 . base of 2nd. If superiorly & posteriorly displaced à ORIF. Tx reattach tendon at navicular Group 2 . Foot FF adduction & inversion then neutral BK WB 2-4 wks. Types III & IV -bending/ compression force . Classification Reudi Algower Type I Type II Type III Distal tibial Fx without displacement Distal tibial fx with significant Distal tibial fx with significant displacement and displacement comminution Tx combination of internal & external fixation. • Type C . does not involve physis Metaphyseal fracture. IV. 5.good if physis separates from metaphysis.ex fix 5-8 wks then ankle fusion Ø Ø Ø Ø Epiphyseal Fractures Physis is radiolucent cartilaginous growth plate. 2 syndesmotic screws are used to correct diastasis & should engage a minimum of 3 cortices. bending. Injury d/t disturbance of physeal blood supply based on fx type . screw placed 3-5cm proximal to ankle joint Pilon Fractures Mechanism of injury – direct impact of talar trochlear against the distal tibial articular surface. Fibula fixated with 3. NWB cast 6wks. Rupture >> PF adducts & subluxes TN joint. Blood – sural. inverts & Adducts foot.Avulsion of PT tendon at the insertion. Mechanism of injury . calcaneus everts Classification 1. hindfoot mobile >> increasing pain Tendon elongated. peroneal & posterior tibial arteries Mechanism of injury – direct laceration – rare eg medial malleolus fx (PER IV). peritendonitis & or Tendon elongated. Physeal injuries comprise 10% of all pediatric fx (4.shear. If blood supply is not disturbed heals in 3 wks. and more frequent in boys. Nerve – posterior tibial. Funk's Classification surgically Based. choparts jt abducts. V – ORIF restoration of jt congruency using smooth k-wires may cross physis. avulsion. 4 steps 1. Type I & II .0mm cancellous screws.Not Tx of choice.older children. Tx Type I & II – closed reduced. chronic or acute stress on degenerated tendon – CVD. may or may not be stable so clinical judgement is indicated. Surgical – Tx of choice 2. axial compression.physeal injury I I III IV V V VI VII VIII Epiphyseal Plate fractures: Salter Harris Same Along the growth plate Above Above the growth plate-exhibits a Thurston Holland Sign (triangular spicule of bone) Werinskoild sign Lower Below the growth plate Through Through the growth plate Really Bad Compaction of the growth plate Zone of Ranvier fracture Epiphyseal fracture. hindfoot 128 . Classification Salter-Harris .younger kids. poor when fx cross the physis.reconstruction of tibial articular surface 3.fibular fracture at the level of the tibiofibular syndesmosis Usually a spiral (SER 2) or oblique fx ( PAB 3) of the fibula. Dorsal – over 2nd & 4th mets. ABH. FDL. crush injuries. Koutsougianis. Long incisions made to depressurize the compartment. puncture. muscle contracture. FDL. indirect.tendon degeneration >> normal foot focal pain proximal to medial malleolus 3-6mths of rest. degenerative. Peroneus Tertius Peroneus Longus Peroneus Brevis Posterior Compartment Superficial: Grastocnemius . TP. Resting compartment pressure . EHL. Medial – ?? Leg .take to OR & perform open fasciotomy ASAP to prevent tissue necrosis & contractures. defect 3-6cm Repair end to end anastomosis with autogenous tendon graft flap. NSAIDS. FDMQ. tendon debridement. NO TOURNIQUET! Protect from infection. Do not elevate limb >> ^ ischemia. Lumbricals FHB. fluoroquinolones Dx – Tenography. deep heel cup 4-6mm Kirby skive Synovectomy. crush. Hydration – IV fluids helps protect kidneys and prevent renal failure from excess myoglobin. Evans. wound packed open & delayed primary closure in 5. tendon length TENDOACHILLES RUPTURE Is anastomosis of tendons of gastrocnemius and soleus and inserts into calcaneus. Koutsougianis.7 days. Stryker) inserted into muscular compartment and measures pressure. Watershed area 2-6cm above insertion. u/s CTand MRI is best. Evaluation and assessment 129 .Ps – Pain out of proportion. Evans. defect >6cm Gastrocnemius recession followed by end to end anastomosis with a free or synthetic tendon graft • • • Classification: Kuwada Type I Partial tendon tear 8 wks cast immunization if tear constitutes less than 50% of tendon Compartment Syndrome Excessisve pressure in the muscular compartments of leg or footà ischemia and death of muscle tissue. or double incisions through – anterolateral compartment for anterior & lateral compartments and posteromedial compartment to decompress both superficial and deep posterior compartments Complications Volkman’s contracture of muscle . Pressure Diagnosis Wick catheter (eg. 80% occur in LE. heel raise pain on medial side d/t DJD FDL transfer under tension. Dwyer. Pink. spontaneous – post traumatic. Paralysis. exercise. muscle ischemia. Symptoms 6. undersurface of navicular via drill Realignment with STJ hole. Surgical . burns. Kidner. Dwyer. area of poor vascularity Mechanism of injury – direct injury – laceration. Magic angle – tendon oriented 55o from main magnetic field will create illusion of a tear in a normal tendon Type II Complete rupture. STJ arthroereisis. Incisions: Foot . BK cast 6wk + PT proximal attachment of TP muscle to arthrodesis . Young. EDL. Anatomy Review Compartments in Leg ** Remember No Medial compartment** Anterior Compartment Lateral Compartment TA. early ankle joint degeneration.single incision over peroneal compartment & all compartments can be decompressed. weakness on deformed & stiff >> fixed flat foot. Intramuscular pressure > 30mmHg for > 8 hours is diagnostic Formula – MAP (mean arterial pressure) – Pulse = ^P (Delta P) if GREATER than 30mmHg normal NO COMPARTMENT SYNDROME Treatment Podiatric Emergency Supportive: Remove all dressings & casts. Soleus. During exercise pressure increases to 50mmHg and then returns to normal in 5-10minutes. Pulses present or pulselessness. orthoses. Anterior and deep posterior compartments are most often involved. Plantaris Deep: FHL. fractures (Calcaneal). FF abduction. ABDM Quadratus Plantae. RF valgus. ADH 4DAB interossei and 3PAD interossei Etiology Trauma. corticosteroid injection.5mmHg. surgery. Physical Medicine and rehabilitation 1. defect < 3cm Typically end to end anastomosis Type III Complete rupture. isolated arthrodesis isolated arthrodesis Stage IV – added by Myerson – rigid hindfoot & valgus angulation of talus.2 dorsal and / or 1 medial. Paresthesia. rest BK cast 3 wks along tendon. Renal failure secondary to myoglobinuria C. tight cast. muscle necrosis. Popliteus Compartment in Foot 1st Layer 2nd Layer 3rd Layer 1 4th Layer FDB. possible augmentation w/ synthetic graft Type IV Complete rupture. pacemakers Therapeutic heat Hot packs. Direct (Galvanic) current: monophasic currents that provide a constant electron flow from the (-) electrode to the (+) with no oscillation or alterationsà major chemical and thermal rx in the tissue leading to muscle stimulation Alternating current: Biphasic or polyphasic current that provides alternating current from the (-) to (+) and v/v. compliance and consistensy of the patient is an issue 2. and ulcer debridement is performed using the mechanical effects of the moving water. pain.0 MHZ is absorbed in superficial tissues. 3. pregnancy. Can be contraindicated in pulsed (greater mechanical or non-thermal effects that are to stimulate healing. fluid collection in Diathermy Conversion of electrical to magnetic to heat energy w/greatest build-up of heat tissues. malignancy. joint effusion. Iontophoresisà use of low voltage DC current to move ions through tissues Excitation of nerve cells. reduce edema by deep immersion using the hydrostatic pressure effect. slows nerve conduction. boney prominence. eye. • Advantages-Modality is applied directly to that body part. with possible nerve damage if over irradiated. Analgesic effects (Peripheral and central) Gate theory of pain control Mucle contraction. with prevention of physical disabilities and with restoration of function to patients disabled with disease. preganancy and pts with peripheral neuropathy Loss of sensations. increase joint mobility. enhance tissue and bone healing 130 . decreases metabolic activity. HBP. penetrates 1-2 cm. Average time of treatment is 6-8 minutes which is dependent upon depth and area affected. It heats tissue with a combo of induced sweating. tenosynovitis. contractures and pain Contraindications Vasospastic dz. Fluidotherapy Deep heating modalities Ultrasound High frequency electrical current causes vibration of crystals in the U/S head producing a sound wave. increase in the extensibility of collagen tissue. Hydrotherapyà Mechanical effects: reduce weight bearing (buoyancy). or electrical currents and the vibration it imposes on the molecules of a tissue. tissue healing. It is the health profession concernes with of health. gradually absorbed and converted to heat with the greatest build up of heat in the: peripheral nerves-à hematomas. Subactue or chronic inflammation Chronic stages of arthritis. or Phonophoresisà use of U/S to drive molecules into the skin for anti-inflam or anesthetic properties (Lidocaine. immediate results can be gained. enhance muscle fiber recruitment. PVD. open wounds. excessive in fatà muscleàboneà peripheral nerves. smooth and skeletal muscle contraction. muscle spasms and spasticity. Acute inflammation. Short Wave Application of high frequency currents (radiowaves) for therapeutic use. pain relief. chronic treatment.0 MHZ is absorbed into deeper tissues. protein synthesis.) or continuous. pain control. over heart. lessens muscle tone. hemorrhagic dz. myositis. ASA. infrared. increase in tissue metabolism. sprains. exposure to gonads. asisstance in the resolution of inflammatory infiltrates Indications Acute inflammation Mechanical trauma. thrombophlebitis. only stimulation of muscle. strains. and fibroblastic and osteoblastic stimulation. Diagnosis • Complete H&Pà working differential diagnosis 3. testicles hydrocortisone. fibrocytic stiffness.Description: Treatment of pain and disease by physical means. decrease in jt stiffness and edema. apophysitis. dexamethasone-not creams-are air filled emulsions and poor transmitters) 1. dislocations. bursitis. it is non-habit forming when need to treat chronic infections • Disadvantages-Time consuming. adjunct in muscle re-education. Treatment (physical medicine modalities) • Modality Cold therapy Description Icing à vasoconstriction. menstruation Electrotherapy Description Physiologic response Clinical use Ambulatory aids Stimulation os muscle tissue using direct or alternating currents. bleeding. reduces muscle spasms and spasticity and produces superficial anesthesiaà reflex vasodiltion: Hunter’s response Vasodilation. tendonitis. peds with Peizoelectric effect: electrical to mechanical energy conversion. reduction in muscle spasm. paraffin baths. penetrates 3-5cm. changes in membrance permeability. arthritis. changes in microcirculation. infections. injury or pain. costly. The sound is transmitted into the tissue. open epiphyseal sites. metal implants. There is no concomitant thermal or chemical rx. muscle pumping action to improve lymphatic and venous flow. boneà muscleà fatà blood. ischemia. UV Superficial heating modalities Monochromatic Infrared Radiation “Anodyne” à stimulates the RBCs and cells of the endothelial lining of the microvasculature to release NO into the blood stream reducing tissue inflammation and promoting wound healing. moist dressings. it is good for absorbing vertical forces Polyurethane foams Thermosetting. leather or synthetic piece of rounded material applied externally to the sole or internally to the innersole with its apex just behind the met heads An angulated sole with its apex just behind the met heads which promotes a rocking action and eliminates the toe break at push off. It is used extensively in the treatment of the diabetic and arthritic foot. it is grindable and it is washabale Polyolefins Polyethylenes-accomodative. restrict or control function of the muscusculoskeletal system UCBL Used for the treatment of flexible flat foot. reducing WB pressure on bony prominences Orthoses-A device used to mechanically assist. commonly used of rrunning or walking shoe insoles. will deform over time and can be used to produce a temporary function device Plastizote #1 soft #2 firm # 3 rigid this is a closed cell. cross-linked polyethylene foam it is a high quality. fabricated around a WB impression of the foot and utilizes a deep heel cup with long high medial and lateral flanges to restrict pronation. closed cell polyethylene foam that is non-allergenic. then advance the weak extremity. Move both crutches forward simultaneously. grindable and washable. Instruction for walkingà crutches should be placed 6-12 in in front and outside both feet. unbreakable and heat moldable TL-2100-tough. Usually posted with EVA or ethylene vinyl acetate acrylnitrile designed to replace Rohadur which is rigid. excellent memory-will not deform if thickness is appropriate for the pts wt. Frequently used with CP or other neurological conditions Blake inversion Originally developed to counteract marked pronatory forces encountered in running. can be used for functional or accomodative orthoses Steel Used for brace components and shoe shanks Aluminum Used for brace components and shoe shanks-lighter weight Rubber Neoprene or Spenco a closed cell expanded rubber material . and HAV pts AFO CROW Bracing and biomaterials Leather Earliest orthotic material. Poron-PPT It is an open cell material made of polyurethane foam It has excelled abilities to reduce direct pressure as well as shear It generally is smooth on one side and abraded on the other making it somewhat uncomfortable when applied directly against the foot Its advantages include the fact that it does not “bottom out” with use. lightweight. Kirby medial heel skive Gait plates Rigid devices with extended medial (induces in toe) or lateral (induces out toe) anterior margins used to alter a child’s gait to achieve a more cosmetic appearance 1st ray cut-out Developed to address the need for independent 1st ray motion and propulsion.ultra high density Polopropylenes-high density thermoplastic. resilient. can and involved limb move together Walker height is set so that the elbows are flexed 20o A rubber. Utilizes a significantly inverted standard off WB cast to produce a heel cup inverted 15o to as much as 75 o. and next advance the strong extremity Use on the opposite side of the injury. enhancing total contact. heat moldable Polydur Graphite 131 . Commonly used with hallux limitus. It is heat moldable.Crutches Canes Walkers Footgear Metatarsal bar Rocker bottom bar or sole Measurement: 1) from the acillary area to the bottom of the heel and add 1 or 2 more inches 2) Measure fomr the anterior fold of the axilla to 6in anterolaterally from the foot. It will quickly mold to foot contours during weight bearing. not heat moldable. It is not very effective as a shock absorber Polyethylene thermoplastics Ortholen-High density Sub-ortholen.
Copyright © 2024 DOKUMEN.SITE Inc.